Skip to main content

Full text of "Vascular-Surgery-Books_1aim.net"

See other formats


Georce Geroulakos 

Hero van Urk 

RobertW. Hobsonll Editors 




# 



• 



f 



u 





Cases,. Questions and Commentaries 
Secord Edition 



© Springer 



Vascular Surgery 



George Geroulakos, Hero van Urk and 
Robert W. Hobson II (Eds) 



Vascular Surgery 

Cases, Questions and Commentaries 



Second Edition 



4h 



Springer 



George Geroulakos, MD, FRCS, DIC, PhD 

President, Section of Vascular Medicine 

Royal Society of Medicine 

London, UK 

and 

Consultant Vascular Surgeon and Senior Lecturer 

Department of Surgery 

Charing Cross Hospital 

London, UK 

Hero van Urk, MD, PhD, FRCS 
Professor of Vascular Surgery 
Erasmus University Medical Center 
Rotterdam, The Netherlands 

Robert W. Hobson II, MD 

Professor of Surgery and Physiology 

Director, Division of Vascular Surgery 

UMDNJ-NJMS (University of Medicine and Dentistry of New Jersey - New Jersey Medical School) 

Newark, New Jersey, USA 

Artwork or chapters marked with £0 symbol throughout the book are original to the 1st edition 
(Geroulakos G, van Urk H, Hobson II RW, Calligaro K. Vascular Surgery: Cases, Questions and 
Commentaries, first edition. Springer London Ltd 2003) and are being republished in this second 
edition. 

British Library Cataloguing in Publication Data 

A catalogue record for this book is available from the British Library 

Library of Congress Control Number: 2005932895 

ISBN-10: 1-85233-963-2 e-ISBN 1-84628-21 1-x 

ISBN-13: 978-1-85233-963-0 

ISBN 1-85233-533-5 1st edition 

Printed on acid-free paper. 

© Springer-Verlag London Limited 2006 

First published 2003 
Second edition 2006 



Apart from any fair dealing for the purposes of research or private study, or criticism or review, as 
permitted under the Copyright, Designs and Patents Act 1988, this publication may only be repro- 
duced, stored or transmitted, in any form or by any means, with the prior permission in writing of 
the publishers, or in the case of reprographic reproduction in accordance with the terms of 
licences issued by the Copyright Licensing Agency. Enquiries concerning reproduction outside 
those terms should be sent to the publishers. 

The use of registered names, trademarks, etc. in this publication does not imply, even in the 
absence of a specific statement, that such names are exempt from the relevant laws and regulations 
and therefore free for general use. 

Product liability: The publisher can give no guarantee for information about drug dosage and 
application thereof contained in this book. In every individual case the respective user must check 
its accuracy by consulting other pharmaceutical literature. 

Printed in China (EXPO/EVB) 

987654321 

Springer Science+Business Media 
springer.com 



This book is dedicated to the memory of 

Polychronia Geroulakos, Senior Sister in the 

Geroulakos Clinic in Sparta, Greece. For 

almost 40 years she looked tirelessly after 

the patients of this institution in an 

exemplary manner leaving a legacy of 

high standards. 




Union Europeenne des Medecins Specialistes 

SECTION AND BOARD 
OF VASCULAR SURGERY 




President of the Section of Vascular Surgery: F. Benedetti-Valentini 
Secretary / Treasurer of the Section and Board of Vascular Surgery: 
M. Cairols 



President of the Board of Vascular Surgery: K. Balzer 
Vice President of the Board of Vascular Surgery: 
A. Nevelsteen 



Barcelona, March 2006 



Vascular Surgery is a discipline that deals with one of the true plagues of the 20th 
century. Moreover, atherothrombosis will continue to be the main cause of death in 
the near future. 

New developments in the investigation, and endoluminal treatment of vascular 
disease have recently attracted significant publicity from the mass media and 
patient groups, and have significantly changed the management of the vascular 
patient. 

The provision of a high quality vascular service is closely linked with the need to 
give residents an appropriate training and to further introduce Vascular Surgery as 
an outstanding specialty. 

The book, "Vascular Surgery; Cases, Questions and Commentaries", by 
Mr. Geroulakos, Prof Hero van Urk and Dr. R W Hobson II, will indeed contribute 
to a better understanding of Vascular Surgery as a specialty that deals with the 
pathology of arteries, veins and lymphatics. The experience and the teaching capa- 
bilities of the authors are unquestionable. 

This book, being so comprehensive, enhances the idea of considering Vascular 
Surgery as an independent entity from other specialties. Before achieving adequate 
competence to deal with the variety of cases shown in the book, the need for an 
appropriate training is obvious. Besides, the present text will help candidates to 
better prepare for the EBSQ-Vasc examination. The book utilises a time proven 
concept for teaching by questions and answers based on real problems, an essential 
part of CME. The book proposes learning following the Socratic method, by exer- 
cising our mind rather than reading told facts. On the other hand, it may improve 
our clinical practice and care of our vascular patients, as it incites Continuous 
Professional Development as a step forward in CME. 

The European Board of Vascular Surgery congratulates the authors for their ini- 
tiative and gladly endorses the book. 

Marc Cairols 

Secretary General 

UEMS Section and Board of Vascular Surgery 



Foreword to the First Edition 



This book is rather unique among textbooks in vascular surgery. Most cover the 
surgical management of vascular diseases, in whole or in part, in standard textbook 
fashion, with the text organized to cover the topics methodically in a didactic 
manner, and supported by tables, illustrations and references. Others have special 
purposes, such as atlases on technique or algorithm based books on decision- 
making. All have their place, but if the educational goals are training of the young 
surgeon, self-assessment and continuing medical education for the practitioner or 
preparation for oral examination, this book fills a special need, and fills it very well 
by breaking away from the didactic approach. 

It has long been recognized by educators that retention of knowledge, i.e. true 
learning, are much better achieved using the Socratic method of questions and 
answers, as opposed to simply reading or being told facts. In this book this 
approach is developed and presented in a very effective manner. In each "chapter", 
one is presented with a case report representing a real life scenario. The case 
reports-scenarios in this book together cover most of vascular surgery experience. 
Following the case report, one is presented with questions and answers based on 
various aspects of the case, forcing the reader to commit to an answer. Whether the 
answer is right or wrong is not critical, in fact getting a wrong answer may be more 
beneficial in terms of correcting knowledge and retaining information. The com- 
mentary and conclusions that follow analyze the choice of answers, correct and 
incorrect, and discuss them in concise, authoritative detail, many of which are truly 
"pearls of information". The conclusion then summarizes the current state of 
knowledge on the clinical issues under consideration. Numerous references are 
included. Together, these components constitute one of the most effective vehicles 
for self-education in vascular surgery today. Importantly, all aspects of manage- 
ment are covered: diagnostic evaluation and appropriate treatment, whether it is 
non-operative or interventional, endovascular or open surgery. 

To accomplish their goals the editors have gathered together a large number of 
experienced contributors, many well-known for their special areas of interest within 
vascular surgery, reflected in the contributions they make to this book. As such, the 
book should be useful to future and practicing vascular surgeons all over the world. 
It is full of statements covering most of the current state of knowledge in vascular 
surgery, and it does so in an entertaining and effective manner. 

Robert B. Rutherford MD, FACS, FRCS 

Emeritus Professor of Surgery 

University of Colorado 

ix 



Preface to the First Edition 



This book is a unique collection of real life case histories written by experts that 
highlight the diversity of problems that may be encountered in vascular surgery. 
Each case scenario is interrupted by several questions that aim to engage the reader 
in the management of the patient and to give him the opportunity to test his knowl- 
edge. The comments reflect to as much as possible the principles of evidence based 
medicine and provide the answers to the questions. 

Several chapters are authored by individuals that contributed to the development 
of innovations in the management and prevention of vascular disease and are of 
interest for both the vascular trainee and the experienced vascular specialist. 

The goal of this book is to help vascular trainees review for Board and other 
examinations as well as to provide vascular surgeons who wish to expand or refresh 
their knowledge with an update and interactive source of information relevant to 
case scenarios that could be encountered in their practice. 

The European Boards in Vascular Surgery is a relatively new examination. 
Although the American Boards in Vascular Surgery were established many years 
earlier, there are no "dedicated" guides to cover the needs of these examinations. 
We hope that our book will provide a helpful hand that does not come from the 
standard text books, but directly from daily practice and therefore contains a high 
content of "how to do it" and "why we do it". The references show the close relation 
between daily practice and "evidence based" practice, and we hope the two are not 
too different. 

We would like to thank all the authors who have contributed generously their 
knowledge and time to this project. 

George Geroulakos 

Hero van Urk 

Keith D Calligaro 

Robert Hobson II 



XI 



Preface to the Second Edition 



The authors' principal objective in the first edition of Vascular Surgery was the pre- 
sentation of the principles of vascular and endovascular surgery through interactive 
real life clinical scenarios. The success of the first edition has been gratifying. We 
have received many suggestions for additions and changes from vascular trainees, 
specialists, and teachers at various institutions in Europe, the United States, and 
other parts of the world. These comments have been well received and have been 
important in improving and expanding the second edition. We wish to acknowledge 
our appreciation and gratitude to our authors and publishers. 

George Geroulakos 

Hero van Urk 

Robert W Hobson II 

London, Rotterdam, and New Jersey 

March 2006 



XIII 



Contents 



Contributors xiii 

I. ARTERIAL ANEURYSMS 

1. Preoperative Cardiac Risk Assessment and Management of Elderly 
Men with an Abdominal Aortic Aneurysm 

Don Poldermans andjeroen J. Bax 3 

2. Abdominal Aortic Aneurysm 

Jean-Pierre Becquemin and Alexandre d'Audiffret 13 

3. Endoluminal Treatment of Infrarenal Abdominal Aortic Aneurysm 

Hence J. M. Verhagen, Geoffrey H. White, Tom Daly and 

Theodossios Perdikides 23 

4. Ruptured Abdominal Aortic Aneurysm 

Jeffreys. Weiss and Bauer E. Sumpio 35 

5. Thoracoabdominal Aortic Aneurysm 

Nicholas J. Morrissey, Larry H. Hollier and Julius H. Jacobson II 45 

6. Aortic Dissection 

Barbara T. Weiss-Muller and Wilhelm Sandmann 57 

7. Popliteal Artery Aneurysm 

Jonathan D. Woody and Michel S. Makaroun 67 

8. Renal Artery Aneurysm 

Lutz Reiher, Tomas Pfeiffer and Wilhelm Sandmann 73 

9a. Anastomotic Aneurysms 

William D. Neary and Jonothan J. Earnshaw 79 

9b. False Aneurysm in the Groin Following Coronary Angioplasty 

Steven S. Kang 87 



II. ACUTE ISCHAEMIA 

10. Acute Thrombosis 

Vikram S. Kashyap and Kenneth Ouriel 97 

xv 



xvi Contents 

1 1 . Arterial Embolism 

Andre Nevelsteen 107 

12. Blast Injury to the Lower Limb 

Paul H. B. Blair, Adrian K. Neill and Christopher T. Andrews 115 

13. Endoluminal Treatment of Traumatic Arteriovenous Fistula of the 
Axillary Artery 

Jonathan D. Woody and Rodney A. White 125 



III. MANAGEMENT OF CHRONIC ISCHAEMIA OF THE 
LOWER EXTREMITIES 

14. Cardiovascular Risk Factors and Peripheral Arterial Disease 

Stella S. Daskalopoulou and Dimitri P. Mikhailidis 133 

15. Angioplasty for Critical Arterial Stenosis 

Lars Norgren 141 

16. Lower Limb Claudication due to Iliac Artery Occlusive Disease 

Fabien Koskas and Marcus J. Brooks 147 

17. Erectile Dysfunction due to Aortic Disease 

Ralph G. DePalma 157 

18. Bypass to the Popliteal Artery 

Jeannie K. Chang, Keith D. Calligaro and Matthew J. Dougherty 161 

19. Chronic Critical Limb Ischemia 

Enrico Ascher and Anil P. Hingorani 167 

20. Popliteal Artery Entrapment 

Luca di Marzo and Norman M. Rich 173 

21. Adventitial Cystic Disease of the Popliteal Artery 

Bernard H. Nachbur and Jon Largiader 181 

22. The Obturator Foramen Bypass 

Andries J. Kroese and Lars E. Staxrud 191 

23. Diabetic Foot 

Mauri J. A. Lepantalo, Milla Kallio and Anders Alback 201 



IV. SURGERY OF THE MAJOR BRANCHES OF THE 
INFRADIAPHRAGMATIC AORTA 

24. Chronic Visceral Ischaemia 

George Geroulakos and William L. Smead 215 

25. Acute Mesenteric Ischaemia 

Jonathan S. Refson and John H. N. Wolfe 221 

26. Renovascular Hypertension 

David Bergqvist and Martin Bjorck 231 



Contents xvii 

V. MANAGEMENT OF PORTAL HYPERTENSION 

27. Management of Portal Hypertension 

Yolanda Y. L. Yang and J. Michael Henderson 239 



VI. MANAGEMENT OF EXTRACRANIAL 
CEREBROVASCULAR DISEASE 

28. Management of Patients with Carotid Bifurcation Disease 

Wesley S. Moore 251 

29. Carotid Endarterectomy and Cranial Nerve Injuries 

Christos D. Liapis and John D. Kakisis 259 

30. Paragangliomas of the Head and Neck 

Johanna G. H. van Nes, Sylvia C. dejong, Marc R. H M. van Sambeek 

and Hero van Urk 267 

31. Vertebrobasilar Ischemia: Embolic and Low-flow Mechanisms 

Ramon Berguer 277 



VII. NEUROVASCULAR CONDITIONS OF THE UPPER EXTREMITY 

32. Neurogenic Thoracic Outlet Syndrome 

Richard J. Sanders 289 

33. Thoracoscopic Sympathectomy 

Samuel S. Ahn, Huck A. Mandel and Kyung M. Ro 297 

34. Acute Axillary/Sub clavian Vein Thrombosis 

Jarlis Wesche, Torbjorn Dahl and Hans 0. Myhre 305 

35. Raynaud's Phenomenon 

Ariane L. Herrick 313 



VIII. PREVENTION AND MANAGEMENT OF COMPLICATIONS 
OF ARTERIAL VASCULAR SURGERY 

36. Aortofemoral Graft Infection 

Christopher P. Gibbons 323 

37. Aortoenteric Fistulas 

David Bergqvist 337 



IX. VASCULAR ACCESS 

38. The Optimal Conduit for Hemodialysis Access 

Frank T. Padbergjr, Robert W. Zickler and Joseph M. Caruso 345 

39. Acute Ischaemia of the Upper Extremity Following Graft 
Arteriovenous Fistula 

Miltos K. Lazarides and Vasilios D. Tzilalis 359 



xviii Contents 

X. AMPUTATIONS 

40. Amputation in an Ischaemic Limb 

Mohideen M. Jameel and Kingsley P. Robinson 367 

41. Congenital Vascular Malformation 

Byung-Boong Lee 377 



XL MANAGEMENT OF VENOUS DISORDERS 

42. Deep Venous Thrombosis 

Fahad S. Alasfar, Dwayne Badgett and Anthony J. Comerota 395 

43. Primary Varicose Veins 

Michael Dialynas and Stephen G. E. Barker 403 

44. Venous Ulcers Associated with Deep Venous Insufficiency 

Seshadri Raju 413 

45. Venous Ulcers Associated with Superficial Venous Insufficiency 

Gudmundur Danielsson and Bo Eklof. 423 

46a. Iliofemoral Venous Thrombosis 

William P. Paaske 433 

46b. Iliofemoral Deep Venous Thrombosis (During Pregnancy) 

Anthony J. Comerota 439 



XII. LYMPHOEDEMA 

47a. Management of Upper Extremity Lymphoedema with Microsurgical 
Lymphovenous Anastomosis 

Corradino Campisi and Francesco Boccardo 453 

47b. Management of Upper Extremity Lymphoedema with Liposuction 

Hdkan Brorson 465 

Index 477 



Contributors 



Samuel S. Ahn, MD 

UCLA Gonda Vascular Center 
Los Angeles, CA, USA 

Fahad S. Alasfar, MD 

Department of Surgery 
Temple University Hospital 
Philadelphia, PA, USA 

Anders Alback, MD 

Department of Vascular Surgery 
Helsinki University Central Hospital 
Helsinki, Finland 

Christopher T. Andrews, MB ChB, 
FRCS 

Department of Orthopaedic Surgery 
Royal Victoria Hospital 
Belfast, UK 

Enrico Ascher, MD, FACS 

The Vascular Institute of New York® 
Brooklyn, NY, USA 

Dwayne Badgett, MD 

Department of Surgery 
Temple University Hospital 
Philadelphia, PA, USA 

Stephen G. E. Barker, MB BS, BSc, MS, 
FRCS 

Academic Vascular Unit 

Royal Free & University College London 

The Middlesex Hospital 

London, UK 



Jeroen J. Bax, MD, PhD 

Department of Cardiology 
Leiden University Medical Center 
Leiden, The Netherlands 

Jean-Pierre Becquemin, MD 

Department of Vascular Surgery 
Hopital Henri Mondor 
University Paris Val De Marne 
Paris, France 

David Bergqvist, MD, PhD, FRCS 

Department of Surgical Sciences 
Section of Surgery 
University Hospital 
Uppsala, Sweden 

Ramon Berguer, MD, PhD 

Department of Vascular Surgery 
A. Alfred Taubman Health Care Center 
University of Michigan 
Ann Arbor, MI, USA 

Martin Bjdrck, MD, PhD 

Department of Surgery 
University Hospital 
Uppsala, Sweden 

Paul H. B. Blair, MD, FRCS 

Vascular Surgery Unit 
Royal Victoria Hospital 
Belfast, UK 



XIX 



XX 



Contributors 



Francesco Boccardo, MD 

Professorial Unit of Medical Oncology 
University and National Cancer 

Research Institute 
Genoa, Italy 

Marcus J. Brooks, MA, MD, FRCS 

Department of Vascular Surgery 
Charing Cross Hospital 
London, UK 

Hakan Brorson, MD, PhD 

The Lymphedema Unit 
Department of Plastic and 
Reconstructive Surgery 
Malmo University Hospital 
Lund University 
Malmo, Sweden 

Keith D. Calligaro, MD 

Section of Vascular Surgery 
University of Pennsylvania Health 

System 
Pennsylvania Hospital 
Philadelphia, PA, USA 

Corradino Campisi, MD 

Lymphology and Microsurgery Centre 
University School of Medicine and 

Surgery 
Department of Specialist Surgical 

Sciences 
Anaesthesiology and Organ Transplants 
S. Martino Hospital 
Genoa, Italy 

Joseph M. Caruso, MD 

Division of Vascular Surgery, 

Department of Surgery 
New Jersey Medical School 
University of Medicine and Dentistry of 

New Jersey 
Newark, NJ, USA 

Jeannie K. Chang, MD 

Section of Vascular Surgery 
University of Pennsylvania Health 

System 
Pennsylvania Hospital 
Philadelphia, PA, USA 



Anthony J. Comerota, MD, FACS 

Jobst Vascular Center 
Toledo, OH, USA 

Torbjern Dahl, MD 

Department of Surgery 
University Hospital of Trondheim 
Trondheim, Norway 

Tom Daly, FRACS 

Department of Vascular Surgery 
Royal Prince Alfred Hospital 
Sydney, Australia 

Gudmundur Danielsson, MD, PhD 

Department of Vascular Diseases 
Malmo University Hospital 
Malmo, Sweden 

Stella S. Daskalopoulou, MSc, DIC, MD, 
FASA 

Department of Clinical Biochemistry 
(Vascular Disease Prevention Clinics) 
and 

Department of Surgery 

Royal Free Hospital 

Royal Free and University College 
School of Medicine 

London, UK 

Alexandre d'Audiffret, MD 

Morristown, TN, USA 

Ralph G. DePalma, MD, FACS 

Surgical Service 
Department of Veteran Affairs 
The Health Sciences 
Washington, DC, USA 

Michael Dialynas, MS, FRCS 

Academic Vascular Unit 

Royal Free & University College London 

The Middlesex Hospital 

London, UK 

Matthew J. Dougherty, MD 

Section of Vascular Surgery 
University of Pennsylvania Health 

System 
Pennsylvania Hospital 
Philadelphia, PA, USA 



Contributors 



XXI 



Jonothan J. Earnshaw, MBBS, DM, 
FRCS 

Department of Surgery 
Gloucestershire Royal Hospital 
Gloucester, UK 

Bo Eklof, MD, PhD 

John A. Burns School of Medicine 
University of Hawaii 
Honolulu, HI, USA, and 
University of Lund 
Sweden 

George Geroulakos, MD, FRCS, DIC, 
PhD 

Department of Surgery 
Charing Cross Hospital 
London, UK 

Christopher P. Gibbons, MA, DPhil, 
MCh, FRCS 

Department of Vascular Surgery 
Morriston Hospital 
Swansea, UK 

J. Michael Henderson, MD 

Division of Surgery 
Cleveland Clinic Foundation 
Cleveland, OH, USA 

Ariane L. Herrick, MD, FRCP 

Rheumatic Diseases Centre 
University of Manchester 
Hope Hospital 
Salford, UK 

Anil P. Hingorani, MD 

The Vascular Institute of New York® 
Brooklyn, NY, USA 

Robert W. Hobson II, MD 

Division of Vascular Surgery 
University of Medicine and Dentistry of 

New Jersey - New Jersey Medical 

School 
Newark, NJ, USA 



Larry H. Hollier, MD 

Louisiana State University Health 

Sciences Center 
School of Medicine 
New Orleans, LA, USA 

Julius H. Jacobson II, MD 

Department of Surgery 

The Mount Sinai School of Medicine 

New York, NY, USA 

Mohideen M. Jameel, MBBS, LRCP, 
LRCS, FRCS, MSc, DIC 

Wrightington, Wigan & Leigh NHS 

Trust, and 
University of Central Lancashire 
Wigan, UK 

Sylvia C. de Jong, MD, PhD 

Department of Surgery 

Erasmus University Medical Centre 

Rotterdam, The Netherlands 

John D. Kakisis, MD 

Vascular Unit, 3rd Department of 

Surgery 
Athens University Medical School 
Athens, Greece 

Milla Kallio, MD 

Department of Vascular Surgery 
Helsinki University Central Hospital 
Helsinki, Finland 

Steven S. Kang, MD 

South Miami Heart Center 
South Miami Hospital 
Miami, FL, USA 

Vikram S. Kashyap, MD, FACS 

Department of Vascular Surgery 
The Cleveland Clinic Foundation 
Cleveland, OH, USA 

Fabien Koskas, MD 

Division of Vascular Surgery 
CHU Pitie-Salpetriere 
Paris, France 



XXII 



Contributors 



Andries J. Kroese, MD, PhD 

Oslo Centre of Vascular Surgery 
Aker University Hospital 
Oslo, Norway 

Jon Largiader, MD 

University Hospital of Zurich 
Zurich, Switzerland 

Miltos K. Lazarides, MD, EBSQvasc 

Department of Vascular Surgery 
Demokritos University Hospital 
Alexandroupolis, Greece 

Byung-Boong Lee, MD, PhD, FACS 

Department of Vascular Surgery 
Georgetown University Hospital 
Reston, VA, USA 

Mauri J. A. Lepantalo, MD, PhD 

Department of Vascular Surgery 
Helsinki University Central Hospital 
Helsinki, Finland 

Christos D. Liapis, MD, FACS, FRCS 

Department of Vascular Surgery 
Athens University Medical School 
Laiko Peripheral General Hospital 
Athens, Greece 

Michel S. Makaroun, MD 

Division of Vascular Surgery 
University of Pittsburgh Medical Center 
Pittsburgh, PA, USA 

Huck A. Mandel, MS 

Division of Vascular Surgery 
University of California Center for the 

Health Sciences 
Los Angeles, CA, USA 

Luca di Marzo, MD, FACS 

Department of Surgery "Pietro 

Valdoni" 
University of Rome "La Sapienza" 
Rome, Italy 



Dimitri P. Mikhailidis, MD, FASA, 
FFPM, FRCP, FRCPath 

Department of Clinical Biochemistry 
(Vascular Disease Prevention Clinics) 
and 

Department of Surgery 

Royal Free Hospital 

Royal Free and University College 
School of Medicine 

London, UK 

Wesley S. Moore, MD 

UCLA Division of Vascular Surgery 
Los Angeles, CA, USA 

Nicholas J. Morrissey, MD 

The Mount Sinai School of Medicine 
New York, NY, USA 

Hans 0. Myhre, MD, PhD 

Department of Surgery 
University Hospital of Trondheim 
Trondheim, Norway 

Bernard H. Nachbur, MD 

University of Berne 
Berne, Switzerland 

William D. Neary, MB ChB, MRCS 

Vascular Surgical Department 
Gloucestershire Royal Hospital 
Gloucester, UK 

Adrian K. Neill, MRCS 

Department of Vascular Surgery 
Royal Victoria Hospital 
Belfast, UK 

Johanna G. H. van Nes, MD 

Department of Surgery 

Erasmus University Medical Centre 

Rotterdam, The Netherlands 

Andre Nevelsteen, MD, PhD, FRCS 

Department of Vascular Surgery 
University Hospital Gasthuisberg 
Leuven, Belgium 



Contributors 



XXIII 



Lars Norgren, MD, PhD, FRCS 

Department of Surgery 
Orebro University Hospital 
Orebro, Sweden 

Kenneth Ouriel, MD 

Department of Vascular Surgery 
The Cleveland Clinic Foundation 
Cleveland, OH, USA 

William P. Paaske, MD, FRCS, FRCSEd, 
FACS 

Department of Cardiothoracic and 

Vascular Surgery 
Aarhus University Hospital 
Aarhus, Denmark 

Frank T. Padberg Jr, MD 

Division of Vascular Surgery 
Department of Surgery 
New Jersey Medical School 
University of Medicine and Dentistry of 

New Jersey 
Newark, NJ, USA 

Theodossios Perdikides, MD 

Vascular and Thoracic Surgery 

Department 
Hellenic Air Force Hospital 
Athens, Greece 

Tomas Pfeiffer, MD 

Klinik fur Gefafichirurgie und 

Nierentransplantation 
Universitatsklinikum Diisseldorf 
Heinrich-Heine-Universitat 
Diisseldorf, Germany 

Don Poldermans, MD, PhD 

Department of Anaesthesiology 
Erasmus Medical Centre 
Rotterdam, The Netherlands 

Seshadri Raju, MD, FACS 

University of Mississippi Medical 

Center 
Flowood, MS, USA 



Jonathan S. Refson, MBBS, MS, FRCS 

Department of Vascular Surgery 
Princess Alexandra Hospital 
Harlow, UK 

Lutz Reiher, MD 

Klinik fur Gefafichirurgie und 

Nierentransplantation 
Universitatsklinikum Diisseldorf 
Heinrich-Heine-Universitat 
Diisseldorf, Germany 

Norman M. Rich, MD, FACS 

Department of Surgery 

Uniformed Services University of the 

Health Sciences (USUHS) 
Bethesda, MD, USA 

Kyung M. Ro, MPh 

Division of Vascular Surgery 
University of California Center for the 

Health Sciences 
Los Angeles, CA, USA 

Kingsley P. Robinson, FRCS 

Centre for Biomedical Engineering 
University of Surrey 
Guildford, UK 

Marc R. H. M. van Sambeek, MD, PhD 

Department of Vascular Surgery 
Erasmus University Medical Center 
Rotterdam, The Netherlands 

Richard J. Sanders, MD 

University of Colorado Health Sciences 

Center 
Denver, CO, USA 

Wilhelm Sandmann, MD 

Department of Vascular Surgery and 

Kidney Transplantation 
University Clinic of Diisseldorf 
Diisseldorf, Germany 

William L. Smead, MD 

Division of General Vascular Surgery 
Ohio State University 
Columbus, OH, USA 



XXIV 



Contributors 



Lars E. Staxrud, MD 

Oslo Centre of Vascular Surgery 
Aker University Hospital 
Oslo, Norway 

Bauer E. Sumpio, MD, PhD 

Department of Vascular Surgery 
Yale University School of Medicine 
New Haven, CT, USA 

Vasilios D. Tzilalis, MD 

Department of Vascular Surgery 
General Military Hospital 
Athens, Greece 

Hero van Urk, MD, PhD, FRCS 

Department of Vascular Surgery 
Erasmus University Medical Center 
Rotterdam, The Netherlands 

Hence J. M. Verhagen, MD, PhD 

Endovascular Program 
University Medical Center-Utrecht 
Utrecht, The Netherlands 

Jeffrey S. Weiss, MD 

Department of Vascular Surgery 
Yale University School of Medicine 
New Haven, CT, USA 

Barbara T. Weiss-Muller, MD 

Department of Vascular Surgery and 

Kidney Transplantation 
University Clinic of Diisseldorf 
Diisseldorf, Germany 

Jarlis Wesche, MD, PhD 

Department of Surgery 
Akershus University Hospital 
L0renskog, Norway 



Geoffrey H. White, MD 

Endovascular Research Unit 
Department of Surgery 
University of Sydney 
Sydney, NSW, Australia 

Rodney A. White, MD 

Department of Surgery 
Harbor - UCLA Medical Center 
Torrance, CA, USA 

John H. N. Wolfe, MS, FRCS 

Regional Vascular Unit 
St Mary's Hospital 
London, UK 

Jonathan D. Woody, MD, FACS 

Division of Vascular Surgery 
University of Pittsburgh Medical Center 
Pittsburgh, PA, USA 

Yolanda Y. L. Yang, MD, PhD 

Department of General Surgery 
Cleveland Clinic Foundation 
Cleveland, OH, USA 

Robert W. Zickler, MD 

Division of Vascular Surgery 
Department of Surgery 
New Jersey Medical School 
University of Medicine and Dentistry of 

New Jersey 
Newark, NJ, USA 



1 . Preoperative Cardiac Risk Assessment 
and Management of Elderly Men with an 
Abdominal Aortic Aneurysm 

Don Poldermans and Jeroen }. Bax 






A 72-year-old male presented with an abdominal aortic aneurysm. He had a 
history of chest pain complaints and underwent percutaneous transluminal coro- 
nary angioplasty (PTCA) 6 years ago. After the PTCA procedure he had no chest 
pain symptoms until 2 years ago. The chest pain complaints are stable and he 
was able to perform moderate exercise, such as a round of golf, in 4.5 hours. 
Physical examination showed a friendly man, with blood pressure 160/70 mm Hg 
and pulse 92 bpm. Examination of the chest revealed no abnormalities of the 
heart. Palpation of the abdomen showed an aortic aneurysm with an estimated 
diameter of 7 cm. The patient was referred to the vascular surgeon. Blood test 
showed an elevated fasting glucose of 10.0 mmol/1 and low-density lipoprotein 
(LDL) cholesterol of 4.1 mmol/1. Electrocardiography showed a sinus rhythm and 
pathological Q-waves in leads V1-V3, suggestive of an old anterior infarction. 



Question 1 

Which of the following statements regarding postoperative outcome in patients 
undergoing major vascular surgery is correct? 

A. Cardiac complications are the major cause of perioperative morbidity and 
mortality. 

B. Perioperative myocardial infarctions are related to fixed coronary artery steno- 
sis in all patients. 

C. Perioperative cardiac events are related to a sudden, unpredictable progression 
of a nonsignificant coronary artery stenosis in all patients. 

D. Perioperative cardiac complications are related to both fixed and unstable coro- 
nary artery lesions. 

This patient experienced angina pectoris in the past. He was successfully treated 
with a PTCA procedure, but recently angina pectoris reoccurred. Because of the 

3 



Vascular Surgery 



40 Atropine 2 mg 









30 








20 I 




Stopping reasons: 




a r\ 






Target heart rate 


5 


10 






Side effects 
Ischemia 





Echocardiography 



+ 



+ 



+ 



Heart rate/Blood pressure 

H 1 1 



H 



10 



13 



16 



19 



minutes 



Fig. 1 .1 . The normal stress protocol, with increasing doses of dobutamine and test endpoints. 





LAD 
RCA 
LCX 



1 = normal 

2 = mild hypokinesia 

3 = severe hypokinesia 

4 = akinesia 

5 = dyskinesia 



4CH 



2CH 



Fig. 1.2. The scoring of the left ventricle for wall motion abnormalities. LAX, long axis; SAX, short axis; 4CH, four 
chambers; 2CH, two chambers; LAD, left anterior descending artery; RCA, right coronary artery; LCX, left circum- 
flex artery. 



Preoperative Cardiac Risk Assessment and Management of Elderly Men 



5 




Fig. 1.3. An example of a normal resting echocardiogram, showing respectively, apical views and one short-axis 
view. 



multiple risk factors and the planned high-risk surgery a dobutamine stress echocar- 
diography was performed. Fig. 1.1 shows the normal stress protocol, with increasing 
doses of dobutamine and test endpoints. In Fig. 1.2 the scoring of the left ventricle 
for wall motion abnormalities is shown. Fig. 1.3 is an example of a normal resting 
echocardiogram, showing respectively, apical views and one short-axis view. In Fig. 
1.4, the different stages of the stress test are shown for the apical four-chamber view: 
rest, low-dose dobutamine, peak dose dobutamine, and recovery. As indicated by 
arrows, the posterior septum shows an outward movement during peak stress, sug- 
gesting dyskinesia, and myocardial ischemia of the posterior septum. 



Question 2 



Postoperative outcome in patients undergoing major vascular surgery has been 
improved in those taking beta-blockers and statins. Medical therapy may reduce the 
need for additional preoperative testing for coronary artery disease as the incidence 
of perioperative cardiac mortality is reduced to less than 1 percent, and may even 
reduce the indications for preoperative coronary revascularization. 

A. Beta-blockers are associated with a reduced perioperative cardiac event rate in 
patients undergoing vascular surgery, both in retrospective and prospective 
studies. 



Vascular Surgery 




Fig. 1.4. The different stages of the stress test of the apical four-chamber view, rest, low-dose dobutamine, peak 
dose dobutamine, and recovery. As shown and indicated with arrows, the posterior septum shows an outward 
movement during peak stress, suggestive of dyskinesia, and also myocardial ischemia of the posterior septum. 

B. Statin use is associated with an improved postoperative outcome. 

C. Statin use is not associated with an increased incidence of perioperative myopa- 
thy. 

D. Beta-blockers and statins are independently associated with an improved post- 
operative outcome. 



Question 3 

Preoperative beta-blocker therapy is widely used. However, the dose and duration 
of preoperative therapy is uncertain. 

A. Beta-blockers should be started preferably 30 days prior to surgery. 

B. Beta-blockers should be initiated several hours before surgery. 

C. Heart rate control should be aimed at a heart rate between 90 and 100 bpm. 

D. Heart rate control should be aimed at a heart rate between 60 and 70 bpm. 

In this patient beta-blockers were started 6 weeks before surgery. Starting dose of 
bisoprolol was 2.5 mg; the dose was increased to 5.0 mg to obtain a resting heart 
between 60 and 70 bpm. 



Preoperative Cardiac Risk Assessment and Management of Elderly Men 7 

Question 4 

Perioperative statin therapy has recently been introduced to improve postoperative 
outcome. 

A. Statins improve postoperative outcome by reducing the cholesterol level. 

B. Withdrawal of perioperative statin therapy is associated with an increased peri- 
operative cardiac event rate. 

C. Perioperative statin use is associated with an increased incidence of myopathy. 

D. Perioperative statin use is associated with a reduced perioperative cardiac event 
rate in vascular surgery patients only. 

Statins were prescribed in this patient, Lescol XL 80 mg daily, at the same time as 
beta-blockers were introduced. 



Question 5 

Preoperative coronary revascularization seems to be an attractive option to improve 
not only direct postoperative outcome in high-risk patients but also long-term sur- 
vival after surgery. 

A. Preoperative coronary revascularization improves postoperative outcome in all 
patients with significant coronary artery disease prior to major vascular surgery. 

B. Preoperative coronary revascularization in patients with one- or two-vessel 
disease is not associated with an improved postoperative outcome compared to 
patients receiving medical therapy. 

C. Preoperative coronary revascularization is associated with an improved 2-year 
outcome compared to medical therapy. 

D. Patients with proven coronary artery disease who are treated medically are at 
increased risk of late coronary revascularization after surgery. After late revas- 
cularization, long-term outcome is similar to that with revascularization prior to 
surgery. 

This 72-year-old male had multiple cardiac risk factors: elderly age, angina pectoris, 
diabetes mellitus, and a previous ML He underwent a noninvasive stress test, dobuta- 
mine stress echocardiography, which showed myocardial ischemia, suggesting left 
anterior descending artery (LAD) disease. Beta-blockers and statins were prescribed 
and continued during surgery. Surgery was uneventful; after 2 years angina pectoris 
complaints increased and a PTCA procedure was successfully performed on the LAD. 



Commentary 

Cardiac complications are the major cause of perioperative morbidity and mortal- 
ity, which may occur in 1-5 percent of unselected patients undergoing major 



8 Vascular Surgery 

vascular surgery [1]. [Q1: A] This high frequency of cardiac complications is related 
to the high prevalence of coronary artery disease; 54 percent of patients undergoing 
major vascular surgery have advanced or severe coronary artery disease and only 8 
percent of patients have normal coronary arteries [2]. Perioperative cardiac compli- 
cations are equally caused by prolonged myocardial ischemia or by coronary artery 
ulaque rupture with subsequent thrombus formation and coronary artery occlusion 
1, 3]. [Q1: B, C, D] Prolonged perioperative myocardial ischemia usually occurs from 
either increased myocardial oxygen demand or reduced supply, or from a combina- 
tion of the two. There are several perioperative factors that can increase myocardial 
oxygen demand including tachycardia and hypertension resulting from surgical 
stress, postoperative pain, interruption of beta-blocker use, or the use sympath- 
omimetic drugs. Decreased oxygen supply, on the other hand, can occur as a result 
of hypotension, vasospasm, and anemia, hypoxia or coronary artery plaque rupture. 
Beta-blockers primarily reduce myocardial oxygen demand, while statins may 
prevent coronary artery plaque rupture. [Q2: A, B] 

Beta-Adrenergic Antagonists 

Several retrospective and prospective clinical trials have shown that perioperative 
use of beta-blockers is associated with reduction in the incidence of postoperative 
myocardial ischemia, nonfatal myocardial infarction and cardiac death [4-6]. 
[Q2: A] The majority of these studies were small in sample size, and the studies were 
designed to explore the protective effect of beta-blockers for the reduction of peri- 
operative myocardial ischemia. To overcome the limitations of these studies two 
randomized clinical trials addressed the issue of perioperative use of beta-blockers 
for the prevention of cardiac death and myocardial infarction. Mangano et al. [7] 
studied the effect of atenolol on mortality and cardiovascular morbidity after non- 
cardiac surgery including vascular surgery. The investigators enrolled and ran- 
domized 200 patients to atenolol (given intravenously before and immediately 
after surgery and orally thereafter for the duration of hospitalization) or placebo. 
No difference was observed in 30-day mortality but mortality was significantly 
lower at 6 months following discharge (0% vs. 8 %, p < 0.001), over the first year 
(3% vs. 14%, p = 0.005), and over 2 years (10% vs. 21%, p = 0.019). The apparent 
lack of a perioperative cardioprotective effect of atenolol in this study was proba- 
bly related to the small sample size, and the fact that patients at low risk for cardiac 
complications were studied. In a more recent study, Poldermans et al. [8] clearly 
demonstrated the cardioprotective effect of perioperative beta-blocker use for the 
reduction of perioperative cardiac death and myocardial infarction in high-risk 
patients undergoing major vascular surgery. In total, 112 high-risk vascular 
patients were selected using a combination of cardiac risk factors and positive 
results on dobutamine stress echocardiography. Patients were then randomly 
assigned to standard care or standard care with bisoprolol use. Bisoprolol was 
started at least 30 days prior to surgery; the dose was adjusted to aim at a resting 
heart rate of 60-70 bpm. [Q3: A, B, C, D] The results showed that the incidence of the 
combined endpoint of cardiac death and myocardial infarction within 30 days of 
surgery was significantly lower in patients using bisoprolol compared to patients in 
the control group (combined endpoint 3.3% in the bisoprolol group vs. 34% in the 
control group). Based on the findings of these studies, beta-blocker use has been 
recommended by the ACC/AHA Guidelines on Perioperative Cardiovascular 



Preoperative Cardiac Risk Assessment and Management of Elderly Men 9 

Evaluation for Noncardiac Surgery in high-risk patients with a positive stress test 
as a level one recommendation [4]. 

3-Hydroxy-3-Methylglutaryl Coenzyme A Reductase Inhibitors (Statins) 

Although perioperative use of beta-blockers has been associated with a significant 
reduction in cardiac mortality and morbidity, still some patients with multiple 
cardiac risk factors and positive stress test results may remain at considerable risk 
for perioperative cardiac mortality [9]. For these patients additional cardioprotec- 
tive medication such as statin use may offer an important addition to preoperative 
risk reduction strategies. The association between statin use and possible reduction 
in perioperative cardiac complications may result from the favorable actions of 
statins on atherosclerosis and from their vascular properties other than those attrib- 
uted to cholesterol lowering [10-12]. [Q4: A, B, C] These so-called pleiotropic effects 
of statins may attenuate coronary artery plaque inflammation and influence plaque 
stability in addition to antithrombogenic, antiproliferative and leukocyte-adhesion 
inhibiting effects [13-15]. All these effects of statins may stabilize unstable coronary 
artery plaques, thereby reducing myocardial ischemia and subsequent myocardial 
damage. 

There are only a few studies that have evaluated the beneficial effects of periop- 
erative statin use in reducing perioperative cardiac complications [16-18]. 
Poldermans et al. [16], using a case-control study design in 2816 patients who 
underwent major vascular surgery, showed that controls more often were statin 
users than cases, which resulted in a fourfold reduction in all-cause mortality 
within 30 days after surgery. This finding was consistent in subgroups of patients 
according to type of vascular surgery, cardiac risk factors and beta-blocker use. 
[Q2: D] Similar to these findings, Durazzo et al. [17] also reported a significantly 
reduced incidence of cardiovascular events within 6 months of vascular surgery in 
patients who were randomly assigned to atorvastatin compared with placebo 
(atorvastatin vs. placebo, 8.3% vs. 26.0%). Finally, the study results of Lindenauer 
et al. [18] indicated that statin use was associated with 28 percent relative risk 
reduction of in-hospital mortality compared to no statin use in 780,591 patients 
undergoing major noncardiac surgery. [Q4: D] The results of these studies are 
important indications of the possible beneficial effect of perioperative statin use. 
However, certain limitations such as the retrospective nature of the study of 
Poldermans et al. and Lindenauer et al., the relatively small sample size (n = 100 
patients) of the study of Durazzo et al., and the lack of information about the 
optimal timing and duration of statin therapy warrant future clinical trials to 
confirm the effectiveness and safety of statin therapy in patients undergoing major 
noncardiac surgery. Initially, statin use was contraindicated in the perioperative 
period as it was thought that drug interactions might increase the incidence of 
myopathy and in combination with analgesics this might even remain asympto- 
matic. However, a recent study showed no increased incidence of myopathy 
among statin users [19]. Statin users undergoing vascular surgery at the Erasmus 
MC were screened for myopathy by measuring creatine kinase (CK) levels at 
regular intervals and checking for clinical symptoms. In 981 patients no relation 
was found between statin use and CK levels. Also, no patient experienced myopa- 
thy symptoms. Importantly, no deleterious effect of temporary statin interruption 
was observed. [Q2: C and Q4: B, C] 



10 Vascular Surgery 

Preoperative cardiac risk evaluation may identify high-risk patients for whom the 
risk of perioperative cardiac complications without further coronary assessment 
and subsequent intervention could be too high. For these patients either percuta- 
neous transluminal coronary angioplasty (PTCA) or coronary artery bypass graft- 
ing (CABG) maybe considered. 



Percutaneous Revascularization 

There have been several studies evaluating the clinical utility of PTCA in high-risk 
patients undergoing major noncardiac surgery including vascular surgery. In the 
studies of Elmore et al. [20] and Gottlieb et al. [21], retrospective data were collected 
of patients who underwent PTCA prior to surgery. These patients were referred for 
PTCA because of the need to relieve symptomatic angina or to treat myocardial 
ischemia identified by noninvasive testing. The findings of these studies indicated 
that the incidence of perioperative cardiac death and myocardial infarction was low, 
but the investigators in these studies failed to use a comparison group of patients 
with coronary artery disease not treated with PTCA. The apparent limitations of 
these studies prompted Posner et al. [22] to conduct their own investigation to 
compare adverse cardiac outcomes after noncardiac surgery in patients with prior 
PTCA, patients with non-revascularized coronary artery disease and normal con- 
trols. The results showed that patients treated with PTCA within 90 days of noncar- 
diac surgery had a similar incidence of perioperative events to matched patients 
with coronary artery disease who had not been revascularized. [Q5: A] Those patients 
who underwent a PTCA procedure 90 days earlier then the day of noncardiac 
surgery had a lower risk of cardiac events than non-revascularized patients but not 
as low as normal controls. Furthermore, the effect of revascularization was limited 
to a reduction in the incidence of angina pectoris and congestive heart failure and 
there was no reduction in the incidence of death and nonfatal myocardial infarc- 
tion. Indeed, the recent findings of the Coronary Artery Revascularization 
Prophylaxis (CARP) trial [23] also showed that coronary revascularization with 
PTCA or CABG prior to vascular surgery in high-risk cardiac stable patients did not 
provide short-term survival benefit or better long-term event-free survival rate. 
[Q5: B, C, D] The findings of the study indicated that patients undergoing coronary 
revascularization prior to vascular surgery had a 3.1 percent mortality rate within 
30 days of vascular surgery compared to a 3.4 percent rate for those not having 
coronary revascularization (p = 0.87). Additionally, the rate of perioperative nonfa- 
tal myocardial infarction as detected by troponin elevation was also similar in 
coronary revascularization patients and patients not undergoing coronary revascu- 
larization (11.6% vs. 14.3%, p = 0.37). Furthermore, the results of the trial also indi- 
cated that coronary revascularization prior to vascular surgery was associated with 
delay or cancellation of the required vascular operation. Apart from these findings, 
it is also important to note that if a PTCA procedure and coronary stent placement 
are performed less than 6 weeks before major noncardiac surgery, the risk of peri- 
operative coronary thrombosis or major bleeding complications may be substan- 
tially increased [24, 25]. Two separate small-scale studies reported an increased rate 
of serious bleeding complications if antithrombotic therapy was continued until the 
time of surgery, and in patients in whom antiplatelet drugs were interrupted one or 
two days before surgery an increased rate of fatal events was observed due to stent 
thrombosis [24, 25]. The risk of these complications persisted for 6 weeks after 



Preoperative Cardiac Risk Assessment and Management of Elderly Men 1 1 

coronary stent placement. Patients who underwent surgery more than 6 weeks after 
coronary stent placement experienced no adverse cardiac events. These observa- 
tions indicate that if PTCA with stenting is planned in the weeks or months before 
noncardiac surgery then a delay of at least 6 weeks should occur before noncardiac 
surgery to allow for completion of the dual antiplatelet therapy and re-endothelial- 
ization of the stent. 

Coronary Artery Bypass Grafting 

The results of the largest retrospective study to date indicated that CABG had a pro- 
tective effect prior to noncardiac surgery [26]. Data for 3368 patients analyzed from 
the Coronary Artery Surgery Study (CASS) registry showed that patients who 
underwent CABG before abdominal, vascular, thoracic, or head and neck surgery 
had a lower incidence of perioperative mortality (3.3% vs. 1.7%) and myocardial 
infarction (2.7% vs. 0.8%) compared with medically treated patients. The largest 
reduction in perioperative mortality was observed in patients with a history of 
advanced angina and in patients with multivessel coronary artery disease. In a more 
recent study, data analyzed from a random sample of Medicare beneficiaries 
showed that preoperative coronary revascularization was associated with a reduc- 
tion in 1-year mortality for patients undergoing aortic surgery but showed no effect 
on mortality in those undergoing infrainguinal procedures [27]. Hassan et al. [28], 
using data from the Bypass Angioplasty Revascularization Investigation, showed 
there was no difference in the incidence of cardiac death and myocardial infarction 
between patients who underwent coronary angioplasty or CABG and subsequent 
noncardiac surgery (coronary angioplasty group, 1.6% vs. CABG group, 1.6%). 
[Q5: A] As mentioned above under 'Percutaneous revascularization', the recent 
findings of the CARP trial showed that high-risk patients randomized to coronary 
revascularization prior to vascular surgery had no better perioperative and long- 
term cardiac complication rates than medically treated patients. Therefore, in the 
light of these findings a decision to proceed with coronary angioplasty and selective 
revascularization before high-risk surgery should be made independent of the need 
for major noncardiac surgery [4]. 



References 

1. Mangano DT. Perioperative cardiac morbidity. Anesthesiology 1990;72(l):153-84. 

2. Hertzer NR, Beven EG, Young JR, et al. Coronary artery disease in peripheral vascular patients. A 
classification of 1000 coronary angiograms and results of surgical management. Ann Surg 
1984;199(2):223-33. 

3. Dawood MM, Gutpa DK, Southern J, Walia A, Atkinson JB, Eagle KA. Pathology of fatal periopera- 
tive myocardial infarction: implications regarding pathophysiology and prevention. Int J Cardiol 
199615;57(l):37-44. 

4. Eagle KA, Berger PB, Calkins H, et al. ACC/AHA guideline update for perioperative cardiovascular 
evaluation for noncardiac surgery - executive summary: a report of the American College of 
Cardiology/ American Heart Association Task Force on Practice Guidelines (Committee to Update 
the 1996 Guidelines on Perioperative Cardiovascular Evaluation for Noncardiac Surgery). 
Circulation 200212;105(10):1257-67. 

5. Eagle KA, Rihal CS, Mickel MC, Holmes DR, Foster ED, Gersh BJ. Cardiac risk of noncardiac surgery: 
influence of coronary disease and type of surgery in 3368 operations. CASS Investigators and 
University of Michigan Heart Care Program. Coronary Artery Surgery Study. Circulation 
199716;96(6):1882-7. 



12 Vascular Surgery 

6. Boersma E, Poldermans D, Bax JJ, et al. Predictors of cardiac events after major vascular surgery: 
role of clinical characteristics, dobutamine echocardiography, and beta-blocker therapy. JAMA 
2001;285(14):1865-73. 

7) Mangano DT, Layug EL, Wallace A, Tateo I. Effect of atenolol on mortality and cardiovascular mor- 
bidity after noncardiac surgery. Multicenter Study of Perioperative Ischemia Research Group. N Engl 
J Med 1996;335(23):1713-20. 

8. Poldermans D, Boersma E, Bax JJ, et al. The effect of bisoprolol on perioperative mortality and 
myocardial infarction in high-risk patients undergoing vascular surgery. Dutch Echocardiographic 
Cardiac Risk Evaluation Applying Stress Echocardiography Study Group. N Engl J Med 
1999;341(24):1789-94. 

9. Devereaux PJ, Leslie K, Yang H. The effect of perioperative beta-blockers on patients undergoing 
noncardiac surgery - is the answer in? Can J Anaesth 2004;51(8):749-55. 

10. Takemoto M, Liao JK. Pleiotropic effects of 3-hydroxy-3-methylglutaryl coenzyme A reductase 
inhibitors. Arterioscler Thromb Vase Biol 2001;21(ll):1712-9. 

11. Huhle G, Abletshauser C, Mayer N, Weidinger G, Harenberg J, Heene DL. Reduction of platelet activ- 
ity markers in type II hypercholesterolemic patients by a HMG-CoA-reductase inhibitor. Thromb 
Res 1999;95(5):229-34. 

12. Hernandez-Perera O, Perez-Sala D, Navarro -Antolin J, et al. Effects of the 3-hydroxy-3-methylglu- 
taryl-CoA reductase inhibitors, atorvastatin and simvastatin, on the expression of endothelin-1 and 
endothelial nitric oxide synthase in vascular endothelial cells. J Clin Invest 1998;101(12):2711-2719. 

13. Stamler JS, Loh E, Roddy MA, Currie KE, Creager MA. Nitric oxide regulates basal systemic and pul- 
monary vascular resistance in healthy humans. Circulation 1994;89(5):2035-40. 

14. Kurowska EM. Nitric oxide therapies in vascular diseases. Curr Pharm Des 2002;8(3):155-66. 

15. van Haelst PL, van Doormaal JJ, May JF, Gans RO, Crijns HJ, Cohen Tervaert JW. Secondary preven- 
tion with fluvastatin decreases levels of adhesion molecules, neopterin and C-reactive protein. Eur J 
Intern Med 2001;12(6):503-9. 

16. Poldermans D, Bax JJ, Kertai MD, et al. Statins are associated with a reduced incidence of peri- 
operative mortality in patients undergoing major noncardiac vascular surgery. Circulation 
2003;107(14):1848-51. 

17. Durazzo AES, Machado FS, Ikeoka DT, et al. Reduction in cardiovascular events after vascular 
surgery with atorvastatin: a randomized trial. J Vase Surg 2004;39(5):967-75. 

18. Lindenauer PK, Pekow P, Wang K, Gutierrez B, Benjamin EM. Lipid-lowering therapy and in-hospi- 
tal mortality following major noncardiac surgery. JAMA 2004;291(17):2092-9. 

19. Schouete O, Kertai MD, Bax J J, et al. Safety of statin use in high-risk patients undergoing major vas- 
cular surgery. Am J Cardiol 2005;95(5):658-660. 

20. Elmore JR, Hallett JW, Jr, Gibbons RJ, et al. Myocardial revascularization before abdominal aortic 
aneurysmorrhaphy: effect of coronary angioplasty. Mayo Clin Proc 1993;68(7):637-41. 

21. Gottlieb A, Banoub M, Sprung J, Levy PJ, Beven M, Mascha EJ. Perioperative cardiovascular morbid- 
ity in patients with coronary artery disease undergoing vascular surgery after percutaneous translu- 
minal coronary angioplasty. J Cardiothorac Vase Anesth 1998;12(5):501-6. 

22. Posner KL, Van Norman GA, Chan V. Adverse cardiac outcomes after noncardiac surgery in patients 
with prior percutaneous transluminal coronary angioplasty. Anesth Analg 1999;89(3):553-60. 

23. McFalls EO, Ward HB, Moritz TE, et al. Coronary-artery revascularization before elective major vas- 
cular surgery. N Engl J Med 2004;351:2795-804. 

24. Kaluza GL, Joseph J, Lee JR, Raizner ME, Raizner AE. Catastrophic outcomes of noncardiac surgery 
soon after coronary stenting. J Am Coll Cardiol 2000;35(5): 1288-94. 

25. Wilson SH, Fasseas P, Orford JL, et al. Clinical outcome of patients undergoing non-cardiac surgery 
in the two months following coronary stenting. Journal of the American College of Cardiology 
2003;42(2):234-40. 

26. Eagle KA, Rihal CS, Mickel MC, Holmes DR, Foster ED, Gersh BJ. Cardiac risk of noncardiac surgery: 
influence of coronary disease and type of surgery in 3368 operations. CASS Investigators and 
University of Michigan Heart Care Program. Coronary Artery Surgery Study. Circulation 
1997;96(6):1882-7. 

27. Fleisher LA, Eagle KA, Shaffer T, Anderson GF. Perioperative- and long-term mortality rates after 
major vascular surgery: the relationship to preoperative testing in the Medicare population. Anesth 
Analg 1999;89(4):849-55. 

28. Hassan SA, Hlatky MA, Boothroyd DB, et al. Outcomes of noncardiac surgery after coronary bypass 
surgery or coronary angioplasty in the Bypass Angioplasty Revascularization Investigation (BARI). 
Am J Med 2001;110(4):260-6. 



2. Abdominal Aortic Aneurysm 

Jean-Pierre Becquemin and Alexandre d'Audiffret 



A 59-year-old man presented with an abdominal aortic aneurysm (AAA) discov- 
ered on duplex scan examination of the abdomen. The AAA was 60-mm large 
and extended to the left common iliac artery. The patient was otherwise asymp- 
tomatic, with no abdominal or back pain. His medical history was significant for 
hypertension controlled by bitherapy, non-insulin-dependent diabetes diagnosed 
5 years previously, claudication with a walking distance of 400 metres, and a 
smoking history of 40 packs/year. He had no history of myocardial infarction 
(MI) or angina pectoris. 

He had a positive family history for an aneurysm. His father underwent 
surgery 20 years earlier for an abdominal aneurysm. He also has a brother who is 
70 years old and a sister who is 55 years old with apparently no health problems 

On examination, the patient was obese. No abdominal mass was palpated. 

A computed tomography (CT) scan was performed (Figs 2.1 and 2.2). 

Routine blood tests were normal except for serum creatinine level, which was 
200 mg/ml. Electrocardiogram (ECG) was normal. 



Question 1 

The AAA of this patient was found during a routine screening. In which group(s) of 
population is duplex scan screening for AAA justified? 

A. Patients with uncomplicated hypertension. 

B. Patients with a family history of aneurysmal disease. 

C. Patients with a smoking history. 

D. Patients with peripheral vascular disease. 

E. Obese patients with vascular risk factors 

F. All men, starting at the age of 50 years. 



13 



14 



Vascular Surgery 





Fig. 2.1 . CT scan demonstrating aortic, left common iliac, and left hypogastric aneurysms. 




Fig. 2.2. Arteriography demonstrating aortic, left common iliac, and left hypogastric aneurysms (shadow of the 
external iliac indicated by arrow). 



Question 2 



With the imaging you have been provided with, is (are) there any reason(s) for per- 
forming an arteriogram? 



Abdominal Aortic Aneurysm 15 

A. No need, CT scan is sufficient. 

B. Arteriogram is mandatory to define the surgical strategy. 

C. Hypogastric aneurysm is a reason for performing an angiogram. 

D. Angiography is required because of the suspicion of lower limb occlusive 
disease. 

E. Serum creatinine level contraindicates angiography. 

Question 3 

To assess cardiac risk several tests can be performed. Which of the following explo- 
rations should be performed first? 

A. None, ECG is sufficient. 

B. Cardiac scintigraphy. 

C. Cardiac echography. 

D. Cardiac echography with dobutamine test. 

E. Coronarography. 

Question 4 

Without treatment this patient is at risk of rupture. Which of the following factors 
have been proved to be associated with an increased risk of rupture? 

A. Diameter >60 mm. 

B. Association with a hypogastric aneurysm. 

C. Diabetic patient. 

D. Lower limb occlusive disease. 

E. Smoking. 

Question 5 

If an operation were being considered, which of the following factors are associated 
with an increased postoperative mortality? 

A. Diameter >60 mm. 

B. Association with an hypogastric aneurysm. 

C. Diabetic patient. 

D. Renal insufficiency. 

E. Smoking. 



16 Vascular Surgery 

Question 6 

With the current information obtained from the case description, what would you 
recommend to the patient? 

A. Duplex scan surveillance every 3 months. 

B. Aorto-bifemoral graft through a midline incision. 

C. Aorto-bifemoral graft through a left retroperitoneal incision. 

D. Aorto-bi-iliac graft through a left retroperitoneal incision. 

E. Stent graft. 

The patient underwent, via a left retroperitoneal approach, an aorto-right 
common iliac-left external iliac bypass with end-to-end anastomosis. The left 
common iliac artery was ligated at its origin. This was possible in this case because 
of the presence of a small proximal common iliac neck. The iliac and hypogastric 
aneurysms were opened, and back bleeding was controlled with an endoaneurysmal 
ligature. In addition, intraoperative cell-saver was used. 

The left retroperitoneal approach was chosen because it permits an easier dis- 
section of large left common iliac aneurysms, especially when associated with 
hypogastric aneurysms. 

The patient's postoperative course was uneventful, and he was discharged on the 
tenth postoperative day. 



Question 7 

During open operation for AAA, cell-saver autotransfusion (CSA) can be used. 
Which of the following statements are correct? 

A. It should be used routinely. 

B. It should be reserved for when the expected blood loss is significant. 

C. It should be substituted in all cases with preoperatively deposited autologous 
blood transfusion. 

D. It presents fewer complications than unwashed cell autotransfusion. 

One month later, the patient returned to the clinic with his 50-year-old brother 
who feared he could be suffering from a similar problem. He also stated that his 
father might have died from ruptured aneurysm. 



Question 8 

Does a genetic predisposition to AAA exist? Describe the pathogenesis of AAA. 



Abdominal Aortic Aneurysm 17 

Question 9 

A duplex scan has been performed on the patient's bother, which detected a 40-mm 
abdominal aneurysm. 

What recommendation(s) would you give this patient's brother? 

A. Serial duplex studies at 3-monthly intervals, and intervention when the diameter 
reaches 5.5 cm. 

B. Serial duplex studies at 6-monthly intervals, and intervention if the diameter 
reaches or exceeds 5 cm. 

C. Serial duplex studies at 12-monthly intervals until the diameter reaches 4.5 cm, 
then every 6 months until the diameter reaches 5 cm, then every 3 months, and 
then intervention when the aneurysm reaches 5.5 cm. 

D. Schedule the patient for surgery as he is a smoker and therefore his aneurysm 
will most likely require intervention. 



Commentary 

The question of the optimal format for population screening for AAA remains 
unanswered. Many studies have attempted to identify high-risk populations in 
order to reduce healthcare costs and maximise the yield. Simon et al. [1] have 
demonstrated a prevalence of AAA of 11 percent in male patients aged 60-75 years 
with a systolic blood pressure greater than 175 mm Hg. No patients with uncompli- 
cated hypertension had an AAA. Claudication was the only cardiovascular compli- 
cation associated independently with AAA (relative risk 5.8). Furthermore, 
preliminary results from the Aneurysm Detection and Management (ADAM) study 
revealed that smoking was the most important risk factor associated with AAA 
(odds ratio (OR) 5.57), followed by a positive family history (OR 1.95), age, height, 
coronary artery disease, atherosclerosis, high cholesterol level and hypertension [2]. 
Similar factors have been identified in the cardiovascular health study of the 
University of Pittsburgh [3]. Mass screening of the population, including all hyper- 
tensive patients, would not be cost-effective or beneficial. In conclusion, white men 
over the age of 60 years with significant hypertension should routinely be offered an 
abdominal ultrasound. [Q1: B, C, D] 

As arteriography does not provide information on aneurysm size, the most 
important argument for routine arteriography as part of the preoperative evalua- 
tion of the patient with AAA is the detection of associated intra-abdominal vascular 
pathologies, especially renal artery stenosis or occlusion, presence of accessory 
renal arteries, coeliac and superior mesenteric stenosis or occlusion, and lower 
extremity disease. 

According to several studies, the association of renal artery stenoses and AAA is 
found in 18-22% of cases [4]. Similarly, there is a prevalence of 25 percent for 
coeliac trunk stenoses and 6 percent for superior mesenteric stenoses in patients 
with AAA [5, 6]. Arteriography is associated with several complications related to 
its invasive nature and contrast load. Spiral CT provides high-quality images and 



18 Vascular Surgery 

accurately identifies renal and visceral artery stenoses; however, the radiation and 
contrast requirements remain high. As a general rule, most vascular surgeons would 
agree on the selective use of arteriography and limit its use to patients with evidence 
of lower-extremity occlusive disease, suprarenal or juxtarenal aneurysms, and 
common or internal iliac artery involvement. Of note, when endovascular treatment 
is considered, an arteriogram with a graduated catheter is usually recommended for 
endograft sizing. However, in institutions where stent grafts are readily available on 
the shelf, intraoperative arteriogram is the first step of the procedure. 

Moderate renal insufficiency is not an absolute contraindication to performing 
either an arteriogram or a stent graft. However, correct patient hydration and a 
small amount of contrast medium are necessary to avoid acute renal insufficiency. 
[Q2: B,C] 

Routine coronary angiography in vascular patients has shown that 60% of them 
have severe coronary artery disease [7]. However, a large randomised study in 
patients with stable angina has clearly demonstrated that preoperative coronary 
bypass or angioplasty does not improve the postoperative and 5-year survival rate 
[8]. Beta-blockers, statins and antiplatelets have all contributed to the reduction of 
cardiac events following major vascular surgery [9]. Thus preoperative exploration 
can be restricted to patients with identified risk factors for severe coronary artery 
disease [10]. In the current case diabetes, renal insufficiency and claudication are 
three of these markers and preoperative cardiac screening should be undertaken. 
Dobutamine echo stress testing is probably the most reliable test [11]. Coronary 
angiography is indicated only when more than three parietal wall segments are 
ischaemic. [Q3: D] 

The natural history of aneurysms and risk of rupture are better understood. A 
cohort study of patients who refused early operation [12] or who were considered to 
be inoperable has shown that the risk of rupture is significant for >5.5-cm 
aneurysms. A population-based study [13] has shown that diameter, FEV1 less than 
1.5 1/min and smoking are strongly correlated with aneurysm growth and rupture. 
[Q4: A, E] 

Analysis of factors predictive of mortality in patients submitted to open repair of 
AAA has shown that age, cardiac status, renal insufficiency and pulmonary status 
are strongly predictive of postoperative complications and deaths [14]. Difficult 
operations are also associated with an increased operative risk mostly related to the 
increased blood loss. Unilateral or bilateral hypogastric aneurysm increased the 
operative risk [14]. [Q5: B, D] 

Surveillance was not recommended due to the aneurysm's size and the relatively 
young age of the patient. 

This patient presented several factors which would make stent grafting a wise 
option. Unfortunately, the anatomy was not favourable. Huge left hypogastric 
aneurysms made a stent graft difficult with a risk of type II endoleak. Hypogastric 
embolisation if performed should have blocked the distal tributaries of the internal 
laic artery, which is a cause of sever buttock complications. Another restricting 
factor for stent graft is the creatinine level. Since CT scan with contrast medium is 
the gold standard for surveillance after stent grafting, renal function impairment is 
a serious risk in this patient. 

Open surgery could have been performed with any of the proposed routes. 
However, median laparotomy was not the best choice due to the increased risk of 
postoperative abdominal wall dehiscence in this obese patient. Aorto-bifemoral 
bypass should be avoided to limit the risk of infection from the groin. 



Abdominal Aortic Aneurysm 1 9 

The left retroperitoneal approach was chosen because it permits an easier 
dissection of large left common iliac aneurysms, especially when associated with 
hypogastric aneurysms. [Q6: D] 

Over the past three decades, with the appreciation of the risk of transfusion- 
related transmission of infectious diseases, a large body of research and instrumen- 
tation has emerged on autotransfusion. The current options are: 

• preoperative deposit of autologous blood; 

• intraoperative salvage and washing of red blood cells (cell-saver autotransfu- 
sion, CSA); 

• intraoperative salvage of whole blood without washing. 

Although both whole-blood autotransfusion (WBA) and CSA are currently in use, 
the magnitude of haemostatic and haemolytic disturbances, as well as the clinical 
side effects, after WBA compared with CSA are still under debate. While Ouriel et al. 
[15] showed the safety of WBA in 200 patients undergoing AAA repair, others have 
demonstrated a lower content of haemolytic degradation products and fewer coagu- 
lation disturbances after retransfusion of cell-saver blood [16]. Nevertheless, despite 
its widespread use, several studies have found that CSA is not cost-effective and 
should be limited to patients who have an expected blood loss of at least 1000 ml, 
which includes patients with large, complicated aneurysms [17, 18]. Finally, transfu- 
sion of pre-donated autologous blood is associated with some of the disadvantages 
of homologous transfusions, namely dilutional hypofibrinogenaemia, thrombocy- 
topenia and hypothermia. [Q7: B, D] 

The causes of AAA are numerous, and may include syphilis, aortic dissection, 
Ehlers-Danlos syndrome and cystic medial necrosis. However, more than 90 
percent of all AAAs are associated with atherosclerosis and are classified as either 
atherosclerotic or degenerative aneurysms. The true aetiology of AAA is most likely 
multifactorial, including genetic predisposition, abnormality of the arterial wall 
proteolytic activity and connective tissue metabolism, and haemodynamic stresses. 

The notion of genetic predisposition to AAA has been demonstrated in several 
reports of familial clustering as well as animal models. It is estimated that 15 
percent of patients presenting with an AAA have a first-degree relative with the 
same condition. Male siblings are at higher risk, but current evidence also supports 
an autosomal dominant pattern of inheritance [19]. 

Multiple studies of the aortic aneurysmal wall have demonstrated an increased 
activity of metalloproteases, which have predominantly collagenolytic and elasti- 
nolytic activity, leading to fragmentation of the elastin network and replacement by 
a non-compliant collagen layer. In addition, deficiencies of tissue inhibitors of 
metalloproteases and alpha- 1 -antitrypsin have also been demonstrated [20]. 

Mechanical factors also play a significant role in the development of AAA. 
A strong association between hypertension and aneurysm expansion has been 
demonstrated. Also, the higher incidence of AAA in Second World War amputees 
places into play the reflective pressure stress [21]. [Q8] 

The management and surveillance of small AAAs has been debated for many 
years. The UK Small Aneurysm Trial has attempted to shade some light on this 
subject [22]. The Trial Participants concluded that early surgical intervention did 
not offer any long-term survival advantages. Their recommendations, based on the 
trial methodology, were serial duplex every 6 months for aneurysms of size 



20 Vascular Surgery 

4-4.9 cm, and every 3 months for aneurysms of size 5-5.5 cm. In another, larger 
analysis, the recommendations were yearly duplex for aneurysms measuring 
4-4.5 cm on the initial scan [23]. [Q9: B, C] Indications and surveillance data for AAA 
remain confusing, however, and the answers to this question appear to represent 
common practice. One matter of concern is the common underestimation of the 
aneurysm size by duplex scan compared with CT scan, which may result in a high- 
risk surveillance period when the aneurysm diameter reaches 5 cm. Finally, accord- 
ing to the UK Small Aneurysm Trial, surgical treatment was unnecessary in only 
25 percent of the patients, with 75 percent in the surveillance group requiring an 
intervention at some time during follow-up. Thus, the surveillance resulted only in 
a delayed intervention in 75 percent of cases. Chronic obstructive pulmonary 
disease (COPD) and continuation of smoking have been associated with aneurysm 
expansion, but the rate of expansion does not justify intervention on 4-cm 
aneurysms [24]. 



References 

1. Simon G, Nordgren D, Connelly S, Schultz PJ. Screening for abdominal aortic aneurysms in a hyper- 
tensive population. Arch Intern Med 1996;156:2084-8. 

2. Lederle FA, for the Aneurysm Detection and Management (ADAM) Veterans Affairs Cooperative 
study group. Prevalence and association of AAA detected through screening. Ann Intern Med 
1997;126:441-9. 

3. Alcorn HG, Wolfson SK Jr, Sutton-Tyrell K, Kuller LH, O'Leary D. Risk factors for abdominal aortic 
aneurysms in older adults enrolled in the cardiovascular health study. Arterioscler Thromb Vase 
Biol 1996;16:963-70. 

4. Valentine RJ, Martin JD, Myers SI, Rossi MB, Clagett GP. Asymptomatic celiac and SMA stenoses are 
more prevalent among patients with unsuspected renal artery stenoses. J Vase Surg 1991;14:195-9. 

5. Brewster DC, Retana A, Waltman AC, Darling RC. Angiography in the management of aneurysms of 
the abdominal aorta. N Engl J Med 1975;292:822-5. 

6. Piquet P, Alimi Y, Paulin M, et al. Anevrisme de l'aorte abdominal et insuffisance renale chronique. 
In: Kieffer E, editor. Les Anevrysmes de l'Aorte Abdominal sous-renale. Paris: Editions AERCV, 
1990. 

7. Hertzer NR, Beven EG, Young JR, O'Hara PJ, Ruschhaupt WF III, Graor RA, et al. Coronary artery 
disease in peripheral vascular patients. A classification of 1000 coronary angiograms and results of 
surgical management. Ann Surg 1984;199:223-33. 

8. McFalls EO, Ward HB, Moritz TE, Goldman S, Krupski WC, Littooy F, et al. Coronary-artery revas- 
cularization before elective major vascular surgery. N Engl J Med 2004;351:2795-804 

9. Kertai MD, Boersma E, Bax JJ, Thomson IR, Cramer MJ, van de Ven LL, et al. Optimizing long-term 
cardiac management after major vascular surgery: role of beta-blocker therapy, clinical characteris- 
tics, and dobutamine stress echocardiography to optimize long-term cardiac management after 
major vascular surgery. Arch Intern Med 2003;163:2230-5. 

10. Kertai MD, Boersma E, Westerhout CM, Klein J, van Urk H, Bax JJ, et al. A combination of statins 
and beta-blockers is independently associated with a reduction in the incidence of perioperative 
mortality and nonfatal myocardial infarction in patients undergoing abdominal aortic aneurysm 
surgery. Eur J Vase Endovasc Surg 2004;28:343-52. 

11. Kertai MD, Boersma E, Bax J J, Heijenbrok-Kal MH, Hunink MG, L'talien GJ, et al. A meta-analysis 
comparing the prognostic accuracy of six diagnostic tests for predicting perioperative cardiac risk in 
patients undergoing major vascular surgery. Heart 2003;89:1327-34 

12. Lederle FA, Johnson GR, Wilson SE, Ballard DJ, Jordan WD, Jr, Blebea J, et al. Rupture rate of large 
abdominal aortic aneurysms in patients refusing or unfit for elective repair. JAMA 2002;287:2968-72. 

13. Brady AR, Thompson SG, Fowkes FG, Greenhalgh RM, Powell JT. Abdominal aortic aneurysm 
expansion: risk factors and time intervals for surveillance. Circulation 2004;110:16-21. 

14. Becquemin JP, Chemla E, Chatellier G, Allaire E, Melliere D, Desgranges P. Peroperative factors 
influencing the outcome of elective abdominal aorta aneurysm repair. Eur J Vase Endovasc Surg 
2000;20:84-89. 



Abdominal Aortic Aneurysm 21 

15. Ouriel K, Shortell CK, Green RM, DeWeese JA. Intraoperative autotransfusion in aortic surgery. 
J Vase Surg 1993;18:16-22. 

16. Bartels C, Bechtel JV, Winkler C, Horsch S. Intraoperative autotransfusion in aortic surgery: com- 
parison of whole blood autotransfusion versus cell separation. J Vase Surg 1996;24:102-8. 

17. Goodnough LT, Monk TG, Sicard G, Satterfield SA, Allen B, Anderson CB. Intraoperative salvage in 
patients undergoing elective abdominal aortic aneurysm repair: an analysis of cost and benefit. 
J Vase Surg 1996;24:213-18. 

18. Huber TS, McGorray SP, Carlton LC, Irwin PB, Flug RR, Flynn TC, et al. Intraoperative autologous 
transfusion during elective infrarenal aortic reconstruction: a decision analysis model. J Vase Surg 
1997;25:984-94. 

19. Majumder PP, St Jean PL, Ferrell RE, Webster MW, Steed DL. On the inheritance of abdominal aortic 
aneurysm. Am J Hum Genet 1991;48:164-70. 

20. Dobrin PB, Baker WH, Gley WC. Elastolytic and collagenolytic studies of arteries: implications for 
the mechanical properties of aneurysms. Arch Surg 1984;119:405-9. 

21. Errington ML, Ferguson JM, Gillespie IN, Connell HM, Ruckley CV, Wright AR. Complete pre-oper- 
ative imaging assessment of abdominal aortic aneurysm with spiral CT angiography. Clin Radiol 
1997;52:369-77. 

22. The UK Small Aneurysm Trial Participants. Mortality results for randomized controlled trial of early 
elective surgery or ultrasonographic surveillance for small abdominal aortic aneurysms. Lancet 
1998;352:1649-55. 

23. Grimshaw GM, Thompson JM, Hamer JD. A statistical analysis of the growth of small abdominal 
aneurysms. Eur J Vase Surg 1994;8:741-6. 

24. Macsweeney STR, Ellis M, Worell PC, Greenhalgh RM, Powell JT. Smoking and growth rate of small 
abdominal aortic aneurysms. Lancet 1994;344:651-2. 



3. Endoluminal Treatment of Infrarenal 
Abdominal Aortic Aneurysm 

Hence }. M. Verhagen, Geoffrey H. White, Tom Daly and 
Theodossios Perdikides 



A 78-year-old male was referred for investigation and management of an asymp- 
tomatic 6.2-cm diameter abdominal aortic aneurysm (AAA), which was diag- 
nosed coincidentally during an abdominal ultrasound examination performed 
for investigation of prostatic symptoms. 

The patient had a significant previous medical history, which included 
ischaemic heart disease, severe chronic obstructive lung disease, and several pre- 
vious laparotomies. He had undergone coronary artery bypass graft (CABG) 
surgery a few years ago. On examination, the patient was haemodynamically 
stable with no abdominal complaints. There was an expansile pulsatile mass 
palpable in his obese abdomen, and all peripheral pulses were palpable. 

Endoluminal repair of the AAA was considered, particularly in view of his high 
risk for open surgical repair. 



Question 1 

What is the optimal method of preoperative aneurysm assessment? 

A. Abdominal colour flow duplex ultrasound. 

B. Contrast-enhanced spiral computed tomography (CT) scan of the abdomen. 

C. Abdominal colour duplex ultrasound and calibrated aortic angiography. 

D. Contrast-enhanced spiral CT scan of the abdomen and calibrated aortic angio 
graphy. 



Question 2 

What is the mean annual risk of rupture of a 6.2-cm abdominal aortic aneurysm? 
A. Less than 5 per cent. 

23 



24 Vascular Surgery 

B. Between 5 and 10 per cent. 

C. Between 10 and 20 per cent. 

D. Greater than 20 per cent. 

Question 3 

In anatomically similar aneurysms: 

A. Open repair is a safer option for high-risk patients. 

B. The benefits of endoluminal repair, in terms of reduced morbidity and mortal- 
ity, only apply in high-risk patients. 

C. The presence of chronic renal failure is an absolute contraindication to endolu- 
minal repair. 

D. Endoluminal repair results in a two-thirds reduction in 30-day operative mor- 
bidity and mortality compared to open aneurysm repair. 

A contrast-enhanced spiral CT scan was obtained, which demonstrated that 
endovascular repair of the AAA was possible (Fig. 3.1). A calibrated aortic 
angiogram was performed to better define some of the anatomical features 
(Fig. 3.2). This showed an infrarenal aneurysm with a maximum diameter of 
62 mm. The neck of the aneurysm was 19 mm in diameter and 22 mm in length. The 
distance from the lowest renal artery to the aortic bifurcation was 125 mm, and 
there was a further distance of 60 mm from the aortic bifurcation to the orifice of 
the internal iliac artery on each side. The neck of the aneurysm was noted to be 
quite angulated. The common iliac arteries were non-aneurysmal but severely tor- 
tuous, with angulations of more than 90° in their midsection. The minimum diame- 
ters of the external iliac and femoral arteries were 9 mm bilaterally. An endoluminal 
repair procedure was planned. 

Question 4 

Which anatomical features limit endoluminal repair? 

A. Length and diameter of the aneurysm neck. 

B. Length and diameter of the aneurysm. 

C. Angle of the neck as well as the angle of the iliac arteries. 

D. Tortuosity and diameter of the iliac arteries. 

Question 5 

For choosing a suitable endoluminal graft, one must: 

A. Take the graft that resembles your measurements most closely. 



Endoluminal Treatment of Infrarenal Abdominal Aortic Aneurysm 



25 




Fig.3.1.(a,b) 



26 



Vascular Surgery 




Fig. 3.1. a Contrast-enhanced CT scan at the level of the neck of the aneurysm, immediately below the renal 
arteries. The neck has a maximum diameter of 19 mm on this image and has no irregular features, such as mural 
thrombus or atheromatous plaque, b Contrast-enhanced CT scan at the level of the sac of the aneurysm, 
showing a typical "target" appearance due to the presence of significant mural thrombus lining the aneurysm 
and contrast filling the flow channel. Maximum diameter 62 mm. c Contrast-enhanced CT scan at the level just 
below the aortic bifurcation, showing two iliac arteries of diameter 1 6-1 7 mm. d Three-dimensional reconstruc- 
tion of a contrast-enhanced CT scan, showing angulation between the neck and the aneurysm of about 45°. The 
iliac arteries are tortuous, with an angulation of at least 90°. 



Endoluminal Treatment of Infrarenal Abdominal Aortic Aneurysm 



27 




Fig. 3.2. a Calibrated aortic angiogram showing the aortic lumen, renal arteries and neck of the aneurysm, b 
Calibrated aortic angiogram showing the lumen of the aorta and both iliacs. Note that the diameter of the iliac 
arteries measures less on the angiogram than on the CT images. 



28 Vascular Surgery 

B. Oversize all diameters by 10 per cent. 

C. Oversize the proximal diameter by 20 per cent and the limb diameters by 
10 per cent. 

D. Undersize all diameters by 10 per cent and balloon-expand them to the proper 
size at the end of the procedure. 

Due to the patient's severely impaired lung function and other risk factors, 
the procedure was performed under epidural anaesthesia. The abdomen and 
both groins were prepared into a sterile field. Common femoral arteries were 
surgically exposed, sheaths were inserted, and wires were put into place under 
fluoroscopy. Angiography was performed to mark exactly the position of the 
renal arteries (Fig. 3.3), and an endoluminal device was implanted successfully 
(Fig. 3.4). 

Question 6 

Whilst deploying the graft, the following need to be considered: 

A. The orientation of the graft. 

B. The location of any renal accessory renal arteries. 

C. The location of the aortic bifurcation. 

D. The location of the bifurcation of the common iliac artery. 




Fig. 3.3. Early on-table angiogram to locate precisely the renal arteries during graft implant procedure. The 
arrow indicates the left renal artery. The device is ideally placed immediately below the orifice of the renal artery 
(note that some designs of endograft incorporate a bare stent that projects over the renal orifices). 



Endoluminal Treatment of Infrarenal Abdominal Aortic Aneurysm 



29 




Fig. 3.4. Angiogram of a successfully deployed graft showing patent renal arteries, the endograft on the outside 
of the contrast-filled lumen, and the absence of an endoleak (flow of blood into the AAA sac). The blush near the 
proximal end of the graft (arrow) is gas inside the superimposed intestine. £Q 



Question 7 

What are the more common complications of endoluminal aneurysm repair? 



Question 8 

Which of the following is the most important finding on follow-up imaging 

A. No change to the lung field on chest X-ray. 

B. The diameter of the aneurysm compared to the original diameter. 

C. The absence of a type II endoleak previously visible on CT. 

D. The position of the graft on abdominal X-ray. 



Commentary 

AAAs are generally a disease of elderly white males. In men, AAAs start to occur at 
the age of 50 years, reaching a peak incidence of about 350/100,000 person-years by 
the age of 80 years. The prevalence of AAAs of at least 3 mm in diameter in men 
over 65 years is 7.6 per cent. In women, AAAs tend to occur a few years later in life. 
The age-adjusted incidence is four to six times higher in men than in woman. Age, 



30 Vascular Surgery 

gender and smoking are the risk factors with the largest impact on AAA prevalence 

in. 

Although ultrasonography is the method of choice for population screening or 
follow-up measurements in patients with known aneurysms, ultrasound imaging 
alone gives insufficient information for preoperative assessment for repair proce- 
dures. For open repair, most surgeons recommend a preoperative imaging study 
with (spiral) CT scanning, which provides accurate information regarding 
aneurysm size and its relationship to branch vessels, as well as any anatomic anom- 
alies [2]. 

Preoperative imaging is even more important when endovascular treatment is 
considered, because patient selection and sizing of the endograft depend on it. With 
contrast-enhanced spiral CT, the dimensions of the proximal neck can be deter- 
mined accurately and the presence of calcification or mural thrombus noted. 
Although the anatomy of the iliac arteries and accessory renal arteries can be 
demonstrated by spiral CT, in most medical centres calibrated aortography is also 
performed to allow accurate measurements. Anteroposterior and lateral views are 
required to demonstrate tortuosity in the neck of the aneurysm and the iliac arteries 
[3]. Three-dimensional reconstructions of contrast-enhanced CT scans are being 
utilised increasingly in order to get a more detailed perception of the actual 
anatomy. These reconstructions may become the standard method for accurate 
sizing of endografts in the near future. [Q1 : D] 

The decision of whether to treat an AAA remains a difficult process in which mul- 
tiple factors play a role. One important factor is the risk of rupture, which, unfortu- 
nately, will always be an estimate, since large numbers of patients with AAAs have 
not been followed up without intervention. Based on currently available data from 
the UK Small Aneurysm Trial, the annual risk of rupture is less than 1 per cent 
when the diameter is 4-5.5 cm, although the validity of this risk estimation was 
compromised by the fact that many patients in this trial received surgery at a diam- 
eter less than 5.5 cm due to other factors. With increasing diameters, the annual 
rupture rates have been estimated to be as follows: 5-6 cm, 5-15%; 6-7 cm, 10-20%; 
7-8 cm, 20-40%; <greater than>8 cm, 30-50% [4]. [Q2:C] 

Recent trials have demonstrated a reduction in 30-day morbidity and mortality 
rate in patients undergoing endoluminal aortic aneurysm surgery compared to the 
traditional open approach [5, 6]. These well-constructed randomised trials showed 
a reduction of approximately 65 per cent in the incidence of moderate to major 
complications following endoluminal repair compared to open repair. Current 
information indicates that the benefit is likely to be greater for high-risk patients. 
The presence of renal failure is not an absolute contraindication to the endovascular 
approach as various precautions can be taken to protect the kidneys, such as intra- 
venous hydration, antioxidant medications or temporary dialysis. The average con- 
trast use for the procedure is 120-150 ml. [Q3: C, D] 

Not all aneurysms are anatomically suitable for endovascular repair. In general, 
all endoluminal grafts need areas of reasonably healthy vessel wall proximally and 
distally to be able to seal off the aneurysm from the blood flow. Most important for 
suitability are the size and morphology of the proximal neck (the segment of aortic 
wall between the lowest renal artery and the beginning of the aneurysm). The neck 
should consist of relatively normal aorta over a length of at least 1.5 cm, and the 
diameter should not exceed 30 mm. Another limitation for endovascular repair is 
the maximum angulation in the neck (should not exceed 60°) and the iliac arteries 
(ideally less than 90°) [7]. [Q4: A, C, D] 



Endoluminal Treatment of Infrarenal Abdominal Aortic Aneurysm 31 

After precise measurements from angiography and CT scanning, the optimal size 
of the graft can be chosen. To anchor the proximal part of the graft firmly onto the 
native aortic wall, a constant radial force of the graft is necessary. To achieve this, it 
is important to oversize the graft. Unfortunately, some recent reports suggest that 
there maybe slow ongoing expansion of the neck of the aneurysm after endolumi- 
nal AAA repair [8]. This may favour a more aggressive oversizing of the proximal 
part of the graft, but this could in turn cause infolding of the graft fabric, which may 
lead to failure to seal the AAA sac ("endoleak"). Current recommendations are to 
oversize the proximal part of the graft by 10-20 per cent and the limb diameters by 
10 per cent. [Q5:C] 

Most current devices for endovascular repair procedures depend on self-expand- 
ing stents or wireforms for attachment to the aortic wall. Most grafts are configured 
so that there is a marking indicating the contralateral limb. Preoperative measuring 
assessment of size is confirmed intraoperatively to prevent problems in length of 
the limbs. After the graft has been implanted, a pigtail angiographic catheter is rein- 
troduced, and a post-procedure digital subtraction angiogram is performed. The 
contrast run is examined closely for the presence of extravasation of contrast, sug- 
gesting an endoleak. In some grafts, the aneurysmal sac will fill temporarily with 
contrast due to porosity of the graft material. A retrograde angiogram through the 
femoral sheaths is used to visualise a seal at the distal end of the graft. [Q6: A, B, C, D] 

Complications of endoluminal AAA repair have been divided into remote/sys- 
temic and local/vascular. The remote/ systemic complications are similar to, but less 
frequent than, those occurring after open AAA repair. The local/vascular complica- 
tions are more specific for the endoluminal repair (Table 3.1). [Q7] 

The follow-up of endoluminal aneurysm repair patients remains important in 
determining the long-term success of aneurysm exclusion. General recommenda- 
tions include a physical examination and abdominal X-ray (AXR) plus contrast- 
enhanced CT scan within 1 week, then 6, 12 and 18 months after operation, and 
then annually. At present there are a number of registries such as Cleveland Clinic, 
and a European collaboration (EUROSTAR) [9] indicating an annual mortality rate 
of more than 1 per cent related to abdominal aortic aneurysms after endovascular 
repair of AAA (EVAR). It appears that 15-25 per cent of deployed grafts will require 
some secondary intervention. The requirement for secondary intervention is 
greater for earlier generation grafts, compared with the newer devices. This appears 

Table 3.1 . Local/vascular complications after endoluminal repair £Q 



Injury to access arteries 

Embolisation 

Distal ischaemia 
Renal failure 

Endoleak 

Type I (proximal or distal attachment zones) 
Type II (lumbar or mesenteric collateral channels) 
Type III (fabric tear or modular dislocation) 
Type IV (porosity leak) 
Endotension 

Graft limb thrombosis 

Groin wound infection 

Conversion to open repair 



32 



Vascular Surgery 



to be independent of the length of time the grafts remain in place. There is, 
however, significant improvement in quality-of-life measurements in patients with 
EVAR compared with open repair up to at least 6 months [10]. The importance of 
the type II endoleaks is unclear. However, they have been known to reappear after a 
period of absence. The position and the integrity of the graft is easily identified on 
plain abdominal X-ray. Whilst the aneurysmal sac can vary over time with endovas- 
cular repair, an expanding aneurysmal sac in the presence or absence (endotension) 
of endoleaks warrants further investigation and treatment. [Q8: B] 



Case Analysis Quiz 



A number of imaging examples are shown in Figs 3.5-3.8. Analyse the anatomical 
suitability of each case for the possibility of endoluminal graft repair of the AAA. In 
particular, determine the favourable and unfavourable features shown. 




Fig. 3.5. Aortic angiogram showing a very favourable anatomy for endovascular repair: the neck is straight and 
long, without irregular features of the wall. In addition, the aneurysm sac is straight, and both iliac arteries are 
non-aneurysmal and relatively straight. 



Endoluminal Treatment of Infrarenal Abdominal Aortic Aneurysm 



33 




Fig. 3.6. In this case, the aortagram shows neck angulation of about 45°, and the neck has a reversed taper con- 
figuration. Both these features are considered to be unfavourable for endoluminal treatment because it is more 
difficult for the device to achieve complete seal and reliable fixation). 




Fig. 3.7. Three-dimensional reconstruction of contrast-enhanced CT scan showing severe iliac angulation: a 
lateral view, b antero-inferior view. The iliac arteries show angulation in at least three different planes. The right 
common iliac artery is also aneurysmal. These features are unfavourable for access of the deployment sheath, 
and are associated with a higher risk of distal endoleak. 



34 Vascular Surgery 




Fig. 3.8. Aortic angiogram showing a 90° angulation within the neck of the aneurysm. This is considered to be 
unsuitable for endovascular repair, since it is difficult to achieve satisfactory seal of the AAA sac or long-term 
attachment of the device. GO 

References 

1. Melton LJ 3rd, Bickerstaff LK, Hollier LH, Van Peenen HJ, Lie JT, Pairolero PC, et al. Changing inci- 
dence of abdominal aortic aneurysms: a population-based study. Am J Epidemiol 1984;120:379-86. 

2. Jaakkola P, Hippelainen M, Farin P, Rytkonen H, Kainulainen S, Partanen K. Interobserver variabil- 
ity in measuring the dimensions of the abdominal aorta: comparison of ultrasound and computed 
tomography. Eur J Vase Endovasc Surg 1996;12:230-7. 

3. Fillinger M. Computed tomography and three-dimensional reconstruction in evaluation of 
vascular disease. In: Rutherford RB, editor. Vascular surgery, 5th edn. Philadelphia: WB Saunders, 
2000;230-69. 

4. Rutherford RB, editor. Vascular surgery, 5th edn. Philadelphia: WB Saunders, 2000. 

5. Greenhalgh RM, Brown LL, Kwong GP, Powell JJ, THompson SG. Comparison of endovascular repair 
with open repair in patients with AAA (EVAR trial 1) 30 day operative mortality results: randomised 
controlled trial. Lancet 2004;364:843-8. 

6. Prinssen M, Verhoeven ELG, Buth J, Cuypers PW, van Sambbek MR, Balm R, et al. A randomized 
trial comparing conventional and endovascular repair of abdominal aortic aneurysms. N Engl J Med 
2004;351:1607-18 

7. Ahn SS, Rutherford RB, Johnston KW, May J, Veith FJ, Baker JD, et al. Reporting standards for 
infrarenal endovascular abdominal aortic aneurysm repair. J Vase Surg 1997;25:405-10. 

8. Prinssen M, Wever JJ, Mali WP, Eikelboom BC, Blankensteijn JD. Concerns for the durability of the 
proximal abdominal aortic fixation from a 2 -year and 3 -year longitudinal computed tomography 
angiography study. J Vase Surg 2001;33:S64-9. 

9. Van Marrewijk CJ, Buth J, Harris PL, Norgren L, Nevelsteen A, Wyatt MG. Significance of endoleaks 
after endovascular repair of abdominal aneurysms: the EUROSTAR experience. J Vase Surg 
2002;35:461-73. 

10. Prinssen M, Buskens E, Blankensteijn JD, DREAM trial participants. Quality of life after endovascu- 
lar and open AAA repair. Results of a randomised trial. Eur J Vase Endovasc Surg 2004;27:121-27. 



4. Ruptured Abdominal Aortic Aneurysm 

Jeffrey S. Weiss and Bauer E. Sumpio 



A 70-year-old white male presents to the emergency department with sudden 
onset of severe back pain. The pain is described as severe and constant without 
alleviating or aggravating symptoms. He has never had pain like this before. He 
denies chest pain, shortness of breath, or loss of consciousness. He denies any 
history of an abdominal aortic aneurysm. His past medical history is significant 
for hypertension, and chronic obstructive pulmonary disease that requires home 
oxygen therapy. He had bilateral inguinal herniorrhaphy some years ago, but has 
never had a laparotomy. 

His vital signs yielded a pulse at 90 bpm and a blood pressure of 1 10/60 mm Hg. 
He is appropriately conversant and appears older than his stated age. He was 
without abdominal tenderness or masses and no bruits were heard; however, his 
belly was slightly obese and the examination was difficult. He has bilaterally 
palpable lower extremity pulses. 



Question 1 

What symptoms are considered the classic presenting triad for ruptured AAA? 

A. Abdominal/back pain, shortness of breath, and a pulsatile mass. 

B. Abdominal/back pain, syncope, and a pulsatile mass. 

C. Abdominal/back pain, nausea, and syncope. 

D. Abdominal/back pain, chest pain, and hematochezia. 

The patient remained stable while the emergency department staff obtained 
laboratory results and cross-matched blood, and performed an electrocardiogram 
(ECG). 



35 



36 Vascular Surgery 

Question 2 

If this patient is considered to have a ruptured AAA, which of the following factors 
does not adversely contribute to prognosis? 

A. Diabetes. 

B. Serum creatinine =1.8 mg/dl. 

C. Age =75 years. 

D. Preoperative blood pressure =80 mm Hg (systolic). 

E. Syncope. 

The patient's ECG shows normal sinus rhythm, the creatinine was 1.7 mg/dl, and 
the hematocrit was 32 percent. He remains hemodynamically stable. Your resident 
feels he is stable enough for a computed tomography (CT) scan (Fig. 4.1). 

Question 3 

Which of the following statements is true? 

A. Patients with unknown AAA history and symptoms should undergo further 
diagnostic imaging if they are hemodynamically stable. 

B. Symptomatic AAA should undergo emergency repair to prevent possible 
rupture. 

C. Patients with an unknown AAA history must have diagnostic imaging 
confirmation of an AAA before proceeding to the operating theatre. 




Fig. 4.1. Non-contrast CT scan of abdomen reveals an aortic aneurysm rupture in a left posterior location with 
extravasation into the retroperitoneum. 



Ruptured Abdominal Aortic Aneurysm 



37 




Yale New Hmn Hwpftal C5-2 Abdfteri 



Map 9 

P«riifl Mtd 
2D OptQflrt 

Ccl76% M»p5 
WF Med 
PRF 1500 Hi 
Flow Oplr Mid V 



02 Nov il Tit 11 Mf IS 

12:21:49 pm Fr #111 115cm 





23.0 

■ 




1 
1 



-23.0 
4 L-m.': 



LRA 



Fig. 4.2. Abdominal ultrasound with duplex color demonstrating rupture of aneurysm at the level of the left 
renal artery with a fluid collection in the left retroperitoneum. 

D. An ECG demonstrating ischemic changes in a patient with epigastric pain, 
hypotension and tachycardia is the sine qua non for a myocardial infarction and 
any operation should be postponed. 

E. CT scans are reserved for elective evaluation of AAA and have no place in the 
work-up of a symptomatic AAA. 

Question 4 

If an ultrasound (Fig. 4.2) was obtained instead of a CT scan, what statements could 
be made regarding this study? 

A. Ultrasound is more reliable than CT scan for the diagnosis of ruptured AAA. 

B. The location of the rupture is typical for most ruptured AAAs. 

C. Ultrasound can be performed quickly at the bedside. 

D. Ultrasound can be used to provide endograft measurements. 

E. Ultrasound is best used in unstable patients to confirm the presence of a known 
AAA. 

After the confirmation of ruptured AAA by radiology, the patient is taken imme- 
diately to the operating room. 



Question 5 

All of the following measures are indicated in the perioperative management of a 
ruptured AAA, except: 

A. Surgical preparation and drape before induction. 



38 Vascular Surgery 

B. Preoperative resuscitation to normal blood pressure. 

C. Passive cooling of the patient. 

D. Heparinization before cross-clamping. 

E. Blood recuperation and autotransfusion devices. 

The patient is prepared and draped, the anesthetic administered, and operation 
commenced. The medical student asks if this could be done via an endovascular 
approach. 

Question 6 

Currently, what are the contraindications for endovascular repair of ruptured 
AAAs? 

A. Infrarenal neck diameter >30 mm. 

B. Infrarenal neck length <10 mm. 

C. Systolic blood pressure <100 mm Hg. 

D. Endograft or "endograft team" not available. 

E. Thrombus present at infrarenal neck. 

The patient was determined to have too large a neck diameter for an endo- 
vascular stent, so you decide to proceed with an open repair. After induction, 
the patient's blood pressure falls to a systolic of 60 mm Hg. A supraceliac clamp is 
quickly placed and the aneurysm exposed. The rupture was contained to the 
retroperitoneum, but is rather large. The supraceliac clamp is moved to an 
infrarenal position after about 10 minutes. Anesthesia quickly catches up and his 
systolic blood pressure rises to 100 mm Hg. The inferior mesenteric artery was 
not patent and the iliac arteries were without aneurysms, allowing a Dacron 
tube graft to be placed. The clamp is slowly removed and he remains hemody- 
namically stable. The bowel appears well perfused and distal pulses are palpable 
before closure. Postoperatively, the patient recovers in the surgical intensive care 
unit. 



Question 7 

The most common complication following repair of ruptured AAAs is? 

A. Aortoenteric fistula. 

B. Bowel ischemia. 

C. Myocardial ischemia. 

D. Atheroemboli. 

E. Acute renal failure. 



Ruptured Abdominal Aortic Aneurysm 39 

He is noted to have a creatinine that rises to 4.7 mg/dl 2 days after operation and 
his urine output falls to less than 100 ml/day. He is eventually placed on intermittent 
hemodialysis because of volume overload. Over the next 2 weeks he is weaned off the 
ventilator, his urine output slowly increases, and his creatinine levels stabilizes at 
2.0 mg/dl. He is discharged to a convalescence facility 19 days after operation. 



Commentary 

The optimal treatment of ruptured abdominal aortic aneurysm (rAAA) is preven- 
tion; unfortunately close to 70 percent of presenting patients have no prior diagno- 
sis [1]. The overall mortality rates for rAAA are 80-90 percent with operative 
mortality around 50 percent [2-4]. Although more than three-quarters of patients 
with an rAAA report either abdominal or back pain, they can present with a myriad 
of symptoms and signs that are both broad and inconsistently present [5]. The triad 
of hypotension, abdominal pain, and a pulsatile mass [Q1: B] are found together in 
only half of cases [6]. A great deal of effort has been applied to identifying perioper- 
ative risk factors for patients who have a decreased survival advantage. Preoperative 
risk factors include: age <75-76 years, hypotension =80-95 mm Hg, creatinine 
= 1.8-1.9 mg/dl, loss of consciousness, ECG ischemia or dysrhythmia, CHF, hemo- 
globin <9 g/dl, base deficit >8, and free rupture [7-10]. [Q2: A] Intraoperative risk 
factors include: blood loss >2-3.5 liters, duration of surgery >200 min, aortic cross- 
clamp time >47 min, lack of autotransfusion devices, bifurcated grafts, and techni- 
cal complications (i.e., left renal vein injury) [11-13]. Postoperative risk factors 
include renal failure, coagulopathy, and cardiac complications. Hardman et al. [10] 
found that possession of three or more preoperative risk factors correlated with 100 
percent mortality. Currently, no recommendation exists to withhold surgery for 
patients with any or all of these risk factors; this decision is made on a case-by-case 
basis, making risk factor analysis useful mostly from the standpoint of guiding 
patient decisions on surgery and family discussions on prognosis. 

Patients who present with symptoms of a rAAA can be divided into two groups 
based on whether or not they have a known AAA (Fig. 4.3) [14]. Unstable patients 
with known AAAs present the least diagnostic challenge as they belong in the oper- 
ating room. In contrast, the unstable patient without known AAA can be the hardest 
to evaluate. If an rAAA is suspected, this patient needs to be assessed expeditiously 
with an ECG as myocardial infarction can often mimic these symptoms. If cardio- 
genic shock is clinically apparent, resuscitation should override emergent surgery; 
however, cardiac ischemia secondary to hypovolemic shock from a rupture needs 
both rapid resuscitation and emergent surgery as the underlying cause of shock is 
the rupture and not the heart. Patients without hemodynamic instability allow 
the examiner the time to proceed with radiological confirmation [15]. [Q3: A] 
Ultrasound is fast and convenient as it allows an examination while resuscitation is 
taking place at the bedside. The sensitivity is as high as 100 percent for detecting an 
AAA, but it is inaccurate on diagnosing rupture (49 percent) [12,16]. This study is 
ideal on hemodynamically stable patients without known AAA, minimal operative 
risk factors, and symptoms or signs suggestive of rupture. [Q4: B, C] In this case, the 
mere presence of an AAA would warrant surgery without delay. CT scans are more 
difficult to obtain and place the patient at some increased risk because of time delay 
and interruption of resuscitation. They are clearly only indicated for patients who 



40 



Vascular Surgery 



Symptoms/Signs of Ruptured Abdominal Aortic Aneurysm 




Stable 



Unstable 



Unstable 



Stable 




Not Ruptured 



Preoperative 
optimization of 
comorbidities, 
OR as soon as 

practicable 



No AAA 



Find another cause 



Not Ruptured 



Fig. 4.3. Algorithm for suspected rAAA. 



are stable and offer the advantage of being able to diagnosis rupture. The groups of 
patients most likely to benefit from CT scan are those with significant comorbidities 
where delay could allow preoperative optimization [17]. The sensitivity and 
specificity of CT scan for diagnosing rupture is quoted to be as high as 94 percent 
and 95 percent, respectively [15]. 

Once the decision to operate has been made, several preoperative measures 
should be undertaken. A natural instinct is to bolus intravenous (IV) fluid in an 
attempt to normalize the blood pressure; this should be avoided. Instead, adopting 
a permissive hypotensive strategy will allow the patient's own physiologic response 
to minimize blood loss [18]. Although there are times when fluids are necessary, 
this strategy can be effective in preventing accelerated blood loss until the aorta is 
clamped or occluded. Every effort should be made to keep the patient warm with 
blankets, raising the operating room temperature, and utilizing warmed IV fluids 



Ruptured Abdominal Aortic Aneurysm 41 

and blood products [8]. The patient should be prepared and draped before induc- 
tion as the loss of sympathetic tone with anesthesia may cause a marginally com- 
pensated patient to collapse. 

A midline laparotomy provides the quickest route of entry and best exposure in 
most cases. A low threshold to obtain supraceliac control will prevent inadvertent 
venous injury, especially in cases with large retroperitoneal hematomas. This 
control is obtained by incising the gastrohepatic ligament and diaphragmatic crura, 
and then bluntly dissecting the periaortic tissue; a preoperative nasogastric tube can 
aid in identification of the laterally positioned esophagus. A clamp or manual pres- 
sure is applied to the supraceliac aorta. The transverse colon is reflected cephalad 
and the small bowel eviscerated. The supraceliac control can then be moved to the 
infrarenal neck after it is carefully dissected out. Systemic heparinization is avoided 
and heparinized saline (10 units/ml) is used locally down both iliacs before balloon 
occlusion. The use of intraoperative blood recuperation and autotransfusion 
devices is crucial in minimizing postoperative mortality by limiting homologous 
blood transfusions [13]. The use of a tube graft, typically knitted Dacron or PTFE, 
will shorten operative times and restore flow sooner than a bifurcated graft; this 
may necessitate leaving aneurysmal iliac arteries alone [14]. [Q5: B, C, D] After com- 
pletion of grafting, bowel and lower extremity perfusion are assessed, usually by 
inspection and Doppler probe. The aneurysm sac is closed around the graft in an 
attempt to prevent later aortoenteric fistulas. Depending on the size of retroperi- 
toneal hematoma and degree of resuscitation, the abdomen may not close easily. In 
these cases, it is best to perform a temporary closure with plans to return to the 
operating room for washout and definitive closure at a later, more stable time. 

The dismal mortality following open repair of rAAA and the expansion of 
endovascular techniques has prompted recent exploration into application of stent 
grafts for primary therapy. Patient candidacy for an endovascular repair of AAA 
(EVAR) is the first hurdle when considering this approach. Measurements to deter- 
mine this are typically done by CT angiography, although the Montefiore group 
have been successful utilizing digital subtraction angiography in two views [19]. The 
concern of sending a potentially unstable patient with known or suspected ruptured 
AAA to the CT scanner was recently addressed by Lloyd et al. [20] from Leicester; 
they found that 87.5 percent of patients survived longer than 2 hours after admis- 
sion, with 92 percent of these patients having systolic blood pressures greater than 
80 mm Hg. Ruptured or symptomatic AAAs are found to have larger infrarenal 
neck diameters and smaller neck lengths [21]. Despite these morphological differ- 
ences, several reports have found amazingly high feasibility rates for EVAR, ranging 
from 46 percent to 80 percent [22, 23]. Dimensional requirements for endografts are 
constantly shifting as new devices improve the field, but currently an infrarenal 
neck =10 mm and a diameter =30 mm are needed [24]. [Q6: A, B, D] The next hurdle 
is availability of an endograft team and the graft itself. The importance of a knowl- 
edgeable and experienced team cannot be overstated as any program without this is 
destined for failure. A variety of grafts are being utilized, with favor towards a 
modular aorto-uniiliac device; this set-up decreases the need for large inventories 
[23, 24]. The Montefiore group have developed an aorto-unifemoral graft which 
they use in conjunction with a crossover femoral-femoral graft [19]. Surprisingly 
few patients are rejected for EVAR secondary to unfavorable hemodynamics. 
Supraceliac balloon occlusion via a brachial or femoral route under fluoroscopic 
guidance can allow proximal aortic control under local anesthesia; a technique 
being utilized by some for control prior to laparotomy in open cases [25]. 



42 Vascular Surgery 

Prospective randomized studies are underway to examine the morbidity and mor- 
tality rates of EVAR with respect to open repair, but preliminary nonrandomized 
results are already favoring this approach [19, 24, 26]. 

The most common complication of rAAA repair is renal failure, followed by ileus, 
sepsis, myocardial infarction, respiratory failure, bleeding, and bowel ischemia [1, 
11]. [Q7: E] Postoperative renal failure has been found by several authors to correlate 
with mortality [1, 11]. Minimizing suprarenal clamp time and use of mannitol 
before cross-clamping the aorta to initiate brisk diuresis may limit renal damage. 
The inflammatory mediators and cytokines released from the shock state, visceral 
hypoperfusion, and massive transfusions associated with open repair can lead to 
multi-organ system failure; the avoidance of supraceliac clamping and lower blood 
loss are some of the potential advantages of the EVAR approach. But EVAR has its 
own unique complications which include endoleaks, graft malfunction, and groin 
wound issues. 



References 

1. Noel AA, Gloviczki P, Cherry KJ, Bower TC, Panneton JM, Mozes GI, et al. Ruptured abdominal 
aortic aneurysms: The excessive mortality rate of conventional repair. J Vase Surg 2001;34:41-6. 

2. Dardik A, Burleyson GP, Bowman H, Gordon TA, Webb TH, Perler BA. Surgical repair of ruptured 
abdominal aortic aneurysms in the state of Maryland: Factors influencing outcome among 527 
recent cases. J Vase Surg 1998;28:413-21. 

3. Heller JA, Weinberg A, Arons R, Krishnasastry KV, Lyon RT, Deitch JS, et al. Two decades of abdom- 
inal aortic repair: have we made any progress? J Vase Surg 2000;32:1091-100. 

4. Bengtsson H, Bergqvist D. Ruptured abdominal aortic aneurysm: A population-based study. J Vase 
Surg 1993;18:74-80. 

5. Rose J, Civil I, Koelmeyer T, Haydock D, Adams D. Ruptured abdominal aortic aneurysms: Clinical 
presentation in Auckland 1993-1997. Aust N Z J Surg 2001;71:341-344. 

6. Wakefield TW, Whitehouse WM, Wu SC, Zelenock GB, Cronenwett JL, Erlandson EE, et al. 
Abdominal aortic aneurysm rupture: statistical analysis of factors affecting outcome of surgical 
treatment. Surgery 1982;91:586-96. 

7. Sasaki S, Yasuda K, Yamauchi H, Shiya N, Sakuma M. Determinants of the postoperative and long- 
term survival of patients with ruptured abdominal aortic aneurysms. Surg Today 1998;28:30-5. 

8. Piper G, Patel N, Chandela S, Benckart DH, Young JC, Collela JJ, et al. Short-term predictors and 
long-term outcome after ruptured abdominal aortic aneurysm repair. Am Surg 2003;63:703-10. 

9. Shackelton CR, Schechter MT, Bianco R, Hildebrand HD. Preoperative predictors of mortality risk in 
ruptured abdominal aortic aneurysm. J Vase Surg 1987;6:583-9. 

10. Hardman DT, Fisher CM, Patel MI, Neale M, Chambers J, Lane R, et al. Ruptured abdominal aortic 
aneurysms: who should be offered surgery? J Vase Surg 1996;23:123-9. 

11. Donaldson MC, Rosenberg JM, Bucknam CA. Factors affecting survival after ruptured abdominal 
aortic aneurysm. J Vase Surg 1985;2:564-70. 

12. Markovic M, Davidovic L, Maksimovic Z, Kostic D, Cinara I, Cvetkovic S, et al. Ruptured abdominal 
aortic aneurysm predictors of survival in 229 consecutive surgical patients. Herz 2004;29:123-9. 

13. Marty- Ane CH, Alric P, Picot MC, Picard E, Colson P, Mary H. Ruptured abdominal aortic 
aneurysm: influence of intraoperative management on surgical outcome. J Vase Surg 1995;22:780-6. 

14. Hallett JW, Rasmussen TE. Ruptured abdominal aortic aneurysm. In: Cronenwett JL, Rutherford RB 
, editors. Decision making in vascular surgery. Philadelphia: Saunders, 2001; 104-7. 

15. Kvilekval KH, Best IM, Mason RA, Newton GB, Giron F. The value of computed tomography in the 
management of symptomatic abdominal aortic aneurysms. J Vase Surg 1990;12:28-33. 

16. Tayal VS, Graf CD, Gibbs MA. Prospective study of accuracy and outcome of emergency ultrasound 
for abdominal aortic aneurysm over two years. Acad Emerg Med 2003;10:867-71. 

17. Sullivan CA, Rohrer MJ, Cutler BS. Clinical management of the symptomatic but unruptured abdom- 
inal aortic aneurysm. J Vase Surg 1990;11:799-803. 

18. Owens TM, Watson WC, Prough DS, Uchida T, Kramer, GC. Limiting initial resuscitation of uncon- 
trolled hemorrhage reduces internal bleeding and subsequent volume requirements. J Trauma 
1995;39:200-209. 



Ruptured Abdominal Aortic Aneurysm 43 

19. Veith FJ, Ohki T, Lipsitz EC, Suggs WD, Cynamon J. Treatment of ruptured abdominal aneurysms 
with stent grafts: a new gold standard?. Semin Vase Surg 2003;16:171-175. 

20. Lloyd GM, Bown MJ, Norwood MG, Deb R, Fishwick G, Bell PR, et al. Feasibility of preoperative 
computer tomography in patients with ruptured abdominal aortic aneurysm: a time-to-death study 
in patients without operation. J Vase Surg 2004;39:788-91. 

21. Lee WA, Huber, TS, Hirneise CM, Berceli SA, Seeger JM. Eligibility rates of ruptured and sympto- 
matic AAA for endovascular repair. J Endovasc Ther 2002;9:436-42. 

22. Reichart M, Geelkerken RH, Huisman AB, van Det RJ, de Smit P, Volker EP. Ruptured abdominal 
aortic aneurysm: endovascular repair is feasible in 40% of patients. Eur J Vase Endovasc Surg 
2003;26:479-86. 

23. Hinchliffe RJ, Braithwaite BD, Hopkinson BR. The endovascular management of ruptured abdomi- 
nal aortic aneurysms. Eur J Vase Endovasc Surg 2003;25:191-201. 

24. Peppelenbosch N, Yilmaz N, van Marrewijk C, Buth J, Cuypers P, Duijm L, et al. Emergency treat- 
ment of acute symptomatic or ruptured abdominal aortic aneurysm. Outcome of a prospective 
intent-to-treat by EVAR protocol. Eur J Vase Endovasc Surg 2003;26:303-10. 

25. Matsuda H, Tanaka Y, Hino Y, Matsukawa R, Ozaki N, Okada K, et al. Transbrachial arterial inser- 
tion of aortic occlusion balloon catheter in patients with shock from ruptured abdominal aortic 
aneurysm. J Vase Surg 2003;38:1293-6. 

26. Lee WA, Huber, TS, Hirneise CM, Berceli SA, Seeger JM. Impact of endovascular repair on early out- 
comes of ruptured abdominal aortic aneurysms. J Vase Surg 2004;40:211-5. 



5. Thoracoabdominal Aortic Aneurysm 

Nicholas }. Morrissey, Larry H. Hollier and 
Julius H. Jacobson II 



A 72-year-old white male presented to his primary-care physician with a 
history of left chest pain for the past month. The pain was dull and constant, 
and radiated to the back, medial to the scapula. He denied new cough or wors- 
ening shortness of breath. He had no recent weight loss, and his appetite was 
good. He has a history of hypertension that was currently controlled medically. 
He had a smoking history of 60 packs a year. In addition, he suffered a 
myocardial infarction (MI) 5 years ago. The patient denied any history of clau- 
dication, transient ischaemic attacks or stroke. He had undergone surgery in 
the past for bilateral inguinal hernias, and he underwent cardiac catheterisa- 
tion after his MI. 

On physical examination, the patient was thin but did not appear malnour- 
ished. Vital signs were respiratory rate 18/min, heart rate 72 bpm, blood pressure 
140/80 mm Hg and temperature 36.8°C. His head and neck examination was 
remarkable only for bilateral carotid bruits. Cardiac examination revealed a 
regular rate and rhythm without murmurs. Abdominal examination revealed no 
bruits and an infrarenal aortic diameter of approximately 2.5 cm by palpation. 
His femoral and popliteal pulses were normal. Posterior tibial pulses were 1 + 
bilaterally, and dorsalis pedis were detectable only by Doppler. Routine blood 
work was unremarkable, and an electrocardiogram (ECG) revealed changes con- 
sistent with an old inferior wall MI and left ventricular (LV) hypertrophy. Chest 
X-ray (Fig. 5.1) was remarkable for a tortuous aorta, which had calcification 
within the wall and appeared dilated. There were no pleural effusions, there was 
some flattening of both hemidiaphragms, and bony structures were normal. 
Lung fields were clear of masses or consolidation. 



Question 1 

Which of the following is the single most likely diagnosis causing this man's pain? 

A. Acute MI. 

B. Acute aortic dissection. 

C. Thoracic aortic aneurysm. 

D. Lung cancer. 

E. Pneumonia. 

45 



46 



Vascular Surgery 




Fig. 5.1 . Chest X-ray demonstrating tortuous and dilated descending thoracic aorta suggestive of a thoracoab- 
dominal aorta. 



Question 2 

Which of the following studies should be performed in this patient in order to plan 
therapy? 

A. Aortography. 

B. Computed tomography (CT) scan of chest. 

C. Carotid duplex studies. 

D. Cardiac stress test. 

E. Arterial blood gas (ABG) analysis. 

The CT scans of the chest and aortagram shown in Figs 5.2 and 5.3 were obtained. 
Findings were consistent with a thoracoabdominal aneurysm without concomitant 
dissection of the aorta. There was no evidence for acute leak or rupture, and the 
maximal diameter of the thoracic aorta was 7.3 cm. 



Question 3 

Briefly describe the Crawford classification system for thoracoabdominal aortic 
aneurysms TAAs). 



Thoracoabdominal Aortic Aneurysm 



47 




Fig. 5.2. CT scan demonstrating aneurysmal dilation of the descending thoracic aorta. 




Fig. 5.3. Aortagram of patient in Fig. 5.2 showing tortuosity of aneurysmal aorta. Note the disparity between 
lumen size and aortic diameter, indicating a significant amount of mural thrombus. 



48 Vascular Surgery 

The patient underwent a cardiac stress test, which was normal. Carotid duplex 
studies revealed bilateral stenoses of less than 50 per cent. ABG analysis showed pH 
7.38, pC0 2 42 and p0 2 76 on room air. 



Question 4 

Which of the following management schemes seems most reasonable for this 
patient? 

A. Observation with annual follow-up chest CT. 

B. Repair of thoracoabdominal aneurysm after bilateral carotid endarterectomies. 

C. Cardiac catheterisation followed by repair of TAA. 

D. Elective repair of TAA. 

The patient is scheduled for elective repair of his TAA. He expresses concern 
about the possibility of complications from the surgery. You explain to him the 
most likely complications related to this surgery. 

Question 5 

List the four most common complications following TAA repair. 

The patient seems most concerned about the risk of postoperative paralysis. You 
explain to him that there are things you can do to decrease his risk of suffering this 
complication, although nothing can eliminate the risk. 

Question 6 

List four technical modifications that maybe beneficial in the prevention of spinal 
cord dysfunction following TAA repair. 

The patient undergoes repair of TAA and tolerates the procedure well. 
Postoperatively, the chest tubes are draining 100-150 cm 3 blood/hour for the first 
3 h. In addition, urine output is steady at 500 cm 3 /h. The patient has transient drops 
in blood pressure to a systolic blood pressure in the 70s, with central venous pres- 
sure dropping to 5 mm Hg. 

Question 7 

(a) Describe the initial work-up and potential correction of the bleeding problem 
described above in order to prevent a return to the operating room, (b) What fluid 
resuscitation approach should be taken to stabilise this patient's haemodynamic 
status? 

The patient's temperature is 34.6°C, international normalised ration (INR) is 1.7 
and prothrombin time (PTT) is 50 s (control, 34 s). Platelet count is 33,000. After 



Thoracoabdominal Aortic Aneurysm 49 

infusion of warm fluids, the use of a warming blanket, and platelet and fresh 
frozen plasma (FFP) transfusions, the parameters return to normal and the 
drainage from the chest tubes decreases to about 10-20 cm 3 /h. On the second 
postoperative day, the patient is noted to have loss of motor function in his lower 
extremities. 



Question 8 

What therapeutic intervention may, if carried out in a timely fashion, restore this 
patient's neurological function partially or fully? 

Following appropriate intervention, the patient's neurological function returns to 
normal. The patient's recovery is otherwise uneventful, and he is discharged on 
postoperative day 8 with clean incisions, intact neurological status and adequate 
analgesia. 

Question 9 

What is this man's approximate predicted 5-year survival? 



Commentary 

TAAs are less common than infrarenal abdominal aortic aneurysms. One popula- 
tion-based study suggested an incidence of 5.9 TAAs per 100,000 person-years [1]. 
Although TAAs are more common in males, the male : female ratio of 1.1-2.1 : 1 is 
not as weighted as the ratio of abdominal aortic aneurysm (AAA). The aetiology of 
TAAs is related to atherosclerotic medial degenerative disease (82 per cent) and 
aortic dissection (17 per cent) in most cases [2]. About 45 per cent of TAAs are 
asymptomatic and detected during work-up of other systems, usually on chest X- 
ray or cardiac echo exam. Patients tend to be older than AAA patients and therefore 
may have more severe comorbidities. When present, symptoms are usually pain 
related to compression of adjacent structures by the aneurysm or cough from com- 
pression/erosion of airways. Fistulisation into the bronchial tree can lead to massive 
haemoptysis, while erosion into the oesophagus can result in upper-gastrointestinal 
bleeding. Acute, severe pain may reflect leak, acute expansion or dissection of the 
aneurysm and may require more urgent evaluation and treatment. The risk factors 
associated with TAA are smoking, hypertension, coronary artery disease, chronic 
obstructive pulmonary disease, and disease in other vascular beds. Syphilitic 
aneurysms are a rare cause of TAA in this era and usually involve the ascending 
aorta. Other causes of vague chest and back pain in a patient such as this include 
myocardial ischaemia, pulmonary neoplasm, acute dissection, pneumonia, and 
bony metastases. [Q1: C] The clinical and X-ray findings in this particular case argue 
against these other possibilities. 

The work-up of patients with TAA requires assessment of the aneurysm extent 
and size, as well as of the condition of the remaining aorta. [Q2: A, B, C, D, E] Before 
any studies are carried out, a thorough history and physical examination, including 
vascular assessment, are needed. Currently, aortography remains an important tool 



50 Vascular Surgery 

for defining the extent of TAA as well as the status of aortic branches, but CT exam 
is the most useful diagnostic test. Helical CT scanning and CT angiography are 
rapidly becoming more precise in terms of delineating extent of aneurysm, as well 
as diameter, presence of dissection, presence of leak, and involvement of aortic 
branches. Magnetic resonance imaging (MRI) and magnetic resonance angiogra- 
phy (MRA) continue to improve and offer benefits over CT such as lack of radia- 
tion and non-nephrotoxic contrast agents. MRA has not yet achieved the 
resolution of conventional angiography, and its use is contraindicated in unstable 
patients. Transoesophageal echocardiography can assess the status of the aortic 
valve as well as cardiac function. Significant aortic insufficiency is a contraindica- 
tion to thoracic aortic cross-clamping unless a shunt or pump is used to bypass 
the left heart. In addition to assessment of the aneurysm, the high incidence of 
comorbidities in this patient population mandates thorough evaluation of cardiac 
as well as pulmonary reserve. Preoperative studies should include EKG and 
cardiac stress testing. Further work-up will be dictated by the presence of positive 
findings. Screening chest X-ray and preoperative ABG will provide information 
regarding pulmonary status. Formal pulmonary function tests should be reserved 
for those patients with evidence of significant pulmonary compromise. Since the 
risk factors for TAA are the same as those for atherosclerotic disease, a careful 
history and physical will dictate whether there is a need to work up disease in 
other beds (carotid, mesenteric, renal, lower extremity). Carotid duplex studies 
maybe done routinely preoperatively and significant stenoses treated before TAA 
repair. The status of the patient's clotting system must be determined and opti- 
mised if necessary. In the absence of indications to carry out other operations first, 
this patient with a TAA of >6 cm should undergo elective repair of his aneurysm. 
[Q4: D] Observation with follow-up imaging studies is dangerous and puts the 
patient at risk of death due to aneurysm rupture. 

The Crawford classification [Q3] is used to characterise TAAs (Fig. 5.4) [3]. 
According to this system, aneurysms beginning just distal to the left subclavian 
artery and involving the aorta up to but not below the renals are termed type I. Type 
II begin aneurysms begin just beyond the left subclavian and continue into the 
infrarenal aorta. Type III aneurysms involve the distal half of the thoracic aorta and 
varying extents of the abdominal aorta, while type IV refers to those aneurysms 
involving the entire abdominal aorta, up to the diaphragm and including the vis- 
ceral segments. This classification scheme has been useful for predicting morbidity 
and mortality following repair of TAAs. In the case of non-dissecting TAA, the four 
types occur with approximately equal frequency. 

The natural history of TAAs is related to size and growth rate. Understanding 
the behaviour of these lesions is of crucial importance when determining treat- 
ment. Crawford's series of 94 TAAs followed for 25 years demonstrated 2-year 
survival of 24 per cent, with about half of deaths due to rupture [4]. This series 
included dissected as well as non-dissected aneurysms. A more recent series of 
non-dissected TAAs revealed rupture rates of 12 per cent at 2 years and 32 per 
cent at 4 years; for aneurysms greater than 5 cm in diameter, rupture rates 
increased to 18 per cent at 2 years [5]. Rupture is very uncommon in aneurysms 
measuring less than 5 cm in diameter. Another risk factor for rupture seems to be 
expansion rate, with aneurysms growing more than 5 mm in 6 months at higher 
risk than those growing more slowly. Survival in nonoperated patients was 52 per 
cent at 2 years and 17 per cent at 5 years. Patients who underwent repair of TAA 
had a 5-year survival of 50 per cent. Another series revealed 61 per cent 5-year 



Thoracoabdominal Aortic Aneurysm 



51 







*-A"V ( ^t?£ 




r 



ii 



in 



IV 



Fig. 5.4. Crawford classification of thoracoabdominal aortic aneurysms, types I — IV. 

Reproduced from Morrissey NJ, Hamilton IN, Hollier LH. Thoracoabdominal aortic aneurysms. In: Moore W, 
editor. Vascular surgery: a comprehensive review. Philadelphia: WB Saunders, 2001 ; 41 7-434, with permission 
from Elsevier. 



survival following TAA repair. Survival decreased to 50 per cent for patients with 
dissecting TAA [6]. [Q9] 

Operative repair is usually through a left thoracotomy with a paramedian abdom- 
inal extension, depending on the distal extent of the aneurysm. A retroperitoneal 
approach to the abdominal segment is used. The distal extent of the aneurysm 
determines which intercostal space will be used for a thoracotomy. The incision is 
in the fourth or fifth intercostal space for type I or high type II TAA, while an inci- 
sion in the seventh, eighth or ninth space is appropriate for types III or IV [7]. 
Careful identification and reimplantation of visceral vessels is important, as is reat- 
tachment of intercostal arteries when feasible. Successful repair of TAA results from 
careful yet quick technique, as well as maintenance of optimal physiology by the 
anaesthesia and surgical teams. Distal aortic perfusion is accomplished either with 
left heart bypass and selective visceral perfusion or an axillary-femoral artery 
bypass before thoracotomy. Distal aortic perfusion manoeuvres are important for 
the prevention of major systemic morbidity following TAA repair. 

Patients undergoing TAA repair frequently are older and have significant cardiac, 
pulmonary and other vascular comorbidities. These factors, combined with the 
magnitude of the operation and extent of aortic replacement, can lead to significant 
rates of mortality and serious morbidity. [Q5] Pulmonary complications remain 
most common and result from a combination of preoperative tobacco use, chronic 
obstructive pulmonary disease (COPD), and the effect of the thoracoabdominal 
incision on postoperative pulmonary mechanics. Reperfusion injury may also lead 



52 



Vascular Surgery 



to pulmonary microvascular injury and subsequent pulmonary dysfunction [8]. 
Cardiac complications remain the next most common, in spite of preoperative 
cardiac optimisation. Avoidance of hypotension, close monitoring perioperatively 
with pulmonary artery catheters, and minimisation of strain on the left ventricle 
can help decrease postoperative cardiac dysfunction. Using the bypass circuit to 
control ventricular afterload can reduce the risk of cardiac complications [9]. Renal 
insufficiency preoperatively increases the risk of postoperative renal failure. 
Minimising ischaemic time, selective renal perfusion during cross-clamping, distal 
aortic perfusion techniques, and avoidance of hypovolaemia are important in pre- 
venting renal failure [10]. 

Perhaps the most devastating complication following TAA repair is paraplegia. 
Despite years of research and development of protective strategies, paraplegia rates 
following TAA repair remain between 5 and 30 per cent, with an average of 13 per 
cent [6]. Risk factors for postoperative paraplegia include extent of aneurysm, 
cross-clamp time, postoperative hypotension, and oversewing of intercostal arter- 
ies. Cross-clamp times of less than 30 min are generally safe, while those in the 
range of 30-60 min are associated with increasing risk; cross-clamp times of more 
than 60 min carry the highest risk for neurological complications (Fig. 5.5). 
Minimising cross-clamp time and avoiding hypotension will decrease the risk of 
paraplegia. Sequential reperfusion of intercostal vessels by moving the cross clamp 
caudally as segments are reimplanted is useful to re-establish flow to these vessels 
quickly. In addition, avoiding prolonged mesenteric ischaemia, which may worsen 
reperfusion injury to the lungs, heart and possibly spinal cord through release of 
cytokines, is beneficial. 

Numerous adjuncts have been studied for their ability to prevent paraplegia. [Q6] 
The use of cerebrospinal fluid (CSF) drainage to keep CSF pressure at less than 
10 mm Hg has been shown to decrease the incidence of postoperative paraplegia 
when combined with distal aortic perfusion and/or moderate hypothermia [11]. 



1.0 








>> 








.i. 0.9 


Safe 


Vulnerable/ 


' Certain 


If - 8 








S g 0.7 








2 - 0.6 








■— o 








8- .2 0.5 








M— 0> 








° - 04 








■*— ' CC 








■= £ 0.3 
1 3 0.2 








o 








£ 0.1 







i 


30 60 9l 




Aortic 


: cross-clamp ti 


me (min) 



Fig. 5.5. Probability of postoperative paraplegia as a function of aortic cross-clamp time. 

Reproduced from Svensson L, Loop F. Prevention of spinal cord ischemia in aortic surgery. In: Yao JT, editor. 
Arterial surgery. New York: Grune & Stratton, 1 988; 273-85, with permission from Elsevier. 



Thoracoabdominal Aortic Aneurysm 53 

Reimplantation of intercostal vessels is most likely beneficial in preventing postop- 
erative paraplegia, provided this manoeuvre does not excessively prolong clamp 
time [12]. Epidural cooling by continuous infusion of cool saline via a catheter has 
been reported to decrease the incidence of paraplegia following TAA repair [13]. 
Preoperative angiographic localisation of the artery of Adamkiewicz followed by 
successful reimplantation of this vessel during surgery has resulted in no neurolog- 
ical sequelae in one series [14]. Patients who did not have preoperative localisation, 
or in whom reimplantation was unsuccessful, had a 50 per cent paraplegia rate. 
These results have not been reproduced, and angiographic localisation has not 
gained widespread acceptance. General anaesthetic agents can also help to prevent 
paraplegia, with propofol being the most protective. When left heart bypass is per- 
formed using pump techniques, moderate hypothermia can be used to protect the 
spinal cord. Other pharmacological adjuncts that may be beneficial include steroids 
and mannitol. Free-radical scavengers and inhibitors of excitatory neurotransmitter 
pathways have shown benefit experimentally but have not been proven clinically 
15]. At present, the best strategy for preventing spinal cord complications appears 
to involve a combination of physiological optimisation of the patient periopera- 
tively, intraoperative use of spinal drainage and some form of distal aortic perfu- 
sion, reimplantation of patent intercostal vessels, and minimisation of cross-clamp 
time. Other protective adjuncts are used based on surgeon preference and experi- 
ence. Fig. 5.6 summarises the pathophysiology and prevention of neurological 
injury following TAA repair. 

Some patients, as in the case we present here, will awake neurologically intact 
only to develop paraplegia hours to days later. [Q8] This phenomenon of delayed- 
onset paraplegia may represent reperfusion injury to areas of the spinal cord at 
risk from intraoperative hypoperfusion. Avoidance of postoperative hypoperfu- 
sion may decrease the incidence of this complication. The epidural catheter is left 
in place for 3 days postoperatively. In cases of delayed-onset paraplegia, 
maintenance of CSF pressure below 10 mm Hg may permit restoration of func- 
tion. There are anecdotal reports of reversal of delayed-onset paraplegia by place- 
ment of an epidural catheter after onset of paralysis and removal of CSF to 
decrease pressure to below lOmmHg [16]. Lowering the CSF pressure may 
increase cord perfusion pressure enough to rescue the threatened regions of neu- 
ronal tissue. Lowering the CSF pressure to below 5 mm Hg may cause intra- 
cerebral haemorrhage, therefore the pressure must be monitored closely and 
maintained in the safe range. 

Repair of a TAA represents a major physiological insult. Excellent anaesthesia 
care and monitoring are essential components of a successful operation. 
Postoperatively, large volumes of urine output must be replaced on a 1 : 1 basis in 
order to avoid hypovolaemia. Use of warmed, balanced electrolyte solutions is pre- 
ferred. Hypocoagulability in the postoperative period is usually related to incom- 
plete replacement of clotting factors and hypothermia. [Q7] In addition, 
supracoeliac aortic clamping has been shown to result in a state of fibrinolysis that 
may exacerbate bleeding [17]. The aneurysm itself can be responsible for chronic 
coagulation factor consumption and a subsequent increased tendency to periopera- 
tive hypocoagulability [18]. Ongoing bleeding after TAA repair may require reoper- 
ation, and results in an increase in major morbidity and mortality. It is important to 
ensure that the PTT and partial thromboplastin times are corrected with plasma 
transfusions. Platelets should be replaced if thrombocytopoenia occurs in the face 
of ongoing bleeding. Hypothermia is a serious problem and can lead to failure of 



54 



Vascular Surgery 



Decreased spinal 
artery pressure 





Aortic cross clamping 


















Expeditious surgery to shorten clamp time 




«*- 






-► 



Increased spinal 
CSF pressure 



Maintain proximal aortic blood pressure 

DAP (shunt or bypass) 

Intercostal artery reimplantation 



CSF drainage 
Avoid venous hypertension 
Minimise reperfusion injury 



Decreased spinal 

cord perfusion 

pressure 

I 



Reduce spinal cord metabolism 

Moderate hypothermia 

Barbiturates (thiopental) 

Dextrose-free IV fluids 

Propofol 



I 



Spinal cord ischaemia 





Spinal cord ischaemia 










Membrane stabilisers (steroids) 

Free-radical scavengers (mannitol, propofol) 

Calcium channel blockers (nimodipine) 

Opiate receptor antagonists (naloxone, nalmefene) 

Excitatory neurotransmitter antagonists (propofol) 










Spinal cord reperfusion 












Postoperative CSF drainage 





I 



Neurological injury 



Fig. 5.6. Algorithm describing pathophysiology of spinal cord dysfunction following thoracoabdominal 
aneurysm surgery. Potential preventive interventions are also outlined. 

Reproduced from Morrissey NJ, Hamilton IN, Hollier LH. Thoracoabdominal aortic aneurysms. In: Moore W, 
editor. Vascular surgery: a comprehensive review. Philadelphia: WB Saunders, 2001 ; 41 7-434, with permission 
from Elsevier. 



Thoracoabdominal Aortic Aneurysm 55 

coagulation. Since hypothermia is often used intraoperatively as a spinal cord pro- 
tective measure, it may persist as a problem postoperatively. Aggressive correction 
with warm fluids, blood products and warming blankets is needed to restore nor- 
mothermia and proper function of coagulation as well as other enzymatic systems. 
Reoperation is reserved for ongoing significant bleeding following correction of 
coagulopathy and hypothermia. Reoperation for bleeding results in mortality rates 
of 25 per cent or greater in these patients [19]. 



References 

1. Bickerstaff LK, Pairolero PC, Hollier LH, Melton LJ, Van Peenen HJ, Cherry KJ, et al. Thoracic aortic 
aneurysms: a population based study. Surgery 1982;92:1103-8. 

2. Panneton JM, Hollier LH. Nondissecting thoracoabdominal aortic aneurysms: part I. Ann Vase Surg 
1995;9:503. 

3. Crawford ES, Crawford JL, San HJ, Coselli JS, Hess KR, Brooks B, et al. Thoracoabdominal aortic 
aneurysms: preoperative and intraoperative factors determining immediate and long term results of 
operations in 605 patients. J Vase Surg 1986;3:389-404. 

4. Crawford ES, DeNatale RW. Thoracoabdominal aortic aneurysm: observations regarding the natural 
course of the disease. J Vase Surg 1986;3:578-82. 

5. Cambria RA, Gloviczki P, Stanson AW, Cherry K, Bower TC, Hallet JW, Pairolero PC. Outcome and 
expansion rate of 57 thoracoabdominal aortic aneurysms managed nonoperatively. Am J Surg 
1995;170:213-17. 

6. Panneton JM, Hollier LH. Dissecting descending thoracic and thoracoabdominal aortic aneurysms: 
part II. Ann Vase Surg 1995; 9:596-605. 

7. Hollier LH. Technical modifications in the repair of thoracoabdominal aortic aneurysms. In: 
Greenlagh RM, editor. Vascular surgical techniques. London: WB Saunders, 1989; 144-51. 

8. Paterson IS, Klausner JM, Goldman G, Pugatch R, Feingold H, Allen P, et al. Pulmonary edema after 
aneurysm surgery is modified by mannitol. Ann Surg 1989;210:796-801. 

9. Hug HR, Taber RE. Bypass flow requirements during thoracic aneurysmectomy with particular 
attention to the prevention of left heart failure. J Thorac Cardiovasc Surg 1969;57:203-13. 

10. Kazui T, Komatsu S, Yokoyama H. Surgical treatment of aneurysms of the thoracic aorta with the aid 
of partial cardiopulmonary bypass: an analysis of 95 patients. Ann Thorac Surg 1987;43:622-7. 

11. Safi HJ, Hess KR, Randel M, Iliopoulos DC, Baldwin JC, Mootha RK, et al. Cerebrospinal fluid 
drainage and distal aortic perfusion: reducing neurologic complications in repair of thoracoabdom- 
inal aortic aneurysm types I and II. J Vase Surg 1996;23:223-9. 

12. Safi HJ, Miller CC 3rd, Carr C, Iliopoulos DC, Dorsay DA, Baldwin JC. Importance of intercostal 
artery reattachment during thoracoabdominal aortic aneurysm repair. J Vase Surg 1998;27:58-68. 

13. Cambria RP, Davison JK, Zannetti S, LTtalien G, Brewster DC, Gertler JP, et al. Clinical experience 
with epidural cooling for spinal cord protection during thoracic and thoracoabdominal aneurysm 
repair. J Vase Surg 1997;25:234-43. 

14. Webb TH, Williams GM. Thoracoabdominal aneurysm repair. Cardiovasc Surg 1999;7:573-85. 

15. Wisselink W, Money SR, Crockett DE, Nguyen JH, Becker MO, Farr GH, Hollier LH. Ischemia-reper- 
fusion of the spinal cord: protective effect of the hydroxyl radical scavenger dimethylthiourea. J Vase 
Surg 1994;20:444-50. 

16. Hollier LH, Money SR, Naslund TC, Proctor CD Sr, Buhrman WC, Marino RJ, et al. Risk of spinal 
cord dysfunction in patients undergoing thoracoabdominal aortic replacement. Am J Surg 
1992;164:210-14. 

17. Gertler JP, Cambria RP, Brewster DC, Davison JK, Purcell P, Zannetti S, et al. Coagulation changes 
during thoracoabdominal aneurysm repair. J Vase Surg 1996;24:936-45. 

18. Fisher DF, Yawn DH, Crawford ES. Preoperative disseminated intravascular coagulation caused by 
abdominal aortic aneurysm. J Vase Surg 1986;4:184-6. 

19. Svensson LG, Crawford ES, Hess KR, Coselli JS, Safi HJ. Experience with 1509 patients undergoing 
thoracoabdominal aortic operations. J Vase Surg 1993;17:357-70. 



6. Aortic Dissection 



Barbara T. Weiss-Miiller and Wilhelm Sandmann 



Dissection: Stanford A 

A 68-year-old woman spontaneously and suddenly developed severe retrosternal 
pain during her holiday in Turkey. Without knowing the diagnosis, she flew 
home 2 days later. Computed tomography (CT) scans taken immediately after 
arrival revealed a dissection of the ascending aorta, the aortic bow and the 
descending aorta. 



Question 1 

How would you classify the aortic dissection? 

A. Stanford A dissection. 

B. Stanford B dissection. 

C. de Bakey I dissection. 

D. de Bakey II dissection. 

E. de Bakey III dissection 

On the same day, she underwent an emergency operation. The dissected ascend- 
ing aorta with the entry of dissection was incised in a cardiopulmonary bypass and 
replaced by a graft using the in-graft technique. The aortic valve was patent and 
remained in situ. For reconstruction of the aortic root, the sandwich technique was 
used. Two Teflon strips were placed externally and into the true lumen to reattach 
the dissected membrane to the aortic wall. The aortic graft was then sutured into the 
reconstructed aortic root. 



57 



58 Vascular Surgery 

Question 2 

Which of the following statements are wrong? 

A. Stanford A dissections should be treated medically. 

B. Stanford A dissections should undergo operation immediately. 

C. Stanford B dissections without ischaemic complications should be treated med- 
ically. 

D. Stanford B dissections require operative intervention immediately. 

E. Stanford A dissections require an aortic stent graft immediately. 

The postoperative course was uneventful at the beginning. However, 3 days later, 
renal function deteriorated and the patient required haemofiTtration. Moreover, the 
patient developed severe hypertension and had to be treated with three different 
antihypertensive drugs. Contrast CT scans revealed that the right kidney was 
without function due to an old hydronephrosis, while the left renal artery was prob- 
ably dissected. Furthermore, the patient developed left leg ischaemia and was trans- 
ferred to our centre. We explored the abdomen via the transperitoneal approach. 
The pulsation of the left iliac artery was weak due to aortic and left iliac dissection. 
Infrarenal aorto-iliac membrane resection was performed to restore the blood flow 
to the extremities. Then the left renal artery was explored; the renal artery dissec- 
tion was found to extend towards the hilus of the kidney. 

Revascularisation was achieved with a saphenous vein interposition graft placed 
between the left iliac artery and the distal left renal artery (Fig. 6.1). 

Question 3 

Which of the following statements are correct? 

A. Complications of Stanford A dissection are aortic valve insufficiency and perfo- 
ration into the pericardium. 

B. Stroke is a typical complication of Stanford B dissection. 

C. Paraplegia is a typical complication of aortic dissection. 

D. Most patients with Stanford B dissections die of aortic perforation. 

E. Typical complications of aortic dissection are organ and lower-extremity 
ischaemia. 

The postoperative course was uneventful. The patient recovered promptly from 
the operative intervention, while renal function and blood pressure improved sub- 
stantially. Urine production and laboratory findings became normal, and only one 
antihypertensive drug (a beta-blocker) was necessary to maintain normal blood 
pressure. The postoperative angiography showed a patent iliac-renal interposition 
graft and normal perfusion of the left kidney (Fig. 6.1). CT scans taken 2 years later 
displayed a hypertrophic, well-functioning left kidney, while the right kidney was 
small and hydronephrotic (Fig. 6.2). 



Aortic Dissection 



59 




Fig. 6.1 . a Left common iliac artery, b Left renal artery saphenous vein bypass. 



Dissection: Stanford B 



A 54-year-old woman was admitted to another hospital with the provisional 
diagnosis of a myocardial infarction (MI). She experienced a sudden chest pain. 
Some hours later, she developed paraesthesia in both legs, which improved spon- 
taneously. Subsequently, she felt abdominal discomfort and developed diarrhoea 
and vomiting. The patient had been normotensive throughout her life, but now 
she required five different antihypertensive drugs to stabilise blood pressure. 
Some laboratory data were abnormal, including leucocytes, transaminases, lactic 
dehydrogenase and lactate. Duplex sonography and transoesophageal echocar- 
diography revealed an aortic dissection of the thoracic and abdominal aorta 
beginning distal to the left subclavian artery; blood flow into the visceral arteries 
and the right renal artery was reduced. Contrast CT scans confirmed Stanford B 
aortic dissection. 



60 



Vascular Surgery 




Fig. 6.2. CT scans taken 18 months after operative intervention show a well-functioning, hypertrophic left 
kidney and a small, hydronephrotic right kidney. Note the dissected but non-dilated aorta. 



Question 4 

What diagnostic methods are involved in acute aortic dissection? 

A. Computed tomography. 

B. Magnet resonance imaging. 

C. Angiography. 

D. Transoesophageal echocardiography. 

The patient was first treated medically with parenteral therapy and antihyperten- 
sive drugs (including beta-blockers). Under this management, clinical outcome and 
laboratory findings improved, but 3 weeks later the patient deteriorated again and 
developed severe right upper abdominal pain. 

She was referred to our hospital for operation. CT scans displayed the aortic dis- 
section and a dissected superior mesenteric artery. The true aortic lumen was very 
small and partially thrombosed (Fig. 6.3). Abdominal exploration via the transperi- 
toneal approach revealed borderline ischaemia of all intra-abdominal organs due to 
aortic dissection. The dissection had affected the coeliac trunk, the superior mesen- 
teric artery and the right renal artery. The right upper abdominal pain was caused 
by an ischaemic cholecystitis. The gallbladder had to be removed. The para-aortic 
tissue displayed severe inflammation; therefore no fenestration and membrane 



Aortic Dissection 



61 




Fig. 6.3. Aortic dissection, with a small, partially thrombosed "true" aortic lumen and dissected superior mesen- 
teric artery. 



resection could be carried out. Instead, intestinal and renal blood flow was restored 
by a 12-mm Dacron graft, which was placed end to side into the left iliac artery and 
end to end to the coeliac trunk. The superior mesenteric artery was implanted 
directly into the Dacron graft, while the right renal artery was attached by means of 
a saphenous vein interposition graft (Fig. 6.4). 

Question 5 

What techniques are used to restore blood flow to the visceral organs and extremi- 
ties following ischaemia from aortic dissection? Which of the following statements 
are wrong? 

A. Aortic stent graft. 




Fig. 6.4. Extra-anatomical reconstruction with a Dacron graft, which was placed end to side between the left 
common iliac artery and end to end to the coeliac trunk. The superior mesenteric artery was implanted directly 
into the graft, while the right renal artery was implanted via the interposition of a saphenous vein. The left renal 
artery originates from the aorta. 



62 Vascular Surgery 

B. PTA of organ and limb arteries and stenting. 

C. Aortic fenestration and membrane resection. 

D. Cardiopulmonary bypass. 

E. Extra-anatomic revascularisation, e.g. axillo-femoral bypass. 

The complication of postoperative retroperitoneal bleeding from the recon- 
structed right renal artery had to be managed by relaparotomy and single vascular 
stitches, and clinical recovery was delayed. The patient required 4 months of reha- 
bilitation until she had regained her previous health status. At this point, digestion 
and renal function had recovered, laboratory findings became normal, and hyper- 
tension had to be treated with only one drug (beta-blocker). Postoperative 
angiographies showed good perfusion of all visceral and renal arteries via the 
Dacron graft (Fig. 6.4). 



Commentary 

The life-threatening aortic dissection starts with an intimal tear (entry) in the 
ascending aorta (Stanford A, de Bakey I or II) or distally to the left subclavian artery 
(Stanford B, de Bakey III). De Bakey II dissection affects the ascending aorta only, 
while de Bakey I and III dissections also involve the descending aorta [1, 2]. [Q1: A, 
C] Most patients with acute aortic dissection present with severe chest pain, which 
might be misinterpreted as acute MI [3, 4]. 

Echocardiography, particularly by the transoesophageal approach, is a reliable 
and rapid method for diagnosis of aortic dissection and differentiation into 
Stanford A or B type [5]. Nevertheless, the evaluation of organ arteries and their 
blood flow by ultrasound maybe difficult in acute dissection. In our opinion, con- 
trast thoracic and abdominal CT scans, especially using the spiral technique, are 
appropriate diagnostic methods for determining the extension of dissection and the 
relation of its dissecting membrane to major branches of the aorta. The perfusion of 
abdominal organs, and often of their arteries, can be seen easily. In the case of 
organ malperfusion, angiography may be helpful to determine whether the 
ischaemia is caused by the dissecting membrane of the aorta or whether the dissec- 
tion extends into the organ arteries [6]. Magnetic resonance imaging (MRI) or mag- 
netic resonance angiography (MRA) are effective alternatives in the diagnosis of 
patients with dissection and renal failure [7]. [Q4: A-D] 

Without treatment, the prognosis of acute aortic dissection is very poor. In 1958, 
Hirst et al. reviewed 505 cases of aortic dissection and found that 21 per cent of 
patients died within 24 h of onset and only 20 per cent survived the first month [3]. 
Causes of death in patients with Stanford A dissection include intrapericardial and 
free intrapleural rupture, acute aortic valve insufficiency, and, to a minor extent, 
cerebral and coronary malperfusion. In patients with type B dissections, free 
rupture of the aorta is less frequent. Dissection of the descending aorta may lead, in 
about 30 per cent of cases, to obstruction of visceral, renal and extremity arteries, 
resulting in visceral ischaemia, renal insufficiency and acute limb ischaemia, which 
may be lethal without prompt and adequate therapy [8-10]. [Q3: A, C, E] 

To improve the natural course of the disease, in 1955 de Bakey et al. started to 
treat acute aortic dissections surgically. Within only a few years, they had developed 



Aortic Dissection 63 

the current principles of operative intervention in acute Stanford A dissection with 
replacement of the ascending aorta by a graft in cardiopulmonary arrest. Their 
results were outstanding, with an overall mortality of 21 per cent [1, 11]. 

However, the surgical experiences of other workgroups were not so successful. 
Therefore, Wheat et al. developed a new medical treatment with ganglionic block- 
ers, sodium nitroprusside or beta-blockers to influence the hydrodynamic forces of 
the bloodstream based on the theory that blood pressure and the steepness of the 
pulse wave are propagating the dissecting haematoma [12]. In 1979, a meta-analysis 
of 219 patients with acute aortic dissection from six centres revealed that Stanford A 
patients treated medically had a mortality of 74 per cent, whereas 70 per cent of 
patients survived after surgical therapy. On the other hand, in patients with acute 
type B dissection, drug therapy alone had a survival rate of 80 per cent, whereas 50 
per cent died after operative intervention [13]. Therefore in most centres, current 
therapy for acute dissection type Stanford A is surgical [14-17], and for uncompli- 
cated Stanford B dissection it is medical [18-22]. [Q2: A, D, E] 

An acute dissection, involving the ascending aorta, should be considered a surgi- 
cal emergency. The aim of operative intervention is to prevent or treat dilation or 
rupture of the aortic root, and to maintain aortic valve function. The following 
reconstructive approach is recommended: in patients in whom the root is not 
involved by dissection, a tubular graft is anastomosed to the sinotubular ridge. In 
the presence of commissural detachment, the valve is resuspended before supra- 
commissural graft insertion. If the aortic valve is affected by congenital or acquired 
abnormalities, then it is generally replaced [15]. 

Patients with acute uncomplicated Stanford B dissection should be treated med- 
ically. Careful monitoring is obligatory, while antihypertensive drugs, such as beta- 
blockers [23], and analgesics are administered. The aim of treatment is to stabilise 
the dissected aortic wall within 2 weeks and to prevent further extension of dissec- 
tion or perforation. Careful clinical and laboratory examinations are necessary to 
detect symptoms of organ or extremity malperfusion in time. Limb, renal and vis- 
ceral ischaemia can be observed frequently, but paraplegia due to malperfusion of 
intercostal arteries is rare [6, 8-10]. 

If peripheral vascular complications occur, several therapeutic strategies are pos- 
sible. Newer publications describe endovascular procedures, for example emer- 
gency aortic stenting to close the "entry" and the false aortic lumen [24-27]. 
Ultrasound-guided endovascular catheter aortic membrane fenestration was per- 
formed to restore the blood flow to the aortic branches. Dilation and stenting of dis- 
sected organ or iliac arteries were performed to resolve stenosis and restore blood 
flow [28-30]. These new therapeutic methods need to be evaluated in long-term 
follow-up. 

Aortic surgery in the acute stage of aortic dissection is a dangerous procedure. 
The dissected aortic wall is extremely friable and does not hold sutures well. 
Therefore we, and many other centres, try to leave the aorta itself untouched and to 
restore organ or extremity blood flow by extra-anatomical bypass procedures. 
Extra-anatomical revascularisation also becomes necessary if the aortic branches 
themselves are dissected [6, 8, 15]. Normally, we use one common iliac artery as the 
donor vessel for extra-anatomical bypass grafting, but the distal lumbar aorta might 
also be suitable. If only one aortic branch requires revascularisation, then the iliac- 
visceral bypass is performed with the saphenous vein (Fig. 6.1). If two or more 
branches are affected, then a Dacron graft is used and the visceral arteries can 
be implanted into the graft directly or via interposition of the saphenous vein 



64 Vascular Surgery 

(Fig. 6.4). Blood flow to the legs can be restored with a femoral-femoral crossover 
bypass or with an axillo-(bi)-femoral graft. If several organ arteries are occluded by 
the aortic dissecting membrane, and the visceral arteries are undissected, then 
abdominal aortic fenestration and membrane resection combined with thrombec- 
tomy of the organ arteries can also be performed [31-34]. We prefer the latter to 
treat paraplegia caused by acute aortic dissection. [Q5: D] 

Our only indication for total aortic replacement in the acute stage of dissection is 
aortic penetration or perforation. 



References 

1. De Bakey ME, Henly WS, Cooley DA, et al. Surgical management of dissecting aneurysm of the aorta. 
J Thorac Cardiovasc Surg 1965;49:130. 

2. Daily PO, Trueblood HW, Stinson EB, Wuerflein RD, Shumway NE. Management of acute aortic dis- 
sections. Ann Thorac Surg 1970;10:237-47. 

3. Hirst AE, Johns VJ, Kime SW. Dissecting aneurysm of the aorta: a review of 505 cases. Medicine 
1958;37:217. 

4. De Bakey ME, McCollum CH, Crawford ES, et al. Dissection and dissecting aneurysms of the 
aorta: twenty year follow up of five hundred twenty seven patients treated surgically. Surgery 
1982;92:1118-34. 

5. Nienaber CA, von Kodolitsch Y, Nicolas V, et al. The diagnosis of thoracic aortic dissection by non- 
invasive imaging procedures. N Engl J Med 1993;328:1-9. 

6. Miiller BT, Grabitz K, Fiirst G, Sandmann W. Die akute Aortendissektion: Diagnostik und Therapie 
von ischamischen Komplikationen. Chirurg 2000;7 1:209. 

7. Nienaber CA, von Kodolitsch Y. Bildgebende Diagnostik der Aortenerkrankungen. Radiologic 
1997;37:402. 

8. Cambria RP, Brewster DC, Gertler J, et al. Vascular complications associated with spontaneous aortic 
dissection. J Vase Surg 1988;7:199-209. 

9. Da Gama AD. The surgical management of aortic dissection: from university to diversity, a continu- 
ous challenge. J Cardiovasc Surg 1991;32:141. 

10. Fann JI, Sarris GE, Mitchell RS, et al. Treatment of patients with aortic dissection presenting with 
peripheral vascular complications. Ann Surg 1990;212:705-13. 

11. De Bakey ME, Cooley DA, Creech O. Surgical considerations of dissecting aneurysm of the aorta. 
Ann Surg 1955;142:586. 

12. Wheat MW, Palmer RF, Bartley TD, Seelmann RC. Treatment of dissecting aneurysm of the aorta 
without surgery. J Thorac Cardiovasc Surg 1965;49:364. 

13. Wheat MW, Wheat MD, Jr. Acute dissecting aneurysms of the aorta: diagnosis and treatment - 1979. 
AmHeartJ1980;99:373. 

14. Borst HG, Laas J, Frank G, Haverich A. Surgical decision making in acute aortic dissection type A. 
Thorac Cardiovasc Surg 1987;35:134. 

15. Borst HG, Heinemann MK, Stone CD. Indications for surgery. In: Surgical treatment of aortic dissec- 
tion. New York: Churchill Livingstone, 1996;103. 

16. Heinemann M, Borst HG. Kardiovaskulare Erkrankungen des Marfan Syndroms. Dt arztebl 
1996;93B:934. 

17. Vecht RJ, Bestermann EMM, Bromley LL, Eastcott HHG. Acute dissection of the aorta: long term 
review and management. Lancet 1980;i:109. 

18. Bavaria JE, Brinster DR, Gorman RC, et al. Advances in the treatment of acute type A dissection: an 
integrated approach. Ann Thorac Surg 2002;74: SI 848. 

19. Vecht RJ, Bestermann EMM, Bromley LL, Eastcott HHG. Acute aortic dissection: historical perspec- 
tive and current management. Am Heart J 1981;102:1087. 

20. Fradet G, Jamieson WR, Janusz MT, et al. Aortic dissection: a six year experience with 17 patients. 
Am J Surg 1988;155:697-700. 

21. Glower DD, Speier RH, White WD. Management and long-term outcome of aortic dissection. Ann J 
Surg 1990;214:31. 

22. Hashimoto A, Kimata S, Hosada S. Acute aortic dissection: a comparison between the result of 
medical and surgical treatments. Jpn Circ J 1991;55:821. 



Aortic Dissection 65 

23. Shores J, Berger KR, Murphy EA, Pyeritz R. Progression of aortic dilatation and the benefit of long- 
term beta-adrenergic blockade in Marfan's syndrome. N Engl J Med 1994;330:1335. 

24. Nienaber CA, Fattori R, Lund G, et al. Nonsurgical reconstruction of thoracic aortic dissection by 
stent-graft replacement. N Engl J Med 1999;340:1539-45. 

25. Dake MD, Kato N, Mitchell RS, et al. Endovascular stent-graft replacement for treatment of acute 
aortic dissection. N Engl J Med 1999;340:1546. 

26. Leurs LJ, Bell R, Drieck Y, et al. Endovascular treatment of thoracic aortic diseases: combined 
experience from EUROSTAR and United Kingdom thoracic Endograft registries. J Vase Surg 
2004;40:670-80. 

27. Hansen CJ, Bui H, Donayre CE. Complications of endovascular repair of high-risk and emergent 
descending thoracic aortic aneurysms and dissections. J Vase Surg 2004;40:228-34. 

28. Chavan A, Hausmann D, Dresler C, et al. Intravasal ultrasound guided percutaneous fenestration of 
the intimal flap in the dissected aorta. Circulation 1997;96:2124-7. 

29. Farber A, Gmelin E, Heinemann M. Transfemorale Fensterung und Stentimplantation bei aorto-ili- 
akaler Dissektion. Vasa 1995;24:389. 

30. Slonim SM, Nyman U, Semba CP, Miller DC, Mitchell RS, Dake MD. Aortic dissection: percutaneous 
management of ischemic complications with endovascular stents and balloon fenestration. J Vase 
Surg 1996;23:241-51. 

31. Gurin D, Bulmer JW, Derby R. Dissecting aneurysm of the aorta: diagnosis of operative relief of 
acute arterial obstruction due to this cause. N Y State J Med 1935;35:1200. 

32. Elefteriades JA, Hammond GL, Gusberg RJ, Kopf GS, Baldwin JC. Fenestration revisited. A safe and 
effective procedure of descending aortic dissection. Arch Surg 1990;125:786-90. 

33. Harms J, Hess U, Cavallaro A, Naundorf M, Maurer PC. The abdominal aortic fenestration proce- 
dure in acute thoraco-abdominal aortic dissection with aortic branch artery ischemia. J Cardiovasc 
Surg (Torino) 1998;39:273-80. 

34. Webb TH, Williams GM. Abdominal aortic tailoring for renal, visceral and lower extremity mal- 
perfusion resulting from acute aortic dissection. J Vase Surg 1997;26:474. 



7. Popliteal Artery Aneurysm 

Jonathan D. Woody and Michel S. Makaroun 



A 77-year-old male presented to the emergency department with a 2-day history 
of increased swelling and pain in the left knee and distal thigh. He was unable to 
extend the knee without severe pain. On examination, the patient had a large 
pulsatile mass in the left popliteal fossa and distal thigh. 



Question 1 

Which of the following is the initial diagnostic test of choice for popliteal artery 
aneurysm? 

A. Magnetic resonance imaging (MRI). 

B. Arteriography. 

C. Duplex ultrasonography. 

An arteriogram was obtained, which revealed bilateral popliteal artery 
aneurysms, right common iliac artery aneurysm, and multiple aneurysms of the left 
internal iliac artery (Figs 7.1 and 7.2). 

Question 2 

Which of the following statements about popliteal artery aneurysm is false? 

A. It is bilateral in approximately 60 percent of cases. 

B. Approximately 80 percent of patients with bilateral popliteal artery aneurysm 
have another aneurysm elsewhere. 

C. Duplex ultrasonography is the initial diagnostic test of choice. 

D. Rupture of popliteal artery aneurysm occurs frequently. 

67 



68 



Vascular Surgery 




Fig. 7.1. Arteriogram demonstrating aneurysms of the right common iliac artery and left internal iliac 
artery. 



Question 3 

Which of the following are indications for operative treatment of popliteal artery 
aneurysm? 

A. Size greater than 2 cm. 

B. Swelling or pain in the affected leg. 

C. Aneurysm of any size with mural thrombus. 

D. Distal embolization. 



Popliteal Artery Aneurysm 



69 




Fig. 7.2. Arteriogram demonstrating bilateral popliteal artery aneurysms. 



Treatment 



The patient underwent operative exploration of the left popliteal fossa through a 
medial approach. After proximal and distal control was obtained, the aneurysm was 
opened and all collateral flow into the aneurysm was obliterated. An interposition 
vein graft was placed from the distal superficial femoral artery to the distal popliteal 
artery. The patient subsequently underwent surgical treatment of the right popliteal 
artery aneurysm and endovascular treatment of the right common iliac artery 
aneurysm. 



70 Vascular Surgery 

Question 4 

Which of the following statements regarding the treatment of popliteal artery 
aneurysm is false? 

A. Bypass and exclusion of the aneurysm is safe and effective. 

B. Resection and endoaneurysmorrhaphy is safe and effective. 

C. The medial approach is the preferred route of exposure. 

D. Endoluminal stent grafting may be indicated in a high-risk patient. 

E. For acute thrombosis, surgical thrombectomy has a higher limb salvage rate 
than thrombolysis followed by surgical reconstruction. 

Question 5 

All of the following are true regarding the treatment of acute thrombosis of a 
popliteal artery aneurysm except: 

A. Limb loss occurs in up to 50 percent of cases. 

B. Thrombolysis improves the run-off and increases limb salvage. 

C. Thrombolysis alone is adequate treatment. 

D. Surgical thrombectomy is inferior to thrombolysis followed by surgical 
reconstruction. 



Commentary 

Popliteal artery aneurysm is the most common type of peripheral aneurysm; the 
vast majority are caused by atherosclerosis. The typical patient is a male in his sixth 
or seventh decade. Popliteal artery aneurysm occurs bilaterally in approximately 60 
percent of cases [1]. Approximately 80 percent of patients with bilateral popliteal 
artery aneurysm have other aneurysms elsewhere, and 1 percent of patients with 
abdominal aortic aneurysm (AAA) have popliteal artery aneurysm [2]. 

Diagnosis of popliteal artery aneurysm is relatively straightforward. 
Examination reveals a prominent pulsation or a pulsatile mass in the popliteal 
fossa. Duplex ultrasonography is the initial diagnostic test of choice because it 
can differentiate an aneurysm from other masses in the popliteal fossa and it 
can determine whether thrombus is present in the aneurysm. [Q1: C] 
Angiography is not sensitive for the documentation of an aneurysm, but it is 
crucial for evaluating the status of the distal arterial system. If the diagnosis can 
be made on physical examination, then one can proceed directly to angiography 
for operative planning. 

The majority of patients with popliteal artery aneurysm are symptomatic at the 
time of diagnosis. Distal embolization is the most common form of presentation, 
followed by pain and/or swelling of the affected extremity due to compression of the 



Popliteal Artery Aneurysm 71 

adjacent nerves and veins. Rupture is rare and usually not life threatening. [Q2: D] 
However, rupture can lead to limb loss in 50-75% of cases due to associated limb 
ischemia [3]. 

Indications for operation include size of 2 cm or greater, aneurysm of any size 
with mural thrombus, any patient with pain and/or swelling due to compression of 
adjacent structures, and any symptoms of embolization. While some surgeons 
prefer to delay operation in asymptomatic patients until the aneurysm reaches 3 cm 
in size, we believe that an aggressive approach is justified because the majority of 
patients ultimately become symptomatic. Some authors advocate an aggressive 
approach to any small, asymptomatic popliteal artery aneurysm containing mural 
thrombus due to the high frequency of embolization in these cases [4]. [Q3: A, B, C, D] 
It is important to note that thromboembolization is the most common form of pre- 
sentation. Accepted forms of surgical treatment include bypass with complete 
exclusion of the aneurysm, bypass with partial or complete resection of the 
aneurysm if compressive symptoms are present, or endoaneurysmorrhaphy like the 
open repair of AAA. The medial approach is preferred over the posterior approach 
because it allows for better exposure of the distal superficial femoral artery and 
tibioperoneal trunk, easier access to the saphenous vein, and the ability to perform 
distal bypass if it becomes necessary. Autogenous vein is the preferred conduit since 
it provides better patency than prosthetic graft. Endoluminal stent grafting has not 
been proven, and theoretically it would seem to be inferior to surgical repair due to 
the small size of the stent graft required and the lower patency rates of prosthetic 
graft in this position. Nevertheless, it may be an option in a patient who is at high 
risk for a surgical procedure. 

Distal embolization can cause tissue loss, but the more troubling consequence is 
the destruction of the run-off bed. This results in decreased patency of bypass grafts 
and an increased rate of limb loss. Further, there is a significant reduction in limb 
salvage and graft patency in symptomatic patients compared with asymptomatic 
patients. Numerous studies indicate that virtually all patients will become sympto- 
matic and that graft patency and limb salvage approach 100 percent in elective 
reconstruction [1-6]. This demonstrates the increased risk of complications and 
limb loss in untreated patients and underscores the importance of early treatment. 

Acute thrombosis is a particularly vexing problem. While some may present with 
only claudication due to the previous development of collateral flow, many present 
with threatened limb loss. In the setting of acute thrombosis, the combined rate of 
primary and early amputation is 50 percent and operative mortality is 5 percent [7]. 
Thrombolysis followed by surgical treatment yields better graft patency and limb 
salvage than surgical thrombectomy. [Q4: E] Most likely, this is due to some degree 
of clearing of the run-off bed. Thrombolysis alone is inadequate treatment since 
surgical reconstruction is required to obliterate the aneurysm. [Q5: C] 



References 



Carpenter JP, Barker CF, Roberts B, Berkowitz HD, Lusk EJ, Perloff LJ. Popliteal artery aneurysms: 
current management and outcome. J Vase Surg 1994;19:65-72. 

Whitehouse WM, Jr, Wake?eld TW, Graham LM, Kazmers A, Zelenock GB, Cronenwett JL, et al. Limb- 
threatening potential of arteriosclerotic popliteal artery aneurysms. Surgery 1983;93:694-9. 
Varga ZA, Locke-Edmunds JC, Baird RN. A multicenter study of popliteal aneurysms. J Vase Surg 
1994;20:171-7. 



72 Vascular Surgery 

4. Ascher E, Markevich N, Schutzer RW, Kallakuri S, Jacob T, Hingorani AP. Small popliteal artery 
aneurysms: are they clinically significant? J Vase Surg 2003;37:755-60. 

5. Lowell RC, Gloviczki P, Hallett JW, Jr, Naessens JM, Maus TP, Cherry KJ, Jr, et al. Popliteal artery 
aneurysms: the risks of nonoperative management. Ann Vase Surg 1994;8:14-23. 

6. Anton GE, Hertzer NR, Beven EG, O'Hara PJ, Krajewski LP. Surgical management of popliteal 
aneurysms: trends in presentation, treatment and results from 1952 to 1984. J Vase Surg 
1986;3:125-34. 

7. Reilly MK, Abbott WM, Darling RC. Aggressive surgical management of popliteal aneurysms. Am J 
Surg 1983;145:498-502. 



8. Renal Artery Aneurysm 



Lutz Reiher, Tomas Pfeiffer and Wilhelm Sandmann 



A 45-year-old woman presented with a 10-year history of arterial hypertension. 
After initially successful conservative therapy with two antihypertensive drugs, 
arterial blood pressure was not controlled well during the last months. To 
exclude a renovascular origin of hypertension, an angiography was performed, 
which showed fibrodysplastic disease of the right renal artery with several 
stenotic segments and aneurysms (Fig. 8.1). 



Question 1 

Which of the following statements regarding renal artery aneurysm (RAA) is 
correct? 

A. It has a marked female preponderance. 

B. It is usually diagnosed during examination for flank pain. 

C. It may cause arterial hypertension. 

D. It typically leads to proteinuria by compression of the renal vein. 

E. It can cause haematuria in rare cases. 



Question 2 

Which statements about the aetiology of the RAA are true ? 

A. The most frequent underlying diseases of RAA are aortic coarctation with con- 
comitant disease of the renal artery and renal artery dissection. 

B. Fibromuscular dysplasia of the renal artery may present with renal artery steno- 
sis (RAS), RAA or both. 

C. Arteriosclerosis is a frequent cause of RAA. 

73 



74 



II 



RECHTS 



Vascular Surgery 

B-l'IHR-30 
1:59=36 



P, 




Oil 



I : : 



Fig. 8.1 . Selective intra-arterial renal artery angiography revealed RAA combined with renal artery stenosis due 
to fibromuscular dysplasia. £3 



D. Some RAA present with inflammation of the arterial wall. 

E. The incidence of RAAs is increased in Ehlers-Danlos syndrome and Marfan's 
syndrome 



Question 3 

Which risks of the spontaneous course of the RAA should you explain to your 
patient? 

A. The RAA may rupture and lead to a life-threatening bleeding. 

B. The risk of rupture decreases during pregnancy and childbirth. 

C. Hypertension in RAA may be caused by concomitant stenosis of the renal artery 
or its branches. 

D. In cases of RAA and hypertension the angiography of the renal artery always 
shows an additional RAS. 



Renal Artery Aneurysm 75 

E. The RAA may be a source of embolisation leading to a loss of renal function. 

Question 4 

Which of the following statements regarding the indication of renal artery repair 
(RAR) for RAA is correct? 

A. There is an indication for RAR only in cases of symptoms other than 
hypertension. 

B. There is no reason to perform RAR in women of childbearing age if there is no 
arterial hypertension. 

C. There is a good indication for RAR if a concomitant RAS is found. 

D. There is a good indication for RAR only if the RAA is larger than 5.5 cm. 

E. There is an indication for RAR in patients presenting with RAA and hyperten- 
sion even if an additional RAS is not detectable. 

For RAR, a midline abdominal incision was performed for direct access to the 
infrarenal aorta, where an end-to-side anastomosis was performed with a segment 
of the patient's greater saphenous vein. After Kocher's manoeuvre, the distal renal 
artery was transected and anastomosed to the saphenous vein, which had been 
placed on the renal hilus dorsal to the inferior vena cava. Good results were shown 
by postoperative angiography (Fig. 8.2). At re-examination 3 years after the opera- 
tion, the patient had a normal blood pressure without antihypertensive medication. 

Question 5 

Which of the following statements regarding the management of RAA is correct? 

A. Replacement of the diseased renal artery by prosthetic graft is the RAR of first 
choice. 

B. Protection of the kidney against ischaemic injury is performed only during ex 
situ reconstruction of the renal artery. 

C. RAA exclusion and aortorenal vein graft interposition, or RAA resection and 
end-to-end anastomosis or aneurysmorrhaphy, are valuable methods for RAR. 

D. Ex situ repair of the renal artery may be needed in cases presenting with lesions 
of the distal branch arteries. 

E. Tailoring of RAA often leads to recurrent aneurysmatic dilation of the renal 
artery. 



Commentary 

RAAs do not usually cause symptoms, and generally they are diagnosed accidentally 
during work-up for hypertension, as in our patient. In rare cases, flank pain has 



76 



Vascular Surgery 




Fig. 8.2. Postoperative angiography demonstrates a patent aortorenal venous graft. 



been described as the initial symptom, which may be due either to the size of the 
RAA or to a renal artery dissection. Rupture of the aneurysm into the urinary tract 
will lead to haematuria. [Q1: A, C, E] The underlying disease is most frequently dys- 
plasia of the arterial wall followed by arteriosclerosis. In our case, fibromuscular 
dysplasia was found to be the aetiology of the RAA. Rare causes of RAA may be 
atypical aortic coarctation with concomitant disease of the renal arteries, inflamma- 
tion of the arterial wall, dissection or trauma, or disorders of the elastic and colla- 
gen fibres (i.e. Ehlers-Danlos syndrome or Marfan's syndrome). [Q2: B, C, D, E] 

RAA is found about twice as often in the right renal artery as in the left. Selective 
angiography often reveals concomitant RAS of mainstem and segmental arteries, 
and segmental arteries may also be aneurysmal. Concomitant renal artery dissec- 
tion is rare. 

Rupture of RAA, development or deterioration of arterial hypertension, and loss 
of renal function by thrombosis or embolisation, are impending spontaneous con- 
sequences of RAA. 



Renal Artery Aneurysm 77 

As with all arterial aneurysms, rupture is a possible complication of RAA. While 
Tham et al. [1] experienced no rupture of RAA in 69 patients who had been treated 
conservatively during a mean observation time of 4.3 years, Henriksson et al. [2] 
observed RAA rupture in four cases (10.2 percent), and at the time of rupture only a 
nephrectomy could be performed. There are several case reports about RAA 
rupture in pregnancy and childbirth [3-5], and one author found the probability of 
RAA rupture during pregnancy to be as high as 80 percent [6]. 

As high arterial blood pressure is in itself a risk factor for rupture of arterial 
aneurysms of any localisation, one can argue that hypertension per se is an indica- 
tion to remove an RAA. Hypertension was found in 90 percent of all patients with 
ruptured RAA [7]. 

The larger the diameter of the RAA, the more likely the danger of rupture seems 
to be, which can be explained by Laplace's law. However, RAAs of any diameter can 
rupture. In one patient cohort [8], the smallest (1 cm) and the largest (16.5 cm) 
RAAs ruptured. 

About eighty percent of patients with RAA have arterial hypertension [9, 10]. If 
RAA is accompanied by RAS on the same or the contralateral side, as in our patient, 
then it is reasonable to remove both, with the intention to improve hypertension 
and eliminate the risk of rupture. However, an ipsilateral stenosis maybe missed by 
angiography due to overprojection of the aneurysm. Furthermore, aneurysmal 
disease includes not only dilation of vessels but also elongation, which might cause 
kinking with a relevant stenosis [11]. [Q3: A, C, E] 

There is an absolute indication to remove RAAs in all patients with arterial 
hypertension with and without concomitant RAS and in women of childbearing age. 
[Q2: C] RAAs with a diameter greater than 2 cm should be removed, even if there is 
no hypertension. There are good long-term results for autologous RAR; therefore, 
there is a relative indication for operation in younger patients without hypertension 
and concomitant RAS with RAA of diameter of 1 cm or more. [Q4: C, E] 

The most promising method of RAR is by autogenous reconstruction. Methods of 
RAR are replacement of the renal artery by the greater saphenous vein, resection of 
diseased sections and reanastomosis. The autoplastic reconstruction by tailoring 
(synonym: aneurysmorrhaphy) is another appropriate technique. Although the 
aneurysmatic wall is only resected partially, recurrent RAAs have not been 
observed. The in situ reconstruction is less traumatic, but ex situ repair of the renal 
artery may be necessary in cases in which not only the distal mainstem artery but 
also the segmental arteries are involved. [Q5: C, D] 

If arterial repair is restricted to renal arteries only, and if concomitant repair of 
the aorta is not necessary, then a postoperative mortality of less than 1 percent can 
be expected. Postoperative morbidity is due to temporary kidney insufficiency, graft 
thrombosis, bleeding, thrombosis and pancreatitis. Affected kidneys can be pre- 
served in more than 85 percent of cases. The number of patients who benefit from 
surgical therapy in terms of improvement of arterial hypertension varies consider- 
ably between authors, ranging from 5 to 50 percent and from 25 to 62 percent, 
respectively [12]. 



References 

1. Tham G, Ekelund L, Herrlin K, Lindstedt EL, Olin T, Bergentz SE. Renal artery aneurysms. Natural 
history and prognosis. Ann Surg 1983;197:348-52. 



78 Vascular Surgery 

2. Henriksson C, Lukes P, Nilson AE, Pettersson S. Angiographically discovered, non-operated renal 
artery aneurysms. Scand J Urol Nephrol 1984;18:59-62. 

3. Rijbroek A, v. Dijk HA, Roex AJM. Rupture of renal artery aneurysm during pregnancy. Eur J Vase 
Surg 1994;8:375-6. 

4. Smith JA, Macleish DG. Postpartum rupture of a renal artery aneurysm to a solitary kidney. Aust N Z 
J Surg 1985;55:299-300. 

5. Whiteley MS, Katoch R, Kennedy RH, Bidgood KA, Baird RN. Ruptured renal artery aneurysm in the 
first trimester of pregnancy. Eur J Vase Surg 1994;8:238-9. 

6. Love WK, Robinette MA, Vernon CP. Renal artery aneurysm rupture in pregnancy. J Urol 
1981;126:809-11. 

7. Abud O, Chelile GE, Sole-Balcells F. Aneurysm and arteriovenous malformation. In: Novick AC, 
Scoble J, Hamilton G, editors. Renal vascular disease. London: Saunders, 1996;35-46. 

8. Hupp T, Allenberg JR, Post K, Roeren T, Meier M, Clorius JH. Renal artery aneurysm: surgical indi- 
cations and results. Eur J Vase Surg 1992;6:477-86. 

9. Martin RSD, Meacham PW, Ditesheim JA, Mulherin JL, Jr, Edwards WH. Renal artery aneurysm: 
selective treatment for hypertension and prevention of rupture. J Vase Surg 1989;9:26-34. 

10. Brekke IB, Sodal G, Jakobsen A, Bentdal O, Pfeffer P, Albrechtsen D, et al. Fibro-muscular renal 
artery disease treated by extracorporeal vascular reconstruction and renal autotransplantation: 
short- and long-term results. Eur J Vase Surg 1992;6:471-6. 

11. Poutasse EF. Renal artery aneurysms. J Urol 1975;113:443-9. 

12. Pfeiffer T, Reiher L, Grabitz K, Griinhage B, Hafele S, Voiculescu A, et al. Reconstruction for renal 
artery aneurysm: operative techniques and long-term results. J Vase Surg 2003;37:293-300. 



9a. Anastomotic Aneurysms 



William D. Neary and Jonothan }. Earnshaw 



A 70-year-old woman presented with bilateral pulsatile groin masses (Fig. 9a. 1). 
Six years ago, she had an elective aorto-bifemoral graft for a 6-cm abdominal 
aortic aneurysm involving both iliac arteries, from which she made a full recov- 
ery. She first found the larger, right-sided mass 4 months ago, and she had noted 
gradual enlargement since then. She had no symptoms of claudication or leg 
ischaemia. Her past medical history included a myocardial infarction (MI) 18 
months ago, but without limitation to her exercise tolerance. 

On examination, the patient appeared well. There was a well-healed midline 
laparotomy scar from the previous operation. Abdominal examination was unre- 
markable, and there were no bruits on auscultation. Two well-defined expansile 
masses were palpable in the middle third of the femoral scars, measuring approx- 
imately 2 cm on the left and 4 cm on the right. The masses were not tender. There 
was no evidence of compromise in the distal circulation, and all pulses were 
palpable. Duplex imaging was performed, which identified anastomotic false 
aneurysms in both groins, measuring 1.8 cm on the left and 3.5 cm on the right. 



Question 1 

Which of the following statements regarding the aetiology of anastomotic false 
aneurysms are correct? 

A. Anastomotic false aneurysms occur in 3-5 per cent of anastomoses to the 
femoral artery in the groin. 

B. Seventy per cent are found in the groin. 

C. Primary degeneration of the arterial wall is an aetiological factor. 

D. Continued smoking is an aetiological factor. 

E. At reoperation, approximately one-third will be found to be infected with path- 
ogenic bacteria. 



79 



80 



Vascular Surgery 




Fig. 9a. 1 . Female patient with bilateral anastomotic aneurysms from an aortobifemoral graft. 



Question 2 

The patient wished to know the risks of leaving the aneurysm alone. Rank the 
potential complications of anastomotic aneurysms in order of frequency. 

A. Rupture. 

B. Embolisation. 

C. Pressure symptoms. 

D. Pain. 

E. Secondary haemorrhage. 



Question 3 

Which of the following non-operative treatments are also available? 

A. Embolisation. 

B. Ultrasound-guided compression. 

C. Thrombin injection. 

D. Intravascular stent graft. 

The larger of the two aneurysms was repaired surgically. The previous surgical 
incision was reopened and extended. A large false aneurysm was confirmed; the 
graft appeared to have become detached from the artery. There were no signs of 
infection. The aneurysm was replaced by straight 8-mm gelatine-coated woven 



Anastomotic Aneurysms 81 

Dacron interposition graft (soaked in rifampicin solution 10 mg/ml) taken end to 
end from the old graft and sutured end to side over the common femoral bifurca- 
tion. The thrombus and old graft were sent for microbiology. The patient made a 
good postoperative recovery. All bacterial cultures were negative, so perioperative 
antibiotic prophylaxis was stopped after 48 h. 

Question 4 

Rank the following surgical procedures in order of use for the management of anas- 
tomotic aneurysm in the groin: 

A. Resuture or local repair. 

B. Ligation and bypass. 

C. Prosthetic patch. 

D. Vein patch. 

E. Interposition graft. 

This patient at 2-year follow-up had no evidence of recurrence of the pseudoa- 
neurysm in her right groin. An ultrasound scanning of her left groin revealed that 
the left pseudoaneurysm remained 2 cm in maximum diameter. 

Question 6 

The following statements are true or false. 

A. Surgery cures 50 per cent of all anastomotic aneurysms. 

B. Surgery cures 90 per cent of all anastomotic aneurysms. 

C. Surgery cures 50 per cent of all recurrent anastomotic aneurysms. 

D. Surgery cures 90 per cent of all recurrent anastomotic aneurysms. 

E. Long-term follow-up of retroperitoneal anastomotic aneurysms is not 
necessary. 



Commentary 

The incidence of anastomotic aneurysms is increasing, due primarily to the 
increased frequency of prosthetic vascular reconstructions involving groin anasto- 
mosis. The overall incidence following vascular anastomoses is about 2 per cent, but 
this increases to 3-8 per cent when the anastomosis involves the femoral artery 
[1-4]. Although they are most common after prosthetic bypass, anastomotic 
aneurysms occasionally occur after vein bypass, semi-closed endarterectomy, and 
open endarterectomy with a vein patch. Anastomotic aneurysms can occur any- 
where, but they frequently develop near to a joint. About 80 per cent occur at the 
groin [1], presumably due to movement-related strains. [Q1: A,C, D, E] 



82 



Vascular Surgery 



Primary factors 



Arterial 

degeneration 

65% 




Secondary factors 

Arterial weakness 

Endarterectomy 

Poor suture technique 

Reoperative surgery 

Hyperlipidaemia 

Smoking 

Distal disease progression 

Poststenotic dilation 

Steroid therapy 

Radiotherapy 




Suture failure 

5% 




Increased forces across the anastomosis 



Hypertension 

Anastomotic tension 

Compliance mismatch 

Dacron dilation 

High outflow resistance 

Hip joint motion 

Trauma 



Fig. 9a.2. Aetiology of anastomotic aneurysms. 



The aetiology is summarised in Fig. 9a. 2; there are three primary factors and a 
number of secondary factors. One of the first documented causes was suture failure, 
when braided silk was employed [5]. Since monofilament sutures have been used, 
suture failure has become a less common factor, although occasionally reported dis- 
asters highlight the importance of careful suture handling to avoid cracking [6]. 

Arterial degeneration is the most common primary factor. The disease process 
that mandated the bypass continues after its insertion [1, 7, 8]. Histologically, a 
chronic inflammatory response can be identified at an anastomosis [9]. 

Secondary factors are numerous and compound the process of arterial degenera- 
tion [10]. Poor technique, failing to suture all layers of the artery, use of Dacron, 
and the need for endarterectomy all weaken the arterial graft complex [1]. 
Hypertension and high outflow resistance may theoretically increase strains at the 
anastomosis, together with physical disruption from both hip motion and post- 
stenotic dilation as the graft passes under the inguinal ligament [9]. These and other 
factors can cause compliance mismatch, which may also be a factor [8]. 

Anastomotic aneurysms can be caused by local infection. Infection with high-vir- 
ulence bacteria, such as Staphylococcus aureus, usually presents early with clinical 
graft infection. Late anastomotic rupture is often caused by low-virulence 
organisms, such as Staphylococcus epidermidis. Up to 30 per cent of anastomotic 
aneurysms can be shown to harbour pathogenic bacteria at reoperation [7]. This 
has implications for surgical repair (see below). [Q2: D, C, B, A, E] 



Anastomotic Aneurysms 83 

Indications for Surgical Intervention 

Surgery for anastomotic aneurysms is aimed at controlling symptoms or preventing 
the onset of complications. Symptoms of pain are associated with the enlarging 
mass or pressure on adjacent structures, such as the femoral nerve. Complications 
may be local or distal. The enlarging aneurysm may occlude the underlying vessel, 
causing distal ischaemia. Emboli associated with flow disruption maybe propagated 
distally. Aneurysm rupture represents the greatest worry but is relatively rare. 

Complications are related to aneurysm size. Therefore, conservative management 
maybe undertaken if the aneurysm is small and easily accessible, and demonstrates 
no evidence of progressive enlargement or symptoms. Aneurysms less than 2 cm in 
diameter can be observed safely [1]. Above this size, the incidence of complications 
rises and surgical intervention should be considered. However, the medical state of 
the patient may necessitate selective aneurysms larger than 2 cm being managed 
conservatively by watchful waiting. 

False aneurysms caused iatrogenically following direct arterial puncture must be 
differentiated from anastomotic aneurysms because their treatment differs substan- 
tially. False aneurysms following sterile arterial puncture maybe treated by arterial 
compression under duplex imaging [11]. More recently, injection of thrombin into 
these false aneurysms has been shown to be safe and effective, even in anticoagu- 
lated patients [12]. This technique is not suitable for anastomotic aneurysms. Other 
radiological techniques may be used selectively for false aneurysms in inaccessible 
positions, such as the renal or subclavian arteries, where coil embolisation maybe 
used to occlude the feeding vessel [13]. Again, this is rarely suitable for anastomotic 
aneurysms. Occasionally, it is possible to insert a covered stent across an anasto- 
motic aneurysm to produce local aneurysm thrombosis [14, 15] and to maintain 
normal distal flow. The ideal indication for this is intra-abdominal aortoiliac anas- 
tomotic aneurysms, where reoperation carries substantial risk. It is important that 
these non-operative techniques are not used in situations where there is any risk 
that the false aneurysm is due to infection. The most common site for anastomotic 
aneurysm is the groin, where non-operative techniques have not been found to be 
effective. The groin is also easily accessible for surgery, so direct operation is the 
usual intervention in this situation. [Q4: A, B, C, D] [Q3: A, B, C, D] 

Surgery for Anastomotic Aneurysms 

Surgical repair should be undertaken in fit patients with large or symptomatic anas- 
tomotic aneurysms. Local repair is usually possible in non-infected aneurysms, 
although graft replacement may be necessary. If infection is the cause of the 
aneurysm, then more extensive repairs with ligation and remote bypass or replace- 
ment of the entire initial graft maybe needed [16]. 

Anastomotic aneurysms usually occur in arteriopathic patients. Careful preoper- 
ative planning is needed to make the patient as fit as possible. General anaesthesia is 
needed to allow adequate exposure, and the surgery is carried out under antibiotic 
and heparin cover. Once vascular control above and below the aneurysm has been 
obtained with minimal dissection, the aneurysm should be opened, along with the 
entire abnormal artery. Occlusion balloon catheters are often helpful in obtaining 
vascular control in this situation. The false aneurysm is usually resected and the 
ends of the graft and artery freshened for reanastomosis. Interposition grafting is 



84 Vascular Surgery 

likely to be needed to ensure that the new anastomosis is created without tension. 
Autologous saphenous vein is the graft of choice, although often polytetrafluoro- 
ethylene (PTFE) or Dacron may be better for size matching. [Q4: E, C, B, A, B] 
Retroperitoneal anastomotic aneurysms present more of a challenge. Proximal 
aortic anastomotic aneurysms may require supracoeliac clamping or balloon occlu- 
sion catheters [17]. Aneurysms associated with the distal portion of an aortoiliac 
graft may present late and catastrophically, illustrating the potential importance of 
monitoring these grafts for a prolonged period [18]. 

Infection in Anastomotic Aneurysms 

Some 80 per cent of anastomotic aneurysms occur in the groin, and they have the 
highest incidence of infection as their primary cause; approximately 30 per cent 
contain pathogenic bacteria. A high level of clinical suspicion of infection must be 
maintained, and Gram staining of all clots and removed graft should be carried out 
as a matter of routine. Perioperative antibiotics should be continued until results 
are available. 

The diagnosis of infection is usually obvious if the graft is surrounded by pus. If 
the graft is frankly infected, it should be excised completely with an extra-anasto- 
motic bypass to restore the distal circulation with prolonged, high-dose antibiotic 
cover. An obturator bypass may be used for an infected femoral false aneurysm. 
Aortic stump oversewing and axillobifemoral grafting can treat the (fortunately 
rare) infected aortic anastomotic aneurysm. Morbidity and mortality rates are 
high. 

Grafts with a more indolent level of infection that becomes apparent only after 
microbiological investigation maybe treated less radically. It is safest to assume 
that all femoral anastomotic aneurysms are contaminated. If prosthetic material is 
needed for repair, then measures used to reduce the chance of reinfection include 
the use of a rifampicin-soaked, gelatine-coated Dacron graft and gentamicin 
beads laid in close proximity. The reinfection rate after such procedures is 10 per 
cent [19]. 

Outcome 

Outcome depends on the initial site of the aneurysm and any confounding factors 
[20]. As the most common site for anastomotic aneurysms, the femoral artery has 
one of the highest successful outcomes. About 90 per cent of cases will have success- 
ful surgery, and those that have a recurrence still have a 90 per cent success rate 
from a second or subsequent operation. In comparison, anastomotic aneurysms 
that are intra-abdominal have a high complication rate when repaired surgically. A 
small anastomotic aneurysm in a superficial position can be monitored by ultra- 
sound initially, and can then be monitored safely by repeated examination by a clin- 
ician or the motivated patient. The success rate of operation at these sites is good 
[21]. Retroperitoneal aneurysms require long-term ultrasound follow-up. [Q6: F,T, F, 
T, F] If possible, minimally invasive techniques should be used for repair to avoid 
the high morbidity and mortality associated with surgery (providing infection is not 
present). In patients fit for surgery, excision and graft interposition has excellent 
long-term results. 



Anastomotic Aneurysms 85 

References 

1. Szilagyi DE, Smith RF, Elliott JP, Hageman JH, Dall'Olmo CA. Anastomotic aneurysms after vascular 
reconstruction: problems of incidence, etiology and treatment. Surgery 1975;78:800-16. 

2. Satiani B. False aneurysms following arterial reconstruction. Surg Gynecol Obstet 1981;152:357-63. 

3. Millili JJ, Lanes JS, Nemir P. A study of anastomotic aneurysms following aortofemoral prosthetic 
bypass. Ann Surg 1980;192:69-73. 

4. Waibel P. False aneurysm after reconstruction for peripheral arterial occlusive disease. Observations 
over 15-25 years. Vasa 1994;23:43-51. 

5. Moore WS, Hall AD. Late suture failure in the pathogenesis of anastomotic false aneurysms. Ann 
Surg 1970;172:1064-8. 

6. Berridge DC, Earnshaw JJ, Makin GS, Hopkinson BR. A ten-year review of false aneurysms in 
Nottingham. Ann R Coll Surg Engl 1988;70:253-6. 

7. Wandschneider W, Bull O, Deneck H. Anastomotic aneurysms: an unsolvable problem. Eur J Vase 
Endovasc Surg 1988;2:115-19. 

8. Gayliss H. Pathogenesis of anastomotic aneurysms. Surgery 1981;90:509-15. 

9. Sladen JG, Gerein AN, Miyagishima RT. Late rupture of prosthetic aortic grafts. Am J Surg 
1987;15:453-8. 

10. De Monti M, Ghilardi G, Sgroi G, Longhi F, Scorza R. Anastomotic pseudoaneurysm, true para-anas- 
tomotic aneurysm and recurrent aneurysm following surgery for abdominal aortic aneurysm. Is a 
unifying theory possible? Minerva Cardioangiol 1995;43:367-73. 

11. Hajarizadeh H, LaRosa CR, Cardullo P, Rohrer MJ, Cutler BS. Ultrasound guided compression 
of iatrogenic femoral psuedoaneurysm: failure, recurrence and long term results. J Vase Surg 
1995;22:425-30. 

12. Kang SS, Labropoulos N, Mansour MA, Michelini M, Filliung D, Baubly MP, et al. Expanded indica- 
tions for ultrasound-guided thrombin injection of pseudoaneurysms. J Vase Surg 2000;31:289-98. 

13. Un?aker R. Transcatheter embolisation of arterial aneurysms. Br J Radiol 1986;59:317-24. 

14. Manns RA, Duffield RG. Intravascular stenting across a false aneurysm of the popliteal artery. Clin 
Radiol 1997;52:151-3. 

15. Brittenden J, Gillespie I, McBride K, Mclnnes G, Bradbury AW. Endovascular repair of aortic 
pseudoaneurysms. Eur J Vase Endovasc Surg 2000;19;82-4. 

16. Clarke AM, Poskitt KR, Baird RN, Horrocks M. Anastomotic aneurysms of the femoral artery: aetiol- 
ogy and treatment. Br J Surg 1989;76:1014-16. 

17. Ernst CB. The surgical correction of arteriosclerotic femoral aneurysm and anastomotic aneurysm. 
In: Greenhalgh RM, Mannick JA, editors. The cause and management of aneurysms. London: WB 
Saunders, 1990;245-56. 

18. Treiman GS, Weaver FA, Cossman DV, Foran RF, Cohen JL, Levin PM, et al. Anastomotic false 
aneurysms of the abdominal aorta and the iliac arteries. J Vase Surg 1988;8:268-73. 

19. Earnshaw JJ. Anastomotic/false aneurysms. In: Horrocks M, editor. Arterial aneurysms: diagnosis 
and management. Bath: Butterworth Heinemann, 1995;209-21. 

20. Ylonen K, Biancari F, Leo E, Rainio P, Salmela E, Lahtinen J, et al. Predictors of development of anas- 
tomotic femoral pseudoaneurysms after aortobifemoral reconstruction for abdominal aortic 
aneurysm. Am J Surg 2004;187:83-7. 

21. Woodburn K. False aneurysms. In: Earnshaw J J, Parvin S, editors. Rare vascular disorders. Tfm 
Publishing Ltd 2005 (in press). 



9b. False Aneurysm in the Groin Following 
Coronary Angioplasty 

Steven S. Kang 



A 70-year-old female with a history of hypertension developed chest pair and was 
admitted to a local hospital. Heparin was administered intravenously. Later that 
day, she underwent coronary angiography, which showed a critical stenosis of 
the left anterior descending artery. The lesion was treated with angioplasty and 
stent placement. The right femoral artery sheath was left in place overnight, and 
heparin was continued. The following morning after stopping heparin, the sheath 
was removed and a FemoStop device was placed over the groin for 4 h. Heparin 
was then restarted. 

The next day, the patient was without any chest pain, but she did have mild 
discomfort in the right groin. There was a large hematoma in the right groin. The 
overlying skin had ecchymosis. The femoral pulse was prominent, and popliteal 
and pedal pulses were normal. A systolic bruit was heard over the femoral artery. 



Question 1 

What test should be obtained at this time? 

A. Computed tomography scan with intravenous contrast. 

B. Duplex ultrasound. 

C. Magnetic resonance angiogram. 

D. Contrast arteriogram. 

A false aneurysm was suspected and confirmed by duplex ultrasound examina- 
tion. It was arising from the common femoral artery (CFA). The flow cavity mea- 
sured 3 cm in diameter (Fig. 9b. 1). 



87 



88 



Vascular Surgery 




Fig. 9b.1 Duplex ultrasound demonstrates a false aneurysm arising from the common femoral artery. £Q 

Question 2 

The incidence of postcatheterization false aneurysms in the groin is higher under 
which of the following situations? 

A. Puncture of the CFA instead of the superficial femoral artery (SFA). 

B. Use of larger sheaths. 

C. Use of postprocedural anticoagulation. 

D. Patients with hypertension. 

E. Manual compression versus mechanical compression with a FemoStop after 
catheter removal. 



Question 3 

Which of the following statements about postcatheterization false aneurysms is/are 
true? 

A. Urgent surgical repair is indicated. 

B. This aneurysm is likely to undergo spontaneous thrombosis if observed. 

C. Spontaneous thrombosis is less common in patients who are anticoagulated. 

D. They may cause deep venous thrombosis. 



False Aneurysm in the Groin Following Coronary Angioplasty 89 

Heparin was discontinued and ultrasound-guided compression repair 
(UGCR) was attempted. 



Question 4 

Which are the disadvantages of UGCR? 

A. Thrombosis of the underlying artery is a frequent complication. 

B. Most patients find it painful. 

C. It is less successful in patients who are anticoagulated. 

D. Approximately 30 percent of successfully thrombosed false aneurysms recur. 

Due to patient discomfort, intravenous morphine and midazolam were adminis- 
tered. After 60 min of compression, the false aneurysm still had flow. It was decided 
not to persist. The hospital did not have any experience with ultrasound-guided 
thrombin injection. After discussion with our vascular surgery service, the patient 
was transferred to our hospital for thrombin injection. 

Question 5 

Which of the following statements regarding ultrasound-guided thrombin injection 
is/are true? 

A. It requires direct injection of thrombin into the neck of the false aneurysm. 

B. It involves simultaneous compression of the false aneurysm. 

C. It is less painful but less effective than UGCR. 

D. It works well in anticoagulated patients. 

E. It is appropriate only for femoral false aneurysms. 

Thrombin solution (1000 units/ml) was loaded into a small syringe and a 
22-gauge spinal needle was attached. Under ultrasound guidance, the needle was 
placed into the center of the false aneurysm (Fig. 9b. 2) and 0.3 ml thrombin was 
injected slowly. Within 15 s, the false aneurysm was thrombosed completely (Fig. 
9b. 3). The procedure was tolerated well. Flow in the underlying artery was pre- 
served and pedal pulses were intact. As the patient was otherwise stable, she was 
discharged soon afterwards. 

Question 6 

What are the reported complications of thrombin injection? 

A. Anaphylaxis. 

B. Intra-arterial thrombosis. 



90 



Vascular Surgery 




Fig. 9b.2. The tip of the needle is visible within the false aneurysm cavity. 




Fig. 9b.3. The aneurysm is completely thrombosed 15 seconds after thrombin injection. 



False Aneurysm in the Groin Following Coronary Angioplasty 91 

C. Prolonged urticaria. 

D. Mad cow disease. 



Commentary 



A false aneurysm after catheterization is suspected when there is a hematoma, espe- 
cially an enlarging one, at the puncture site hours or days after the procedure. There 
is often significant ecchymosis of the overlying skin. There may be a bruit, but a 
continuous bruit is usually associated with an arteriovenous fistula. There may be 
pain or neuralgia, and the site is often tender. A pulsatile mass is usually palpable, 
but a simple hematoma overlying the artery may give the same impression. Only a 
minority of false aneurysms are diagnosed unequivocally by physical examination. 
The diagnosis of a femoral false aneurysm has become very easy with duplex 
ultrasound. [Q1:B] 

The incidence of postcatheterization femoral false aneurysms varies from less 
than 0.5 percent to more than 5 percent [1]. Some of the factors that increase the 
likelihood of false aneurysm formation include larger sheaths, longer procedure 
times, multiple catheter exchanges, and peri- and postprocedure anticoagulation. 
Puncture of the superficial femoral or deep femoral artery instead of the CFA is 
found to be associated with higher rates of false aneurysm formation. Direct manual 
compression after catheter removal is better than compression devices, such as the 
FemoStop or C-clamp. Patient characteristics that may increase the likelihood of 
false aneurysm formation include atherosclerosis of the punctured artery, obesity 
and hypertension. [Q2: B, C, D] 

The potential complications of untreated false aneurysms are well known. 
Rupture is the most dramatic and life-threatening complication. Compression of 
surrounding tissues can cause pain, neuropathy, venous thrombosis, and necrosis 
of the overlying skin. Thrombosis of, or embolization into, the femoral artery may 
occur. Infection of these false aneurysms is less common. Because of these potential 
outcomes, early surgical repair had been advocated in the past. However, in the 
1990s, several series showed that the majority of small false aneurysms will develop 
spontaneous thrombosis [2-4]. It is less likely to occur with larger false aneurysms 
or in patients who are on anticoagulants. [Q3: C, D] Thrombosis may occur within 
days, or it may take weeks. Once thrombosis occurs, the false aneurysm is then a 
simple hematoma that gets resorbed slowly over time. The defect in the artery heals 
uneventfully in most cases. 

In 1991, Fellmeth et al. [5] described the method of UGCR of postcatheteriza- 
tion femoral false aneurysms and arteriovenous fistulas. The ultrasound trans- 
ducer is used to apply downward pressure on the neck of the false aneurysm to 
arrest flow. Pressure is maintained until the blood in the aneurysm becomes 
thrombosed. After the introduction of UGCR, numerous reports were published 
verifying the efficacy and overall safety of this procedure [6-9]. The typical 
success rate was between 60 and 90 percent. There were only a few published 
complications, including thrombosis of the underlying artery or the femoral 
vein from the compression, rupture during compression, rupture after success- 
ful compression, skin necrosis caused by prolonged pressure on the skin, and 
vasovagal reactions. Therefore, UGCR was shown to be a good alternative to 



92 Vascular Surgery 

surgical repair or observation, and most centers made it the initial treatment 
method. 

There are several disadvantages to the procedure. It is time-consuming, requiring 
an average of 30-60 min of compression. In most hands, the results are significantly 
poorer for patients on anticoagulants [10]. The recurrence rate is about 4-11 
percent, but it is as high as 20 percent for anticoagulated patients [6]. About 10 
percent of patients cannot be treated with UGCR because they have false aneurysms 
that are not compressible or cannot be compressed without also collapsing the 
underlying artery, which would increase the chance of arterial thrombosis. For most 
patients, the compression is painful, and intravenous sedation or analgesia is often 
necessary. Some patients have required epidural or general anesthesia to allow com- 
pression. Applying compression is also very uncomfortable for the operator. [Q4: B, C] 

Various endovascular treatments have been described for false aneurysms that 
have failed compression. They usually require catheterization of the feeding artery 
or false aneurysm from a remote access site. Embolization coils can be used to 
occlude the neck or to fill the cavity of the false aneurysm [11, 12]. Stent grafts can 
be placed in the femoral artery to exclude the false aneurysm, but late occlusion of 
the grafts is not uncommon [13]. They certainly should not be the initial method of 
treatment. However, for false aneurysms arising from other, less easily accessible 
arteries, these techniques may have a role. 

Because of the shortcomings of UGCR, we developed a new method of treating 
false aneurysms with ultrasound-guided thrombin injection [14, 15]. Thrombin 
causes the cleavage of fibrinogen into fibrin, which then polymerizes into a solid. It 
is the final product of the coagulation cascade, and this reaction occurs naturally 
whenever blood clots. Thrombin has been used topically for many years to control 
surface bleeding in the operating room. Our technique is as follows: The ultra- 
sound transducer is centered over the false aneurysm. Thrombin at a concentra- 
tion of 1000 U/ml is placed into a small syringe, and a 22-gauge spinal needle is 
attached. The needle is inserted at an angle into the false aneurysm along the same 
plane as the transducer, and the tip is positioned near the center of the false 
aneurysm. About 0.5 ml thrombin solution is injected slowly into the false 
aneurysm. Within seconds, thrombosis of the false aneurysm is seen. The proce- 
dure is not painful, and patients do not require any analgesia or sedation. We allow 
patients to get out of bed immediately after treatment, and outpatients are sent 
home soon after the procedure. 

So far, we have had great success with this procedure. We have treated 165 false 
aneurysms. Most (149) developed after groin puncture. There were also false 
aneurysms in six brachial, three subclavian, two radial, two tibial, one distal SFA, 
and one superficial temporal arteries, and in one arm arteriovenous fistula. Forty- 
seven patients were anticoagulated at the time of thrombin injection. It was initially 
successful in 161 of 165 patients. The other four (all femoral) had partial thrombo- 
sis. One of these had complete thrombosis 3 days later when brought back for 
repeat injection. Three had surgical repair. There were early recurrences in 12 
patients who had initial successful thrombin injection. Seven were reinjected suc- 
cessfully at the time the recurrence was diagnosed. One had spontaneous thrombo- 
sis several days after recurrence was identified. Four had surgical repair. Overall, 
only 7 of 165 required surgical repair. There were three complications. A brachial 
artery false aneurysm had injection of thrombin directly into its neck, which caused 
thrombosis of the brachial artery. A femoral false aneurysm had a relatively large 
volume of thrombin injected and developed a thrombus in the posterior tibial 



False Aneurysm in the Groin Following Coronary Angioplasty 93 

Table 9b.1 . Results of ultrasound-guided thrombin injection 





Cases 


Successes (percent) 


Complications 


Current 


165 


158(96) 


3 


Khoury [18] 


131 


126(96) 


3 


Paulson [19] 


114 


110(96) 


4 


Maleux [20] 


101 


99 (98) 





Mohler[21] 


91 


89 (98) 


1 


La Perna [22] 


70 


66 (94) 





TOTAL 


672 


648 (96) 


11(1.6) 



artery. Both of these thromboses resolved after intravenous heparin. A femoral false 
aneurysm with a short neck that was about 10 mm wide had partial thrombosis of 
the aneurysm. Further injection was not able to thrombose the remaining cavity but 
instead caused a tail of thrombus to form in the SFA. The patient underwent surgi- 
cal thrombectomy and repair of the aneurysm. [Q5: D] 

Our results show that intra-arterial thrombosis after thrombin injection is 
uncommon. The high concentration of thrombin results in almost immediate con- 
version of the solution into a solid (thrombus) when it mixes with relatively stag- 
nant blood. Since the neck of the false aneurysm is usually much narrower than the 
aneurysm cavity, the thrombus cannot enter the artery. As long as the volume of the 
thrombin injected does not approach or exceed the volume of the false aneurysm, 
which may result in forcing some of the solution out of the cavity, then the risk of 
native artery thrombosis should be small. It is likely to be higher when the neck is 
very wide. Other complications that have been reported include single cases of ana- 
phylaxis [16] and prolonged urticaria [17]. Repeated exposure to bovine thrombin 
can also lead to development of antibodies to bovine factor V, which may cross- 
react with autogenous factor V, causing hemorrhagic complications. [Q6: A, B, C] 

Many others have also had good results with this procedure. In the largest series, 
the success rate is around 96 percent and the complication rate less than 2 percent 
(Table 9b. 1). Given its simplicity, efficacy, and safety, ultrasound-guided thrombin 
injection should be considered the initial treatment of choice for postcatheteriza- 
tion false aneurysms. 



References 

1. Skillman JJ, Kim D, Bairn DS. Vascular complications of percutaneous femoral cardiac interventions. 
Incidence and operative repair. Arch Surg 1988;123:1207-12. 

2. Kent KC, McArdle CR, Kennedy B, Bairn DS, Anninos E, Skillman JJ. A prospective study of the clin- 
ical outcome of femoral pseudoaneurysms and arteriovenous fistulas induced by arterial puncture. 
J Vase Surg 1993;17:125-31. 

3. Kresowik TF, Khoury MD, Miller BV, Winniford MD, Shamma AR, Sharp WJ, et al. A prospective 
study of the incidence and natural history of femoral vascular complications after percutaneous 
transluminal coronary angioplasty. J Vase Surg 1991;13:328-33. 

4. Toursarkissian B, Allen BT, Petrinec D, Thompson RW, Rubin BG, Reilly JM, et al. Spontaneous 
closure of selected iatrogenic pseudoaneurysms and arteriovenous fistulae. J Vase Surg 
1997;25:803-8. 

5. Fellmeth BD, Roberts AC, Bookstein JJ, Freischlag JA, Forsythe JR, Buckner NK, et al. 
Postangiographic femoral artery injuries: nonsurgical repair with US-guided compression. 
Radiology 1991;178:671-5. 



94 Vascular Surgery 

6. Cox GS, Young JR, Gray BR, Grubb MW, Hertzer NR. Ultrasound-guided compression repair 
of postcatheterization pseudoaneurysms: results of treatment in one hundred cases. J Vase Surg 
1994;19:683-6. 

7. Hajarizadeh H, LaRosa CR, Cardullo P, Rohrer MJ, Cutler BS. Ultrasound-guided compression of 
iatrogenic femoral pseudoaneurysm failure, recurrence, and long-term results. J Vase Surg 
1995;22:425-30. 

8. Hertz SM, Brener BJ. Ultrasound-guided pseudoaneurysm compression: efficacy after coronary 
stenting and angioplasty. J Vase Surg 1997;26:913-16. 

9. Hood DB, Mattos MA, Douglas MG, Barkmeier LD, Hodgson KJ, Ramsey DE, et al. Determinants of 
success of color-flow duplex-guided compression repair of femoral pseudoaneurysms. Surgery 
1996;120:585-8. 

10. Hodgett DA, Kang SS, Baker WH. Ultrasound-guided compression repair of catheter-related femoral 
artery pseudoaneurysms is impaired by anticoagulation. Vase Surg 1997;31:639-44. 

11. Jain SP, Roubin GS, Iyer SS, Saddekni S, Yadav JS. Closure of an iatrogenic femoral artery pseudoa- 
neurysm by transcutaneous coil embolization. Catheter Cardiovasc Diagn 1996;39:317-19. 

12. Pan M, Medina A, Suarez DL, Romero M, Hernandez E, Segura J, et al. Obliteration of femoral 
pseudoaneurysm complicating coronary intervention by direct puncture and permanent or remov- 
able coil insertion. Am J Cardiol 1997;80:786-8. 

13. Thalhammer C, Kirchherr AS, Uhlich F, Walgand J, Gross CM. Postcatheterization pseudoaneurysms 
and arteriovenous fistulas: repair with percutaneous implantation of endovascular covered stents. 
Radiology 2000;214:127-31. 

14. Kang SS, Labropoulos N, Mansour MA, Baker WH. Percutaneous ultrasound guided thrombin injec- 
tion: a new method for treating postcatheterization femoral pseudoaneurysms. J Vase Surg 
1998;27:1032-8. 

15. Kang SS, Labropoulos N, Mansour MA, Michelini M, Filliung D, Baubly MP, et al. Expanded indica- 
tions for ultrasound-guided thrombin injection of pseudoaneurysms. J Vase Surg 2000;31:289-98. 

16. Pope M, Johnston KW. Anaphylaxis after thrombin injection of a femoral pseudoaneurysm: recom- 
mendations for prevention. J Vase Surg 2000;32:190-1. 

17. Sheldon PJ, Oglevie SB, Kaplan LA. Prolonged generalized urticarial reaction after percutaneous 
thrombin injection for treatment of a femoral artery pseudoaneurysm. J Vase Interv Radiol 
2000;11:759-61. 

18. Khoury M, Rebecca A, Greene K, Rama K, Colaiuta E, Flynn L, et al. Duplex scanning-guided throm- 
bin injection for the treatment of iatrogenic pseudoaneurysms. J Vase Surg 2002;35:517-21. 

19. Paulson EK, Nelson RC, Mayes CE, Sheafor DH, Sketch MH, Jr, Kliewer MA. Sonographically guided 
thrombin injection of iatrogenic femoral pseudoaneurysms: further experience of a single institu- 
tion. AJR Am J Roentgenol. 2001;177:309-16. 

20. Maleux G, Hendrickx S, Vaninbroukx J, Lacroix H, Thijs M, Desmet W, et al. Percutaneous injection 
of human thrombin to treat iatrogenic femoral pseudoaneurysms: short- and midterm ultrasound 
follow-up. Eur Radiol 2003;13:209-12. 

21. Mohler ER 3rd, Mitchell ME, Carpenter JP, Strandness DE, Jr, Jaff MR, Beckman J A, et al. 
Therapeutic thrombin injection of pseudoaneurysms: a multicenter experience. Vase Med. 
2001;6:241-4. 

22. La Perna L, Olin JW, Goines D, Childs MB, Ouriel K. Ultrasound-guided thrombin injection for the 
treatment of postcatheterization pseudoaneurysms. Circulation. 2000;102:2391-5. 



10. Acute Thrombosis 



Vikram S. Kashyap and Kenneth Ouriel 



A 60-year-old retired nurse presents to the emergency department with a cold 
and painful right foot. She has an extensive history of peripheral vascular disease 
including bilateral iliac stenting for atherosclerotic occlusive disease. Prior to the 
onset of symptoms, she could walk at least a quarter of a mile without pain. Her 
past history includes a stroke years ago that she has recovered from, hyperlipi- 
demia, coronary artery disease, and smoking. She has not been recently hospital- 
ized or undergone any invasive procedure or operation. 

On examination, her pulse is 90 bpm, and her blood pressure is 115/65. Her 
heart sounds reveal a regular rhythm. She has a normal left femoral pulse and the 
left foot is warm and well perfused. The right limb has no palpable pulses and the 
right foot is pale and cool. She can move the foot, but the toes are insensate. 
There is a venous Doppler signal in the right foot, but no arterial signal. 



Question 1 

Native arterial or graft thrombosis can be differentiated from embolic occlusion by 
the following: 

A. The presence of palpable pulses in the contralateral extremity. 

B. A history of cardiac arrhythmias. 

C. The degree of profound ischemia in the affected extremity. 

D. The location of the occlusion. 

E. All of the above. 

Question 2 

What is the SVS/ISCVS category of limb ischemia in this patient? 

A. Category I. 

97 



98 



Vascular Surgery 



B. Category Ha. 

C. Category lib. 

D. Category III. 



Question 3 

In acute embolism, the sequence of events is: 

A. Paralysis, pain, paresthesia, pulselessness, pallor. 

B. Pulselessness, pain, pallor, paresthesia, paralysis. 

C. Pulselessness, pain, pallor, paralysis, paresthesia. 

The patient is taken to the endovascular suite and an angiogram is performed via a 
contralateral femoral approach. This reveals an occluded right iliac stent (Fig. 10.1) 




Fig. 10.1 . Aortography via a left femoral approach documents a right iliac thrombosis in the setting of prior iliac 
stenting. 



Acute Thrombosis 



99 




Fig. 10.2. Arteriography of the right leg documents reconstitution of the common femoral artery with a normal 
run-off via both anterior and posterior tibial arteries (not shown). 



with reconstitution of the femoral bifurcation (Fig. 10.2) and normal outflow to the 
foot. 



Question 4 

Treatment options for this patient include: 

A. Anticoagulation with heparin and Coumadin. 

B. Operative thrombectomy. 

C. Extra-anatomic bypass. 

D. Aortofemoral bypass. 

E. Mechanical thrombectomy, thrombolysis, and endovascular intervention. 

F. Intravenous thrombolysis. 



100 



Vascular Surgery 



Question 5 

After thrombolysis, long-term outcome is predicated on: 

A. The thrombolytic agent used. 

B. Unmasking a "culprit lesion" that is treated via either endovascular or surgical 
means. 

C. The length of thrombolysis. 




Fig. 10.3. A hydrophilic wire is used to safely cross the thrombosed segment and this arteriogram confirms 
catheter placement in the native circulation distal to the iliac thrombosis. 



Acute Thrombosis 



101 



D. Assuring all acute thrombus is lysed. 

E. The arterial outflow. 

This patient underwent successful thrombolysis. A combination of a hydrophilic 
wire and catheter was used to cross the acute thrombotic occlusion ("guidewire tra- 
versal test") and gain access to the native femoral system (Fig. 10.3). Thrombolysis 
(agent: alfimeprase, Nuvelo, Inc., investigational use only) was performed through a 
multi-side hole infusion catheter and at 2 hours after initiation, significant throm- 
bus resolution had occurred (Fig. 10.4). The patient was monitored closely and 
returned to the endovascular suite for a follow-up study. 

Contrast injection through the sheath demonstrated a stenotic lesion and percu- 
taneous balloon angioplasty of the distal right iliac system was performed (Fig. 
10.5) with restoration of blood flow into the right limb without any evidence of 
distal embolization. The patient had palpable femoral, popliteal, and pedal pulses 
at the completion of the procedure. She was carefully observed for the develop- 
ment of compartment syndrome. She was discharged 3 days after admission with 




Fig. 10.4. After 2 hours of thrombolysis (alfimeprase, Nuvelo, Inc., investigational use only), approximately 50 
percent of the thrombus burden is dissolved. 



102 



Vascular Surgery 




Fig. 10.5. After complete thrombolysis, a distal iliac stenosis is uncovered and treated with balloon angioplasty. 
This lesion was presumably the etiology of the acute thrombosis. 



an ankle-brachial index of 0.9. The patient has remained free of symptoms for over 
6 months. 



Commentary 

The etiology of acute limb ischemia can be classified into two groups. Thrombotic 
events occur in the setting of native arterial disease or bypass graft stenoses. In con- 
trast, embolic phenomena usually occur in normal vessels and tend to lodge at arte- 
rial bifurcations [1]. Thrombotic occlusions are thought to represent progression of 
atherosclerotic disease and occur at sites along the arterial tree and most notably 
the superficial femoral artery (SFA) at the adductor canal. In comparison, autolo- 
gous grafts fail at sites of intimal hyperplasia, or fibrotic valves. Due to preexisting 



Acute Thrombosis 103 

collaterals, native arterial thrombosis seldom presents with the profound ischemia 
seen with embolic ischemia. The presence of palpable pulses on the contralateral 
limb and a history of cardiac arrhythmia assist in differentiating acute embolus 
from thrombotic occlusions [1]. 

All of the factors listed can help in differentiating embolic from thrombotic 
occlusions. [Q1: E] Often, a definitive diagnosis cannot be made preoperatively. 
However, identifying an embolic source for acute limb ischemia is helpful for both 
the acute and long-term management of the patient. 

Clinical classification and diagnosis of acute occlusion of the lower extremity is 
based on the symptoms of the patient. The severity of symptoms is associated with 
the extent of the occlusion and the presence of preexisting vessels. Patients with 
thrombotic occlusions from underlying disease of the SFA at the adductor canal 
may only experience worsening claudication while embolic events are usually asso- 
ciated with rapid onset and severe ischemia because of the lack of preexisting collat- 
eral flow. Limb ischemia has been classified into three categories by an SVS/ISCVS 
ad hoc committee, based on severity of ischemia [2]. Category I limbs are viable, not 
immediately threatened and have no motor or sensory loss. There are clearly 
audible arterial Doppler signals in the foot. Category II includes threatened limbs 
where salvage maybe possible with timely intervention. Importantly, this category 
is divided into two subgroups, a and b, which distinguish the time interval neces- 
sary for treatment. Category Ha patients require prompt treatment whereas cate- 
gory lib patients need immediate therapy to prevent amputation. In category II, 
audible venous Doppler signals are present, but there is no arterial signal in the 
foot. Category Ha patients have minimal sensory loss and have no motor loss. 
However, patients with category lib ischemia have muscle weakness, and sensory 
loss encompasses more than the toes. Category III is characterized by irreversible 
ischemia with profound and permanent neuromuscular damage where amputation 
is the only recourse. 

This patient has category Ha limb ischemia characterized by mild sensory loss 
and lack of distal arterial signals in the foot. [Q2: B] 

The sequence of clinical events in patients with lower extremity ischemia is often 
predictable. [Q3: B] Most patients with acute ischemia, especially of an embolic 
nature, will have pulselessness followed by pain and pallor. Paresthesia indicates 
sensory nerve ischemia and occurs usually from 1 to 3 hours after the onset of acute 
ischemia. Paralysis indicates motor nerve damage that is often irreversible. In the 
setting of acute ischemia without collateral flow, paralysis occurs approximately 6 
hours after the onset of ischemia [3]. Any motor dysfunction should be seen as a 
worrisome sign and should prompt urgent intervention. Poikilothermia indicates 
that the foot or limb has approximated ambient temperature. In these irreversible 
cases (category III), amputation is the only option and often has to be done quickly 
to avoid systemic complications. 

Both the diagnosis and localization of acute arterial occlusion is based upon the 
findings on physical examination and imaging studies. A "waterhammer" pulse 
signifies outflow obstruction, as observed with a common femoral embolus. By con- 
trast, calcified vessels are common with thrombosis from underlying atherosclerotic 
disease. Multiple options are available for localization of the occlusion. Noninvasive 
testing with segmental pressures, pulse volume recording, and measurement of the 
ankle brachial indices can provide a baseline study for comparison after treatment. 
Both vertical and horizontal pressure gradients of 30 mm Hg or more in the lower 
extremity can accurately identify the site of occlusion. Duplex ultrasonography can 



104 Vascular Surgery 

also be utilized to examine the femoral and popliteal vessels and localize area of 
occlusion. Other causes such as thrombosed popliteal aneurysm can be easily diag- 
nosed in this manner. Magnetic resonance angiography and CT angiography are 
emerging as noninvasive techniques for arteriographic imaging and localization of 
thrombosis. However, angiography remains the gold standard for localization of 
arterial occlusion. As importantly, angiography allows percutaneous access to the 
site of thrombosis and an array of treatment options for restoring blood flow to the 
limb. 

Treatment for limb ischemia has evolved over the past two decades with advances 
in both pharmacologic therapy and endovascular options. [Q4: B, C, E] In patients 
with acute limb ischemia secondary to iliac occlusion, operative thrombectomy of 
the occluded iliac system maybe feasible. In the setting of profound ischemia and a 
diseased iliac artery precluding successful thrombectomy, extra-anatomic bypass 
can be performed to provide expeditious blood flow into the ischemic limb. In these 
cases, either femoral-femoral bypass or axillo-femoral bypass can be contemplated 
depending on the inflow source. Aortofemoral bypass is a very durable option for 
patients with chronic occlusion and chronic ischemia of the limb. However, in 
patients with acute ischemia ill-prepared for major surgery, proceeding with direct 
reconstruction with the aorta as the inflow is sometimes hazardous. Multiple 
endovascular devices are available in the setting of acute thrombosis. Percutaneous 
mechanical thrombectomy with thrombolysis either via the power-pulse technique 
or via a standard infusion often quickly resolves the acute ischemia. Continued 
thrombolytic infusion is required for complete resolution of thrombus. Often, a 
"culprit" lesion will be unmasked by dissolving all of the acute thrombus, allowing 
percutaneous treatment of the offending lesion. Systemic thrombolytic therapy has 
been used to treat peripheral arterial occlusions, but results have been disappoint- 
ing owing to a significant incidence of bleeding complications. Currently, systemic 
therapy is usually used for venous thromboembolic states. Regional intravascular 
infusion of the lytic agent avoids some of the systemic complications and is largely 
used for peripheral arterial thromboses and graft occlusions. Because a systemic 
lytic state may occur with prolonged regional intravascular thrombolytic therapy, 
patient selection is critical. Absolute contraindications include active internal bleed- 
ing, recent surgery or trauma to the area to be perfused, recent cerebrovascular 
accident, or documented left heart thrombus [3]. Relative contraindications include 
recent surgery, gastrointestinal bleeding or trauma, severe hypertension, mitral 
valve disease, endocarditis, hemostatic defects, or pregnancy. 

Several multicenter trials have examined groups of patients treated with surgical 
therapy or thrombolysis. [Q5: B, D, E] The Rochester trial randomized patients to 
surgery or thrombolysis and demonstrated a lower mortality in the thrombolysis 
group [4]. Following successful thrombolysis, unmasked "culprit lesions" were 
treated with angioplasty or surgery of a lesser magnitude, thereby reducing the 
severity of the intervention and overall morbidity. The finding of a lesion that pre- 
cipitated the thrombosis is critical to avoiding re-thrombosis. The STILE trial 
(Surgery versus Thrombolysis for Ischemia of the Lower Extremity) compared 
optimal surgical therapy to intra-arterial catheter-directed thrombolysis for native 
arterial or bypass graft occlusions [5, 6]. Stratification by duration of ischemic 
symptoms revealed that patients with ischemia of less than 14 days' duration had 
lower amputation rates with thrombolysis and shorter hospital stays, while patients 
with ischemia for longer than 14 days who were treated surgically had less ongoing 
or recurrent ischemia and trends toward lower morbidity. At 6 months, amputa- 



Acute Thrombosis 105 

tion-free survival was improved in patients with acute ischemia treated with throm- 
bolysis, but patients with chronic ischemia had lower amputation rates when 
treated surgically. Fifty-five percent of patients treated with thrombolysis had a 
reduction in magnitude of their surgical procedure. Of note, no difference was seen 
between the use of rt-PA and urokinase [5]. 

A multicenter, randomized, prospective trial comparing thrombolysis to surgery 
for acute lower extremity ischemia of less than 14 days' duration has been carried 
out [7]. The Thrombolysis or Peripheral Arterial Surgery trial (TOPAS) randomized 
757 patients to surgery or thrombolytic therapy. The most effective dose for uroki- 
nase was determined to be 4000 U/min with complete thrombolysis in 71 percent of 
patients. After successful thrombolytic therapy, either surgical or endovascular 
intervention was performed on the lesion responsible for the occlusion if found. 
When compared to the surgical arm, the one-year limb salvage rates and mortality 
were not statistically different. However, although no statistical differences between 
the two groups were seen with respect to amputation-free survival, thrombolysis 
was associated with a reduction in the number and magnitude of open surgical 
interventions over a one year follow-up period. 

Unlike thrombolysis in coronary or venous systems, dissolution of the larger 
peripheral arterial thrombi requires direct infusion of thrombolytic agent into the 
clot. The thrombosed artery or bypass graft must be accessed with a wire, followed 
by placement of an infusion system into the thrombus. Following successful throm- 
bolysis, any unmasked lesion can be addressed with balloon angioplasty and stent- 
ing or with an open surgical procedure. Even when a surgical procedure is 
necessary, it can usually be performed electively, in a well-prepared patient, and is 
often of a lesser magnitude than what would have been required without thrombol- 
ysis. Thrombolytic therapy is an effective option for selected patients with acute 
thrombotic occlusion. 



References 

1. Blaisdell FW, Steele M, Allen RE: Management of acute lower extremity arterial ischemia due to 
embolism and thrombosis. Surgery 1978;84:822-34. 

2. Rutherford RB, Baker JD, Ernst C, et al. Recommended standards for reports dealing with lower 
extremity ischemia: Revised version. J Vase Surg 1997;26:517-38. 

3. Kashyap VS, Quinones-Baldrich WJ. Principles of thrombolytic therapy. In: Rutherford RB, editor. 
Vascular surgery, 5th edn. Philadelphia, PA: WB Saunders, 2000;457-75. 

4. Ouriel K, Shortell CK, DeWeese JA, et al. A comparison of thrombolytic therapy with operative revas- 
cularization in the initial treatment of acute peripheral arterial ischemia. J Vase Surg 1994;19:1021-30. 

5. The STILE Investigators. Results of a prospective randomized trial evaluating surgery versus throm- 
bolysis for ischemia of the lower extremity, The STILE Trial. Ann Surg 1994;220:251-68. 

6. Weaver F, Camerato A, Papanicolau G, et al. Surgical revascularization versus thrombolysis for non- 
embolic lower extremity native artery occlusions: results of a prospective randomized trial. The STILE 
Investigators. J Vase Surg 1996;24:513-23. 

7. Ouriel K, Veith FJ, Sasahara AA. A comparison of recombinant urokinase with vascular surgery as 
initial treatment for acute arterial occlusion of the legs. N Engl J Med 1998;338:1105-11. 



11. Arterial Embolism 



Andre Nevelsteen 



A 65-year-old man presented with acute severe pain in his right leg. Medical 
history revealed non-insulin-dependent diabetes mellitus for 3 years and a 
myocardial infarction (MI) some 5 years ago. The pain in the right leg developed 
suddenly over 6 hours without associated trauma and became worse over time. 
On admission, the right leg looked pale distally from the level of the knee. There 
was loss of light touch sensation on examination of the foot. The patient had 
difficulties in wiggling the toes. Plantarflexion and dorsiflexion of the toes were 
still possible. Palpation of the calf showed soft but tender muscles. Clinical exam- 
ination of the abdomen showed no abnormalities. There was no pulsating mass. 
Irregular but bounding pulsations were felt in the right femoral artery. Popliteal 
artery and tibial artery pulsations were absent. Normal pulsations were felt in the 
left popliteal and posterior tibial artery. 



Question 1 

What is the aetiology of arterial embolism? 

A. The aetiology of arterial embolism is most frequently unknown. 

B. The most frequent cause of arterial embolism is cardiac valve destruction by 
rheumatic heart disease or endocarditis. 

C. The most frequent cause of arterial embolism is atrial fibrillation in association 
with atherosclerotic heart disease. 

D. Deep venous thrombosis might represent a rare cause of arterial embolism. 

E. Arterial embolism is most frequently seen in the presence of increased blood 
viscosity. 

With the diagnosis of acute arterial ischaemia in mind, a full dose of intravenous 
heparin was administered immediately. 



107 



108 Vascular Surgery 

Question 2 

What is the place of heparin in the treatment of arterial embolism? 

A. Heparin can dissolve an arterial embolus, avoiding the need for subsequent 
operation. 

B. Heparin will avoid subsequent arterial thrombosis, which can complicate treat- 
ment of arterial embolism. 

C. Heparin will avoid subsequent arterial thrombosis, which can complicate treat- 
ment of arterial embolism. In addition, heparin will prevent recurrent emboli. 

D. The use of heparin is contraindicated since it may lead to fragmentation of an 
arterial embolism and induce microembolisation in the peripheral arteries. 

A chest film X-ray showed no abnormalities. Electrocardiogram (ECG) revealed 
atrial fibrillation and signs of an old MI. Laboratory studies were normal. Duplex 
examination showed a thrombotic occlusion of the right femoral bifurcation and 
the superficial femoral artery. A weak flow sign was present in the popliteal artery. 
The tibial arteries were not visualised. 



Question 3 

The preferred treatment of arterial embolism is: 

A. Local excision of the vessel and reconstruction with interposition graft. 

B. Continued heparinisation and wait and see. 

C. Simple Fogarty catheter embolectomy with peroperative angiographic control. 

D. Simple Fogarty catheter embolectomy, but percutaneous aspiration throm- 
boembolectomy might be a good alternative in selected cases. 

After placement of a central venous catheter, the patient was taken to the operat- 
ing theatre and the right femoral bifurcation was exposed under local anaesthesia. A 
transverse arteriotomy confirmed complete thrombotic occlusion of the femoral 
bifurcation. There was good inflow. Thrombi were removed from the femoral bifur- 
cation, and pulsatile backflow was obtained from the profunda femoris artery. 

Multiple thrombi were removed from the superficial femoral artery and the 
popliteal artery after several passages of Fogarty embolectomy catheters numbers 3 
and 4. Intraoperative angiography showed good patency of the superficial, popliteal 
and peroneal arteries. The anterior tibial artery was completely occluded. The pos- 
terior tibial was patent in its first portion but occluded distally. A small catheter was 
inserted into the popliteal artery, and 350,000 units of urokinase were infused as a 
dripping infusion over 30 min. Repeated angiography showed further clearance of 
the posterior tibial artery to the level of the ankle joint. The anterior tibial artery 
was still occluded. It was decided to accept the situation. The arteries were flushed 
with a diluted heparinised saline solution, and the transverse arteriotomy was 
closed with the aid of a Dacron patch. Sodium bicarbonate was administered intra- 
venously before reperfusion. 



Arterial Embolism 109 

Question 4 

Reperfusion syndrome after arterial embolectomy: 

A. Will never be seen after peripheral but only after aortic embolism. 

B. Cannot be prevented medically. 

C. Will be prevented by early ambulation. 

D. Is induced by metabolic acidosis and myoglobinuria. 

Postoperatively, the foot was well vascularised and the patient was able to wiggle 
his toes almost normally. Pulsations were felt in the posterior tibial artery. 
Intravenous heparin was continued. Brisk diuresis was maintained with mannitol 
and alkalisation of the urine. Repeated laboratory studies showed no evidence of 
acidosis or hyperkalaemia. 



Question 5 

Fasciotomy: 

A. Has become obsolete and swelling of the limb should be treated by elevation and 
bed rest. 

B. Is best routinely performed in any patient, treated for arterial embolism of the 
lower limbs. 

C. The indication to fasciotomy needs to be based on objective parameters such as 
the presence of reperfusion syndrome and postoperative compartmental pres- 
sure measurements. 

D. In daily practice, the indication for fasciotomy is most frequently based on indi- 
vidual preference and clinical feeling. 

Six hours postoperatively, the patient developed significant limb swelling with 
augmentation of pain, venous hypertension and sensory impairment of the foot. A 
perifibular fasciotomy to decompress all four compartments was performed under 
general anaesthesia. Afterwards, the swelling subsided and the fasciotomy wound 
was closed in a delayed primary fashion after 1 week. 



Question 6 

With the pre- and peroperative diagnosis in mind: 

A. The patient should be placed under antiplatelet therapy postoperatively in order 
to prevent another episode of embolism. 

B. Heparin and oral anticoagulants remain the treatment of choice during the 
postoperative period. 



110 Vascular Surgery 

C. Subsequent investigation with regard to the source of the embolus is not neces- 
sary, because this will not change the medical treatment. 

D. Postoperative investigation with regard to the source of embolism can be 
limited to cardiac examinations such as echocardiography and Holter 
monitoring. 

Abdominal ultrasound performed postoperatively showed atheromatosis of the 
abdominal aorta but no aneurysmal dilatation. Transthoracic and transoesophageal 
echocardiography revealed no ventricular aneurysm or intracardiac thrombi. 
Holter monitoring for 24 h confirmed atrial fibrillation. Pathological examination of 
the retrieved emboli was compatible with ordinary thrombotic material. Cultures 
were negative. The problem of atrial fibrillation was handled medically. Oral anti- 
coagulation was initiated, and the patient was discharged after 10 days. Six months 
later, there were no repeat episodes of acute ischaemia. 



Commentary 

Acute ischaemia due to arterial embolism represents a limb-threatening event. 
Although the carotid or intracranial vessels may be involved in a minority of the 
cases, the upper or lower extremities are involved in 70-80 per cent in most series of 
arterial embolisation [1]. The lower extremity is involved five times as frequently as 
the upper extremity, and the sites of embolic occlusion are most often related to 
major arterial bifurcations. The common femoral bifurcation is the most frequent 
site of embolic occlusion, usually noted in 30-50 per cent of all cases [2]. In total, 
the femoral and popliteal arteries are involved more than twice as often as the aorta. 

The heart is by far the predominant source of arterial emboli, seen in 80-90 per 
cent of cases [3]. Atrial fibrillation is present in approximately 70 per cent of 
patients. Previously, it was most frequently the reflection of rheumatic heart 
disease. Since the incidence of rheumatic heart disease has declined steadily over 
the last 50 years, atrial fibrillation is now associated most frequently with athero- 
sclerotic heart disease. 

MI is the second common cause of peripheral embolisation. Left ventricular 
mural thrombus occurs in 30 per cent of acute transmural infarcts. Clinically 
evident embolism is seen in only 5 per cent of these patients [4]. One should be 
aware, however, that silent MI maybe present in up to 10 per cent of patients with 
peripheral emboli, and that embolisation may be the presenting symptom of an 
acute infarction. Apart from the acute period, MI may also cause emboli after 
longer intervals. This is usually due to areas of hypokinesis or ventricular aneurysm 
formation. Although most emboli occur within 6 weeks of MI, much longer inter- 
vals may be noted. 

Other cardiac diseases are associated less frequently with peripheral emboli. 
Thromboemboli can, however, arise from prosthetic cardiac valves or from vegeta- 
tions on the mitral or aortic valve leaflets. Endocarditis should certainly be ruled 
out. Finally, intracardiac tumours, such as atrial myxoma, may also give rise to clin- 
ically evident embolic events. 

Non-cardiac sources of peripheral emboli are noted less frequently. Major emboli 
may arise from aneurysms of the aorta or less frequently from the femoropopliteal 
vessels [5]. With upper-extremity emboli, one should be aware of unsuspected tho- 



Arterial Embolism 111 

rack outlet syndrome and aneurysmal deformation of the subclavian artery. 
Paradoxical emboli might be seen with deep venous thrombosis in association with 
a patent foramen ovale. Primary or secondary lung tumours might invade the pul- 
monary veins, causing tumour emboli. Finally, apart from rare causes such as 
foreign body embolisation, it should be recognised that the source of embolisation 
will remain inapparent in some 10 per cent of patients [2]. [Q1:C, D] 

The diagnosis of acute ischaemia caused by arterial embolism is usually straight- 
forward. The most typical signs are characterised by the "five Ps": pulselessness, 
pain, pallor, paraesthesia and paralysis. The level of occlusion is determined by the 
presence or absence of palpable pulses. Once the diagnosis of acute arterial 
ischaemia has been made, 5000 units of heparin are administered intravenously. 
This is not meant as effective treatment but it prevents the propagation and frag- 
mentation of the thrombus. Concomitant venous thrombosis, which can occur with 
prolonged severe arterial ischaemia, might also be avoided. Heparin administration 
allows time for diagnosis, evaluation and, if necessary, treatment of cardiac 
disturbances. [Q2: B] 

Fogarty catheter embolectomy remains the treatment of choice in most patients 
with peripheral embolisation [6]. The procedure is usually carried out under local 
anaesthesia and is effective in cases of major emboli. All retrieved emboli should be 
sent for pathological and microbiological examination. The operative result should 
be checked by intraoperative fluoroscopy or angioscopy. Remaining thrombi in the 
distal vessels can be approached directly or by intraoperative thrombolysis [7]. 
Thrombolytic therapy or percutaneous aspiration thromboembolectomy (Fig. 11.1) 
maybe used as alternatives to Fogarty catheter embolectomy in selected cases with 
no motor dysfunction or profound sensory loss [8, 9]. [Q3: C, D] 

All patients undergoing revascularisation of an acutely ischaemic limb are at risk 
of ischaemia reperfusion syndrome. This was first emphasised by Haimovici [10], 
described under its most grave form as the myonephropathic-metabolic syndrome. 
This reperfusion syndrome is the consequence of muscular hypoxia and the associ- 
ated metabolic changes. A prolonged period of ischaemia results in accumulation 
of potassium, lactic acid, myoglobin and other cellular enzymes, leading to a 
significant fall in blood pH due to anaerobic metabolism, paralysis of the sodium 
potassium cellular pump and rhabdomyolysis [11]. Acute washout of these prod- 
ucts may lead to hyperkalaemia and metabolic acidosis, resulting in myocardial 
depression or dysrhythmias. Myoglobin and other products of skeletal muscle 
breakdown can precipitate within the kidney and result in acute renal failure. 
Myoglobinuria is the first sign. [Q4: D] These problems should be anticipated with 
bicarbonate and/or calcium intravenously just before reperfusion. Induction of 
forced diuresis with mannitol and alkalisation of the urine might avoid acute renal 
failure. In addition, mannitol also acts as a scavenger of oxygen-derived free radi- 
cals, which are an important intermediary in ischaemia reperfusion injury [12, 13]. 
It is clear, therefore, that the patient should be monitored carefully postoperatively 
with regard to electrolyte changes, development of metabolic acidosis and urinary 
output. 

Another problem following revascularisation of an acute ischaemic limb might be 
significant limb swelling. This may result in secondary muscle or nerve injury, 
venous compression, further oedema and compartment syndrome, leading to arter- 
ial compression and secondary ischaemia. To avoid this, the surgeon might prefer 
to perform a fasciotomy in conjunction with the embolectomy procedure [14]. 
Alternatively, the extremity can be assessed immediately and at regular intervals 



112 



Vascular Surgery 




Fig. 11.1. a Embolic occlusion of the left popliteal artery; treatment consisted of percutaneous aspiration 
thromboembolectomy. b Normal patency of the popliteal, anterior tibial and peroneal arteries. 



postoperatively for evolving compartment syndrome. As described in different text- 
books, there are several ways of performing an adequate fasciotomy. The most 
important point here is that all four compartments should be decompressed. 

Although concomitant fasciotomy can be preferable in some cases of prolonged 
acute ischaemia, the more conservative approach might avoid unnecessary fas- 
ciotomy and unaesthetic scars. Since a Fogarty catheter embolectomy can easily be 
carried out under local anaesthesia, this wait-and-see approach eliminates the need 
for systematic general anaesthesia, particularly for patients in a poor general 
condition [15]. 

Despite the fact that the value of postoperative compartmental pressure measure- 
ments has been documented by several teams [16, 17], the decision regarding subse- 
quent fasciotomy is most frequently based upon individual preferences and prior 
clinical experience. [Q5: D] 

Every effort should be made in the postoperative period to minimise the inci- 
dence of recurrent emboli. The patient should be treated with heparin or oral anti- 
coagulants until the source of the embolus has been taken care of. [Q6: B] If 



Arterial Embolism 113 

extensive investigation fails to show any correctable source, then long-term anti- 
coagulation is indicated, except in the case of major contraindications. 



References 

1. Panetta T, Thompson JE, Talkinton CM, Garrett WV, Smith BL. Arterial embolectomy: a 34-year 
experience with 400 cases. Surg Clin North Am 1986;66:339. 

2. Thompson JE, Sigler L, Raut PS, Austin DJ, Patman RD. Arterial embolectomy: a 20-year experience. 
Surgery 1970;67:212-20. 

3. Mills JL, Porter JM. Basic data related to clinical decision making in acute limb ischemia. Ann Vase 
Surg 1991;5:96. 

4. Keating EC, Gross SA, Schlamowitz RA. Mural thrombi in myocardial infarctions. Am J Med 
1983;74:989. 

5. Reber PU, Patel AG, Stauffer E, Muller MF, Do DD, Kniemeyer HW. Mural aortic thrombi: an impor- 
tant cause of peripheral embolization. Vase Surg 1999;30:1084-9. 

6. Abbott WM, Maloney RD, McCabe CC, Lee CE, Wirthlin LS. Arterial embolism: a 44 year perspec- 
tive. Am J Surg 1982;143:460-4. 

7. Beard JD, Nyamekye I, Earnshaw JJ, Scott DJ, Thompson JF. Intraoperative streptokinase: a useful 
adjunct to balloon-catheter embolectomy. Br J Surg 1993;80:21-4. 

8. Heymans S, Vanderschueren S, Verhaeghe R, Stockx L, Lacroix H, Nevelsteen A, et al. Outcome and 
one year follow-up of intra- arterial staphylokinase in 191 patients with peripheral arterial occlusion. 
Thromb Haemost 2000;83:666-71. 

9. Sniderman KW, Kalman PG, Quigley MJ. Percutaneous aspiration embolectomy. J Cardiovasc Surg 
(Torino) 1993;34:255. 

10. Haimovici H. Muscular, renal and metabolic complications of acute arterial occlusions: myonephro- 
pathic-metabolic syndrome. Surgery 1979;85:461. 

11. Fischer RD, Fogarty TJ, Morrow AG. Clinical and biochemical observations of the effect of transient 
femoral artery occlusion in man. Surgery 1970;68:323. 

12. Rubin BB, Walker PM. Pathophysiology of acute skeletal muscle injury: adenine nucleotide metabo- 
lism in ischemic reperfused muscle. Semin Vase Surg 1992;5:11. 

13. Pattwell D, McArdle A, Griffiths RD, Jackson MJ. Measurement of free radical production by in vivo 
microdialysis during ischemia/reperfusion injury to skeletal muscle. Free Radic Biol Med 
2001;30:979-85. 

14. Padberg FT, Hobson RWII. Fasciotomy in acute limb ischemia. Semin Vase Surg 1992;5:52. 

15. Rush DS, Frame SB, Bell RM, Berg EE, Kerstein MD, Haynes JL. Does open fasciotomy contribute to 
morbidity and mortality after acute lower extremity ischemia and revascularization? J Vase Surg 
1989;10:343-50. 

16. Whitesides TE, Heckman MM. Acute compartment syndrome: update on diagnosis and treatment. J 
Am Acad Orthop Surg 1996;4:209-18. 

17. Janzing HMJ. The acute compartment syndrome, a complication of fractures and soft tissue injuries 
of the extremities. A clinical study about diagnosis and treatment of the compartment syndrome. 
Doctoral thesis, Leuven University, 1999. 



1 2. Blast Injury to the Lower Limb 

Paul H. B. Blair, Adrian K. Neill and 
Christopher T. Andrews 



A 40-year-old male was admitted to the emergency room approximately one and 
a half hours after sustaining a blast injury to both lower limbs. He had been 
resuscitated at his local accident and emergency department prior to transfer. On 
arrival, his pulse was 120 bpm and his blood pressure 80/40 mm Hg. 

Examination revealed that the patient had sustained significant blast injuries 
to both lower limbs with no obvious torso injuries. The left leg had sustained 
neurovascular damage above and below the knee with concomitant bone and soft 
tissue injury; there was no tissue perfusion below the knee. On the right side 
there was a large wound in the thigh extending anteriorly to the knee joint with 
profuse bleeding; bony fragments could be seen in the wound and the right foot 
was pale with no palpable pulses and slight reduction in sensation. 



Question 1 

The priorities for the care of this patient include: 

A. Secure an airway, commence oxygen therapy and obtain adequate intravenous 
(IV) access. 

B. Complete a full survey of the patient before transferring for further management. 

C. Wait for blood result before deciding on transfer out of the emergency room. 

D. Transfer the patient to theatre for definitive management during primary 
resuscitation. 

E. Discuss treatment options with relatives. 

Question 2 

Which of the following are "hard" signs of vascular injury? 

A. Limb pain. 

115 



116 Vascular Surgery 

B. Absence of pulses. 

C. Pallor or cyanosis. 

D. Cool to the touch. 

E. Bruit or thrill. 



Question 3 

Which of the following statements relating to angiography are true? 

A. Angiography should be performed in all patients to target surgery. 

B. Angiography may be a useful tool in trauma patients with no hard signs of 
vascular injury. 

C. Angiography is reserved for stable patients. 

D. Angiography should only be performed in a radiology department. 

E. The patient's pre-morbid condition should not influence the decision to 
perform angiography. 

Question 4 

For how long will the lower limb tolerate ischaemia? 

A. 20-30 minutes. 

B. 90-120 minutes. 

C. 6-8 hours. 

D. 16-20 hours. 

E. 24-36 hours. 

The patient was resuscitated as per ATLS (advanced trauma life support) proto- 
col. Supplementary oxygen was administered in addition to obtaining additional IV 
access. Pressure dressings were applied to the open wounds and further assessment 
revealed an injury to the patient's right hand; no other significant injuries were 
present. The patient was transferred to the operating theatre. 

Question 5 

What are the primary aims of surgery in such a case? 

A. To control life-threatening haemorrhage. 

B. To prevent end-organ ischaemia. 

C. To restore vascular continuity. 



Blast Injury to the Lower Limb 117 

D. To preserve limb function. 

E. To detect occult injuries. 

Question 6 

What factors will influence the decision to perform an amputation? 

A. Patient's age. 

B. Mechanism of injury. 

C. Time to treatment. 

D. Degree of contamination. 

E. All of the above. 

Question 7 

Which of the following statements about complex vein repair are true? 

A. Complex vein repair should never be undertaken in the trauma patient. 

B. Complex vein repair should only be performed in the absence of major arterial 
injury. 

C. Complex vein repair should be used to improve venous return in unstable 
patients. 

D. Complex vein repair may prevent long-term limb dysfunction. 

E. Intraluminal venous shunting is an acceptable intraoperative temporising 
measure. 

In the operating theatre, under general anaesthesia, the patient was placed in the 
supine position. The lower abdomen and both legs were prepared and draped 
widely and intravenous broad spectrum antibiotics were administered. Closer 
examination revealed that the left leg had sustained extensive injuries. The foot and 
distal calf were cold, pale and mottled. There was a compound injury to the left 
femur and tibia with complete disruption of the superficial femoral artery, 
superficial femoral vein and extensive injury to the sciatic nerve. It was decided that 
primary amputation of the left limb was required. On examination of the right leg 
there was complete disruption of the distal superficial/popliteal artery, a ragged lac- 
eration of the popliteal vein and significant bruising to branches of the sciatic nerve. 
There was a shrapnel injury to the right hand involving the thumb and middle 
finger. 

Immediate surgical steps were as follows: (i) a proximal thigh tourniquet was 
placed on the left leg to arrest haemorrhage prior to formal amputation. The lacera- 
tion to the right lower leg was then extended distally to facilitate exposure of the 
neurovascular structures. Control of the superficial femoral and below-knee 
popliteal artery was obtained and a careful distal embolectomy performed. A Javid 



118 



Vascular Surgery 




Fig. 12.1. Extended wound, medial aspect of right leg with a temporary intraluminal shunt between superficial 
femoral and below-knee popliteal arteries. 



shunt was then placed between the right superficial femoral artery and right below- 
knee popliteal vessel (Fig. 12.1). Significant bleeding from a large defect in the 
popliteal vein occurred following shunt insertion; this was repaired using a lateral 
suture. The long saphenous vein was harvested from the left leg, prior to perform- 
ing above-knee amputation. While the left above-knee amputation was being per- 
formed, the orthopaedic surgeons carefully assessed the right lower limb and placed 
a temporary fixation device traversing the right knee joint (Fig. 12.2). Having 
obtained bony stability, with an external fixator device, the temporary intraluminal 
shunt was removed and a definitive bypass performed using reversed left long 
saphenous vein graft. Formal fasciotomy was performed of the right lower leg using 
a standard lateral and medial approach; distal pulses were confirmed in the right 
foot. Further debridement of necrotic muscle was performed and the wound on the 
medial aspect was partially closed; the anterolateral wounds were debrided and irri- 
gated, as were the fasciotomy sites, with sterile dressings being applied to both. 




Fig. 12.2. A multidisciplinary approach. Bony stabilisation of right leg (after temporary intraluminal shunt place- 
ment) by the orthopaedic surgeons, simultaneous with left above-knee amputation by the vascular surgeons. 



Blast Injury to the Lower Limb 

Question 8 



119 



In the absence of obvious haemorrhage, when is it appropriate to reinspect the 
wounds in the postoperative period? 

A. 1-2 hours. 

B. 4-6 hours. 

C. 12-16 hours. 

D. 24-48 hours. 

E. 5+ days. 

Postoperatively the patient was transferred to the intensive care unit where the 
right limb was elevated to reduce swelling. The right foot was left exposed to allow 
access for pedal pulses. Broad spectrum IV antibiotics were continued in addition 
to standard prophylaxis for deep vein thrombosis, and urine was checked for myo- 
globinuria. The patient was returned to the operating theatre within 48 hours for 
wound inspection and change of dressing. Eventually skin coverage of the right 
limb was obtained using a combination of split skin grafting and healing by delayed 
primary intention. Over the next few months the patient required complex 
orthopaedic surgery including the use of an Ilizarov frame device (Fig. 12.3). He was 
fitted with an above-knee prosthesis for his left leg and is now fully independent 
(Fig. 12.4). 



Commentary 

Lower limb injuries, due to penetrating trauma, can be devastating and occasionally 
may distract the clinician from less obvious but potentially life-threatening injuries 
to the head, neck and torso. It is important that some form of resuscitation protocol 
is followed such as the ATLS system to detect less obvious injuries. Time is of the 




Fig. 12.3. Recovery. Healed traumatic and fasciotomy wounds after skin grafting; Ilizarov frame still in place. 



120 



Vascular Surgery 




Fig. 12.4. Rehabilitation. An excellent result for limb salvage (right leg) and learning to function with a prosthe- 
sis (left). 



essence when managing vascular injuries. While delays rarely occur in patients with 
obvious haemorrhage, it is the prompt instigation of life-saving measures and 
ongoing diagnosis in parallel with transfer to the operating theatre for definitive 
care that reduces morbidity and mortality. [Q1 : A, D] 

The clinical manifestations of vascular injury have traditionally been divided into 
"hard" and "soft" signs (Table 12.1). [Q2: B, E] 



Table 1 2.1 . Signs of vascular injury. Updated 



Hard signs 



Soft signs 



Absent pulse 

Bruit or thrill 

Haematoma (large or expanding) 

Distal ischaemia 



Haematoma (small) 

History of haemorrhage at scene 

Peripheral nerve deficit 



Blast Injury to the Lower Limb 121 

In general, preoperative arteriography may be used in the following situations: 
(1) to confirm the site and extent of vascular injury in stable patients whose clinical 
signs and symptoms are equivocal; and (2) to exclude vascular injury in patients 
with no hard signs, but who are considered to be at risk because of the proximity of 
the injury. The majority of patients with penetrating extremity trauma and the pres- 
ence of a single hard sign should be transferred directly to the operating theatre. 
Possible exceptions to this rule include stable patients with multiple levels of injury, 
extensive bone or soft tissue injury, blast or shotgun injuries, potential injuries to 
the subclavian or axillary arteries and the pre-existence of peripheral vascular 
disease. Some centres report excellent results with emergency room angiography [1] 
while recent advances in endovascular technique facilitate high-quality imaging in 
the operating theatre. [Q3: B, C] 

Inadequate tissue perfusion due to major vessel disruption is aggravated by 
hypovolaemic shock and associated bone and soft tissue injury. The resulting fall in 
tissue p0 2 increases capillary membrane permeability, with increased exudation of 
fluid into the interstitial space. Compromised muscle fibres swell within the fascial 
compartments, causing further resistance to blood flow, and swelling becomes trau- 
matic when arterial repair and restoration of flow brings about reperfusion injury. 
The degree of reperfusion injury depends on the duration of ischaemia, and is 
mediated by the generation of free radicals, activation of neutrophils, and produc- 
tion of arachidonic acid metabolites. Eventually, the microvascular bed of the 
extremity may undergo widespread thrombosis [2]. It is generally accepted that a 
warm ischaemia time of more than 6-8 hours makes limb survival unlikely. [Q4: C] 
To achieve optimal results from emergency vascular repair, and to avoid complica- 
tions such as compartment syndrome or contracture due to prolonged warm 
ischaemia and reperfusion injury, surgical exploration should be undertaken 
expeditiously. 

A patient with complex lower limb injuries should be placed in a supine position 
on an operating table suitable for on-table angiography, if required, when clinical 
stability has been reached. Some form of warming device should be employed to 
maintain adequate body temperature. In lower limb trauma, both limbs should be 
prepared from umbilicus to toes; donor saphenous vein harvesting maybe required 
from the contralateral limb, particularly if ipsilateral venous injury is suspected. 
Careful attention should be given to correct hypothermia, blood loss, electrolyte 
imbalance and coagulopathy. 

The principal aims of emergency vascular surgery are to control life-threatening 
haemorrhage and prevent end-organ ischaemia. [Q5: A, B] An assistant should 
control haemorrhage using a pressure dressing until the patient is prepared and 
draped appropriately. Haemorrhage control can be difficult if the proximal vessels 
are not immediately apparent, and the use of a cephalad incision through virgin ter- 
ritory may be a reasonable alternative to obtain rapid proximal control. Care should 
be taken when making additional incisions, particularly if it seems likely that plastic 
surgery will be required at a later date. When access to the proximal or distal vessel 
is difficult, temporary control can be gained by careful cannulation and inflation of 
an embolectomy catheter. It is important that the surgeon cooperates fully with the 
anaesthetist during surgery as it may be necessary to pack the wound for a few 
minutes to facilitate intravenous fluid resuscitation before proximal vascular 
control can be obtained. Complex lengthy operations should be avoided in unstable 
patients and damage limitation surgery should be considered in patients with 
significant metabolic acidosis, coagulopathy and/or hypothermia. 



122 Vascular Surgery 

The use of a temporary intraluminal vascular shunt should be considered in the 
majority of limb vascular injuries and is particularly important in complex cases 
with associated bone and soft tissue injury. 

Temporary shunts for arterial and venous injuries have been employed in Belfast 
since the late 1970s [2]. A considerable body of evidence continues to support the 
use of these intravascular shunts in the management of both penetrating and blunt 
major vascular trauma [3-6]. Before securing the shunt between the proximal and 
distal arteries, a careful embolectomy should be performed to remove any throm- 
bus in the distal vessel. If a venous injury is encountered, then an additional shunt 
can be employed to facilitate venous return. In the absence of coagulopathy or 
ongoing haemorrhage we use intravenous heparin routinely. Recent evidence has 
shown clearly that delayed renewal of venous flow in combined arterial and venous 
injury compounds ischaemia-reperfusion injury and causes remote lung injury [7]. 
The advantages of shunting artery and vein are the early restoration of blood flow 
and venous return, respectively, thus avoiding the complications of prolonged 
ischaemia and ischaemia-reperfusion injury while ensuring that an optimal vascu- 
lar repair can be performed. 

In patients with concomitant fractures, accurate internal or external fixation of 
the fracture can be performed with the shunt secured carefully with sloops before 
definitive vascular repair is performed. This avoids the dilemma of unnecessary 
haste for both the orthopaedic and vascular surgeons, ensures that a vein graft will 
be of optimal length, and eliminates the risk of graft disruption during fracture 
manipulation. Autologous vein is our preferred bypass conduit in the majority of 
cases because of its durability and suitability in a potentially contaminated wound. 
Satisfactory results, however, have been reported using synthetic grafts and in criti- 
cally ill, unstable patients this maybe a preferable option [8]. 

The acute management of high energy limb trauma can be challenging and 
significant morbidity and mortality can occur following failed attempts at limb salvage. 
A number of scoring systems have been devised in an attempt to assist the clinician's 
decision to either amputate or perform a limb-salvage procedure [9-13]. In each of the 
systems, a score is assigned based on a range of differing criteria including patient age, 
"mechanism of injury", time to treatment, degree of shock, warm ischaemia time and 
the presence of local injuries to the following structures: major artery, major vein, 
bone, muscle, nerve, skin, and degree of contamination. [Q6: E] All of these scoring 
systems demonstrate a much higher degree of specificity than sensitivity and are more 
useful in highlighting the patients who should be considered for a limb-salvage proce- 
dure, than identifying those who should proceed straight to primary amputation. 
Indeed a number of studies have challenged their use at all [14, 15]. 

It is the authors' opinion that scoring systems can help the surgeon perform a 
detailed assessment of a complex limb injury. However, the decision to perform a 
primary amputation must be judged individually in each case. Extensive nerve 
injuries have a particularly poor prognosis and it is important that such injuries, 
where possible, are documented before taking the patient to the operating theatre. 
The patient's life should never be put at risk in a futile attempt to save a severely 
compromised limb. Where possible, additional specialties such as orthopaedics and 
plastic surgery should be involved in the decision to perform a primary limb ampu- 
tation, particularly in a case of upper limb trauma. 

Venous injuries can be difficult to manage. Prior to World War II, the traditional 
treatment for lower extremity venous injuries was ligation. This custom was chal- 
lenged by Debakey & Simeone [16] in 1946 with an analysis of WWII battle injuries. 



Blast Injury to the Lower Limb 123 

Since then a number of clinical and laboratory investigations have confirmed that 
ligation of major veins in conjunction with repair of a traumatically injured arterial 
system leads to significantly poorer clinical outcomes, such as decreased function or 
even limb loss [17, 18]. Where possible vein repair should be attempted, particularly 
in the presence of significant lower limb arterial injury, in an attempt to reduce 
venous hypertension and associated morbidity. While there are few data regarding 
the long-term outcome of venous repairs, it is the authors' impression that main- 
taining venous patency, in the initial few days after injury, can significantly help 
reduce acute post-injury swelling. If the superficial femoral vein requires ligation, it 
is important to maintain patency of the ipsilateral long saphenous and profunda 
femoris veins. Complex vein repair should never be attempted in unstable patients 
who have sustained major blood loss and have significant problems with hypother- 
mia and coagulopathy. In more stable patients, however, temporary intraluminal 
venous shunting can facilitate the construction of larger calibre panel grafts 
obtained from the contralateral long saphenous vein. [Q7: D, E] 

Postoperative management of patients with complex limb injuries is critically 
important. The majority of these patients have been transferred immediately to the 
operating theatre and it is important that a thorough search for occult injuries is per- 
formed on admission to the intensive care unit. These patients are at risk of develop- 
ing multiple organ dysfunction syndrome as a result of their large transfusion 
requirements and likely reperfusion injury sustained [19, 20]. It is important that the 
vascular surgeon communicates clearly with the staff in the intensive care unit 
regarding the presence or absence of distal pulses, to ensure that vascular repair 
remains patent. Young trauma patients with normal blood pressure and temperature 
should have a palpable distal pulse. If there is any doubt regarding the integrity of the 
vascular repair, the dressings should be removed and a careful assessment performed 
by a vascular surgeon using handheld Doppler and/or portable ultrasound device. 

Wounds should be reinspected 24-48 hours after initial surgery and at that stage 
definitive plastic surgery may be required to obtain soft tissue and skin cover. 
[Q8: D] Some centres advocate a selective policy with regard to fasciotomy based on 
compartmental pressures, while many continue to advocate a more liberal policy 
based on clinical grounds. Prolonged ischaemia time, combined arteriovenous 
injuries, complex injuries including bone and soft tissue destruction and crush 
injuries remain absolute indications for fasciotomy. The avoidance of compartment 
syndrome and restoration of limb function far outweigh the low morbidity associ- 
ated with liberal use of fasciotomy. These patients are at significant risk of wound 
and other nosocomial infections and prolonged antibiotic use may be required. 

The management of patients with complex injuries can be difficult; however, timely 
surgery and the involvement of a multidisciplinary team can produce rewarding 
results. One possible criticism of the above care could be failure to use the great toe, 
from the amputated left lower limb, to replace the patient's right thumb. 



References 



1. Itani KM, Burch JM, Spjut-Patrinely V, Richardson R, Martin RR, Mattox KL. Emergency center arte- 
riography. J Trauma 1992;32(3):302-6; discussion 306-7. 

2. Barros D'Sa AA. How do we manage acute limb ischaemia due to trauma? In: Greenhalgh RM, 
Jamieson CW, Nicolaides AN, editors. Limb salvage and amputation for vascular disease. London: 
WB Saunders, 1998. 



124 Vascular Surgery 

3. D'Sa AA. A decade of missile-induced vascular trauma. Ann R Coll Surg Engl 1982;64(l):37-44. 

4. Elliot J, Templeton J, Barros D'Sa AA. Combined bony and vascular trauma: a new approach to treat- 
ment. J Bone Joint Surg Am 1984;66B:281. 

5. Barros D'Sa AA. The rationale for arterial and venous shunting in the management of limb vascular 
injuries. Eur J Vase Surg 1989;3(6):471-4. 

6. Barros D'Sa AA, Moorehead RJ. Combined arterial and venous intraluminal shunting in major 
trauma of the lower limb. Eur J Vase Surg 1989;3(6):577-81. 

7. Harkin DW, D'Sa AA, Yassin MM, Young IS, McEneny J, McMaster D, et al. Reperfusion injury is 
greater with delayed restoration of venous outflow in concurrent arterial and venous limb injury. Br 
JSurg2000;87(6):734-41. 

8. Lovric Z, Lehner V, Kosic-Lovric L, Wertheimer B. Reconstruction of major arteries of lower extrem- 
ities after war injuries. Long-term follow up. J Cardiovasc Surg (Torino) 1996;37(3):223-7. 

9. Howe HR, Jr, Poole GV, Jr, Hansen KJ, Clark T, Plonk GW, Koman LA, et al. Salvage of lower 
extremities following combined orthopedic and vascular trauma. A predictive salvage index. Am 
Surg 1987;53(4):205-8. 

10. Johansen K, Daines M, Howey T, Helfet D, Hansen ST, Jr. Objective criteria accurately predict ampu- 
tation following lower extremity trauma. J Trauma 1990;30(5):568-72; discussion 572-3. 

11. Helfet DL, Howey T, Sanders R, Johansen K. Limb salvage versus amputation. Preliminary results of 
the Mangled Extremity Severity Score. Clin Orthop Relat Res 1990(256):80-6. 

12. Russell WL, Sailors DM, Whittle TB, Fisher DF, Jr, Burns RP. Limb salvage versus traumatic amputa- 
tion. A decision based on a seven-part predictive index. Ann Surg 1991;213(5):473-80; discussion 
480-1. 

13. McNamara MG, Heckman JD, Corley FG. Severe open fractures of the lower extremity: a retrospec- 
tive evaluation of the Mangled Extremity Severity Score (MESS). J Orthop Trauma 1994;8(2):81-7. 

14. Bonanni F, Rhodes M, Lucke JF. The futility of predictive scoring of mangled lower extremities. J 
Trauma 1993;34(1):99-104. 

15. Durham RM, Mistry BM, Mazuski JE, Shapiro M, Jacobs D. Outcome and utility of scoring systems 
in the management of the mangled extremity. Am J Surg 1996;172(5):569-73; discussion 573-4. 

16. Debakey ME, Simeone FA. Battle injuries of arteries in World War II: analysis of 2471 cases. Ann 
Surg 1946;123:534-79. 

17. Nanobashvili J, Kopadze T, Tvaladze M, Buachidze T, Nazvlishvili G. War injuries of major extrem- 
ity arteries. World J Surg 2003;27(2):134-9. 

18. Kuralay E, Demirkilic U, Ozal E, Oz BS, Cingoz F, Gunay C, et al. A quantitative approach to lower 
extremity vein repair. J Vase Surg 2002;36(6):1213-8. 

19. Defraigne JO, Pincemail J. Local and systemic consequences of severe ischemia and reperfusion of 
the skeletal muscle. Physiopathology and prevention. Acta Chir Belg 1998;98(4): 176-86. 

20. Foex BA. Systemic responses to trauma. Br Med Bull 1999;55(4):726-43. 



13. Endoluminal Treatment of Traumatic 
Arteriovenous Fistula of the Axillary 
Artery 

Jonathan D. Woody and Rodney A. White 



A 21 -year-old male sustained a gunshot wound to the anterior right chest. The 
entrance wound was at the midclavicular line. He was hemodynamically stable 
upon arrival at the emergency department. He underwent initial resuscitation 
with intravenous fluids. There was no evidence of hemothorax or pneumothorax 
on chest X-ray. The right radial pulse was present but diminished when com- 
pared with the left. The patient had minor neurological symptoms in the right 
upper extremity, which consisted mainly of weakness of the interosseous muscles 
of the hand. An arteriogram was obtained, which revealed an arteriovenous 
fistula (AVF) of the right axillary artery and vein (Fig. 13.1a). The patient 
remained hemodynamically stable throughout the evaluation period. 



Question 1 

Acquired AVF is most commonly the result of: 

A. Penetrating trauma. 

B. Percutaneous puncture. 

C. Erosion of arterial aneurysm. 

D. Periarterial abscess. 

E. Neoplasm. 



Question 2 

Which of the following are possible complications of AVF? 

A. Bacterial endarteritis at the site of AVF. 

B. Peripheral arterial insufficiency. 

125 



126 



Vascular Surgery 




3 JUN,3S 11: 13: 33 



067-76-35 



? CM ? . 5^ 5. D -S PHR 
FROZEN EMI 
HALL 4QD PR-L54 55* 
GATE 2-SCM 




(iii) 



UiyfcUUl , 



Fig. 13.1. a Pre-treatment and b post-treatment images of an AVF of the right axillary artery after a gunshot 
wound, i Arteriography, ii intravascular ultrasound, and iii duplex grey-scale ultrasound demonstrate the AVF 
before treatment and the complete exclusion of the fistula after treatment. (Reprinted from J Vase Surg, vol. 24, 
White RA, Donayre CE, Walot I, et al., Preliminary clinical outcome and imaging criterion for endovascular pros- 
thesis development in high-risk patients who have aortoiliac and traumatic arterial lesions, pages 569-571, © 
1 996, with permission from The Society for Vascular Surgery.) 



C. Venous congestion, venous valvular insufficiency, venous stasis, venous vari- 
cosities and edema formation. 

D. Tachycardia. 

E. Cardiomegaly. 

F. Congestive heart failure. 



Endoluminal Treatment of Traumatic Arteriovenous Fistula of the Axillary Artery 



127 




: 



4 



>> 



(i) 



3 



* fli*n* .km or 
■ BT"H» . 71? at 






>u- 






(ii) 




(iii) 



Fig. 13.1. (continued) 



Question 3 

What is the gold standard for the diagnosis of AVF? 

A. Magnetic resonance imaging (MRI)/magnetic resonance angiography (MRA), 

B. Computed tomography (CT). 

C. Color-flow duplex ultrasound. 

D. Arteriography. 



128 Vascular Surgery 

Treatment 

The patient was transferred to the endovascular operating suite and placed under 
general anesthesia. He was positioned on the interventional table with his right arm 
on an arm board. A surface ultrasound was performed, which confirmed the location 
of the AVF. The right arm and chest were prepared into a sterile field, and the 
abdomen and upper legs were prepared into a separate sterile field. The right 
brachial artery and vein were surgically exposed and isolated. A sheath was inserted 
percutaneously into the right femoral artery. Under fluoroscopic guidance, a 
guidewire was passed through the femoral sheath into the right axillary artery. It was 
then passed across the injured segment of the right axillary artery and into the right 
brachial artery to the surgical site. The wire was removed through a small transverse 
arteriotomy, and a hemostatic sheath was inserted into the artery over the wire. 

The length of the axillary artery was interrogated with intravascular ultrasound. 
The diameter of the axillary artery was 7 mm. A 4-cm segment of deep brachial vein 
was obtained and sutured to an appropriately sized Palmaz stent. This was crimped 
onto an expandable balloon and, under fluoroscopic guidance, passed over the 
guidewire to the site of injury. The balloon was expanded, and the endoluminal 
device was deployed. The balloon was deflated and removed. The axillary artery was 
again interrogated with intravascular ultrasound, which demonstrated excellent 
apposition between the stent and the arterial wall. Complete exclusion of the fistula 
was documented with both intravascular ultrasound and arteriography (Fig. 13.1b). 
Flow was then restored to the arm. The arm wound was closed surgically, and pres- 
sure was applied to the femoral puncture site. Before awakening the patient, surface 
ultrasound confirmed the closure of the fistula and normal flow through the axillary 
artery. 



Question 4 

Which of the following is not considered to be one of the core principles of treat- 
ment of AVF? 

A. Complete closure of the fistula. 

B. Restoration of normal arterial flow. 

C. Ligation of the arterial inflow 

D. Restoration of normal venous flow. 



Commentary 

AVF is an abnormal communication between an artery and vein. By far the most 
common cause of acquired AVF is trauma. This is usually the result of a penetrating 
injury secondary to a knife or bullet, but it can also result from blunt trauma. 
Iatrogenic injury from percutaneous puncture can also cause AVF. In addition, AVF 
can be caused by erosion of an arterial aneurysm into an adjacent vein, by a peri- 
arterial abscess or by a neoplasm, although these are rare. [Q1: A] 



Endoluminal Treatment of Traumatic Arteriovenous Fistula of the Axillary Artery 129 

Potential complications of AVF can be considered on both a peripheral and a sys- 
temic level. Peripheral effects may include peripheral arterial insufficiency, dilated 
and thickened proximal arterial wall (predisposing to infection, i.e. bacterial endar- 
teritis), venous congestion, venous valvular insufficiency, venous stasis, varicosities, 
edema, and skin and soft tissue changes. Systemic effects may include increased 
cardiac output, tachycardia, cardiomegaly, congestive heart failure, increased pulse 
pressure, decreased diastolic pressure, increased blood volume, increased central 
venous pressure, and increased stroke volume. [Q2: A, B, C, D, E, F] 

The diagnosis of AVF is relatively straightforward. An audible bruit and palpable 
thrill are often found, especially in peripheral lesions. The gold standard for diagno- 
sis is arteriography. The most important point is to delineate clearly the anatomy of 
the fistula so that treatment may be planned. Other diagnostic modalities include 
color-flow duplex ultrasound, CT, and MRI and/or MRA. [Q3: D] 

Treatment of AVF is best carried out early rather than late. The longer an AVF is 
present, the more likely the patient will be to develop complications. Further, the 
longer it exists, the more likely it is to develop significant collateral flow, which 
increases the complexity of the repair. The principles of repair, whether conven- 
tional or endovascular, are the same: complete closure of the fistula and restoration 
of normal arterial and venous flow. [Q4: C] Conventional repair can be undertaken in 
a number of ways but will not be described here since conventional repair is not the 
focus of this chapter. 

The first report of successful endovascular repair of an arterial injury was by 
Becker et al. [1]. Further reports by Parodi and Barone [2], Marin et al. [3] and May 
et al. [4] have demonstrated a high rate of technical success with persistent fistula 
closure, excellent long-term patency, and a low incidence of complications. 
Endovascular treatment of AVF is ideal because most of these injuries are localized 
to a small segment of artery with normal vessel wall on each end. Endovascular 
treatment of these lesions also avoids the major morbidity of conventional open 
repair, which can be significant. Endovascular access is achieved at a site remote 
from the area of injury and can be performed percutaneously in many cases. There 
is little damage to surrounding structures since only the affected vessel is manipu- 
lated. If endovascular treatment is successful, then the benefits for the patient 
include a shorter hospitalization period and a quicker return to normal activity. The 
only major concern is long-term patency and function of the endoluminal stent 
graft, since these studies are not yet available. However, due to the relatively short 
length of the stent graft, its placement in large-diameter vessels, and the high flow 
rates in these vessels, it is reasonable to assume that the long-term patency would be 
good [5]. It is of note that our patient was hemodynamically stable. Although 
endovascular technology continues to evolve and improve, it must be emphasized 
that a hemodynamically unstable patient who has sustained penetrating trauma 
should undergo immediate open surgical exploration. 



References 



1. Becker GJ, Benenati JF, Zemel G, Sallee DS, Suarez CA, Roeren TK, Katzen BT. Percutaneous place- 
ment of a balloon-expandable intraluminal graft for life-threatening subclavian arterial hemorrhage. 
J Vase Interv Radiol 1991;2:225-9. 

2. Parodi JC, Barone HD. Transluminal treatment of abdominal aortic aneurysms and peripheral arteri- 
ovenous fistulas. Paper presented at the 19th Annual Montefiore Medical Center/ Albert Einstein 



130 Vascular Surgery 

College of Medicine Symposium on Current Critical Problems and New Technologies in Vascular 
Surgery, New York, November 1992. 

3. Marin ML, Veith FJ, Panetta TF, Cynamon J, Barone H, Schonholz C, Parodi JC. Percutaneous trans- 
femoral insertion of a stented graft to repair a traumatic femoral arteriovenous fistula. J Vase Surg 
1993;18:299-302. 

4. May J, White G, Waugh R, Yu W, Harris J. Transluminal placement of a prosthetic graft-stent device 
for the treatment of a subclavian artery aneurysm. J Vase Surg 1993;18:1056-9. 

5. Donayre CE. Endovascular treatment of traumatic arteriovenous fistulas and pseudoaneurysms and of 
arterial occlusive disease. In: White RA, Fogarty TJ, editors. Peripheral endovascular interventions, 
2nd edn. New York: Springer, 1999;371-81. 



14. Cardiovascular Risk Factors and 
Peripheral Arterial Disease 

Stella S. Daskalopoulou and Dimitri P. Mikhailidis 



A 62-year-old man with intermittent claudication was referred for vascular risk 
factor modification. He had no history of myocardial infarction (MI) or stroke. 
He was smoking 20 cigarettes/day. His family history was negative for premature 
vascular events. He was not taking any medication. He was advised to start 
aspirin 75 mg/day, but he stopped taking these tablets because of "stomach dis- 
comfort". The patient's total cholesterol was 228 mg/dl (5.9 mmol/1). 

His blood pressure required treatment with amlodipine and a thiazide 
diuretic. The patient eventually stopped smoking after referral to the smoking 
cessation clinic in our hospital. 



Question 1 

Which of the following investigations would you order? 

A. Fasting serum glucose. 

B. Urine glucose to make a diagnosis of diabetes mellitus. 

C. Fasting serum triglycerides. 

D. Fasting serum high-density lipoprotein cholesterol (HDL-C). 

E. Thyroid function tests. 

A. Requesting a fasting serum glucose level is an essential test in all patients with 
vascular disease. In this case the fasting glucose was 87 mg/dl (4.8 mmol/1); this is 
satisfactory. 

Interpretation of fasting glucose values: 
There are three categories in which a patient can be placed relative to fasting serum 
glucose levels: 

• Normal: fasting glucose =110 mg/dl (=6.0 mmol/1). 



133 



134 Vascular Surgery 

Table 14.1. Features of metabolic syndrome*. Updated 



1 . Abdominal obesity (waist circumference): 

Men>102cm 
Women >88 cm 

2. Triglycerides 

=150mg/dl(=1.7mmol/l) 

3. High-density lipoprotein cholesterol (HDL-C): 

Men<40mg/dl(<1.0mmol/l) 
Women <50 mg/dl (<1 .3 mmol/l) 

4. Blood pressure: <130/<85 mm Hg 

5. Fasting glucose: =] 10 mg/dl (=6.0 mmol/l) 



• According to the National Cholesterol Education Program (NCEP) Adult Treatment Panel (ATP) III guidelines [1 ], any 3 or more 
of these 5 features are diagnostic of the metabolic syndrome. Other factors that may coexist in these patients include a family 
history of type 2 diabetes, South Asian ethnicity, decreased physical activity, smoking, elevated serum urate levels and 
evidence of fatty liver (abnormal levels of aminotransferases, ALT/AST). 

There is evidence (post-hoc analysis) from the Scandinavian Simvastatin Survival Study (4S) that IFG and diabetic patients 
benefit from treatment with simvastatin [2]. More recently, a trial in type 2 diabetic patients without established vascular 
disease showed a beneficial effect of atorvastatin 1 mg/day (vs. placebo) in reducing the risk of first cardiovascular events, 
including stroke [3]. 

Both diabetes and metabolic syndrome are common in patients with peripheral arterial disease (PAD) [4]. Furthermore, 
both diabetes and PAD are considered as coronary heart disease (CHD) equivalent and need to be treated aggressively [1], 

• Impaired fasting glucose (IFG): fasting glucose 110-125 mg/dl (6.0-6.9 mmol/l). 

• Diabetes mellitus: fasting glucose =126 mg/dl (=7.0 mmol/l). 

IFG is associated with an increased risk of vascular events and conversion to dia- 
betes mellitus. Furthermore, a glucose level in the IFG range can be one of the fea- 
tures of the metabolic syndrome (also known as insulin resistance or Reaven's 
syndrome) [1] (Table 14.1). 

B. This patient's urine was tested when he was first seen in outpatients. The renal 
threshold for glucose is a serum level of about 180-200 mg/dl (10-11 mmol/l). 
Therefore, testing urine for glucose will not detect IFG or early/mild diabetes. 
Clinicians must not rely on a urine glucose test to exclude IFG or early diabetes. In 
view of the serum glucose value (see A, above), it is not surprising that the urine 
glucose test was negative. However, testing the urine was an opportunity to exclude 
proteinuria, another indicator of vascular risk. 

C. The fasting triglyceride level in this patient was 141 mg/dl (1.6 mmol/l) - this is 
satisfactory. 

Interpretation of fasting triglyceride values: 
There has been considerable confusion regarding the importance of triglycerides. 
There are several reasons for this, including: 

• Interactions with other lipid variables: serum triglyceride and HDL-C levels are 
inversely related. HDL-C is a "protective" lipoprotein. 

• Interactions with potential risk factors: elevated serum triglyceride levels are 
associated with impaired fibrinolysis and possibly elevated plasma levels of 
fibrinogen. Both type 2 diabetes and metabolic syndrome are associated with 
raised serum triglyceride levels. 



Cardiovascular Risk Factors and Peripheral Arterial Disease 1 35 

Table 14.2. Secondary causes of hypertriglyceridaemia/hypercholesterolaemia. Updated 



Excessive alcohol intake 

Diabetes mellitus 

Hypothyroidism 

Some types of liver disease 

Some types of renal disease 

Obesity/diet 

Drugs: beta-blockers, thiazides, oestrogens, anabolic steroids, corticosteroids, tamoxifen, protease 

inhibitors, retinoids, ciclosporin 



• Triglyceride levels vary considerably within any one individual: this variability 
includes the fact that fasting triglycerides may be considerably lower than non- 
fasting levels in some patients. There is evidence that postprandial triglyceride 
levels also predict vascular risk, but this measurement is not easily standardised. 
Therefore, assessment of triglyceride status is best represented by a fasting 
sample (14-h overnight fast; water only allowed). 

The current opinion is that fasting serum triglyceride levels are independent vas- 
cular risk factors [5]. Hypertriglyceridaemia is often associated with secondary 
causes that aggravate the patient's tendency to this type of dyslipidaemia (Table 
14.2). These causes need to be addressed. 

Fasting triglyceride levels are defined in the NCEP ATP III guidelines [1]: 

• Borderline high: 150-199 mg/dl (1.7-2.2 mmol/1). 

• Moderately elevated: 200-499 mg/dl (2.3-5.6 mmol/1). 

• Severe hypertriglyceridaemia: =500 mg/dl (=5.6 mmol/1). 

According to these guidelines [1], the treatment priority for cases with severe 
hypertriglyceridaemia shifts from LDL-C to the triglyceride levels. This is because of 
the increased risk of acute pancreatitis associated with severe hypertriglyceridaemia 
[1]. For milder hypertriglyceridaemia, the priority for treatment remains the LDL-C 
level [1]. 

D. The fasting HDL-C level in this patient was 46 mg/dl (1.2 mmol/1) - this is satis- 
factory. 

Interpretation of fasting HDL-C values: 
A raised HDL-C level is a protective factor, whatever the levels of other lipid vari- 
ables [1, 6, 7]. The recent NCEP ATP III guidelines [1] recommend that HDL-C 
levels should ideally be =40 mg/dl (=1.0 mmol/1) [1, 6, 7]. A low HDL-C level is also 
predictive of the risk of stroke [8, 9]. The importance of HDL-C in reducing the risk 
of vascular events is supported by the findings of a secondary prevention trial 
(VA-HIT) [9]. 

E. The thyroid function tests were normal. 

It is useful to routinely assess thyroid function in dyslipidaemic patients. This is 
because hypothyroidism is not uncommon and it is associated with dyslipidaemia 
(see Table 14.2). There is also some evidence showing that hypothyroid patients are 



136 Vascular Surgery 

Table 14.3. CHD equivalents according to the NCEP ATP III guidelines [1]. Updated 



Peripheral arterial disease 

Abdominal aortic aneurysm 

Symptomatic carotid artery disease 

Diabetes mellitus 

Multiple risk factors conferring a calculated risk for a vascular event >20 per cent over the next 1 years 



more likely to have "muscle-related" side effects if they are given a statin. 
Hypothyroidism can also be difficult to spot unless the clinical features are obvious. 
Replacement with thyroxine is usually associated with a beneficial change in the 
lipid profile and body weight. 



Question 2 

What drug would you use to treat this patient's dyslipidaemia? What are your target 
levels? 

The main target for lipid-lowering treatment is the LDL-C level. Since PAD is 
considered a coronary heart disease (CHD) equivalent [1] (Table 14.3), the LDL-C 
target is 100 mg/dl (2.6 mmol/1) in the USA [1] and 96 mg/dl (2.5 mmol/1) in Europe 
[10]. The NCEP ATP III guidelines were revised in 2004 to include an optional LDL- 
C target of 70 mg/dl (1.8 mmol/1) for very high-risk patients [11]. 

As explained above, the HDL-C and triglyceride levels are secondary targets. 

A full fasting lipid profile should be obtained before making any decision regard- 
ing treatment. In the case presented above, the fasting values were: total cholesterol 
= 228 mg/dl (5.9 mmol/1), HDL-C = 46 mg/dl (1.2 mmol/1), LDL-C = 155 mg/dl 
(4.0 mmol/1) and triglycerides = 141 mg/dl (1.6 mmol/1). 

The drug of choice is a statin to achieve the LDL-C target. Statins also improve 
HDL-C and triglyceride levels, although these latter effects maybe small. 

Three statins have extensive trial-based evidence in terms of reduced mortality: 
atorvastatin, pravastatin and simvastatin. There is also evidence that treatment with 
statins decreases morbidity and mortality and improves symptoms in patients with 
PAD [4,12]. 

There is convincing evidence that statins reduce the risk of stroke [12-14]. 
Several studies have also shown that aggressive lipid lowering is associated with a 
reduced progression of atherosclerotic carotid artery disease [13, 14]. Patients with 
PAD are likely to have some degree of carotid artery disease. PAD is also a strong 
predictor of the risk of stroke. 



Question 3 

What modifiable risk factors would you like to address in a high-risk patient, as in 
this case? 



Cardiovascular Risk Factors and Peripheral Arterial Disease 1 37 

Smoking 

Smoking cessation is of paramount importance. The vast majority of PAD patients 
are, or have been, smokers. Furthermore, smoking is associated with adverse effects 
on several variables that predict vascular events. For example, smoking can lower 
serum HDL-C levels, raise serum triglyceride levels, increase insulin resistance and 
elevate plasma fibrinogen concentrations [15]. Plasma fibrinogen is an independent 
predictor of the risk of MI and stroke. Smoking may even predict the progression of 
PAD and graft occlusion after infrainguinal bypass surgery [16]. There is evidence 
that the vascular risk is greater in smokers than in non-smokers, despite the use of 
statins [17]. 

There is a need to establish smoking cessation clinics where specialist care can be 
delivered. All clinicians should try to motivate patients to quit by spending a few 
minutes explaining why smoking is harmful to them. In PAD, quitting may improve 
claudication and reduce the risk of vascular events. 

Antiplatelet Agents 

This patient could not tolerate aspirin. It is estimated that this problem arises in 
10-15 per cent of patients who are prescribed aspirin. There are several alternatives: 

• "Cover" aspirin with a proton pump inhibitor (e.g. omeprazole). 

• Eradicate Helicobacter pylori infection, if present. 

• Use clopidogrel: the effectiveness of clopidogrel is based on the findings of 
major trials (e.g. CAPRIE, CREDO and CURE), but there is no study specifically 
designed to assess the effectiveness of this drug in PAD [18]. However, patients 
with PAD had significantly fewer events on clopidogrel than on aspirin in the 
CAPRIE trial. Unfortunately, this conclusion is limited by the fact that patient 
subgroup analysis was not included in the original trial protocol [18]. 

Due to his intolerance of aspirin, this patient was prescribed clopidogrel 
75 mg/day. He tolerated this antiplatelet agent without any problems. 

Blood Pressure 

Strict control of blood pressure in high risk patients is essential [19]. In order to 
achieve this objective, there may be a need to use several antihypertensive drugs. 
Some general recommendations are appropriate: 

• Several experts suggest that angiotensin-converting enzyme (ACE) inhibitors 
and angiotensin II receptor blockers should be avoided or used with caution in 
PAD because these patients may have renal artery stenosis. If an ACE inhibitor 
or angiotensin II receptor blocker is used, the plasma creatinine concentration 
should be monitored soon after starting treatment. 

• There is still some debate as to whether beta-blockers adversely affect the circu- 
lation in the lower limbs of patients with PAD. It would appear reasonable, 
however, to use a beta-blocker in post-MI patients with PAD. 



138 Vascular Surgery 

• Some blood pressure drugs exert beneficial or adverse effects on lipid levels, 
haemostatic factors and perhaps more importantly, the long-term risk of devel- 
oping diabetes. 

Glucose Status 

This topic was discussed above. It is also important to note that if the patient is dia- 
betic, the blood pressure targets become stricter, especially if proteinuria is present. 

Lipids 

This topic has been discussed above. 

Emerging Risk Factors 

These factors [1,4] include: 

• Lipoprotein (a) (Lp(a)): there is evidence that Lp(a) is a marker of vascular risk, 
especially in patients with a raised serum LDL-C. Raised Lp(a) levels may also 
predict the risk of restenosis after surgery for PAD [16]. Serum Lp(a) levels are 
difficult to lower, but the risk associated with this abnormality may decrease if 
the LDL-C level is markedly reduced. Correcting hypothyroidism is associated 
with a fall in serum Lp(a) levels. Similarly, postmenopausal hormone replace- 
ment therapy (HRT) may reduce serum Lp(a) concentrations. There are, as yet, 
no intervention trials to show that lowering serum Lp(a) levels (e.g. by using 
high doses of nicotinic acid) is associated with fewer vascular events. 

• Homocysteine: raised plasma levels of homocysteine are thought to predict vas- 
cular risk possibly by acting synergistically with established risk factors. The 
link between homocysteine and PAD appears to be stronger than with CHD [4]. 
There are, as yet, no intervention trials to show that lowering plasma homocys- 
teine levels (e.g. by folic acid, vitamin B12 or B6 supplements) is associated with 
a reduced risk of vascular events. 

• Haemostatic and fibrinolytic factors: there is strong evidence showing that the 
plasma fibrinogen concentration is an independent predictor of vascular risk. 
The levels of this coagulation factor also predict the progression of PAD and 
possibly the risk of restenosis following bypass surgery [16]. Plasma fibrinogen 
levels can be lowered by some fibrates used to treat dyslipidaemia. However, as 
with other emerging risk factors, no trial-based evidence is available to show 
that lowering fibrinogen levels is associated with a decreased risk of vascular 
events. There is less evidence linking fibrinolytic factors with vascular risk. 

• Markers of inflammation (e.g. C-reactive protein, CRP): serum CRP levels 
predict the risk of a vascular event even when there is no vascular disease 
present or when lipid levels are "normal". We do not know whether CRP just 
reflects the inflammatory component of atherosclerosis or whether it is actually 
involved in its pathogenesis. Statins and fibrates lower serum levels of CRP [20]. 
Recent evidence suggests that we should also consider CRP levels (in the high 
sensitivity range) as a target for treatment [21]. 



Cardiovascular Risk Factors and Peripheral Arterial Disease 1 39 

Question 4 

Is it relevant to monitor renal function in this patient? 

Yes, because about 33 per cent of PAD patients have atherosclerotic renal artery 
stenosis [4]. It is therefore important to consider the presence of this condition, 
especially if the renal function tests are abnormal. There is evidence that renal and 
vascular disease progress in parallel [22]. Increased plasma creatinine levels are 
associated with a higher risk of vascular events, even if these values are in the upper 
end of the reference range. There is evidence that statins exert a renoprotective 
action in patients with CHD or PAD [23, 24]. Impaired renal function may con- 
tribute to hyperuricaemia and hyperhomocysteinaemia [25]. These variables may 
predict increased vascular risk. 



References 



1. Expert panel on detection evaluation, and treatment of high blood cholesterol in adults. Executive 
summary of the third report of the National Cholesterol Education Program (NCEP) expert panel on 
detection, evaluation, and treatment of high blood cholesterol in adults (Adult Treatment Panel III). 
JAMA 2001;285:2486-97. 

2. Haffner SM, Alexander CM, Cook TJ, et al. Reduced coronary events in simvastatin-treated patients 
with coronary heart disease and diabetes or impaired fasting glucose levels. Subgroup analyses in the 
Scandinavian Simvastatin Survival Study. Arch Intern Med 1999;159:2661-7. 

3. Colhoun HM, Betteridge DJ, Durrington PN, et al. CARDS investigators. Primary prevention of car- 
diovascular disease with atorvastatin in type 2 diabetes in the Collaborative Atorvastatin Diabetes 
Study (CARDS): multicentre randomised placebo-controlled trial. Lancet 2004;364:685-96. 

4. Daskalopoulou SS, Daskalopoulos ME, Liapis CD, Mikhailidis DP. Peripheral arterial disease: a 
missed opportunity to administer statins so as to reduce cardiac morbidity and mortality. Curr Med 
Chem 2005;12:443-52. 

5. Hokanson JE, Austin MA. Plasma triglyceride level is a risk factor for cardiovascular disease inde- 
pendent of high density lipoprotein cholesterol level: a meta-analysis of the population-based 
prospective studies. J Cardiovasc Risk 1996;3:213-9. 

6. Wood D, Durrington P, Poulter N, Mclnnes G, Rees A, Wray R, on behalf of the Societies. Joint 
British recommendations on prevention of coronary heart disease in clinical practice. Heart 
1998;80(suppl2):Sl-29. 

7. Sacks FM, for the Expert Group on HDL Cholesterol. The role of high-density lipoprotein (HDL) 
cholesterol in the prevention and treatment of coronary heart disease: Expert Group 
Recommendations. Am J Cardiol 2002;90:139-43. 

8. Rizos E, Mikhailidis DP. Are high density lipoprotein (HDL) and triglyceride levels relevant in stroke 
prevention? Cardiovasc Res 2001;52:199-207. 

9. Rubins HB, Robins SJ, Collins D, et al. Gemfibrozil for the secondary prevention of coronary heart 
disease in men with low levels of high-density lipoprotein cholesterol. N Engl J Med 1999;341:410-7. 

10. De Backer G, Ambrosioni E, Borch-Johnsen K, et al. Third Joint Task Force of European and Other 
Societies on Cardiovascular Disease Prevention in Clinical Practice. European guidelines on cardio- 
vascular disease prevention in clinical practice. Eur Heart J 2003;24:1601-10. 

11. Grundy SM, Cleeman JI, Merz CN, et al. National Heart, Lung, and Blood Institute; American College 
of Cardiology Foundation; American Heart Association. Implications of recent clinical trials for the 
National Cholesterol Education Program Adult Treatment Panel III guidelines. Circulation 
2004;110:227-39. 

12. Heart Protection Study Collaborative Group. MRC/BHF Heart Protection Study of cholesterol lower- 
ing with simvastatin in 20,536 high-risk individuals: a randomised placebo-controlled trial. Lancet 
2002;360:7-22. 

13. Cheng KS, Mikhailidis DP, Hamilton G, Seifalian AM. A review of the carotid and femoral intima- 
media thickness as an indicator of the presence of peripheral vascular disease and cardiovascular 
risk factors. Cardiovasc Res 2002;54:528-38. 



140 Vascular Surgery 

14. Rantanen K, Tatlisumak T. Secondary prevention of ischemic stroke. Curr Drug Targets 
2004;5:457-72. 

15. Tsiara S, Elisaf M, Mikhailidis DP. Influence of smoking on predictors of vascular disease. Angiology 
2003;54:507-30. 

16. Cheshire NJW, Wolfe JHN, Barradas MA, Chambler AW, Mikhailidis DP. Smoking and plasma 
fibrinogen, lipoprotein (a) and serotonin are markers for postoperative infrainguinal graft stenosis. 
Eur J Vase Endovasc Surg 1996;11:479-86. 

17. Milionis HJ, Rizos E, Mikhailidis DP. Smoking diminishes the beneficial effect of statins: observa- 
tions from the landmark trials. Angiology 2001;52:575-87. 

18. Robless P, Mikhailidis DP, Stansby G. Systematic review of antiplatelet therapy for the prevention of 
myocardial infarction, stroke or vascular death in patients with peripheral vascular disease. Br J Surg 
2001;88:787-800. 

19. Chobanian AV, Bakris GL, Black HR, et al. National Heart, Lung, and Blood Institute Joint National 
Committee on Prevention, Detection, Evaluation, and Treatment of High Blood Pressure; National 
High Blood Pressure Education Program Coordinating Committee. The Seventh Report of the Joint 
National Committee on Prevention, Detection, Evaluation, and Treatment of High Blood Pressure: 
the JNC 7 report. JAMA 2003;289:2560-72. 

20. Tsimihodimos V, Miltiadous G, Daskalopoulou SS, Mikhailidis DP, Elisaf MS. Fenofibrate: metabolic 
and pleiotropic effects. Curr Vase Pharmacol 2005;3:87-98. 

21. Ridker PM, Cannon CP, Morrow D, et al. Pravastatin or Atorvastatin Evaluation and Infection 
Therapy-Thrombolysis in Myocardial Infarction 22 (PROVE IT-TIMI 22) Investigators. C-reactive 
protein levels and outcomes after statin therapy. N Engl J Med 2005;352:20-8. 

22. Rahman M, Brown CD, Coresh J, et al. Antihypertensive and Lipid-Lowering Treatment to Prevent 
Heart Attack Trial Collaborative Research Group. The prevalence of reduced glomerular filtration 
rate in older hypertensive patients and its association with cardiovascular disease: a report from the 
Antihypertensive and Lipid-Lowering Treatment to Prevent Heart Attack Trial. Arch Intern Med 
2004;164:969-76. 

23. Athyros VG, Mikhailidis DP, Papageorgiou AA, et al. The effect of statins versus untreated dyslipi- 
daemia on renal function in patients with coronary heart disease. A subgroup analysis of the Greek 
atorvastatin and coronary heart disease evaluation (GREACE) study. J Clin Pathol 2004;57:728-34. 

24. Youssef F, Gupta P, Seifalian AM, Myint F, Mikhailidis DP, Hamilton G. The effect of short-term 
treatment with simvastatin on renal function in patients with peripheral arterial disease. Angiology 
2004;55:53-62. 

25. Daskalopoulou SS, Athyros VG, Elisaf M, Mikhailidis DP. Uric acid levels and vascular disease. Curr 
Med Res Opin 2004;20:951-4. 



1 5. Angioplasty for Critical Arterial Stenosis 

Lars Norgren 



A 62-year-old man, formerly a shopkeeper and a heavy smoker (26 cigarettes 
daily until 5 years ago when he succeeded in giving up), presented with a history 
of leg pain. Since retirement 2 years ago, he had started going for walks in the 
nearby countryside. From the start, he could walk for 5 miles, but he felt some 
pain in his left calf, which reduced after slowing down. 



Question 1 

Which of the following statements are correct? 

A. This symptom is typical for both intermittent claudication and knee joint 
arthrosis. 

B. This man presents exceptional symptoms: in the vast majority of cases, both legs 
are affected. 

C. Intermittent claudication affects about 5 per cent of the male population over 
the age of 55 years. 

D. This man should be advised to stop walking to reduce the risk of pain. 

E. The symptom is consistent with an occlusion or a critical stenosis of the 
superficial femoral artery (SFA). 

The man did not seek advice at that stage, but 2 years later, he suddenly experi- 
enced more severe pain in the left calf when he woke up in the morning. This hap- 
pened 3 days before presentation. The pain disappeared slowly, but walking was 
then restricted to about 20 m, at which point the pain started. He felt some numb- 
ness in the toes, and on awaking in the morning there was some pain in the foot, 
which disappeared when he stood up. 



141 



142 Vascular Surgery 

Question 2 

What is the most likely reason for the new symptom? 

A. Muscle rupture. 

B. Deep vein thrombosis. 

C. Thrombosis of an artery or collateral artery. 

D. Minor stroke. 

E. Aggravation of knee joint arthrosis. 

At examination, femoral pulses were present but no popliteal or foot pulses could 
be felt. Doppler flow was detected in the left posterior tibial artery, and in the right 
posterior tibial artery and dorsalis pedis artery (DPA). Ankle brachial pressure 
index (ABPI) of the affected leg was 0.4; that of the right leg was 0.8. 

An angiography was performed, depicting lesions as shown in Fig. 15.1, as less 
than 50 per cent stenosis of the left external iliac artery (EIA), two short stenoses of 
more than 70 per cent of the left SFA, and an occlusion of a deep femoral artery 
branch. On the right side, there was one stenosis of the SFA of about 70 per cent. 
There were also short occlusions of the anterior tibial artery bilaterally. 

Immediately following the diagnostic angiography, it was decided to perform a 
balloon percutaneous transluminal angioplasty (PTA) of the stenoses of the left 
SFA. The right leg was left untreated. 

Question 3 

Which of the following statements are correct? 

A. There is normally no indication to treat an asymptomatic limb. 

B. It is never possible to reopen occlusions of the deep femoral artery branches. 

C. A stenosis of less than 50 per cent is not critical and does not normally need to 
be treated. 

D. The critical stenosis of the SFA in our case is probably the primary reason for 
this patient's initial symptoms. 

E. The deep femoral artery occlusion could well be the reason for the aggravation 
of the patient's symptoms. 

At follow-up after 30 days, the patient had improved and could walk his daily 
5 miles, albeit with some tolerable claudication. 

Question 4 

Which investigations are relevant for the follow-up of this case? 
A. Digital subtraction angiography. 



Angioplasty for Critical Arterial Stenosis 



143 




Fig. 15.1. Lesions found on angiography. 1, <50 per cent stenosis of left external iliac artery; 2, two short 
stenoses (<70 per cent) of left superficial femoral artery; 3, occlusion of a left deep femoral artery branch; 4, 
short stenosis (70 per cent) of the right superficial femoral artery; 5, short occlusions of anterior tibial artery 
bilaterally. 



B. ABPI measurement. 

C. Spiral CT angiography. 

D. Duplex investigation. 

E. Laser Doppler flux measurement. 



144 Vascular Surgery 

Question 5 

What advice and adjunctive treatment should be given to this patient? 

A. Continuing exercise, but stopping if pain appears. 

B. Antiplatelet drug treatment. 

C. Antiplatelet drug + warfarin treatment. 

D. Avoid cycling. Hip flexion increases the risk of new femoral stenotic or occlusive 
lesions. 

E. Continuing exercise. 



Commentary 

Intermittent claudication affects about 5 per cent of the male population over 
55 years and about 2.5 per cent of the female population [1]. Calf pain during 
walking, caused by occlusion or stenosis of the SFA, is the most common symptom; 
in the majority of cases, only one leg is symptomatic to start with. [Q1: C, E] Only 
about 25 per cent of patients suffering intermittent claudication deteriorate to criti- 
cal limb ischaemia (CLI) [2, 3]; the majority remain stable or even improve. The 
major risk is cardiovascular complications. 

When a sudden deterioration appears, as in our case, a thrombotic event is most 
likely. [Q2: C] 

An ABPI of 0.8 indicates the presence of asymptomatic peripheral arterial occlu- 
sive disease (PAOD). Symptoms are hidden due to the walking restriction from the 
more severely diseased contralateral leg. An ABPI of 0.4 can be consistent with CLI, 
but symptoms are not convincing. Usually, the definition of chronic CLI requires 
stable symptoms for at least 2 weeks [4]. The symptoms fit more appropriately into 
the definition of acute ischaemia, with a viable, unthreatened leg [5]. 

Critical stenosis produces a measurable pressure gradient and/or reduced blood 
flow. Usually, more than a 75 per cent reduction of the cross-sectional area is 
required, which means more than a 50 per cent reduction of the luminal diameter. 
[Q3: A, C, D, E] Also, a less pronounced grade of stenosis may produce haemodynamic 
disturbances and symptoms, especially during exercise. Two or more stenoses after 
each other may also become significant at a lesser grade of stenosis. 

In principle, non-disabling claudication symptoms do not justify interventional 
treatment. Had this patient sought advice before the acute symptoms appeared, he 
would have been told to continue exercising and to try to continue walking when 
pain appeared. The acute symptoms changed the scene, although the leg was not in 
danger. In this situation, an angiography should be performed soon to enable treat- 
ment. A duplex examination could be carried out first as a screening procedure. 

PTA was performed on the two subsequent and critical SFA stenoses, while the 
uncritical CIA stenosis was left. If in doubt about whether to treat a stenosis, than a 
measurement of the pressure gradient over such a stenosis is valuable. If this gradi- 
ent is less than 10 mm Hg (mean pressure), then there is no indication to dilate. 

The occluded profunda branch is a delicate problem. It might be a thrombotic 
occlusion, preventing collateral blood flow to the distal SFA. One could consider 



Angioplasty for Critical Arterial Stenosis 1 45 

leaving a catheter in the thrombus and starting thrombolysis; however, this decision 
is at the surgeon's discretion and depends on his/her experience. The reopened SFA 
might be sufficient for appropriate perfusion of the lower leg. 

So what evidence exists that PTA is the correct treatment for this case? Level I 
evidence does not exist, but the consensus is that short lesions of less than 7 cm 
should be treated in this way [6]. The indication should be the same as for open 
surgery, namely critical limb ischaemia but also disabling claudication, provided 
that the patient's quality of life would be very much improved by an increased pos- 
sibility to walk. 

Whether PTA is superior to exercise alone is still under debate [7, 8]. There is, 
however, no doubt that PTA is more accepted for iliac lesions than for SFA lesions 
in cases of intermittent claudication. If one accepts PTA, then it has to be clear that 
recurrence of symptoms is not uncommon, and that the duration of the effect of 
treatment is variable. Stents may be deployed following iliac angioplasty but are less 
effective in SFA angioplasty. Improved results are claimed by those who do sub- 
intimal angioplasty. The use of stents in the SFA should be reserved at present as a 
rescue procedure in cases of suboptimal dilation causing dissection or any other 
damage to the vessel. Controlled studies are needed to define the indications of 
stenting, in particular with the new generation of drug diluting stents, in the 
management of peripheral vascular disease. 

Complications of PTA include bleeding at the puncture site (about 2 per cent), 
pseudoaneurysms (0.7 per cent) [9], local vessel damage (see above), and vessel 
rupture (very infrequent). Bleeding at the puncture site is normally treated with 
compression, but surgery may be required. Likewise, pseudoaneurysms can be 
treated by ultrasound-guided compression or thrombin injection. As described, 
local damage to the vessel with dissection may be treated with a stent; local bleeding 
caused by rupture commonly does not require any treatment other than careful 
observation to avoid compartment syndrome due to an expanding haematoma. 

When following up patients after PTA, duplex examination can be of value as it 
can find restenosis long before the ankle blood pressure is reduced, although ABPI 
should be measured regularly. Other investigations are not required, should no 
symptoms reappear that demand treatment. [Q4: B, D] 

All forms on continuing exercise are of importance for all patients after treatment 
for PAOD and no detrimental effects of cycling should be anticipated; antiplatelet 
drugs should be administered for secondary prevention of thrombotic events at the 
treatment site but mainly for coronary and cerebrovascular thrombotic events. 
Warfarin is not indicated. [Q5: B, E] 



References 



Dormandy J, Mahir M, Ascady G, Balsano F, De Leeuw P, Blombery P, et al. Fate of the patient with 
chronic leg ischaemia. J Cardiovasc Surg 1989;30:50-57 

Jelnes R, Gardstang O, Jensen HK, Baekgaard N, Tonnesen KH, Schroeder T. Fate in intermittent clau- 
dication: outcome and risk factors. BMJ 1986;293:1137-40 

Rosenbloom MS, Flanigan DP, Schuler JJ. Risk factors affecting the natural history of intermittent 
claudication. Arch Surg 1988;123:867-70. 
Second European Consensus Document on Chronic Critical Leg Ischemia. Circulation 1991;84 (Suppl 

4). 

Rutherford RB, Baker JD, Ernst C, Jonston KW, Porter JM, Ahn S, Jones DN. Recommended standards 

for reports dealing with lower extremity ischemia: revised version: J Vase Surg 1997;26:517-38. 



146 Vascular Surgery 

6. Management of peripheral arterial occlusive disease. Trans- Atlantic Intersociety Consensus document 
(TASC). J Vase Surg 2000;31 (Part 2):S1-S296. 

7. Norgren L. Is balloon angioplasty indicated for intermittent claudication? - Yes. In: Greenhalgh RM, 
editor. Indications in vascular and endovascular surgery. London: WB Saunders, 1998;285-92. 

8. Bradbury A. Is balloon angioplasty indicated for intermittent claudication? - No. In: Greenhalgh RM, 
editor. Indications in vascular and endovascular surgery. London: WB Saunders, 1998;293. 

9. Criado FJ, Abdul-Khoudoud O, Twena M, Clark NS, Patten P. Outpatient endovascular intervention: is 
it safe? J Endovasc Surg 1998;5:236-9. 



16. Lower Limb Claudication due to Iliac 
Artery Occlusive Disease 

Fabien Koskas and Marcus }. Brooks 



A 63-year-old hypertensive man presented in 1990 with a history of pain devel- 
oping in his left calf and thigh after walking 100 metres. During the preceding 
3 months, following the introduction of a beta-blocker for newly diagnosed 
hypertension, the distance he could walk at a "normal" pace had reduced from 
200 metres. The pain ceased almost immediately after stopping walking and 
appeared again after the same interval. A systemic enquiry was unremarkable. 
He was noted to be an active and life-long heavy smoker. Clinical examination 
revealed a diminished left femoral pulse and absent left popliteal and pedal 
pulses. The abdominal aorta and right leg pulses were normal. No bruits were 
audible in the abdomen or groins. 



Question 1 

Which of the following would be part of your initial management of this patient? 

A. Smoking cessation advice. 

B. Enrolment of the patient in a supervised exercise programme. 

C. Stopping the beta-blocker. 

D. A prescription for warfarin. 

E. A prescription for aspirin. 

Simple advice was given on the importance of exercise as there was no supervised 
exercise programme available. The patient managed to stop smoking completely. 
He returned to the clinic after 3 months and his symptoms had not improved. As 
the patient was shortly to be retiring and was an enthusiastic hunter he was very 
keen for any intervention that might relieve his claudication. 



147 



148 Vascular Surgery 

Question 2 

How would you proceed with your management now he has returned? 

A. No further intervention. 

B. Drug treatment with cilostazol. 

C. Obtain arterial imaging to better define the lesion. 

Question 3 

Which of the following is not an appropriate first line imaging modality? 

A. Duplex scan. 

B. Contrast-enhanced CT scan (CTA). 

C. Contrast-enhanced magnetic resonance angiography (MRA). 

D. Digital subtraction contrast angiogram (IADSA). 

The patient underwent an arterial duplex scan. This scan showed a significant 
stenosis at the left internal iliac origin and a short but tight stenosis of the proximal 
external iliac artery. The contralateral iliac system was found to be free from 
significant disease, as were the femoral, popliteal and crural arteries. Subsequent 
angiographic images are shown in Fig. 16.1. 

Question 4 

Which of the following would you consider as possible interventions? 

A. Aorto-bifemoral bypass graft. 

B. Left aorto-uni-iliac bypass graft. 

C. Right femoral to left femoral cross-over graft. 

D. Percutaneous transluminal angioplasty via a right femoral puncture. 

E. Percutaneous transluminal angioplasty via a left femoral puncture. 

A percutaneous transluminal angioplasty was performed from the left groin 
using a 7-mm balloon. The left leg pulses were restored by the procedure. The 
patient noticed that his claudication disappeared. 

Question 5 

Which of the following statements describe the optimal follow-up for this patient? 
A. Low-dose subcutaneous low-molecular-weight heparin for 3 months. 



Lower Limb Claudication due to Iliac Artery Occlusive Disease 



149 





Fig. 16.1. Images from the digital subtraction angiogram showing a the renal arteries, infrarenal aorta and 
iliac bifurcation, b stenotic lesions at the origin of the left internal iliac artery and in the left external iliac artery, 
and c an oblique projection of the left iliac system. 



150 



Vascular Surgery 



B. The patient is seen regularly, for review of his claudication and control of his 
risk factors. 

C. Serial duplex scanning to detect recurrent stenosis before symptoms occur. 

D. The patient is discharged from follow-up. 

Eleven years later the patient returned complaining of recent onset of erectile 
dysfunction and the return of his left calf claudication. 

Question 6 

What is the likely aetiology of this man's erectile dysfunction? 

In the intervening 10 years the patient had resumed smoking and had undergone 
a coronary artery bypass graft for unstable angina. Two years following the bypass 
his angina had recurred. A coronary angiogram showed that two of three vein grafts 
had occluded, and that his left ventricle function was poor (28 percent ejection frac- 
tion). On examination his left femoral pulse was weak, the distal pulses were absent 
in the left leg and a soft bruit was heard over the right femoral artery. The patient 
insisted on being relieved from his symptoms "no matter the risks" as he had 
married a woman much younger than him. Another angiogram was requested, one 
image from which is shown in Fig. 16.2. The distal run-off (not shown) was pre- 
served in both legs. 




Fig 16.2. Angiogram performed for investigation of the patient's erectile dysfunction and recurrence of left calf 
intermittent claudication. 



Lower Limb Claudication due to Iliac Artery Occlusive Disease 151 

Question 7 

Which of the following is now the preferred intervention? 

A. Aorto-bifemoral bypass graft with revascularisation of both internal iliac 
arteries. 

B. Left aorto-uni-iliac bypass graft with revascularisation of the left internal iliac. 

C. Right femoral to left femoral cross-over graft. 

D. Percutaneous transluminal angioplasty. 

E. No intervention - claudication and impotence are a good way of protecting the 
heart. 

A percutaneous approach was used; a hydrophilic guidewire passed easily 
through the lesion and both the external and internal iliac lesions were treated 
endovascularly (Fig. 16.3). 



Commentary 

The majority of patients with peripheral vascular disease smoke [1]. Cessation of 
smoking slows the rate of progression of peripheral vascular disease and reduces 
the risk of cardiac morbidity and mortality [2]. The prescription of nicotine replace- 
ment therapy is of benefit in patients who find it difficult to quit [3]. The benefit of 
exercise for relieving the symptoms of intermittent claudication has long been 
recognised [4]. The type and frequency of exercise to yield maximum benefit has 
now been examined in a systematic review and Cochrane Collaboration Overview; 
advice alone is of little benefit but supervised exercise programmes (achieving 
maximal walking distance for at least 30 minutes three times a week) can achieve a 
150 percent increase in walking distance or 6-minute increase in walking time [5, 6]. 
A systematic review failed to show any association between beta-blockers and wors- 
ening claudication [7]. If the beta-blocker is stopped another antihypertensive 
agent, such as a calcium channel blocker or ACE inhibitor, should be substituted for 
control of hypertension, as treating hypertension reduces the stroke risk by 38 
percent, cardiovascular risk by 14 percent and peripheral vascular events by 14 
percent [8]. A systematic review by the Anti-platelet Trialists Collaboration has 
proven the benefit of 75-1500 mg aspirin daily in achieving a 25 percent reduction 
in the risk of death, stroke or myocardial infarction [9]. A post-hoc subgroup analy- 
sis of patients with peripheral vascular disease in the CAPRIE trial showed addi- 
tional benefit for clopidogrel [10]. The additional benefit is small (196 patients on 
clopidogrel to prevent one death) and not justified except for the 20 percent of 
patients who are aspirin intolerant. There is no evidence of benefit from warfarin 
11]. It is also important to start the patient on statin therapy as this intervention 
has been shown to achieve an equivalent reduction in morbidity and mortality to 
aspirin [12, 13]. [Q1:A,B,E] 

The patient returns having modified his risk factors and is no better. His claudi- 
cation is affecting his quality of life. The options for management are persistence 
with unsupervised exercise, drug treatment or intervention. Cilostazol is the only 



152 



Vascular Surgery 




Fig 16.3. Images from the second procedure showing a passage of a hydrophilic guidewire across the occluded 
external iliac artery, b an excellent technical result from angioplasty with stent placement in the external iliac 
artery with the wire now positioned in the internal iliac artery and c a completion angiogram. 



Lower Limb Claudication due to Iliac Artery Occlusive Disease 1 53 

drug shown to be effective at relieving the symptoms of intermittent claudication in 
a small randomised trial [14, 15]. However, it is expensive and the effect is short- 
lived. Intervention can only be considered once the anatomy of the underlying 
stenosis is known. As the presenting symptom is intermittent claudication and the 
patient has a weak left femoral pulse with normal right leg pulses we suspect a 
single level left iliac stenosis. It was decided to image the lesion. [Q2: C, (B)] 

The optimal imaging of aortoiliac lesions is dependent on the facilities available. 
It is preferable to first obtain non-invasive images to allow the approach to a lesion 
to be planned, ensure the appropriate equipment is available and obtain the appro- 
priate patient consent . Duplex scanning has become a useful tool for non invasive 
evaluation of aortoiliac occlusive disease [16]. However, duplex in the aortoiliac 
segment is highly dependent on patient's body habitus and experience of the opera- 
tor. A helical multi-detector row (32 or 64 detectors) CT scanner can provide high- 
quality cross-sectional images of the aorta, iliac arteries and even arteries down to 
the feet. CT scans have the advantage to the surgeon of familiarity and show 
calcified vessel walls. The disadvantages of CTA are the risk of contrast-induced 
nephropathy, patient exposure to ionising radiation and the time it takes to refor- 
mat the images [17, 18]. Contrast-enhanced magnetic resonance angiography 
(MRA) can also image the aortoiliac segment down to the feet. It is the investigation 
of choice in patients at risk of contrast-induced renal impairment. In a comparison 
of CTA and MRA in imaging the aorta and iliac segments, sensitivity and specificity 
for the detection of lesions were equivalent. CTA took longer to reformat and 
report; a greater proportion of patients expressed a preference for CTA [19]. MRA is 
contraindicated in patients with pacemakers and ferromagnetic intracranial 
aneurysm clips. Intra-arterial digital subtraction angiography now has a limited 
diagnostic role in the aortoiliac segment. Angiography is invasive and is only per- 
formed if artefacts from previous implants (i.e. stainless steel stents) degrade three- 
dimensional imaging, if direct pressure measurements across a stenosis are 
required or, as in this patient, as the first stage of an invasive procedure following 
non-invasive imaging. [Q3: D] 

The left internal iliac origin and mid-third external iliac artery lesions are 
TransAtlantic Inter-Society Consensus (TASC) type A lesions [20]. The TASC consen- 
sus on the management of type A aortoiliac lesions (Recommendation 32) was for 
endovascular intervention. Surgical options, endarterectomy or bypass, are reserved 
for longer stenoses (5-10 cm) or occlusions [20]. The reported primary technical 
success of angioplasty of type A lesions is 98-99 percent with 60-80 percent patency at 
5 years [21]. The 5-year patency of open procedures is slightly better, 90 percent for 
aorto-bifemoral bypass, but the patient is exposed to the risks of death (2-3 pre cent), 
erectile dysfunction and graft infection [22]. It is a matter of personal preference 
whether a left or right percutaneous approach is used for the angioplasty as the lesion 
is mid-way between the aortic bifurcation and inguinal ligament. [Q4: D or E] 

The optimal management of patients following angioplasty has not been evalu- 
ated in randomised control trials. The risks to the artery are thrombosis, myointi- 
mal hyperplasia and disease progression. All patients should already be on an 
antiplatelet agent. Patients are formally heparinised during the procedure and for 
this short stenosis this is probably adequate. There is no evidence that post- 
procedure low-molecular-weight heparin, or for that matter any pharmacological 
agent (e.g. ticlopidine), is of benefit. Routine graft surveillance has been shown 
to improve the secondary patency of infra-inguinal vein bypass grafts [23]. 
Surveillance has not been evaluated following iliac angioplasty. As myointimal 



154 Vascular Surgery 

hyperplasia and disease progression both occur, it appears prudent, if not manda- 
tory, to follow up patients. This can be done using clinical examination, arterial 
duplex or ankle brachial pressure index (ABPI) measurement. Clinical follow-up is 
cost-effective and is a good way of enforcing a tight control of risk factors. [Q5: B] 

Erectile dysfunction in this setting is probably due to arterial insufficiency result- 
ing from progression of bilateral iliac occlusive disease as the patient has a new 
bruit in the right groin. The association of erectile dysfunction with aortoiliac occlu- 
sive disease was first described in 1814 by Robert Graham [24]. However, it was 
Rene Leriche who in 1940 in Paris operated on a 29-year-old truck driver "who for 
two years had been suffering from claudicatio intermittens with severe cramps in 
the leg musculature already after a few hundred meters of walking, and cramp pains 
also at night. The last weeks before the operation he complained of not being able to 
complete an intercourse, as both erection and ejaculation was disturbed" [25]. [Q6: 
Leriche syndrome] 

The patient has suffered disease progression in the intervening years. He has 
developed a very tight stenosis of the left internal iliac artery, a stenosis or the right 
internal iliac artery origin and complete occlusion of the left external iliac. The left 
external iliac lesion is classified as a TASC type C lesion [20]. The consensus in 1990, 
when the TASC guidelines were drawn up, was that definitive recommendations on 
how to treat such lesions must await more convincing evidence. This situation has 
not changed. The risks of open aortoiliac bypass surgery and endarterectomy have 
already been discussed. Remote iliac endarterectomy using Moll ring strippers 
avoids an abdominal approach and pelvic dissection, has good published technical 
success rates (88-92 percent), and 3-year patency just below that of open 
endarterectomy (60 percent) [26, 27]. A potential development for the future is 
laparoscopic aortoiliac surgery [28]. In this patient a femoral-femoral cross-over 
graft is not advisable because contralateral lesions may impair the graft inflow and 
because this procedure would not address the internal iliac stenoses. Had the 
cardiac antecedents not been present, direct bilateral surgical antegrade revascular- 
ization of the lower limbs and one or both internal iliac arteries would have been an 
excellent solution. However, in the context of unreconstructable coronary artery 
disease and poor left ventricular function, such a solution is too invasive and carries 
the risk of potentially lethal cardiac morbidity. On the other hand, surgical absten- 
tion, although not without justification, seems exaggerated because quality of life is 
often as important as its length among middle-aged and aged patients. [Q7: D] 

Stenting is generally reserved for the management of lesions with a high risk of 
primary failure (i.e. eccentric calcified plaque), primary failure (residual stenosis 
greater than 50 percent or greater than 10 mm Hg pressure gradient), dissection or 
distal embolisation [29]. In this patient, stents were placed because of the recurrent 
occlusion of the external iliac and residual stenosis in the internal iliac artery. The 
patient recovered from his claudication, as well as his impotence, without any 
significant cardiac morbidity and is still doing well 4 months after the procedure. 



References 

1. Kannel WB, Shurtleff D. The Framingham Study. Cigarettes and the development of intermittent 
claudication. Geriatrics 1973;28:61-8. 

2. Mathieson FR, Larsen EE, Wulff M. Some factors influencing the spontaneous course of arterial vas- 
cular insufficiency. Acta Chir Scand 1970;136:303-8. 



Lower Limb Claudication due to Iliac Artery Occlusive Disease 1 55 

3. Joseph AM, Norman SM, Ferry LH, et al. The safety of trans-dermal nicotine as an aid to smoking 
cessation in patients with cardiac disease. N Eng J Med 1996;335:1793-8. 

4. Housley E. Treating claudication in five words. BMJ 1988;296(6635):1483-4. 

5. Gardener AW, Poehlman ET. Exercise rehabilitation programs for the treatment of claudication 
pain. JAMA 1995;274:975-80. 

6. Leng GC, Fowler B, Ernst E. Exercise for intermittent claudication. The Cochrane Database of 
Systematic Reviews 2000, Issue 2. 

7. Radark K, Deck C. Beta-adrenergic blocker therapy does not worsen intermittent claudication in 
subjects with peripheral arterial disease. A meta-analysis of randomised control trials. Arch Intern 
Med 1991;151:1769-76. 

8. Kannel WB, McGhee DL. Update on some epidemiological features of intermittent claudication in 
subjects with peripheral vascular disease. J Am Geriatr Soc 1985;22:13-18. 

9. Anti-platelet Trialists Collaboration. Collaboration overview of randomised trials on antiplatelet 
therapy. 1. Prevention of death, myocardial infarction and stroke by prolonged antiplatelet therapy 
in various catagories of patients. BMJ 1994;308:81-106. 

10. CAPRIE Steering Committee. A randomised, blinded, trial of clopidogrel versus aspirin in patients at 
risk of ischaemic events (CAPRIE). Lancet 1996;348:1328-39. 

11. Visseren FL, Eikelboom BC. Oral anticoagulant therapy in patients with peripheral artery disease. 
Semin Vase Med 2003;3(3):339-44. 

12. Scandinavian Simvastatin Survival Group Study. Randomised trial of cholesterol lowering in 4444 
patients with coronary heart disease: the Scandinavian Simvastatin Survival Study (4S). Lancet 
1994;344:1383-9. 

13. Sacks FM, Pfeffer MA, Moyle LA, et al. For the Cholesterol and Recurrent Events Trial Investigators. 
The effect of pravastatin on coronary events after myocardial infarction in patients with average cho- 
lesterol levels. N Engl J Med 1996;335:1001-9. 

14. Cameron HA, Waller PC, Ramsey LE. Drug treatment of intermittent claudication: a critical analysis 
of the methods and findings of published clinical trials. Br J Clin Pharmacol 1988;26:569-576. 

15. Strandness DE, Dalman RL, Panian S, et al. Effect of cilostazol in patients with intermittent claudica- 
tion: a randomized, double-blind, placebo-controlled study. Vase Endovasc Surg 2002;36(2):83-91. 

16. Van der Zaag ES, Legemate DA, Nguyen T, et al. Aortoiliac reconstructive surgery based upon the 
results of duplex scanning. Eur J Vase Endovasc Surg 1998;16:383-9. 

17. Catalano C, Fraioli F, Laghi A, et al. Infrarenal aortic and lower-limb arterial disease: diagnostic per- 
formance of multi-detector row CT angiography. Radiology 2004:231;555-63. 

18. Nicholson T, Downes M. Contrast nephrotoxicity and iso-osmolar contrast agents: implications of 
NEPHRIC. Clin Radiol 2003;58:659-660. 

19. Willmann JK, Wildermuth S, Pfammater T, et al. Aorto-iliac and renal arteries: prospective intra- 
indervidual comparison of contrast-enhanced three-dimensional MR angiography and multi- 
detector row CT angiography. Radiology 2003;226:798-811. 

20. Dormandy JA. Management of Peripheral Arterial Disease (PAD). TASC Working Group. 
TransAtlantic Inter-Society Consensus (TASC). Eur J Endovasc Surg 2000;19(Suppl A):S1-S244. 

21. Rutherford RB, Durham JA, Kumpe DA. Endovascular intervention for lower extremity ischaemia. 
In: Rutherford RB, editor. Vascular surgery, 4th edn. Philadelphia: WB Saunders, 1995;858-74. 

22. Nevelsteen A, Wouters L, Suy R. Aortofemoral Dacron reconstruction for aorto-iliac occlusive 
disease: a 25 year survey. Eur J Vase Surg 1991;5:179-86. 

23. McCarthy MJ, Olojugba D, Loftus IM, et al. Lower limb surveillance following autologous vein by- 
pass should be life long. Br J Surg 1998;85:1369-72. 

24. Graham R. Case of obstructed aorta. Communicated by Sir G Blane. Medico-Chirurgical 
Transactions, London, 1814. 

25. Leriche R. De la resection du carrefour aortico-iliaque avec double sympathectomie lombaire pour 
thrombose arteritique la l'aorte: le syndrome de Pobliteration termino-aortique par arterite. La 
presse medicale, Paris 1940;48:601-7. 

26. Ricco JB. Unilateral iliac artery occlusive disease; a randomised multi-centre trial examining direct 
revascularisation versus crossover bypass. Ann Vase Surg 1992;16:841-52. 

27. Smeets L, de Borst GJ, Vries JP, et al. Remote iliac artery endarterectomy: seven year results of a less 
invesive technique for iliac artery occlusive disease. J Vase Surg 2003;38:1297-304. 

28. Loggia M, Javerliat I, DiCentra I, et al. Total laparoscopic bypass for aorto-iliac occlusive lesions: 93 
case experience. J Vase Surg 2004;40:899-906. 

29. Tetteroo E, van der Graff Y, Bosch JL, et al. Randomised comparison of primary stent placement 
versus primary angioplasty followed by selective stent placement in patients with iliac artery occlu- 
sive disease. Lancet 1998;18:499-505. 



1 7. Erectile Dysfunction due to Aortic Disease 

Ralph G. DePalma 



A 60-year-old married man presented with sudden onset of erectile failure about 
1 month previously. This occurred in the absence of critical life changes, psycho- 
logical stress, or known intercurrent illness. Initially, the patient was seen by a 
general practitioner and treated with sildenafil in incremental doses up to 
150 mg. This had only a minor effect: ejaculation was maintained but erections 
remained insufficient for penetration. 

Risk factors consisted of mild hypertension controlled by an angiotensin con- 
verting enzyme (ACE) inhibitor, cigarette smoking (discontinued 1 month previ- 
ously), and a history of an infrarenal abdominal aortic aneurysm diagnosed 
8 months before. Ultrasound had measured an abdominal component 3.6 cm in 
diameter compared with a suprarenal aortic diameter of 3 cm. 

The patient was started on intracavernous injection of prostaglandin El 
25 fig/ml, which allowed intermittent functional status. However, he reported 
intermittent failure with this treatment. 

Because of this complaint and the previous history of aneurysm, computed 
tomography (CT) scans and aortography were obtained. The CT scan revealed an 
infrarenal aneurysm 4.5 cm in diameter and a 2.1 -cm aneurysm of the right 
common iliac artery. Aortography revealed bilaterally patent internal iliac arter- 
ies. The diameter of the left common iliac measured 1.8 cm. Penile brachial 
indices were on the right and 1.0 on the left. Penile pulse waves on plethsymo- 
graphy exhibited delayed upstroke and sine wave forms. 



Question I 

Which of the following statements regarding impotence is incorrect? 

A. Most cases respond to medical treatment, i.e. intracavernous injection or oral 
therapy. 

B. Chronic smoking may render the patient refractory to medical treatment. 

C. Impotence may be due to atheroembolism into penile arteries. 

D. Impotence may relate to stretching of the nerves by aneurysm. 

E. Impotence may be due to pelvic arterial occlusive disease. 

157 



158 Vascular Surgery 

Because of the findings of enlargement of the aneurysm and evidence of penile 
brachial occlusion on the right, a decision was made to operate. 

Question 2 

What are the techniques for operation under these circumstances? 

Question 3 

Describe nerve-sparing dissection for aortoiliac involvement and maintenance of 
internal iliac flow. 



Question 4 

Which of the following statements regarding the management of abdominal aortic 
aneurysms in impotence is/are correct? 

A. Aortoiliac bypass beyond involved external iliac arteries usually preserves erec- 
tile dysfunction. 

B. In patients older than 65 years, erectile dysfunction usually improves after 
aneurysm repair. 

C. Concomitant sympathetic nerve injury might result in retrograde ejaculation. 

D. An operation that achieves both nerve sparing and preserves internal iliac flow 
will offer the best chance of continued function. 

At operation, the aortic aneurysm was measured at 4.5 cm and contained scanty 
clot. However, there was a 2-cm aneurysm at the junction of the common and exter- 
nal iliac arteries medially. This pocket contained loose atheromatous debris. 
Potentially, this related to the absent penile Doppler signal on the right. 

Postoperatively, the patient had subjective improvement after reconstruction, 
reporting spontaneous erectile function for several months. He eventually contin- 
ued to have satisfactorily regular intercourse using a vacuum constrictor device. 
There were no changes in the Doppler signals; the plethsymographic wave forms 
improved in amplitude but not form. The right penile Doppler signal remained 
absent. The patient functions well at 48 months and appears satisfied with this 
result. Prevention of further embolisation of the aneurysmal debris possibly pre- 
vented further deterioration of erectile function. Penile arteries can also be exam- 
ined with duplex ultrasonography after intracavernous injections of a vasoactive 
agent. 



Question 5 

Are the results of surgery better with direct penile artery bypass for cases of small- 
vessel disease in young men or for large-vessel occlusive disease in men aged 
50 years or older? 



Erectile Dysfunction due to Aortic Disease 159 



Commentary 



Impotence (erectile dysfunction) is defined as the persistent or repeated inability 
for at least 3 months' duration to attain and/or maintain an erection sufficient for 
satisfactory performance in the absence of an ejaculatory disorder such as prema- 
ture ejaculation [1]. Erectile dysfunction is a symptom, not a disease; it has a variety 
of causes. With the availability of effective oral drugs [2], and the knowledge that a 
minority of men require vascular surgery, treatment usually begins with medical 
therapy [3, 4]. However, diagnosis and therapy are interrelated [5]. In this case, due 
to the abrupt onset of dysfunction, failure to respond to medical therapy [6], and a 
finding of large-vessel disease, the decision was made to investigate the patient 
further [7]. The usual mechanism of erectile dysfunction in large-vessel disease is 
occlusion, not stretching of the nerves. [Q1: D] 

The operation preferred by this author is transabdominal with preservation of 
the sac of the aneurysm and nerve-sparing dissection at the aortic bifurcation. In 
this case, the right iliac limb of the graft was passed through the iliac sac and anas- 
tomosed to the junction of the external and internal iliac arteries in healthy tissue. 
The inferior mesenteric artery was suture ligated from within the sac [8]. [Q2] [Q3] 
[Q4: D] 

Postoperative treatment for erectile dysfunction should be discussed with the 
patient and partner before surgery .In this case, recovery of function was probably 
due partially to embolisation of the right penile arteries, but the patient can now 
function with a vacuum constrictor device. Among 1094 men with the chief com- 
plaint of impotence, only 17 were found to have large-vessel aortoiliac disease as a 
major cause [9]. This type of presentation of erectile dysfunction is uncommon 
[10], but it is important as the results of intervention can be better than small-vessel 
bypass [11] surgery in well-selected cases [12]. 



References 

1. The Process of Care Consensus Panel: Position Paper. The process of care model for evaluation and 
treatment of erectile dysfunction. Int J Impot Res 1999;11:59-74. 

2. DePalma RG. The best treatment for impotence. Vase Surg 1998;32:519-21. 

3. DePalma RG, Olding M, Yu GE, Schwab FI, Drury EM, Miller HC, Massarin EG. Vascular interven- 
tions for impotence: lessons learned. J Vase Surg 1995;21:576-85. 

4. DePalma RG. New developments in the diagnosis and treatment of impotence. West J Med 
1996;164:54-61. 

5. DePalma RG, Emsellem HA, Edwards CM, Druy EM, Shultz SW, Miller HC, Bergsrud D. A screening 
sequence for vasculogenic impotence. J Vase Surg 1987;5:228-36. 

6. Goldstein I, Lue TF, Padma-Nathan H, Rosen RC, Steers WD, Wicker PA. Oral sildenafil in the treat- 
ment of erectile dysfunction. N Engl J Med 1998;338:1397-404. 

7. DePalma RG, Michal V. Point of view: deja vu - again: advantages and limitations of methods for 
assessing penile arterial flow. Urology 1990;36:199-200. 

8. DePalma RG. Vasculogenic impotence in vascular surgery: a comprehensive review. Moore WI, 
editor. Philadelphia: WB Saunders, 1998; 203-10. 

9. DePalma, RG. Vascular surgery for impotence: a review. Int J Impot Res 1997;9:61-7. 

10. Montague DK. Clinical guidelines panel on erectile dysfunction: summary report of the treatment of 
erectile dysfunction. J Urol 1991;156: 2007-11. 

11. Valji K, Bookstein JJ. Diagnosis of arteriogenic impotence: efficacy of duplex sonography as a screen- 
ing tool. Am J Roentgenol 1993;160;65-9. 

12. Hatzichristou DG. Current treatment and future perspectives for erectile dysfunction. Int J Impot 
Res 1998;10 (SupplF):S3-ll. 



18. Bypass to the Popliteal Artery 

Jeannie K. Chang, Keith D. Calligaro and 
Matthew }. Dougherty 



A 62-year-old overweight postal worker presented with complaints of cramps in 
his right calf. He stated that this reproducible pain occurred each time he walked 
50 yards and resolved upon sitting down. He denied tissue loss or rest pain. His 
past medical history was significant for hypertension, hypercholesterolemia and 
tobacco use, as well as coronary revascularization. 

On physical examination, he had bilateral carotid bruits, normal heart examina- 
tion, and a strong right femoral pulse, but absent popliteal and pedal pulses. His 
left lower extremity had a saphenectomy scar. Both extremities had shiny, hair- 
less skin without ulcerations or gangrene. 



Question 1 

Which of the following is not an indication for a bypass to the popliteal artery? 

A. Mild to moderate intermittent claudication. 

B. Non-healing toe ulcer with an ankle brachial index (ABI) of 0.30. 

C. Rest pain. 

D. Symptomatic popliteal aneurysm, entrapment syndrome, or adventitial cystic 
degeneration. 

The patient's blood pressure and cholesterol levels were controlled well by med- 
ication. He lost excess weight, quit smoking, and initiated cilostazol therapy, but to 
no avail. His symptoms persisted and he was so incapacitated that he was unable to 
continue delivering the mail. 

Arteriography was performed, demonstrating patency of the right iliac arteries 
but severe occlusive disease of the superficial femoral artery. There was reconstitu- 
tion of the popliteal artery with two-vessel run-off. The patient consented to a 
femoropopliteal bypass procedure. 



161 



162 Vascular Surgery 

Question 2 

The conduit yielding the best long-term patency for this bypass is: 

A. Dacron. 

B. Autologous vein. 

C. PTFE. 

D. Umbilical vein. 

E. Cryograft vein. 

Question 3 

A distal cuff or patch is most likely worthwhile for which type of bypass? 

A. Femoropopliteal above-knee reversed vein graft. 

B. Femorotibial in situ vein graft. 

C. Femoropopliteal above-knee PTFE. 

D. Femorotibial PTFE. 

E. Femoral-femoral PTFE cross-over graft. 

Femoropopliteal bypass was performed with in situ greater saphenous vein to 
the below-knee popliteal artery. There was resolution of the patient's claudica- 
tion, and he was able to return to work. Unfortunately, he became lost to follow- 
up, and 2 years later he returned with complaints of recurrent claudication in 
his right lower extremity. Neither popliteal nor pedal pulses were palpable. 
Duplex ultrasonography and arteriography demonstrated several sites with ele- 
vated velocities, suggestive of two moderate focal stenoses in the proximal half 
of his bypass graft as well as a severe narrowing at the distal anastomosis. 

Question 4 

What are the treatment options for a failing graft? 

A. Aspirin therapy. 

B. Percutaneous transluminal angioplasty (PTA). 

C. Laser-assisted angioplasty and atherectomy. 

D. Amputation. 

The patient was taken to the operating room, where a longitudinal incision was 
made through the distal portion of his vein graft and popliteal artery. Under 
fluoroscopy, balloon angioplasty of the proximal moderate stenoses was performed, 
with excellent results. Using a small segment of autologous saphenous vein, patch 



Bypass to the Popliteal Artery 1 63 

angioplasty of the distal anastomosis was performed. Completion angiography 
revealed a widely patent graft, and his distal pulses were again appreciated on 
palpation. He was able to resume his usual activities and was seen routinely in the 
vascular clinic. 

Question 5 

The most useful serial postoperative test to assess graft patency and a possible 
failing graft is: 

A. Arteriography. 

B. Pulse volume recordings. 

C. Duplex ultrasonography. 

D. Ankle brachial index. 

E. Magnetic resonance angiography (MRA). 



Commentary 

Mild to moderate intermittent claudication is not an indication for surgical bypass. 
Most (approximately 75 per cent) patients presenting with only intermittent claudi- 
cation have a benign course, remaining stable or improving with conservative mea- 
sures, such as smoking cessation, weight loss and alteration in diet, graduated 
exercise programs, and medical treatment of risk factors (e.g. hypertension, hyper- 
cholesterolemia, diabetes). Claudication is a strong and independent predictor of 
mortality, however, and thus concomitant identification of comorbidities such as 
coronary and cerebrovascular atherosclerotic disease may have significant impact 
on survival. 

Pharmacological therapy may be initiated with rheological agents such as pentox- 
ifylline or cilostazol with variable effect. Antiplatelet therapy is frequently started to 
prevent cardiac or cerebrovascular complications. Only a minority (10-20 percent) 
of patients require surgical reconstruction, and few (3-6 percent) ultimately 
progress to major amputation [1]. 

Revascularization is reserved for patients with disabling claudication or evidence 
of critical ischemia manifest as acute motor or sensory loss, chronic tissue loss or 
rest pain. Other less common etiologies for lower-extremity ischemia may cause 
femoropopliteal occlusion and are occasionally indications for surgical revascular- 
ization. [Q1:A] 

Long-term patency rates are highest when autologous vein is used as conduit. If 
the greater saphenous vein is not available, then lesser saphenous vein, 
femoropopliteal vein, or upper-extremity veins may be acceptable alternatives. The 
advantages of in situ vein bypass grafting include the preservation of the vein's 
nutrient supply and the better size match of the proximal and distal artery to the 
proximal and distal vein. Using reversed vein grafts, however, avoids the endothe- 
lial trauma of valve lysis. Although at times somewhat conflicting, the literature 
does not support the superiority of one technique over the other for femoro- 
popliteal bypasses. 



164 Vascular Surgery 

The use of human umbilical vein [2] or cryopreserved vein has also been 
described with varying success. The latter may be a potential alternative to pros- 
thetic grafts if autologous vein is unavailable, but in below-knee revascularization, 
cryopreserved vein has demonstrated the tendency for aneurysmal degeneration 
and poor long-term patency [3]. 

Prosthetic grafts in the suprageniculate bypass have demonstrated patency rates 
that are comparable with those for autologous vein [4]. The type of prosthetic graft 
is less important than the age of the patient or the size of the conduit [5]. The 
patency of prosthetic grafts to infrageniculate arteries, however, is significantly 
worse than that of autologous vein. Further, the use of composite prosthetic and 
autologous vein does not seem to improve long-term patency compared with pure 
prosthetic grafts [6]. 

Finally, there have been reports of endovascular treatment of femoropopliteal 
atherosclerotic disease, including percutaneously inserted covered stents [7] and 
prosthetic grafts introduced through a femoral arteriotomy and anchored distally 
with stent deployment [8]. Long-term patency with these techniques remains to be 
evaluated. [Q2: B] 

Intimal hyperplasia occurs frequently at the distal anastomosis when a prosthetic 
graft is used for an infrainguinal bypass and compromises its survival. 
Modifications to improve long-term patency include various vein cuffs and patches. 
Using these techniques theoretically improves compliance match between the pros- 
thetic material and the artery at the distal anastomosis. The reduction in turbulence 
minimizes the trauma to the arterial endothelium and decreases its proliferative 
response. 

The Miller cuff was studied in a prospective randomized study to determine its 
potential benefit in improving the patency rate of distal supra- and infrageniculate 
femoropopliteal polytetrafluoroethylene (PTFE) grafts. Although no difference was 
noted in above-knee bypasses with or without vein cuff, a statistically significant 
improvement in patency was observed in below-knee procedures [9]. Similarly, the 
Taylor patch has been reported to improve patency of infrageniculate bypasses [10]. 
[Q3: D] 

Salvage of a bypass graft in the early postoperative period may include strategies 
such as thrombectomy and revision of technical errors. These errors include graft 
kinks, retained valve leaflets, intimal flaps, and residual arteriovenous fistulas in an 
in situ graft. 

Recently, percutaneous endovascular techniques such as balloon angioplasty 
have been utilized with increasing frequency but with equivocal results. Focal 
lesions (less than 20 mm) are more amenable to catheter-based techniques than are 
more diffuse stenoses, but even these favorable lesions may recur. Laser angioplasty 
and atherectomy, however, have not been shown to be beneficial in the preservation 
of failing grafts. 

Thrombolysis may be considered for patients who present with sudden and 
recent onset of symptoms attributable to bypass graft occlusions. For patients with 
chronic graft occlusion, a new bypass graft provides improved clinical outcome, but 
in acute graft occlusion, thrombolysis may improve limb salvage and reduce the 
magnitude of the subsequent surgical procedure [11]. 

For short-segment stenoses, patch angioplasty or interposition within an existing 
vein graft with autologous or prosthetic material may be performed to preserve a 
bypass to the popliteal artery. Although technically simpler and requiring less autol- 
ogous material, patch angioplasty has inferior results when compared with inter- 



Bypass to the Popliteal Artery 1 65 

position [12]. Longer-segment stenoses are preferably treated with interposition or 
jump graft around the area of narrowing. Failing these strategies, however, the cre- 
ation of an entirely new bypass may be required. 

Amputation is reserved for tissue loss or ischemic pain without possible vascular 
reconstruction. Long-term survival of patients requiring major amputation is poor. 
[Q4: B] 

Early graft failure (i.e. within the first 30 postoperative days) is most likely the 
result of a technical error, hypercoagulability, poor distal run-off, or postoperative 
hypotension. In addition to avoidable technical errors, early graft failure may be 
secondary to endothelial trauma, the use of imperfect conduit, or poor surgical 
judgement with regard to the adequacy of inflow or outflow. 

Graft failure within the first few years of surgery is usually attributable to intimal 
hyperplasia. Subsequent graft failure is most frequently secondary to progression of 
atherosclerotic disease. 

It is vital to identify a failing graft before complete occlusion to preserve the 
patency of the graft. Lower-extremity revascularization can be salvaged with simple 
interventions if lesions leading to intimal hyperplasia and hemodynamic compro- 
mise can be identified before graft thrombosis. Revision of stenotic lesions in a 
failing but nonoccluded graft results in superior patency when compared with revi- 
sion of similar lesions in an occluded graft. It also leads to fewer amputations and 
subsequent revisions. Additionally, the repair of a failing graft is less costly than 
emergent revision of a failed graft or of amputation [13]. 

Postoperative duplex ultrasonography detects correctable abnormalities early, 
precludes the need for angiography in many cases, and markedly improves assisted 
primary patency of vein bypass grafts. Recommended surveillance includes initial 
ABI and duplex studies at 1 week, followed by evaluations at 3, 6, 9, 12, 18, and 24 
months, then annually thereafter. High-grade stenoses can be identified and cor- 
rected before thrombosis occurs. Criteria for the diagnosis of a failing graft include 
monophasic signals, peak systolic velocity (PSV) less than 45 cm/s throughout the 
bypass, any PSV greater than 300 cm/s, or a PSV ratio across a stenosis of greater 
than 3.5 [14].[Q5:C] 



References 



1. Illig KA, Ouriel K. Nonoperative treatment of claudication. In: Cameron JL, editor. Current surgical 
therapy, 6th edn. St Louis: Mosby, 1998; 767-70. 

2. Aalders GJ, van Vroonhoven TJMV. Polytetrafluoro ethylene versus human umbilical vein in above- 
knee femoropopliteal bypass: six-year results of a randomized clinical trial. J Vase Surg 
1992;16:816-24. 

3. Martin RS, Edwards WH, Mulherin JL, Edwards WH, Jenkins JM, Hoff SJ. Cryopreserved saphenous 
vein allografts for below-knee lower extremity revascularization. Ann Surg 1994;219:664-72. 

4. AbuRahma AF, Robinson PA, Holt SM. Prospective controlled study of polytetrafluoroethylene 
versus saphenous vein in claudicant patients with bilateral above knee femoropopliteal bypasses. 
Surgery 1999;126:594-602. 

5. Green RM, Abbott WM, Matsumoto T, Wheeler JR, Miller N, Veith FJ, et al. Prosthetic above-knee 
femoropopliteal bypass grafting: five-year results of a randomized trial. J Vase Surg 2000;31:417-25. 

6. LaSalle AJ, Brewster DC, Corson JD, Darling RC. Femoropopliteal composite bypass grafts: current 
status. Surgery 1982;92:36-9. 

7. Henry M, Amor M, Ethevenot G, Henry I, Abdelwahab W, Leborgne E, Allaoui M. Initial experience 
with the Cragg Endopro System 1 for intraluminal treatment of peripheral vascular disease. 
J Endovasc Surg 1994;1:31-43. 



166 Vascular Surgery 

8. Spoelstra H, Casselman F, Lesceu O. Balloon-expandable endobypass for femoropopliteal athero- 
sclerotic occlusive disease. J Vase Surg 1996;24:647-54. 

9. Pappas PJ, Hobson RW, Meyers MG, Jamil Z, Lee BC, Silva MB, et al. Patency of infrainguinal polyte- 
trafluoroethylene bypass grafts with distal interposition vein cuffs. Cardiovasc Surg 1998;6:19-26. 

10. Taylor RS, Loh A, McFarland RJ, Cox M, Chester JF. Improved technique for polytetrafluoroethylene 
bypass grafting: long-term results using anastomotic vein patches. Br J Surg 1992;79:348-54. 

11. Comerota AJ, Weaver FA, Hosking JD, Froehlich J, Folander H, Sussman B, Rosenfield K. Results of 
a prospective, randomized trial of surgery versus thrombolysis for occluded lower extremity bypass 
grafts. Am J Surg 1996;172:105-12. 

12. Bandyk DF, Bergamini TM, Towne JB. Durability of vein graft revision: the outcome of secondary 
procedures. J Vase Surg 1991;13:200-10. 

13. Wixon CL, Mills JL, Westerband A, Hughes JD, Ihnat DM. An economic appraisal of lower extremity 
bypass graft maintenance. J Vase Surg 2000;32:1-12. 

14. Calligaro KD, Syrek JR, Dougherty MJ, Rua I, McAffee-Bennett S, Doerr KJ, et al. Selective use of 
duplex ultrasound to replace preoperative arteriography for failing arterial vein grafts. J Vase Surg 
1998;27:89-95. 



19. Chronic Critical Limb Ischemia 

Enrico Ascher and Anil P. Hingorani 



An 85-year-old male with a history of diabetes, hypertension, hypercholes- 
terolemia, coronary artery bypass, and active tobacco use presented with a gan- 
grenous right first toe. The patient stated that he had no history of trauma to the 
area, and complained of rest pain in the foot. The patient had been in otherwise 
good health since his coronary artery bypass 12 years ago. On physical examina- 
tion, the patient was in no physical distress. The patient had a well-healed 
median sternotomy scar. Auscultation of the heart revealed a regular rate 
without any murmurs. He was obese. Abdominal examination revealed no palpa- 
ble masses. The patient had bilateral femoral and popliteal pulses but no pedal 
pulses. The patient had bilateral, well-healed scars from the greater saphenous 
vein harvest sites. The right gangrenous toe was dry without any evidence of 
infection. 



Question 1 

Which of the following statements regarding chronic lower-extremity ischemia are 
wrong? 

A. If the patient refuses any intervention, then anticoagulation alone may be 
helpful. 

B. The contralateral asymptomatic lower extremity should also undergo angio- 
graphy as there may be severe atherosclerotic disease there as well. 

C. The treatment options remain unchanged if the patient presents with only rest 
pain, ischemic ulcer or claudication. 

D. The patient cannot undergo revascularization without contrast arteriography as 
there are no other alternatives. 

The patient's arterial duplex demonstrated moderate distal right superficial 
femoral artery disease. The ankle brachial indices (ABIs) and pulse volume record- 

167 



168 Vascular Surgery 

ings demonstrated findings consistent with moderately decreased perfusion at the 
calf level and severely decreased perfusion at the ankle and transmetatarsal levels. 
The cardiac review of systems was unremarkable, and a persantine thallium 
obtained 6 months ago revealed no perfusion defects. Electrocardiogram (ECG), 
chest X-ray and routine preoperative blood tests were normal. Venous duplex 
mapping revealed inadequate veins (sclerotic and too small) in the bilateral upper 
and lower extremities. 



Question 2 

Preoperative medications/lifestyle changes that should be added to the patient's 
regimen to reduce his overall cardiovascular risk based upon randomized prospec- 
tive data include: 

A. Aspirin. 

B. A statin. 

C. Angiotensin-converting enzyme inhibitors. 

D. Tobacco cessation. 

E. A beta-blocker. 

Percutaneous angiogram of the right lower extremity demonstrated moderate 
right distal superficial femoral artery stenosis with distal occlusion. The popliteal 
appeared to be severely diseased with occlusion of the tibioperoneal artery and 
proximal anterior tibial artery. The mid-anterior tibial artery reconstituted and ran 
down to the dorsalis pedis artery. No other vessels appeared to be adequate. 

Question 3 

What type of options would you consider for this lower extremity? 

A. Below-knee amputation. 

B. Digital amputation. 

C. Tibial bypass with expanded polytetrafluoroethylene (ePTFE) with a venous 
interposition or fistula. 

D. Tibial bypass with cadaveric vein. 

E. Sympathectomy. 

F. Chelation therapy. 

G. Subintimal angioplasty 

The patient underwent a successful bypass with ePTFE to the anterior tibial 
artery and did stop smoking after the procedure. The patient's toe underwent auto- 
amputation and the rest pain has resolved. He was followed up 2 years after the pro- 
cedure with a patent bypass. 



Chronic Critical Limb Ischemia 169 

Question 4 

What is the patient's long-term prognosis in terms of mortality, graft patency, and 
limb salvage after successful bypass? 

A. The long-term mortality, patency, and limb salvage are about 20 percent and 
therefore are so poor that no intervention should be made. 

B. The mortality and patency are 50 percent at 4-5 years. The limb salvage is 
70 percent at 4 years. If the patient has a reasonable life expectancy and func- 
tional status, he should undergo the revascularization. 

C. The mortality, patency, and limb salvage rates are irrelevant in this age group. 

Question 5 

Which patients would you consider to be inoperable? What treatment options may 
be offered to this subset of patients? 



Commentary 



Indications for revascularization to the tibial vessels are limited to ischemic ulcers, 
gangrene, and rest pain. The long-term patency of the bypass is affected directly by 
continued tobacco use, and the patient should be urged to stop smoking. 
Anticoagulation plays no role as the sole management of this patient. Even though 
the patient may have asymptomatic contralateral disease, there is no role for further 
investigation. Angiography maybe used to visualize both inflow and outflow sites. 
In general, the most distal available inflow site is utilized to shorten the length of the 
graft. Time-delayed imaging maybe required to visualize the calf and foot arteries 
because of reduced flow. The use of magnetic resonance angiography (MRA) has 
proven to be beneficial in identifying patent lower-extremity arteries, particularly in 
view of the recent advances in imaging software and hardware [1-3]. Finally, high- 
resolution duplex imaging has now become a viable alternative for visualization of 
inflow and outflow sites with the added advantages of cost reduction, fewer compli- 
cations associated with angiography, and the ability to identify the least calcified 
artery segment [4-8]. However, both MRA and duplex imaging should be used only 
as preoperative imaging modalities after they have been validated at each center. 
[Q1:A,B,C,D] 

Increasing focus on the perioperative and long-term management of patients 
with peripheral arterial disease has identified that all the factors listed in Question 2 
can significantly reduce the incidence of cardiovascular events in these patients. 
These data have been supported by large multicenter randomized prospective trials 
[9] Therefore, it becomes incumbent on the vascular surgeon to also consider these 
as part of the treatment plan when evaluating a patient with peripheral arterial 
disease. [Q2: A, B, C, D, E] 

Evolution of vascular surgery techniques in the past decade, combined with the 
availability of an adequate venous conduit, has permitted a liberal and aggressive 
approach to salvage ischemic limbs caused by advanced atherosclerosis. This 



170 Vascular Surgery 

approach is epitomized by the construction of arterial bypasses to the terminal 
branches of tibial vessels [10]. However, significant numbers of patients continue to 
face the threat of a major amputation because of insufficient vein necessary to 
perform a totally autogenous bypass to one of the infrapopliteal arteries. In these 
cases, less durable grafts made of prosthetic material must be used if limb salvage is 
to be attempted. Accordingly, several adjunctive techniques have been designed in 
an attempt to improve the poor patency results achieved with prosthetic bypasses. 
These include the administration of immediate and chronic anticoagulants [11], the 
construction of a vein patch or cuff at the distal anastomosis to prevent occlusion 
by intimal hyperplasia [12, 13], and the creation of an arteriovenous fistula to 
increase graft blood flow in high-outflow-resistance systems [14, 15]. Despite initial 
enthusiasm, the results using cadaveric vein have been poor and resulted in its very 
limited use [16, 17]. [Q3: C] If the popliteal artery had been not as diseased, an 
attempt at subintimal angioplasty with angiography or with duplex guidance may 
also be considered [18, 19]. 

The expected long-term mortality of this patient is 24-50 percent at 4-5 years and 
is due mostly to myocardial ischemia [20]. The expected patency of these tech- 
niques is 50-60 percent at 3-4 years [20-23]. The expected limb salvage rates are 
70-80 percent at 3-4 years [20-23]. [Q4: B] 

Based on these data, we would suggest that there is no role for amputation or 
sympathectomy in this particular patient. However, if the patient had prohibitive 
cardiac risks, had nonreconstructable disease, or was already so neurologically 
impaired that the limb was not of any utility to the patient, then observation, 
primary amputation, hyperbaric oxygen therapy or perhaps experimental protocols 
involving angiogenesis factors may be in order. [Q5] 



References 

1. Carpenter JP, Owen RS, Baum RA, Cope C, Barker CF, Berkowitz HD, et al. Magnetic resonance 
angiography of peripheral runoff vessels. J Vase Surg 1992; 16:807. 

2. Cambria RP, Kaufman JA, L'ltalien GJ, Gertler JP, LaMuraglia GM, Brewster DC, et al. Magnetic res- 
onance angiography in the management of lower extremity arterial occlusive disease: a prospective 
study. J Vase Surg 1997;25:380-89. 

3. Hingorani A, Ascher E, Markevich N, Kallakuri S, Hou A, Schutzer R, Yorkovich W. Magnetic reso- 
nance angiography versus duplex arteriography in patients undergoing lower extremity revascular- 
ization: which is the best replacement for contrast arteriography? J Vase Surg. 2004;39(4):7 17-22. 

4. Ascher E, Mazzariol F, Hingorani A, Salles-Cunha S, Gade P. The use of duplex ultrasound arterial 
mapping as an alternative to conventional arteriography for primary and secondary infrapopliteal 
bypasses. Am J Surg 1999;178:162-5. 

5. Mazzariol F, Ascher E, Salles-Cunha SX, Gade P, Hingorani A. Values and limitations of duplex 
ultrasonography as the sole imaging method of preoperative evaluation for popliteal and 
infrapopliteal bypasses. Ann Vase Surg 1999;13:1-10. 

6. Mazzariol F, Ascher E, Hingorani A, Gunduz Y, Yorkovich W, Salles-Cunha S. Lower-extremity 
revascularisation without preoperative contrast arteriography in 185 cases: lessons learned with 
duplex ultrasound arterial mapping. Eur J Vase Endovasc Surg 2000;19:509-15. 

7. Ascher E, Markevich N, Schutzer RW, Kallakuri S, Hou A, Nahata S, et al. Duplex arteriography prior 
to femoral-popliteal reconstruction in claudicants: a proposal for a new shortened protocol. Ann 
Vase Surg 2004;18(5):544-51. 

8. Ascher E, Hingorani A, Markevich N, Schutzer R, Kallakuri S. Acute lower limb ischemia: the value 
of duplex ultrasound arterial mapping (DUAM) as the sole preoperative imaging technique. Ann 
Vase Surg. 2003;17(3):284-9. 

9. Hackam DG. Cardiovascular risk prevention in peripheral artery disease. J Vase Surg 
2005;41(6):1070-3. 



Chronic Critical Limb Ischemia 171 

10. Ascer E, Veith FJ, Gupta SK. Bypasses to plantar arteries and other tibial branches: an extended 
approach to limb salvage. J Vase Surg 1988;8:434-41. 

11. Flinn WR, Rohrer MJ, Yao JST, McCarthy WJ, Fahey VA, Bergan J J. Improved long-term patency of 
infragenicular polytetrafluoroethylene grafts. J Vase Surg 1988;7:685. 

12. Siegman FA. Use of the venous cuff for graft anastomosis. Surg Gynecol Obstet 1979;148:930. 

13. Miller JH, Foreman RK, Ferguson L, Faris I. Interposition vein cuff for anastomosis of prosthesis to 
small artery. Aust N Z J Surg 1984;54:283. 

14. Dardik H, Sussman B, Ibrahim IM, Kahn M, Svoboda J, Mendes K, Dardik I. Distal arteriovenous 
fistula as an adjunct to maintain arterial and graft patency for limb salvage. Surgery 1983; 94:478. 

15. Ascer E, Veith FJ, White-Flo res SA, Morin L, Gupta SK, Lesser ML. Intraoperative outflow resistance 
as a predictor of late patency of femoropopliteal and infrapopliteal arterial bypasses. J Vase Surg 
1987;5:820. 

16. Albertini JN, Barral X, Branchereau A, Favre JP, Guidicelli H, Magne JL, Magnan PE. Long-term 
results of arterial allograft below-knee bypass grafts for limb salvage: a retrospective multicenter 
study. J Vase Surg 2000;31:426-35. 

17. Harris L, O'Brien-Irr M, Ricotta J J. Long-term assessment of cryopreserved vein bypass grafting 
success. J Vase Surg 2001;33:528-32. 

18. Hingorani A, Ascher E, Markevich N, Yorkovich W, Schutzer R, Hou A, et al. The role of the 
endovascular surgeon for lower extremity ischemia. Acta Chir Belg 2004;104(5):527-31. 

19. Ascher E, Marks NA, Schutzer RW, Hingorani AP. Duplex-guided balloon angioplasty and stenting 
for arterial occlusive disease: an alternative in patients with renal insufficiency. J Vase Surg (in 
press). 

20. Neville R, Tempesta B, Sidway A. Tibial bypass for limb salvage using polytetrafluoroethylene and a 
distal vein patch. J Vase Surg 2001;33:266. 

21. Ascher E, Gennaro M, Pollina RM, Ivanov M, Yorkovich WR, Ivanov M, Lorensen E. Complementary 
distal arteriovenous fistula and deep vein interposition: a five-year experience with a new technique 
to improve infrapopliteal prosthetic bypass patency. J Vase Surg 1996;24:134-43. 

22. Kreienberg PB, Darling RC 3rd, Chang BB, Paty PS, Lloyd WE, Shah DM. Adjunctive techniques to 
improve patency of distal prosthetic bypass grafts: polytetrafluoroethylene with remote arteriove- 
nous fistulae versus vein cuffs. J Vase Surg 2000;3 1:696. 

23. Hingorani AP, Ascher E, Markevich N, Schutzer RW, Mutyala M, Nahata S, et al. A ten-year experi- 
ence with complementary distal arteriovenous fistula and deep vein interposition for infrapopliteal 
prosthetic bypasses. J Vase Endovasc Surg (in press). 



20. Popliteal Artery Entrapment 



Luca di Marzo and Norman M. Rich 



A 26-year-old female presented with a 6-year history of cold foot, paraesthesia 
and cramping in both legs after intensive physical training. She was a recre- 
ational bodybuilder and complained of her symptoms mostly after sporting 
activity. Symptoms subsequently became more severe, with cramping requiring 
20 minutes to release after sport. 



Question 1 

What is the presentation of cases with popliteal artery entrapment? 

A. The patient is often sporty with muscular calves. 

B. The patient often complains of rest pain or necrosis. 

C. The patient often complains of mild symptoms with paraesthesia, cold foot and 
cramping after intensive physical training. 

D. Venous complaints are often encountered. 

E. Symptoms due to arterial embolisation are often present. 

The patient smoked 20 cigarettes a day. Her past medical history included pan- 
creatitis when she was 12 years old and tonsillectomy when she was 19 years old. On 
physical examination, she appeared healthy, with both legs appearing athletic. 
Lower-limb pulses were normal, but bilateral pedal pulse reduction was noted after 
calf muscle contraction. A popliteal artery entrapment (PAE) was therefore sus- 
pected, and the patient was sent for noninvasive vascular evaluation. Doppler and 
colour Doppler showed normal posterior tibial and popliteal recordings, with signal 
disappearance on both legs during calf muscle contraction. Doppler examination 
was conducted with the patient supine recording the posterior tibial artery during 
manoeuvre (Fig. 20.1). Colour Doppler was performed, with the patient prone, and 
the sample volume placed in the popliteal artery. Muscular contraction of the calves 
showed an arterial occlusion on colour flow imaging (Fig. 20.2). 



173 



174 
Left 



Vascular Surgery 





Manoeuvre 



xjr~ 



m ~ i ,, ! ^" * ' i_ i n i 





"^" 



Right 



A A A . L 



Manoeuvre 



m \^>~ if ~» ' ~ > ^i " '■ i. 






Fig. 20.1. Continuous-wave Doppler recording the posterior tibial artery during manoeuvre. 




Fig. 20.2. Colour Doppler during muscular contraction of the calves, showing arterial occlusion. 



Diagnosis of bilateral PAE was made. Arteriography was conducted to confirm 
the diagnosis: it showed normal popliteal arteries, with right severe stenosis and left 
occlusion during calf muscle contraction (Fig. 20.3). Magnetic resonance angiogra- 
phy (MRA) was attempted, which demonstrated bilateral popliteal occlusion during 
manoeuvre (Fig. 20.4). 



Question 2 

How will you make the diagnosis of PAE? 

A. Doppler can detect PAE. 

B. Arteriography is only carried out preoperatively to confirm results of ultrasound 
scans. 

C. MRA may be diagnostic in the hands of an experienced practitioner. 

D. Duplex scanning can detect PAE. 

E. Computed tomography (CT) scanning can detect PAE. 



Popliteal Artery Entrapment 



175 




Fig. 20.3. Arteriography showing normal popliteal arteries, with right severe stenosis and left occlusion during 
calf muscle contraction. 



Question 3 

Which of the following statements regarding angiograms of a patient with PAE are 
correct? 



A. Normal angiograms at rest are often encountered in entrapments. 

B. The angiograms show an occlusion or severe stenosis during calf muscles 
contractions. 

C. Three-vessel run-off is often encountered in PAE. 

D. An arterial occlusion is encountered in PAE diagnosed at a late stage. 

E. A post-stenotic aneurysm may be encountered. 



176 



Vascular Surgery 




Fig. 20.4. MRA demonstrating bilateral popliteal occlusion during manoeuvre. 



The patient was considered for bilateral surgical treatment. A posterior approach 
to the popliteal fussa was made through a Z-shaped incision. The medial gastrocne- 
mius muscle had a large accessory head with a lateral and cranial insertion, causing 
bilateral compression of the popliteal artery and vein. This head was resected on 
both legs, without any need for muscular reconstruction. 



Question 4 



Which of the following statements regarding the treatment of PAE are correct? 

A. Musculotendineous sectioning is the treatment of choice in patients with a 
normal popliteal artery. 

B. Vascular reconstruction should be limited to cases with stable arterial 
impairment. 

C. If vascular reconstruction is planned, then the use of autologous vein is 
mandatory. 

D. The posterior approach is recommended to expose all the structures causing 
compression. 



Popliteal Artery Entrapment 1 77 

E. The structure causing PAE must be sectioned completely, as incomplete sectioning 
may cause recurrence. 

Question 5 

Which of the following statements regarding the incidence of entrapment are 
correct? 

A. The medial gastrocnemius muscle is involved in almost 80 per cent of cases of 
PAE. 

B. Venous entrapment is described more often than arterial entrapment. 

C. Venous entrapment is concomitant in 20 per cent of cases of PAE. 

D. More than one structure may be the cause of arterial entrapment. 

E. Classification of arterial entrapment includes 12 different types. 

The postoperative course was uneventful and the patient was discharged 5 days 
after surgery, returning back to normal activity after 3 weeks. Follow-up demon- 
strated complete regression of symptoms. Ultrasound examinations (Doppler and 
colour Doppler) showed normal popliteal flow with negative response to PAE 
manoeuvres 1 month after surgery. The patient is now doing sport (swimming) 
again without any further complaints. 



Commentary 

The first case of PAE was treated surgically in 1959 in a 12-year-old boy 
complaining of claudication after walking 300 m. At surgical exploration, 
Hamming [1] at Leyden University in The Netherlands found an occluded artery 
with an anomalous course medial to the medial gastrocnemius muscle. He 
transected the muscle and performed a successful popliteal artery thromboen- 
darterectomy. 

A previous description of the disease was reported in 1879 by Stuart [2], a 
medical student at the University of Edinburgh. During the dissection of an ampu- 
tated leg of a 64-year-old man, he observed the popliteal artery coursing around the 
medial head of the gastrocnemius muscle and aneurysmal changes in the popliteal 
artery distal to the point of external muscular compression. 

Since than, many case reports have been published. A few authors have published 
small series [3-6]. Unfortunately, the papers that were collected were missing 
details and showed poor patient follow-up [7]. 

In Rome in 1998, the Popliteal Vascular Entrapment Forum was founded. 
Surgeons from around the world with the greatest experience in this field world 
were invited as founding members of the forum. Great effort was addressed to 
collect different series with comparable criteria. The criteria established by the 
Society for Vascular Surgery (SVS) were reviewed and accepted, with some minor 
changes. Common opinion was to consider both arterial and venous entrapment as 
a common disease defined as vascular entrapment. The functional form of 



178 Vascular Surgery 

Table 20.1 . Classification of popliteal vascular entrapment 

Type Features 

I Popliteal artery running medial to the medial head of gastrocnemius 

II Medial head of gastrocnemius attached laterally 

III Accessory slip of gastrocnemius 

IV Popliteal artery passing below popliteal muscle and medial head of gastrocnemius 

V Primary venous involvement 

VI Variants 

F Functional entrapment 



entrapment was discussed. This was first described by Rignault et al. [8] in 1985, 
and describes cases in which the anatomy of the popliteal fussa is normal. 
Symptoms are usually caused by hypertrophy of the muscles determining a com- 
partment syndrome [8, 9]. Functional entrapment was included in the classification 
as type F (Table 20.1). 

Popliteal artery entrapment is no longer a rare disease. It is encountered more 
and more often, particularly in young adults. Athletes practising sports causing 
hypertrophy of the limb muscles are at higher risk due to an anomalous relation- 
ship of the popliteal artery and its surrounding musculotendineous structures. The 
artery is compressed each time the leg moves, causing peripheral ischaemia during 
intensive exercise. With time, this intermittent arterial trauma may give rise to 
stable arterial damage, with occlusion or post-stenotic aneurysm. Early diagnosis 
and treatment play an important role in limiting surgical treatment to the section- 
ing of the structure causing the arterial compression. [Q1: A, C] 

The diagnosis of PAE is based primarily on ultrasound scanning. Both continu- 
ous-wave Doppler and colour Doppler are able to detect the presence of an arterial 
compression due to entrapment. The manoeuvres to be performed are well 
described and are able to detect suspected cases [7]. Great care should be taken to 
suspect early cases of PAE in patients complaining of minor symptoms (paraesthe- 
sia, cold foot and cramping after intensive physical training). Arteriography is 
limited to cases with positive ultrasound examinations, and it requires great care in 
repeating the manoeuvres to confirm the popliteal compression. MRA may be diag- 
nostic, but it needs latest-generation apparatus and the input of a radiologist with 
great experience in both the disease and the imaging method. At present, MRA 
cannot be considered a first-choice examination, but it is reasonable to assume that 
in the future it will substitute angiography in the diagnosis of entrapment. [Q2: A, B, 
C,D][Q3:TrueA,B,C,D,E] 

Surgical treatment consists of sectioning the musculotendineous structure 
causing the entrapment. The anomalous structure needs to be sectioned entirely 
in order to avoid recurrence of the entrapment due to hypertrophy of the 
remaining anomalous muscle. It is important to remember that complete expo- 
sure of the popliteal fussa is obtained through a posterior approach. The medial 
approach limits the view of the medial gastrocnemius muscle. In our opinion, 
this exposure should be limited to cases in which the arterial impairment is 
extended to the tibial vessels and a distal reconstruction needs to be planned. 
However, early diagnosis allows surgical treatment to be limited to the muscle 
sectioning, which should be considered the first-choice treatment. When a 
popliteal severe stenosis, occlusion or aneurysm is present, then an arterial 



Popliteal Artery Entrapment 1 79 

reconstruction is indicated. In this case, we recommend the use of autologous 
material to reconstruct the artery. This improves the long-term patency rate. 
Great effort should be paid for alternative vein preparation when the saphenous 
vein is unavoidable. [Q4: A, B, C, D, E] 

The medial gastrocnemius muscle is often the cause of compression. However, 
more than 20 different anatomical variants have been described, and sometimes 
multiple and complex structures maybe associated with the medial gastrocnemius 
muscle in causing PAE. The popliteal vein is involved in the compression in 20 per 
cent of cases affected by PAE. Moreover, isolated popliteal vein entrapment is 
described with increasing frequency in the literature. [Q5: A, C, D] 



References 

1. Hamming JJ. Intermittent claudication at an early age due to anomalous course of the popliteal artery. 
Angiology 1959;10:369-70. 

2. Stuart PTA. Note on a variation in the course of the popliteal artery. J Anat Physiol 1879;13:162. 

3. Bouhoutsos J, Daskalakis E. Muscular abnormalities affecting the popliteal vessels. Br J Surg 
1981;68:501-6. 

4. Rich NM, Collins GJ, McDonald PT, Kozloff L, Claget PG, Collins JT. Popliteal vascular entrapment. Its 
increasing interest. Arch Surg 1979;114:1377-84. 

5. Di Marzo L, Cavallaro A, Mingoli A, Sapienza P, Tedesco M, Stipa S. Popliteal artery entrapment syn- 
drome: the role of early diagnosis and treatment. Surgery 1997;122:26-31. 

6. Levien L, Veller MG. Popliteal artery entrapment syndrome: more common than previously recog- 
nized. J Vase Surg 1999;30:587-98. 

7. Di Marzo L, Cavallaro A, Sciacca V, Mingoli A, Stipa S. Natural history of entrapment of the popliteal 
artery J Am Coll Surg 1994;178:553-6. 

8. Rignault DP, Pailler JL, Lunel F. The "functional" popliteal entrapment syndrome. Int Angiol 
1985;4:341-3. 

9. Turnipseed WD, Pozniak M. Popliteal entrapment as a result of neurovascular compression by the 
soleus and plantaris muscles. J Vase Surg 1992;15:285-94. 



21. Adventitial Cystic Disease of the 
Popliteal Artery 

Bernard H. Nachbur and Jon Largiader 



Casel 

A 49-year-old female presented with a 3-week history of left calf intermittent claudi- 
cation at 150 m, which had occurred suddenly and without preliminary herald signs. 
The patient was a nonsmoker and had no risk factors, such as hypertension, dia- 
betes or hyperlipidaemia. She was engaged in regular sporting activity, playing 
tennis all year round and skiing in the winter. She thought at first that it might be a 
strained muscle and would subside spontaneously. This did not happen and she 
sought medical advice. 

At clinical examination, the popliteal and pedal pulses of the left leg were 
barely palpable and were absent after exercise. Angiological examination of the 
right leg was normal. The ankle systolic pressure at the right side was 128 mm Hg 
with a slight rise to 132 mm Hg after exercise. On the left side, ankle systolic 
pressure at rest was 88 mm Hg with a post-exercise reduction to 58 mm Hg. 
On duplex sonography, a 5-cm long polycystic swelling surrounding the left 
popliteal artery was found to be the cause of occlusion of the popliteal artery. The 
superficial femoral artery and the infrapopliteal arteries showed no trace of 
atherosclerotic disease. Ultrasonography demonstrated that the content of the 
cyst was clear and homogeneous. No other cause for popliteal occlusion was 
found. 



Question 1 

What is the aetiology of this condition? 

An angiogram (Fig. 21.1) showed a 3-cm long subtotal occlusion of the proximal 
popliteal artery suggesting medial compression, an eccentric form of occlusion rem- 
iniscent of an hourglass stenosis (scimitar sign). The top frame of the cross-section 
of the computed tomography (CT) scan performed at the same time shows an 
adventitial cyst of approximately 1.5 cm in diameter adjacent to the artery, actually 
within the arterial wall. 

181 



182 



Vascular Surgery 










OIS3LJ5 JATI^B-. V|MU 

IF 3 *■ * 

Tuoai 



• I T 

est % 

3jJ3T JPfGE* c- 



E HQTftHQE 
etQJ ZHflH 15H3U* 



ei t 
sap- a 





Fig. 21.1. Hourglass-shaped subtotal occlusion of the middle portion of the popliteal artery (scimitar sign) 
caused by compression by a cyst in the arterial wall, which can be seen in the top panel of the cross-section of 
the CT scans. 



Adventitial Cystic Disease of the Popliteal Artery 1 83 

Question 2 

Which of the following statements regarding adventitial cystic disease are correct? 

A. It affects only the popliteal artery. 

B. It can occur elsewhere, such as in arteries near the hip, wrist or ankle joints. 

C. It presents with initial signs of acute occlusive disease. 

D. It usually begins with intermittent claudication. 

E. It can be elicited by loss of pedal pulses during hyperextension of the leg. 

F. The cyst is calcified and contains atheromatous material. 

G. The cyst contains a viscous gelatinous fluid. 

The popliteal artery was laid free posteriorly through a S-shaped popliteal inci- 
sion. The arterial wall contained a cyst filled with a gelatinous mucoid yellowish 
substance. The occluded arterial segment was resected and replaced by interposi- 
tion of a segment of saphenous vein. Fig. 21.2 shows the popliteal artery before and 
after surgery with complete normalisation of patency. 



Case 2 

A 49-year-old woman complained of sporadic episodes of intermittent claudica- 
tion of varying intensity [1]. At times, she could walk freely; at other times, after 
physical exercise with bending of the knee, intermittent claudication would occur 
after walking distances of 200-300 m. Angiography revealed only discrete semilu- 
nar narrowing of the middle portion of the popliteal artery, as shown in Fig. 21.3 
(scimitar sign). At the time of this examination, the patient had hardly any 
complaints. 



Question 3 

Adventitial cystic disease of the popliteal artery can be diagnosed reliably by: 

A. Duplex coloured sonography. 

B. Injection of indium 111 and scintigraphy. 

C. The semilunar sign (scimitar sign) or hourglass sign at angiography. 

D. A meniscus-shaped proximal occlusion at angiography. 

E. T2-weighted magnetic resonance imaging (MRI). 

F. Systolic bruit in the hollow of the knee. 

G. Intravascular ultrasound imaging. 
H. CT scanning. 



184 



Vascular Surgery 




Fig. 21.2. Popliteal adventitial cyst before and after segmental resection and interposition of a segment of 
autologous vein. 



Question 4 



What are the treatment options? 

The popliteal artery was laid free posteriorly through a popliteal incision. The 
arterial wall was surrounded by a 5-cm long polycystic tumour in the centre of 
which was a 3-mm wide stem that could be followed to the knee joint. A fine probe 
was introduced for injection of contrast medium. The cyst took the appearance of a 
Baker cyst, which was filled with a jelly-like yellowish mucoid substance. The cyst 



Adventitial Cystic Disease of the Popliteal Artery 



185 




Fig. 21.3. Angiography of the popliteal artery, with a discrete semilunar deformity (arrow pointing to the scim- 
itar sign). At the time of this angiography, the patient was in momentary clinical remission. 



was found to be lying in the outer layers of the adventitia and was removed easily 
without causing any damage to the artery itself (Figs 21.4 and 21.5). 

The varying clinical presentation of intermittent claudication in this case can be 
explained by pressure changes occurring within the cyst during different physical 
activity [1]. Histologically, the wall of the cyst consisted of collagenous connective 
tissue covered on the inside by a single interrupted or several layers of cuboid cells 



186 



Vascular Surgery 






. 


■ 


i 
i 


' : 


ti ' ' ? a 






< 


<■'* 






R «rtST 

* i p- * 




* 

V 


w 


k t.'n 


N* 










■v _ 


V *i 





Fig. 21.4. f/eftj The whole extent of the 6-cm long cyst surrounding the popliteal artery. 
Fig. 21.5. (right)lhe perivascular cyst being resected, with the artery remaining intact. 



■ 





Jl P . "'i» 



Fig. 21.6. The wall of the cyst covered on the inside by a single interrupted or several layers of cuboid cells akin 
to synovial mesothelium. 



Adventitial Cystic Disease of the Popliteal Artery 



187 



akin to synovial mesothelium [2] (Fig. 21.6). The stem connecting with the knee 
joint had a similar structure. The lumen of both cyst and stem contained viscous 
basophil fluid; they are therefore best likened to ganglions. 



Commentary 

Trauma has been ruled out overwhelmingly on the grounds that the disease would 
be seen predominantly in people engaged in competitive sports: this is not the case. 
All cases of adventitial cystic disease reported in the literature have occurred in 
nonaxial vessels during limb differentiation and development. It is therefore postu- 
lated that during limb bud development, cell rests derived from condensations of 
mesenchymal tissue destined to form the knee, hip, wrist or ankle joints are incor- 
porated into the nearby and adjacent nonaxial vessels from vascular plexuses 
during the same stage of development, and in close proximity to the adjacent con- 
densing joint structures [3]. It is postulated further that these cell rests are then 
responsible for the formation of adventitial cystic disease in adult life, when mucoid 
material secreted results in a mass lesion within the arterial or venous wall [3]. 
Fig. 21.7 shows a row of cross-sections of a resected and totally occluded popliteal 




Fig. 21.7. Cross-section through an artery with a large adventitial cyst and compression of the arterial lumen of 
the resected popliteal artery. 



188 Vascular Surgery 

segment. In this case, the cyst is clearly in the midst of the arterial wall and does not 
appear to be located in the adventitia. 

According to the hypothesis of Levien and Benn [3], popliteal adventitial cystic 
disease manifests itself in adults. Early cases manifest in the third decade, but most 
cases occur in the fourth and fifth decades; it occurs less frequently in later stages of 
life [4]. The male : female ratio is about 5 : 1. In summary, there is little doubt that 
popliteal cystic disease is congenital. [Q1] 

Popliteal adventitial cysts are located mostly in outer levels, i.e. in the adventitia 
of the popliteal artery, but they may also occur in the common femoral artery adja- 
cent to the hip joint along the iliofemoral axis, in locations near the elbow or the 
wrist, and in veins [5]. A total of 45 extrapopliteal localisations have been described. 
These extrapopliteal locations account for 20-25% of all cases of adventitial cystic 
disease. Carlsson et al. [6] have also observed adventitial cystic disease in the 
common femoral artery. [Q2: B, D, G] 

Because of the sometimes varying degree of intermittent claudication or occasional 
disappearance of symptoms, the disease can be mistaken for a popliteal entrapment 
syndrome. Noninvasive techniques have vastly improved diagnosis. Duplex coloured 
scanning followed by T2-weighted MRI now appear to be the best choice. Both 
methods are capable of visualising the cyst surrounding the popliteal artery and 
ruling out the popliteal entrapment syndrome [4]. Koppensteiner et al. [7] have 
shown that intravascular ultrasound imaging can reliably identify adventitial cystic 
disease as well. Digital subtraction angiography is necessary to define the degree of 
stenosis or the length of occlusion. Stenotic lesions have an hourglass appearance or 
present with a semilunar impression (the scimitar sign) [4]. [Q3: A, C, E, G, H] 

The treatment options depend on the degree of stenosis and whether the popliteal 
artery is occluded. In the case of total occlusion, most authors have resorted to total 
resection of the affected popliteal arterial segment with interposition either of autol- 
ogous vein or ring-enforced polytetrafluoroethylene (PTFE) grafts. The initial 
success rate is reportedly almost 90 per cent [4]. 

If the cyst lies within the adventitia and surrounds and compresses the artery 
without having given rise to total occlusion, as in our second case, then the artery 
does not have to be resected if the cyst can be removed entirely [1]. Partial removal 
of the cyst is thought to bear the risk of recurrence [1]. If a connecting stem usually 
accompanied by a small collateral artery is present, then this should be resected at 
the level of the knee capsule to avoid recurrence [2]. The initial success rate in 68 
cases treated accordingly is 94 per cent [4]; in our own experience, it was successful 
in case 2 described above [1]. 

There is the possibility of resecting only part of the artery, e.g. the medial vascular 
aspect that bears the cyst, and then replacing the wall defect with a vein patch. This 
approach has been used in a small number of patients, with success in three of four 
cases [4]. Percutaneous transluminal angioplasty (PTA) has been performed just 
once, and failed. PTA should therefore probably be discarded as an treatment option. 

An interesting series of seven cases has been reported by Do et al. [8]. They for- 
warded a 14-gauge needle with real-time ultrasonic guidance transcutaneously directly 
into the cyst and aspirated its contents in cases presenting with stenosis only (but not 
in the presence of total occlusion). This was carried out on an outpatient basis, with a 
100 per cent success rate. Follow-up colour duplex sonography performed between 1 
and 32 months after the procedure showed no recurrent stenosis [8]. 

While the method of percutaneous aspiration of a popliteal cyst guided by ultra- 
sonography is appealing because it can be done on an outpatient basis and mini-inva- 



Adventitial Cystic Disease of the Popliteal Artery 1 89 

sively, the question of recurrence is not settled since the cyst remains in place; hence 
the capacity to form mucinous substance remains and with it the possibility of recur- 
rence. Although Do et al. know of no recurrence in their cases followed up for 1-32 
months, there is a definite need for a more systematic long-term follow-up, which 
should be conducted in all cases in which the cyst has not been removed by resection. 

There is the occasional report of percutaneous clot lysis of occluded popliteal 
arteries followed by aspiration of the contents of the cysts. This method was 
reported by Samson and Willis [9] to be successful, but its reliability has not been 
proven by others. There is hardly a valid contraindication against surgical removal 
of an occluded popliteal segment in the presence of occlusion, and this is probably 
the method of choice that offers the greatest chances for complete recovery. 

Finally, there are reports of spontaneous resolution of the popliteal cysts [10, 
1 1 ] . It must be assumed, therefore, that occasionally cysts can burst or their con- 
tents escape into the periarticular space. This mechanism has been surmised by 
Soury et al. [10]. 

In conclusion, the treatment of choice remains surgical resection, either of the 
cyst alone if it surrounds the artery or of the occluded segment if total occlusion 
and appositional thrombosis has occurred. In this case, vein graft interposition 
should be performed. In expert hands, percutaneous transluminal aspiration has 
been shown to be efficacious. [Q4] 



Acknowledgement 

Special thanks go to Professor Jon Largiader, who offered the documentation of the 
two patients operated on by him at the University Hospital of Zurich. 



References 

1. Largiader J, Leu HJ. Sogenannte zystische Adventitiadegeneration der Arteria poplitea mit 
Stielverbindung zum Kniegelenk. Vasa 1984;13:267-72. 

2. Leu HJ, Largiader J, Odermatt B. Pathogenesis of the so-called adventitial degeneration of peripheral 
blood vessels. Virchow Arch A 1984;404:289-300. 

3. Levien LJ, Benn CA. Adventitial cystic disease: a unifying hypothesis. J Vase Surg 2000;28:193-205. 

4. Tsolakis IA, Walvatne CS, Caldwell MD. Cystic adventitial disease of the popliteal artery: diagnosis 
and treatment. Eur J Vase Endovasc Surg 1998;15:188-94. 

5. Chakfe N, Beaufigeau M, Geny B, Suret-Canale MA, Vix J, Groos N, et al. Extra-popliteal localizations 
of adventitial cysts. Review of the literature. J Mai Vas 1997;22:79-85. 

6. Carlsson S, Sandermann J, Hansborg N. Adventitial cystic disease in the common femoral artery. 
Ann Chir Gynaecol 2001;90:63-4. 

7. Koppensteiner R, Katzenschlager R, Ahmadi A, Staudacher M, Horvat R, Polterauer P, Ehringer H. 
Demonstration of cystic adventitial disease by intravascular ultrasonic imaging. J Vase Surg 
1996;23:534-6. 

8. Do DD, Braunschweig M, Baumgartner I, Furrer M, Mahler F. Adventitial cystic disease of the 
popliteal artery: percutaneous guided aspiration. Radiology 1997;2303:743-6. 

9. Samson RH, Willis PD. Popliteal artery occlusion caused by cystic adventitial disease: successful by 
urokinase followed by non-resectional cystotomy. J Vase Surg 1990;12:591-3. 

10. Soury P, Riviere J, Watelet J, Peillon C, Testart J. Spontaneous regression of a sub -adventitial cyst of 
the popliteal artery. J Mai Vase 1995;20:323-5. 

11. Owen ER, Speechly-Dick EM, Kour NW, Wilkins RA, Lewis JD. Cystic adventitial disease of the 
popliteal artery - a case of spontaneous resolution. Eur J Vase Surg 1990;4:319-21. 



22. The Obturator Foramen Bypass 

Andries }. Kroese and Lars E. Staxrud 



A 62-year-old man presented with a 2-week history of continuous pain in the left 
lower abdomen radiating to the groin. For several weeks, he had complained of 
general malaise, including tiredness and poor appetite, and diarrhoea once or 
twice per day. His general practitioner palpated a pulsating, tender mass in the 
left groin and referred him to the department of vascular surgery at the nearby 
university hospital. Three years previously, he had been operated upon with a 
Dacron aorto-bifemoral bypass for critical ischaemia and intermittent claudica- 
tion in the left and right lower limbs, respectively. 

On admission, the patient was in a relatively good general condition, although 
his body temperature was 38.5°C, pulse rate was 96 bpm, and blood tests showed 
an elevated sedimentation rate, C-reactive protein (CRP) and leucocyte count. 
Palpation of the left iliac fossa was slightly painful. The inguinal swelling was 
covered by erythematous skin and was estimated to be approximately 4 cm in 
diameter. 



Question 1 

What is the most likely diagnosis at this stage? 

A. False aneurysm. 

B. Infected Dacron graft. 

C. Lymphadenitis. 

D. Incarcerated inguinal or femoral hernia. 

E. Incarcerated obturator hernia. 

Based on the clinical signs and symptoms, treatment with broad-spectrum anti- 
biotics was started. 



191 



192 Vascular Surgery 

Question 2 

Which of the following investigations should be carried out to confirm the diagno- 
sis, and in what order? 

A. Duplex scanning. 

B. Arteriography. 

C. Computed tomography (CT) scanning with aspiration of perigraft fluid for 
Gram staining and culture. 

D. Magnetic resonance imaging (MRI). 

E. Leucocyte-labelled scintigraphy. 

F. Surgical exploration. 

Ultrasonography revealed that the Dacron graft and femoral arteries were not 
pathologically dilated but that the anastomotic site was surrounded by fluid. Some 
of this perigraft fluid was aspirated and was found to contain coagulase-negative 
staphylococci (CNS). Antibiotic treatment was adjusted accordingly. 

Question 3 

Vascular graft infection in the groin may be treated without resecting the graft itself 
when there is: 

A. Less than 2.5-cm diameter false aneurysm formation. 

B. An infected anastomosis, but without bleeding. 

C. A thrombosed graft. 

D. No septicaemia. 

MRI and CT scanning revealed that only the left limb of the bifurcation graft was 
infected, most likely only in the groin, involving the site of the anastomosis. 

Question 4 

What treatment options, in addition to antibiotics, are available for the manage- 
ment of an infected vascular graft in the groin? 

A. Excision with or without a revascularisation procedure. 

B. Repeated extensive wound debridement, and insertion of gentamicin mats. 

C. Debridement, skin closure, and insertion of a closed irrigation system. 

D. Debridement and muscle flap transposition. 

E. None; use long-term antibiotic treatment only. 



The Obturator Foramen Bypass 193 

Since the proximal limit of graft infection could not be ascertained, it was decided 
to operate on the patient with a partial graft resection. Because the indication for 
primary operation had been critical ischaemia due to multilevel atherosclerotic 
disease, revascularisation was planned. Therefore, a preoperative angiography was 
performed, which showed signs of progressive atherosclerosis as compared with 
previous angiograms. The proximal part of the left superficial femoral artery was 
occluded, whereas the distal part was patent. Of the crural arteries, only the poste- 
rior tibial was patent. The profunda femoral artery was patent but peripherally 
stenotic. In the right lower extremity, the superficial femoral artery was occluded, 
but the profunda artery and three crural arteries were patent, although partially 
stenotic. Based on these findings, an obturator foramen bypass (OFB) on the left 
side was planned. 

Under general anaesthesia, an 8-mm ring-reinforced polytetrafluoroethylene 
(PTFE) graft was implanted as an OFB between the proximal part of the limb of the 
previously implanted Y-graft and the distal superficial femoral artery. During the 
same operation, the distal part of the infected graft was resected. 

Question 5 

What is the most common indication for an OFB procedure? 

A. Infected femoral (false) aneurysm. 

B. Revascularisation with extensive local trauma. 

C. Tissue scarring in the groin subsequent to radical tumour surgery, radiation or 
burns. 

D. Sciatic artery aneurysm exclusion. 

E. Infection confined to the distal part of an aortofemoral bypass graft. 

Question 6 

Describe briefly how you would perform an OFB procedure. 

After a hypotensive period on the first postoperative day, the left lower limb 
showed clinical signs of increased ischaemia. Blood pressure at the ankle was 60 
mm Hg and the ankle brachial pressure index (ABPI) was 0.4 - slightly lower than 
preoperatively. Duplex scanning could not rule out a technical defect of the OFB, for 
example kinking. Therefore, an angiography via the right groin was performed, 
which did not show any major technical defects (Fig. 22.1). Subsequently anticoagu- 
lation therapy with warfarin was started. 

Question 7 

What is the least frequent complication of an OFB? 

A. Urinary bladder injury. 

B. Injury of the obturator nerve and blood vessels. 



194 



Vascular Surgery 




Fig. 22.1 . Postoperative angiography of an OFB without signs of technical defects. 



C. Kinking of the graft due to erroneous transmuscular tunnelling. 

D. Infection of the obturator graft. 

E. Bleeding, thrombosis. 

F. Injury of the internal iliac artery. 

The further postoperative course was uneventful. Two weeks later, the patient 
was discharged with complaints of claudication in the left lower extremity and a 
walking distance of approximately 50 yards. Oral antibiotics were to be continued 
for 3 months and anticoagulation indefinitely. 



Question 8 

What alternative revascularisation procedures after removal of an infected vascular 
graft in the groin may be considered? 

A. Subintimal angioplasty of the native iliac artery. 

B. Semi-closed endarterectomy (ring-stripping) of the iliac artery. 

C. Axillofemoral bypass by lateral route. 

D. Subvulvular bypass. 

E. Subscrotal bypass. 

F. Bypass with autologous vein. 



The Obturator Foramen Bypass 195 

Commentary 

In patients with a vascular prosthesis anastomosed to the external iliac or common 
femoral artery, presenting with a painful tumour in the groin, the primary tentative 
diagnosis should be infected graft. Alternative diagnoses include non-infected false 
aneurysm, incarcerated inguinal, femoral or obturator hernia, and lymphadenitis. 
[Q1:B] 

Preoperative Measures 

Even though positive cultures may be lacking, treatment with intravenous broad- 
spectrum antibiotics, including those against anaerobic microorganisms, are initi- 
ated on clinical suspicion of graft infection alone. Late vascular graft infections may 
be caused by CNS, low virulent bacteria that are often difficult to diagnose by stan- 
dard techniques [1]. 

Preoperatively, it is crucial to obtain as much information as possible about the 
extent of graft infection. Duplex scanning ultrasonography is an appropriate first 
modality to evaluate perigraft or other groin masses. CT scanning is more effective 
in the diagnosis of aortic graft infection, especially when combined with aspiration 
of perigraft fluid for Gram staining and aerobic and anaerobic cultures [2]. MRI can 
be even more reliable [3]. However, optimal diagnostic accuracy maybe obtained 
by combining CT or MRI with indium-labelled leucocyte scintigraphy [4]. Duplex 
scanning and arteriography do not play significant roles in establishing the diagno- 
sis of a vascular graft infection, but they are used for diagnosing graft occlusion, 
false aneurysm formation and anastomotic bleeding, and for planning the revascu- 
larisation procedure. In certain cases of inguinal graft infection, contrast sinogra- 
phy maybe appropriate to investigate the extent of infection. Finally, when vascular 
graft infection is suspected despite negative diagnostic tests, surgical exploration of 
the graft is necessary to detect the presence of perigraft fluid or to confirm whether 
the graft is incorporated in tissue. It is generally accepted that firm in-growth of 
surrounding tissue into the vascular prosthesis excludes the presence of graft infec- 
tion. Although CT scanning and MRI can be very helpful in delineating the bound- 
aries of infection preoperatively, the final judgement concerning the extent of 
infection is usually made intraoperatively. [Q2: A, C, D, E, F] 

If only the distal part of the graft is infected, there are several therapeutic options 
in addition to antibiotics. [Q3: A, B, D] If the proximal part of the graft is also 
infected, then it should be removed entirely. If a revascularisation procedure is war- 
ranted, then an extra-anatomic bi- or unilateral axillofemoral bypass maybe estab- 
lished, preferably as a first-stage procedure before the entire infected graft is 
removed. 

In the majority of cases, for example if the graft is occluded and the limb is viable, 
no vascular reconstruction is required [5]. In cases of limited infection, with no 
signs of anastomotic bleeding or septicaemia, then local treatment without graft 
resection maybe attempted: wound debridement, irrigation, the use of gentamicin- 
containing collagen mats, and muscle transposition may be alternative ways of 
treating inguinal vascular graft infections [6]. [Q4: A, B, C, D] 

If only the distal part of an aortofemoral prosthesis has to be removed, and revas- 
cularisation is necessary, then OFB is a very good alternative. It is not a common 
operation and comprises less than 0.5 per cent of all arterial reconstructions [7]. 



196 Vascular Surgery 

Since Shaw and Baue [8] introduced this procedure, published results of OFB rarely 
comprise more than 10 patients [7, 9-13]. However, vascular surgeons should be 
familiar with its indications and technique when addressing challenging revascular- 
isation problems in a hostile groin. 

The Concept of the Obturator Foramen Bypass 

The rationale behind this operation is based on creating an arterial conduit 
from the aortoiliac segment to the superficial femoral, popliteal or deep femoral 
artery, depending on run-off conditions, while avoiding contaminated, infected 
or destroyed tissues in the groin. By routing the vascular graft through the obtu- 
rator foramen, dorsally to the hip joint, in a layer between the adductor magnus 
and longus muscles, the area of the femoral triangle is circumvented. 
Autologous saphenous vein has been shown to give satisfying results, reducing 
the danger of secondary graft infection [14]. However, since the saphenous vein 
may be too narrow and/or too short, in most cases an externally reinforced 
Dacron or PTFE graft is used, especially since these conduits offer greater resis- 
tance against compression and kinking. Under special circumstances, the obtu- 
rator bypass can be performed as a cross-over ilioprofunda procedure using the 
contralateral iliac artery as the inflow site, the graft being routed through the 
prevesical space of Retzius [15]. 

The main indication for this procedure (80 per cent of cases) is infection confined 
to the distal iliac and inguinal part of an aortofemoral bypass graft [1]. Other indi- 
cations include the need for a revascularisation procedure in cases of infected 
femoral aneurysm, extensive local trauma [16], tissue scarring in the groin subse- 
quent to radical tumour surgery, and/or therapeutic radiation or burns [17, 18]. The 
obturator bypass has also been used in rare cases for revascularisation of sciatic 
artery aneurysm exclusion [19]. [Q5: E] 

Obturator Foramen Bypass Technique 

The patient lies in the prone position, usually with the hip and knee joints slightly 
flexed, abducted and externally rotated. Some surgeons prefer to have the hip joint 
overextended a little to facilitate the tunnelling manoeuvre through the obturator 
foramen. The operation is usually performed under general anaesthesia, sometimes 
combined with epidural anaesthesia to relieve postoperative pain. In all cases, a 
urinary catheter should be in place, since urinary bladder injury is a potential 
danger of this operation. [Q6] 

If the indication for surgery is an infected prosthetic vascular graft in the groin, 
then it is an advantage to determine in advance whether a reconstruction is neces- 
sary. Thus, the sterile part of the operation, establishing a new vascular conduit, can 
be done first [1]. The infected groin is sealed off with occlusive drape. Through a 
longitudinal paramedian incision, the proximal part of the graft is approached 
transperitoneally. Retroperitoneal access is a good alternative if one is certain that 
the infection is limited to the inguinal area. The involved graft limb is dissected 
proximally, close to the bifurcation. Firm incorporation of the graft in the sur- 
rounding tissue and a negative Gram stain of perigraft fluid indicate that the proxi- 
mal part of the graft can be preserved [20]. The graft limb is then transected, and 
the distal part is closed by sutures and pushed down towards the inguinal ligament. 



The Obturator Foramen Bypass 



197 




Fig. 22.2. The principle of OFB. 

Reprinted from Kroese AJ and Rosen L. In: Greenhalgh RM and Fowkes FGR, editors. Trials and tribulations of 
vascular surgery. London: WB Saunders, page 22, © 1 996, with permission from Elsevier. 



The overlying peritoneum is oversewn to separate the proximal graft from the infec- 
tious area. A ringed PTFE graft of diameter 6 or 8 mm is anastomosed in an end-to- 
end (Fig. 22.2) or end-to-side fashion to the proximal limb of the bifurcation graft. 
By careful blunt and sharp dissection, and with the aid of a large-blade self-retain- 
ing retractor, the ureter and bladder are identified. The pelvic organs are pushed 
gently towards the midline, rendering access to the obturator foramen. The sharp 
edge of the opening in the obturator fascia is usually identified easily by digital pal- 
pation on the anteromedial aspect of the foramen. This opening is dilated with long, 
slim grasping forceps with a blunt tip, taking care not to damage the obturator 
artery, vein and nerve that curve around the lateral edge of the foramen. 
Alternatively, other designs of blunt tunnellers can be used. It is therefore prudent 
to lead the forceps through the foramen bimanually, palpating where the tip of the 
forceps is to meet the fascial opening. We prefer tunnelling through the obturator 
foramen from below, in a plane anteriorly to the adductor magnus muscle and pos- 
teriorly to the pectineus, adductor longus and brevis muscles. Some surgeons 
choose to do this manoeuvre from the retroperitoneal space downwards [21]. The 
PTFE graft may be irrigated retrogradely with heparinised saline to ensure 
unrestricted flow. 

Through an incision in the thigh, medial to the sartorius muscle, the femoro- 
popliteal or profunda femoral artery is exposed for the distal anastomosis, which is 
usually performed in an end-to-side fashion. The profunda femoral artery is situ- 
ated anteriorly to the adductor magnus and brevis muscles, covered partially by the 
adductor longus muscle. By retracting the superficial femoral vessels and the vastus 



198 Vascular Surgery 

medialis muscle laterally, a dense fascia between the adductor longus and the vastus 
medialis is exposed. This fascia is incised, thereby severing the attachment of the 
adductor longus to expose the profunda vessels. The overlying profunda vein is 
often divided and ligated to simplify the approach towards the profunda artery [22, 
23]. 

After closing the abdominal and thigh incisions, the patient is redraped and 
the infected groin is exposed. Swabs are taken for bacterial culture. Necessary 
debridement is performed, the infected anastomosis is excised, and the femoral 
artery is closed with a running monofilament suture. The infected graft is 
removed by withdrawing it under the inguinal ligament from the retroperitoneal 
space. Finally, the wound is irrigated lavishly before closing it over a suction 
catheter. 

Perioperative complications occur in approximately 7 per cent of cases [8, 12, 24]. 
Bleeding from obturator vessels can be prevented by adhering to sound surgical 
principles. 

Perforation of the urinary bladder, vagina or sigmoid colon by faulty tun- 
nelling of the graft is a serious complication that may lead ultimately to loss of 
limb [25,26]. 

Since the obturator bypass is threatened by infection, long-term postoperative 
antibiotic treatment is advised. Although the duration is debatable, a period of 6-12 
weeks can usually be agreed upon. Graft thrombosis may lead to severe ischaemic 
symptoms, and may even threaten the viability of the lower limb, since important 
collateral vessels in the inguinal region may have been sacrificed during the previ- 
ous operation. Gluteus muscle necrosis may also compound this critical situation. 
Therefore, thrombectomy or thrombolysis of the thrombosed OFB graft should be 
attempted without delay. [Q7: F] 

The obturator bypass in the management of infected vascular grafts seems to be 
a valuable procedure [12, 27]. However, long-term results with this operation in 
terms of patency, limb salvage and survival rates are difficult to evaluate because 
the studies are usually small and include cases with different indications for obtu- 
rator bypass. However, the majority of patients suffer from symptomatic periph- 
eral arterial disease. In a review of the literature, perioperative mortality rates 
varied between zero and 14 per cent. Survival rates after 1 and 5 years were 81 and 
61 per cent, respectively. Secondary patency rates for PTFE prostheses at 1 and 5 
years were 71 and 52 per cent, respectively. Short-term limb salvage rates up to 
76-85 per cent [1] and a 5-year salvage rate of 55 per cent could be achieved [12]. 
The results depend on the indication for operation and are better in patients 
without atherosclerosis. In patients with atherosclerosis, graft patency depends 
on factors such as run-off conditions and the progression of the underlying 
atherosclerosis. 

There are several other options for revascularisation after the removal of an 
infected vascular graft in the groin, including semi-closed endarterectomy (ring- 
stripping) or balloon angioplasty of the native iliac artery, axillofemoral bypass by 
lateral route avoiding the infected groin [28, 29], and subscrotal bypass [30]. 
However, the OFB gives better results than bypass through these alternative extra- 
anatomical routes. If the groin is not grossly infected, then an autologous bypass of 
saphenous [31] or femoral vein [32] or thrombectomised femoral or iliac artery may 
be placed in situ without causing major problems, although the danger of future 
graft rupture is always present [33]. In addition, in situ revascularisation with a 
rifampicin-impregnated graft may give satisfactory results [34]. [Q8: A, B, C, E, F] 



The Obturator Foramen Bypass 199 

Although the obturator bypass procedure is not used frequently, it should be a 
part of the vascular surgeon's armamentarium. It may be effective in solving a 
difficult revascularisation problem in the groin, if performed appropriately. 



References 

1. Kroese AJ, Rosen L. What is the optimal treatment for the infected vascular graft? In: Greenhalgh 
RM, Fowkes FGR, editors. Trials and tribulations in vascular surgery. London: WB Saunders, 
1996;17-34. 

2. Low RN, Wall SD, Jeffrey RB, Jr, Sollitto RA, Reilly LM, Tierney LM, Jr. Aorto-enteric fistula and 
perigraft infection: evaluation with computed tomography. Radiology 1990;175:157-62. 

3. Spartera C, Morettini G, Petrassi C, Marino G, Minuti U, Pavone P, et al. The role of MRI in the 
evaluation of aortic graft healing, perigraft fluid collection and graft infection. Eur J Vase Surg 
1990;4:69-73. 

4. Prats E, Banzo J, Abos MD, Garcia-Lopez F, Escalera T, Garcia-Miralles M, et al. Diagnosis of pros- 
thetic vascular graft infection by technetium-labelled leukocytes. J Nucl Med 1994:35:1303-7. 

5. Lorentzen JE, Nielsen OM, Arendrup H, Kimose HH, Bille S, Andersen J, et al. Vascular graft infec- 
tion: an analysis of 62 graft infections in 2411 consecutively implanted synthetic vascular grafts. 
Surgery 1985;98:81-6. 

6. Kretschmer G, Niederle B, Huk I, Karner J, Piza-Katzer H, Polterauer P, Walzer LR. Groin infections 
following vascular surgery: obturator bypass versus biologic coverage - a comparative analysis. Eur 
J Vase Surg 1989;3:25-9. 

7. Sautner T, Niederle B, Herbst F, Kretschmer G, Polterauer P, Rendl KH, Prenner K. The value of 
obturator bypass. A review. Arch Surg 1994;129:718-22. 

8. Shaw RS, Baue AE. Management of sepsis complicating arterial reconstructive surgery. Surgery 
1963;53:75-86. 

9. Prenner KV, Rendl KH. Indications and techniques of obturator bypass. In: Greenhalgh RM, editor. 
Extra-anatomic and secondary arterial reconstructions. London: Pitman Books, 1982;201-21. 

10. Erath HG, Jr, Gale SS, Smith BM, Dean RH. Obturator foramen grafts: the preferable alternate route? 
Ann Surg 1982;48:65-9. 

11. Pearce WH, Ricco JB, Yao JS, Flinn WR, Bergan JJ. Modified technique of obturator bypass in failed 
or infected grafts. Ann Surg 1983;197:344-7. 

12. Nevelsteen A, Mees U, Deleersnijder J, Suy R. Obturator bypass: a sixteen year experience with 55 
cases. Ann Vase Surg 1987;1:558-63. 

13. Geroulakos G, Parvin SD, Bell PRF. Obturator foramen bypass, the alternative route for sepsis in the 
femoral triangle. Acta Chir Scand 1988;154:111-12. 

14. Panetta T, Sottiurai VS, Batson RC. Obturator bypass with nonreversed translocated saphenous vein. 
Ann Vase Surg 1989;3:56-62. 

15. Atnip RG. Crossover ilioprofunda reconstruction: an expanded role for obturator foramen bypass. 
Surgery 1991;110:106-8. 

16. Stain SC, Weaver FA, Yellin AE. Extra-anatomic bypass of failed traumatic arterial repairs. J Trauma 
1991;31:575-8. 

17. Donahoe PK, Froio RA, Nabseth DC. Obturator bypass graft in radical excision of inguinal neo- 
plasm. Ann Surg 1967;166:147-9. 

18. Wood RFM. Arterial grafting through the obturator foramen in secondary haemorrhage from the 
femoral vessels. Angiology 1982;33:385-92. 

19. Urayama H, Tamura M, Ohtake H, Watanabe Y. Exclusion of a sciatic artery aneurysm and an obtu- 
rator bypass. J Vase Surg 1997;26:697-9. 

20. Padberg FT, Smith SM, Eng RHK. Accuracy of disincorporation for identification of vascular graft 
infection. Arch Surg 1995;130:183-7. 

21. Pearce WH, Ricco JB, Yao JS, Flinn WR, Bergan JJ. Modified technique of obturator bypass in failed 
or infected grafts. Ann Surg 1983;197:344-7. 

22. Nunez AA, Veith FJ, Collier P, Ascer E, Flores SW, Gupta SK. Direct approaches to the distal portions 
of the deep femoral artery for limb salvage bypasses. J Vase Surg 1988;8:576-81. 

23. Millis JM, Ahn SS. Transobturator aorto-profunda femoral artery bypass using the direct medial 
thigh approach. Ann Vase Surg 1993;7:384-90. 

24. Det RF, Brands LC. The obturator foramen bypass: an alternative procedure in iliofemoral artery 
revascularisation. Surgery 1981;89:543-7. 



200 Vascular Surgery 

25. Sheiner NM, Sigman H, Stilman A. An unusual complication of obturator foramen arterial bypass. 
J Cardiovasc Surg 1969;10:303-14. 

26. Szilagyi DE, Smith RF, Elliott JP, Vrandecic MP. Infection in arterial reconstruction with synthetic 
grafts. Ann Surg 1972;176:321-6. 

27. Lai TMD, Huber D, Hogg J. Obturator foramen bypass in the management of infected prosthetic vas- 
cular grafts. Aust N Z J Surg 1993;63:811-14. 

28. Leather RP, Karmody AM. A lateral route for extra-anatomical bypass of the femoral artery. Surgery 
1977;81:307-9. 

29. Trout HH, Smith CA. Lateral iliopopliteal arterial bypass as an alternative to obturator bypass. Ann 
Surg 1982;48:63-4. 

30. Baird RN. Subscrotal bypass for the infected groin. In: Greenhalgh RM, editor. Vascular and 
endovascular techniques. London: WB Saunders, 1994;257-9. 

31. Scriven MW, Oshodi TO, Lane IF. Saphenous vein grafting in aortic graft infection: a new answer to 
and old challenge. Eur J Vase Endovasc Surg 1995;10:258-60. 

32. Nevelsteen A, Lacroix H, Suy R. Autogenous reconstruction with the lower extremity deep veins: an 
alternative treatment of prosthetic infection after reconstructive surgery for aortoiliac disease. J Vase 
Surg 1995;22:129-34. 

33. Ehrenfeld WK, Wilbur BG, Olcott CN, Stoney RJ. Autogenous tissue reconstruction in the manage- 
ment of infected prosthetic grafts. Surgery 1979;85:82-92. 

34. Young RM, Cherry KJ, Jr, Davis PM, Gloviczki P, Bower TC, Panneton JM, Hallett JW, Jr. The results 
of in situ prosthetic replacement for infected aortic grafts. Am J Surg 1999;178:136-40. 



23. Diabetic Foot 



Mauri }. A. Lepantalo, Milla Kallio and Anders Alback 



A 54-year-old smoker with type 2 diabetes of 7 years duration had a minor abra- 
sion to the lateral aspect of the left fifth toe. The patient was known to have 
hypertension, nephropathy and retinopathy, and he was overweight. His gly- 
caemic control was good following recent addition of insulin to his oral medica- 
tion. The superficial ulcer did not bother the patient, and it was initially followed 
up in his local healthcare centre. Two months later, the patient was referred to a 
community hospital because of infection and suspicion of osteomyelitis. He now 
had an infected ulcer lateral to the head of the fifth metatarsal, with a discharge. 
Plain X-ray films showed suspected osteomyelitis. Dorsalis pedis and posterior 
tibial pulses were reported to be present. The C-reactive protein (CRP) level was 
31 mg/1, leucocytes 14.8 x 10 9 /1, and blood glucose 12 mmol/1. 



Question 1 

What condition(s) are likely to be responsible for the foot problem? 

A. Infection. 

B. Atherosclerotic macroangiopathy. 

C. Diabetic microangiopathy. 

D. Neuropathy. 

Question 2 

What is the simplest tool available in the surgery or outpatient clinic to detect 
osteomyelitis? 

A. Plain X-ray films. 

B. Clinical examination with blunt nasal probe. 

201 



202 Vascular Surgery 

C. Magnetic resonance imaging. 

D. Computer tomography. 

Question 3 

What simple tools are available in the surgical outpatient clinic to assess angiopathy? 

A. Palpation of foot arterial pulses. 

B. Examination of audible signal with hand-held continuous wave Doppler. 

C. Ankle pressure measurement. 

D. Duplex scanning of lower extremity arteries. 

Question 4 

What simple tools are available in the surgery or outpatient clinic to assess 
neuropathy? 

A. Monofilament sensation testing. 

B. Achilles tendon reflex. 

C. Tuning fork testing. 

D. Electroneuromyography (ENMG). 

The patient was admitted to the medical ward for treatment of his infected foot. 
Despite the administration of intravenous antibiotic treatment, later modified 
according to the results of bacterial cultures, the infection progressed. One week 
after admission, lateral and superficial plantar compartments were drained opera- 
tively on the lateral side of the fifth metatarsal head and between the fourth and fifth 
metatarsal heads. Abundant pus was obtained, and the fifth metatarsal head was 
observed to be soft. The operative wound was left open. The infection seemed to 
subside, and the patient was discharged after a 16-day admission with oral clin- 
damycin treatment and local wound care. 

Question 5 

What major problems were neglected at this point? 

A. Presence of osteomyelitis. 

B. Presence of ischaemia. 

C. The wound was left without coverage with split thickness skin grafting. 

D. The weight-bearing wound area of the foot was not protected with a cast. 



Diabetic Foot 



203 




Fig. 23.1. Foot at the time of admission to the vascular unit. 



Despite continuous antibiotic treatment and local treatment of the open lesion on 
the lateral aspect of the foot, the situation worsened over the next 2 months and the 
patient was readmitted to the hospital. The patient had fever and his CRP level was 
123 mg/1. The serum creatinine was 1.6 mg/dl. An immediate wound debridement 
and amputation of the fourth toe was performed, after which the patient was admit- 
ted to a vascular surgical unit (Fig. 23.1). There was a faint popliteal pulse with no 
other pulses palpated distally. Ankle brachial indices (ABIs) were 1.35 and 1.21. The 
patient could not feel the touch of the monofilament on the plantar surface of the 
great toe or the first and fifth metatarsal heads. 



Question 6 

How would you further examine the circulation non-invasively or invasively? 

A. Toe pressure measurement. 

B. Ankle pressure measurements and pulse wave recordings. 

C. Treadmill test with pressure measurements. 

D. Duplex scanning of distal arteries. 

E. Magnetic resonance angiography. 

F. Digital subtraction angiography. 

The toe pressures were 73 mm Hg on the right side and 29 mm Hg on the left 
side. A selective angiography was obtained the next day (Fig. 23.2). 



204 



Vascular Surgery 




Hi 



Fig. 23.2. Angiography of the left lower limb. 




\ 3 :■ 1 



* 



^^Ha^^HMB^H 



Ay 



Diabetic Foot 205 

Question 7 

What angiographic findings typical of diabetes can you see? 

A. Normal aortoiliac segments. 

B. Haemodynamically non-significant occlusive disease of crural vessels. 

C. Significant occlusive disease of crural vessels. 

D. Severe occlusive disease of all foot vessels. 

E. Patent foot vessel. 



Question 8 

What treatment strategy would you prefer? 

A. No possibilities for reconstruction. Choose the best medical treatment, then wait 
and see. 

B. No possibilities for reconstruction. Foot-level amputation up to bleeding 
tissue. 

C. Below-knee amputation. 

D. Possible acute debridement, reconstruction to pedal artery, and further wound 
excision later. 

E. No wound excision and reconstruction to pedal artery until the wounds are 
clean. 



Question 9 

If you consider vascular reconstruction, what would be your preferred inflow site in 
this patient? 

A. Common femoral artery. 

B. Superficial femoral artery. 

C. Popliteal artery. 

A popliteopedal reconstruction was made 5 days after admission to the vascular 
surgical unit. The great saphenous vein was used in situ with the supragenicular 
popliteal artery as a recipient vessel. Despite achieving an acceptable initial flow of 
51 ml/min, the graft thrombosed the next day and a thrombectomy and a revision of 
the graft was made. A narrow segment below the knee was replaced with a reversed 
proximal great saphenous vein under angioscopic control. A flow of 110 ml/min was 
measured with transit time flowmetry. 



206 



Vascular Surgery 



Question 10 

Which of the following methods are adequate for intraoperative control? 

A. Angiography alone. 

B. Doppler alone. 

C. Flowmetry alone. 

D. Flowmetry with a method giving morphological information. 

E. Intraoperative duplex scanning alone. 

The postoperative ABI was 0.97. Wound excision and three-ray amputation of the 
lateral toes were performed 2 days after revascularisation. The patient was dis- 
charged 2 weeks after admission and transferred to the community hospital. Split 
thickness skin grafting was performed there. The patient was discharged home with 
a heel-sandal (a sandal in which the body weight is borne only by the heel), antibi- 
otic treatment for one more week, and local wound care. The healing of the wound 
progressed well. Six weeks after the vascular reconstruction, the patient was pre- 




Fig. 23.3. Foot at 1 -year follow-up. 



Diabetic Foot 207 

scribed insoles. He also used a silicon piece correcting the position of the second toe 
(Fig. 23.3). 

At 1-year follow-up, ABI was greater than 1.3/0.91 and toe pressures were 
65/55 mm Hg. Duplex surveillance findings indicated a possible vein graft stenosis. 

Question 1 1 

What are the findings indicating vein graft stenosis in the duplex examination? 

A. Midgraft peak systolic velocity (PSV) of less than 45 cm/s. 

B. V2/V1 ratio greater than 3 (V2, PSV at the site of the maximum stenosis; VI, 
PSV in the normal graft adjacent to the stenosis). 

C. Maximum PSV greater than 300 cm/s. 

D. End-diastolic flow velocity (EDV) greater than 20 cm/s. 

A control angiography was performed, but no severe stenosis was found (Fig. 
23.4). 



Commentary 

This case illustrates the problems related to delayed diagnosis and treatment of dia- 
betic neuroischaemic foot. The aetiology of diabetic foot ulceration and infection is 
multifactorial. Our patient evidently had infection and probably also neuropathy. 
Neuropathy often abolishes sensation, and an unpleasant odour and discharge may 
be the first signs of infection to the patient, especially if the lesion is situated on the 
plantar aspect of the foot. The role of microangiopathy in diabetic foot is not 
confirmed, but ischaemia due to atherothrombotic disease often plays a major role 
[1].[Q1:A,B,D] 

The simplest method is to examine the ulcer with a blunt nasal probe. If it hits the 
bone, then osteomyelitis is most likely. The diabetic wounds should be classified 
systematically according to a precise system, such as the Armstrong classification 
(Table 23.1), which takes into account both the depth of the lesions and the pres- 
ence of ischaemia and infection [2]. Plain X-ray films are of limited value and 
magnetic resonance imaging (MRI) is the most reliable tool for diagnosis of 
osteomyelitis [3]. [Q2: B] 

The patient was reported to have palpable distal pulses at one time but not at 
another time. Furthermore, the popliteal pulse was reported to be palpable and ABI 
to be normal. Palpation of foot pulses is not a fully reproducible observation, and 
they maybe considered normal if both tibialis posterior and dorsalis pedis pulses 
are clearly felt [4]. If either is not palpated, non-invasive evaluation is necessary. It 
is far more difficult to palpate the popliteal pulse, and it has been suggested that if 
an inexperienced palpator feels the popliteal pulse, this indicates an aneurysm. 
Systolic pressure measurements taken at the level of the ankle by a Doppler device 
are the most common non-invasive method for assessment of atherothrombotic 
disease. However, the results maybe biased due to the presence of mediasclerosis, 
which is present in 15-40% of diabetics [5]. Incompressible arteries may allow the 



208 



Vascular Surgery 



Fig. 23.4. Control angiography 
after 1 -year follow-up. 



Diabetic Foot 209 

Table 23.1. Classification of diabetic foot lesions by grading and staging according to the depth of the lesion 
and the presence of infection and ischaemia, as proposed by Armstrong et al. [2]. Updated 



Depth 

Grade 0: pre- or post-ulcerative site which has healed 

Grade I: superficial wound through the epidermis or epidermis and dermis which does not penetrate 

to tendon, capsule or bone 
Grade II: wound which penetrates to tendon or capsule 
Grade III: wound which penetrates to bone or joint 
Infection and ischaemia: 

Stage A: clean wound 

Stage B: non-ischaemic infected wound 

Stage C: ischaemic non-infected wound 

Stage D: ischaemic infected wound to tendon, capsule or bone 



signal to be heard in cuff pressures as high as the patient tolerates. In patients with 
mediasclerosis, the ABI typically exceeds 1.15 [6]. The audible Doppler signals may 
help the examiner, as an open inflow channel gives high-pitched biphasic signals 
but collateral flow around an occlusion usually gives only a low-pitched monopha- 
sic murmur. [Q3: A, B, C] 

Symptoms of neuropathy include loss of sensation, hyperaesthesia and burning, 
and aching pain, which are often worse at night [7]. Many patients with severe neu- 
ropathy are asymptomatic. Achilles tendon reflex, monofilament sensation testing 
and 128-Hz tuning fork testing are other recommended clinical tests [8]. [Q4: A, B, C] 

The primary diagnostic work-up in this case was clearly deficient. The patient 
obviously had osteomyelitis, which would have necessitated prompt drainage and 
amputation. Furthermore, the role of ischaemia should also have been evaluated 
and corrected within 3-5 days after proper drainage. [Q5: A, B] 

The Doppler-derived pressures were clearly pseudohypertensive due to the arter- 
ial wall stiffness. Pseudohypertension affects digital arteries far less frequently, and 
therefore toe pressures are more reliable. A pulse volume recording at the ankle can 
also help to detect mediasclerosis. Another method is to measure systolic blood 
pressure at the ankle with Doppler but without an occluding cuff - the pole test [9]. 
The examiner listens to the Doppler signals of the supine patient while the foot is 
elevated gradually until the signals disappear. The scale in the pole gives the pres- 
sure at the ankle (0.75 x pressure (cm) equals the pressure (mm Hg)). In centres 
where duplex scanning of distal arteries can be done by trained validated examiners 
this method would preferably be the next investigation. Magnetic resonance angiog- 
raphy is also a method of choice if high-quality images are available and especially 
when the patient suffers from marked nephropathy. On the other hand in centres 
where technically demanding endovascular procedures can be done during diag- 
nostic contrast angiography this is - as in our case - the primary imaging tech- 
nique. [Q6: A, B, D, E, F] 

Atherosclerotic changes in diabetes are typically situated in femoral and crural 
arteries, or only in crural arteries, in contrast to non-diabetic patients who tend to 
have the first symptoms from the obliteration of the aortic bifurcation. Despite 
proximal crural artery occlusion, the pedal arteries maybe patent, as was the dor- 
salis pedis artery in this patient. [Q7: A, C, E] 

The treatment strategy is affected strongly by the presence and severity of 
infection. A superficial ulceration maybe only the tip of the iceberg. There maybe 



210 Vascular Surgery 

penetration, hidden to the eye, into deep tissues. Vigorous debridement must be 
carried out to establish the degree of penetration and to remove all necrotic tissue 
[3]. Fulminant infection may necessitate guillotine amputation. The bypass can 
often be performed 3-5 days after debridement. If ischaemia plays a major role and 
the infection is quiescent, then revascularisation can, in selected cases, be per- 
formed first. Vascular reconstruction can be performed in as many as 90 per cent of 
diabetic patients with atherothrombotic disease [10]. The best outflow vessel in con- 
tinuity with the foot should be selected [11]. Diabetes is not considered to affect the 
outcome of graft patency, although female diabetic patients are reported to have 
worse outcome regarding patency and leg salvage [12]. In limbs with large tissue 
defects, a microvascular free muscle flap transfer can be used for defect coverage in 
conjunction with long bypass [13]. [Q8: D] 

Short bypasses do well if the inflow artery is not compromised, as in our patient. 
Although the above-knee popliteal artery gave better results as the inflow vessel 
than the below-knee popliteal artery in our own series [14], the question is not 
settled. [Q9: C] 

Angiography is the gold standard for intraoperative monitoring. The accuracy of 
flowmetry is affected strongly by the reproducibility of the method. In contrast to 
older methods, transit-time flowmetry, which does not require information on the 
diameter of the vessels, has proven to be very accurate [15]. Despite this, it gives 
only flow values and does not inform about the morphology. The present case 
clearly shows how the typically narrow segment of the great saphenous vein below 
the knee was missed despite good flow during the initial hyperaemia. In this area, 
there was an intimal tear caused by the valvulectomy catheter. Unfortunately, an 
angioscope was not used in the first operation. An angioscope visualises the inner 
surface of the vessel, whereas intravascular ultrasound is better for detecting 
changes within the vessel wall. Doppler and duplex maybe used for intraoperative 
monitoring as well. Doppler gives only haemodynamic information, whereas duplex 
gives a combination of anatomical and haemodynamic information. There is no 
best method for intraoperative monitoring, but the optimal method would be to 
have both haemodynamic and morphological information. [Q10: A, D, E] 

As there is a 30 per cent risk of developing neointimal hyperplasia and graft steno- 
sis within the first postoperative year, duplex surveillance is considered an essential 
part of postoperative care. All the suggested duplex criteria are indicative of vein 
graft stenosis, but none of them can be 100 per cent sensitive in detecting stenosis 
[16]. Our case demonstrates that using liberal duplex criteria, false positive findings 
are easily encountered as the angiography was deemed normal. [Q1 1 : A, B, C, D] 



References 

1. LoGerfo FW, Coffman JD. Vascular and microvascular disease in the diabetic foot: implications for 
foot care. N Engl J Med 1984;311:1615-19. 

2. Armstrong DG, Lavery LA, Harkless LB. Validation of a diabetic wound classification system. 
Diabetes Care 1998;21:855-9. 

3. Levin ME, O'Neal LW. The diabetic foot. St Louis: Mosby, 1983. 

4. Lundin M, Wiksten JP, Perakyla T, Lindfors O, Savolainen H, Skytta J, Lepantalo M. Distal pulse 
palpation: is it reliable? World J Surg 1999;23:252-5. 

5. Lehto S, Niskanen L, Suhonen M, Ronnemaa T, Laakso M. Medial artery calcification. A neglected 
harbinger of cardiovascular complications in non-insulin-dependent diabetes mellitus. Arterioscler 
Thromb Vase Biol 1996;16:978-83. 



Diabetic Foot 211 

6. Takolander R, Rauwerda JA. The use of non-invasive vascular assessment in diabetic patients with 
foot lesions. Diabet Med 1996;13:S39-42. 

7. Veves A, Sarnow MR. Diagnosis, classification and treatment of diabetic peripheral neuropathy. Clin 
Pod Med Surg 1995;12:19-30. 

8. International Working Group on the Diabetic Foot. International consensus on the diabetic foot. 
Netherlands: International Working Group on the Diabetic Foot, 1999. 

9. Smith FCT, Shearman CP, Simms MH, Gwynn BR. Falsely elevated ankle pressures in severe leg 
ischaemia: the pole test - an alternative approach. Eur J Vase Surg 1994;8:408-12. 

10. Reiber GE, Lipsky BA, Gibbons GW. The burden of diabetic foot ulcers. Am J Surg 1998;176 (Suppl 
2A):5S-10S. 

11. LoGerfo FW, Gibbons GW, Pomposelli FB, Campbell DR, Miller A, Freeman D, Quist WC. Trends in 
the care of the diabetic foot, expanded role of arterial reconstruction. Arch Surg 1992;127:617-21. 

12. Lepantalo M, Tukiainen E. Combined vascular reconstruction and microvascular muscle flap trans- 
fer for salvage of ischaemic legs with major tissue loss and wound complications. Eur J Vase 
Endovasc Surg 1996;12:1-5. 

13. Luther M, Lepantalo M. Femorotibial reconstructions for chronic critical leg ischaemia: influence on 
outcome by diabetes, gender and age. Eur J Vase Endovasc Surg 1997;13:569-77. 

14. Biancari F, Kantonen I, Alback A, Ihlberg L, Lehtola A, Lepantalo M. Popliteo-to-distal bypass grafts 
for leg ischaemia. J Cardiovasc Surg 2000;41:281-6. 

15. Alback A, Makisalo H, Nordin A, Lepantalo M. Validity and reproducibility of transit time 
flowmetry. Ann Chir Gynaecol 1996;85:325-31. 

16. Sladen JG, Reid JD, Cooperberg PL, Harrison PB, Maxwell TM, Riggs MO, Sanders LD. Color flow 
duplex screening of infrainguinal grafts combining low and high velocity criteria. Am J Surg 
1989;158:107-12. 



24. Chronic Visceral Ischaemia 

George Geroulakos 



A 68-year-old woman presented with a 19-month history of generalised abdomi- 
nal pain. Initially, she experienced the pain following meals, but subsequently the 
pain became persistent. Over this period of time, she lost 12 kg in weight. For the 
last few months before admission, she started having diarrhoea once to twice per 
day. There was no blood or mucus in the stool. Her past medical history included 
partial gastrectomy 17 years earlier for benign disease. 

On examination, the patient looked cachectic. Her abdomen was slightly dis- 
tended, and the bowel sounds were increased. There was a high-pitched epigas- 
tric bruit. Routine blood tests were normal. 



Question 1 

Which is the likely diagnosis for our patient on the basis of the available informa- 
tion so far? 

A. Cancer of the pancreas. 

B. Peptic ulcer. 

C. Subacute intestinal obstruction secondary to adhesions. 

D. Mesenteric angina. 

E. Cancer of the large bowel. 

Faecal fat measurement was 17.6 g/day (normal value <6 g/day). Gastroscopy was 
performed, which showed features compatible with atrophic gastritis. This was fol- 
lowed by computed tomography (CT) scanning of the abdomen, which reported 
that the pancreas could not be defined well as a result of paucity of retroperitoneal 
fat. In addition, CT showed non-specific thickening of the small-bowel loops. 
Endoscopic retrograde cholecystopangreatography (ERCP) was performed, which 
ruled out pancreatic pathology. A small-bowel enema did not demonstrate any 
significant findings. A colonoscopy was performed, which showed two isolated 

215 



216 



Vascular Surgery 




Fig. 24.1. Colonoscopic view of an isolated ulcer in the ascending colon in a patient with chronic visceral 
ischaemia. 



ulcers in the ascending colon (Fig. 24.1) and raised the possibility of ischaemic 
colitis. Fig. 24.2 shows the lateral aortogram of our patient, which showed occlusion 
of the coeliac artery and 95 per cent stenosis of the superior mesenteric artery. A 
diagnosis of chronic visceral ischaemia was made. 




Fig. 24.2. Lateral aortogram demonstrating occlusion of the coeliac artery and a 95 per cent stenosis of the 
superior mesenteric artery. 



Chronic Visceral Ischaemia 217 

Question 2 

Which of the following statements regarding chronic visceral ischaemia is correct? 

A. It has a marked male preponderance. 

B. As described in our patient, it usually takes more than 1 year from the first pre- 
sentation of the symptoms until the final diagnosis is made. 

C. It presents clinically as an undiagnosed malignancy. 

D. Symptoms occur when at least one of the three visceral arteries has significant 
disease. 

E. It may cause malabsorption. 

The patient underwent antegrade revascularisation via a ninth rib extraperitoneal 
thoracoabdominal approach of the coeliac artery and the superior mesenteric 
artery. An 8-mm Dacron graft was used as a conduit. 



Question 3 

Which of the following statements regarding the management of this patient are 
false? 

A. The best patency can be achieved using a venous conduit. 

B. Revascularisation of the coeliac artery was unnecessary, and equally good 
results could have been achieved with revascularisation of only the superior 
mesenteric artery. 

C. Surgical revascularisation should not have been considered in this elderly, frail 
patient because it has an excessive mortality rate of about 30 per cent in most 
series. 

D. Percutaneous transluminal angioplasty (PTA) should have been the method of 
choice. 

The postoperative recovery of the patient was uneventful. She was discharged 
home on the eighth postoperative day. Six months later, she was asymptomatic and 
had gained 5 kg in weight. However, at 12 months the patient presented to the out- 
patient clinic with recurrent postprandial abdominal pain. A duplex examination 
showed that the graft to superior mesenteric artery anastomosis had more than 
60 per cent stenosis and the graft to coeliac artery anastomosis was occluded. 

Question 4 

What would you advise your patient? 

A. Reoperation aiming to revascularise the coeliac artery and place a patch on the 
graft to superior mesenteric artery anastomosis. 



218 Vascular Surgery 

B. Angioplasty and stenting of the graft to superior mesenteric artery anastomosis. 

C. Conservative management advising the patient to take small and frequent 
meals. 

The patient underwent angioplasty and stenting of the graft to superior mesen- 
teric artery anastomosis with an excellent technical and clinical result. Twenty- four 
months following this procedure the patient remains asymptomatic. 



Commentary 

As described in our patient, the clinical picture of chronic visceral ischaemia 
includes abdominal pain with or without diarrhoea and weight loss. The diagnosis 
of chronic visceral ischaemia is in doubt if the patient has no significant decrease in 
total body mass. The abdominal pain occasionally radiates to the back. The pain of 
visceral ischaemia has similarities to that of carcinoma of the stomach, pancreatic 
carcinoma and peptic ulceration. Diarrhoea may be explained by the increased 
motility of the bowel induced by the ischaemia; it may also be secondary to malab- 
sorption. [Q1:A,B,D,E] 

Other symptoms that may be seen include nausea and vomiting, which have been 
associated with gastric motility disorders caused by ischaemia [1]. An epigastric 
bruit may or may not be present. Our group and others have reported a marked 
female patient distribution of this condition [2-4]. The reason for this peculiar sex 
distribution remains undetermined. However, it has been suggested that it could be 
the result of the inclusion of cases of Takayasu's aortitis in reports of atherosclerotic 
chronic visceral ischaemia [5]. Takayasu's aortitis closely mimics atherosclerosis of 
the abdominal aorta and has a marked female predominance. 

The time from the onset of symptoms to diagnosis is usually more than 12 
months [6]. The diagnosis of chronic visceral ischaemia is a clinical one. As shown 
clearly in our case, contrast studies, abdominal ultrasound, endoscopy and CT are 
not essential to the diagnosis but will prove important in eliminating other sources 
of abdominal discomfort. In all instances, lateral views of biplane aortography 
demonstrate visceral occlusive lesions compatible with the diagnosis. As a result of 
an abundant network of collateral vessels, clinical symptoms are present when at 
least two of the three visceral arteries have significant disease. There are known 
asymptomatic cases with all three visceral arteries thrombosed, thus emphasising 
the fact that chronic visceral ischaemia cannot be diagnosed exclusively on the basis 
of X-rays. [02:6,0, E] 

Techniques of revascularisation include transection and reimplantation, bypass 
grafting, endarterectomy and balloon angioplasty with or without stent placement. 
There is no consensus regarding the best surgical approach for the treatment of 
chronic visceral ischaemia. This condition is encountered infrequently, and it is 
unlikely that a single centre can treat enough patients and accumulate sufficient 
experience to develop principles of treatment by demonstrating significant differ- 
ences between the various mesenteric revascularisation strategies. Bypass grafting is 
the most common type of visceral revascularisation performed; it may originate 
from several different locations, including the supracoeliac aorta, the infrarenal 



Chronic Visceral Ischaemia 219 

aorta and the common iliac arteries. Regardless of the bypass technique used, the 
status of the donor artery is critical to success [7]. The distal thoracic aorta is 
usually free of atherosclerotic disease and is an excellent origin of a short antegrade 
bypass to the superior mesenteric artery. The bypass is placed in the direction of 
normal blood flow, thus reducing anastomotic turbulence. In addition, this design 
eliminates the possibility of kinking and thrombosis by compression or traction 
from the overlying intestinal mesentery, which may be observed with retrograde 
grafts originating from the infrarenal aorta or the iliac arteries. The distal portion of 
the thoracic aorta may be approached from the abdomen through division of the 
crura. 

There is no uniform agreement about the graft material of choice. In early 
reports, vein grafts had patency rates inferior to synthetic grafts [9, 10]. More recent 
reports described the use of either autogenous veins or prosthetic grafts with excel- 
lent long-term function and no difference in patency rates [11, 12]. In our case, we 
used synthetic Dacron bypass because it is always available, spares the patient from 
the morbidity of one or more incisions for the harvesting of the vein, and provides 
good early and long-term results. 

Aorto-superior mesenteric artery bypass alone is usually sufficient to provide 
good symptomatic relief as a result of the extensive collateral circulation, even when 
all three visceral arteries are occluded. Hollier et al. [13] have shown that complete 
revascularisation in multivessel disease resulted in a late recurrence of 11 per cent, 
while when one of three stenotic vessels was revascularised the recurrence rate was 
50 per cent. They concluded that it is preferable to revascularise as many vessels as 
possible to provide the best chance of long-term relief. 

Most recent series report an acceptable operative mortality rate ranging from 3 to 
8 per cent. 

Our patient could have been considered for angioplasty of the superior mesen- 
teric artery. There is limited information about the selection of patients for PTA and 
the long-term results. A number of small series reported early technical success in 
70-100 per cent of patients. Recurrence after PTA is greater than after surgery. In a 
series of 18 patients with chronic visceral ischaemia who had a technically success- 
ful angioplasty, symptomatic recurrence occurred in three (20 per cent) patients at 
a mean interval of 28 months, while three patients had only partial symptomatic 
relief [14]. Angioplasty is safer than surgery, although fatal distal embolisation and 
fatal superior mesenteric artery dissection with thrombosis and bowel infarction 
have been reported. At present, it should probably be limited to high-risk patients. 
PTA of the visceral arteries can be difficult when performed via the femoral artery 
approach. Access through the brachial and axillary artery may be easier, but this 
route does not guarantee technical success. [Q3: False A, B, C, D] 

Recurrent visceral ischaemia is not uncommon after primary visceral revascular- 
isation for chronic visceral ischaemia. In a large series of 109 patients who under- 
went primary visceral revascularisation at the University of California, San 
Francisco over a period of 38 years, 19 patients had recurrent visceral ischaemia, 12 
(11 per cent) patients had recurrent chronic visceral ischaemia, and seven (6.4 per 
cent) had acute visceral ischaemia [15]. The minimally invasive nature of the 
endovascular techniques and the increased complication rate of reoperations 
renders the endovascular approach a reasonable first option in properly selected 
patients with recurrent symptoms [16]. [Q4: B] 



220 Vascular Surgery 

References 

1. Babu SC, Shah PM. Celiac territory ischemic syndrome in visceral artery occlusion. Am J Surg 
1993;166:227-30. 

2. Geroulakos G, Tober JC, Anderson L, Smead WL. Antegrade visceral revascularisation via a thoraco- 
abdominal approach for chronic visceral ischaemia. Eur J Vase Endovasc Surg 1999;17:56-9. 

3. Zelenock G, Graham LM, Whitehouse WM, Erlandson EE, Kraft RO, Lindenauer SM, Stanley JC. 
Splanchnic arteriosclerotic disease and intestinal angina. Arch Surg 1990;115:497-501. 

4. Geelkerken RH, van Bockel JH, De Ross WK, Hermans J, Terpstra JL. Chronic mesenteric vascular 
syndrome. Results of reconstructive surgery. Arch Surg 1991;126:1101-6. 

5. Lande A. Abdominal Takayasu's aortitis, the middle aortic syndrome and atherosclerosis. Int Angiol 
1998;17:1-9. 

6. Schneider PA, Ehrenfeld WK, Cunningham CG, Reilly LM, Goldstone J, Stoney RJ. Recurrent chronic 
visceral ischaemia. J Vase Surg 1992; 15:237. 

7. Rheudasil JM, Stewart MT, Schellack JV, Smith RB, Salam AA, Perdue GD. Surgical treatment of 
chronic mesenteric arterial insufficiency. J Vase Surg 1988;8:495-500. 

8. Kazmers A. Operative management of chronic mesenteric ischaemia. Ann Vase Surg 
1998;12:299-308. 

9. Rob C. Surgical diseases of the celiac and mesenteric arteries. Arch Surg 1966;93:21-30. 

10. Stoney RJ, Ehrenfeld WK, Wylie EJ. Revascularization methods in chronic visceral ischaemia caused 
by atherosclerosis. Ann Surg 1977;186:468-76. 

11. Bauer GM, Millay DJ, Taylor LM, Porter JM. Treatment of chronic visceral ischaemia. Am J Surg 
1984;148:138-44. 

12. WD, McCarthy WJ, Bresticker MR, Pearce WH, Schneider JR, Golan JF, Yao JST. Mesenteric artery 
bypass: objective patency determination. J Vase Surg 1995;21:729-41. 

13. Hollier LH, Bernatz PE, Pairolero PC, Spencer Payne W, Osmundon PJ. Surgical management of 
chronic intestinal ischaemia. A reappraisal. Surgery 1981;90:940-6. 

14. Allen RC, Martin GH, Rees CR, Rivera FJ, Talkington CM, Garrett WV, et al. Mesenteric angioplasty 
in the treatment of chronic intestinal ischaemia. J Vase Surg 1996;24:415-23. 

15. Schneider DB, Schneider PA, Reilly LM, Ehrenfeld WK, Messina LM, Stoney RJ. Reoperation for 
recurrent chronic visceral ischaemia. J Vase Surg 1998;27:276-86. 

16. Robless P, Belli AM, Geroulakos G. Endovascular versus surgical reconstruction for the management 
of chronic visceral ischaemia: a comparative analysis. In: Geroulakos G, Cherry K, editors. Diseases 
of the visceral circulation. London: Arnold, 2002;108-118. 



25. Acute Mesenteric Ischaemia 



Jonathan S. Refson and John H. N. Wolfe 



A 78-year-old woman presented to the emergency department with a 12-h 
history of sudden-onset abdominal pain. She had vomited after the pain started, 
and she had also had two episodes of diarrhoea. Until this time, she had been 
well, although she was known to be in atrial fibrillation and took digoxin 125 mg 
daily. 

On examination, she was distressed and obviously in pain. Baseline observa- 
tions revealed a pulse of 110 bpm, irregularly irregular, blood pressure of 
95/60 mm Hg, respiratory rate of 28 breaths/min, and temperature of 37.3°C. Her 
chest was clear, heart sounds were normal (irregular rhythm), and the jugular 
venous pressure was not elevated. Abdominal examination was unremarkable, 
with a soft abdomen and minimal tenderness despite severe pain, and normal 
bowel sounds. 

The investigations shown in Table 25.1 were performed by the admitting 
surgeon. 

Electrocardiogram (ECG) revealed atrial fibrillation with no other acute 
changes. Erect chest X-ray revealed normal lung fields and no free gas under the 
diaphragm. Abdominal radiography was unremarkable except for minimal 
small-bowel distension. 



Question 1 

Which of the following is the most unlikely diagnosis? 

A. Acute ulcerative colitis. 

B. Pancreatitis. 

C. Mesenteric venous thrombosis. 

D. Acute mesenteric ischaemia (AMI). 

E. Diabetic ketoacidosis. 



221 



222 Vascular Surgery 

Table 25.1 . Investigations performed by the admitting surgeon. Updated 



Investigation 


Finding 


Urinalysis 


No abnormality 


Biochemistry 


Na + 139mmol/l 




K + 4.6mmol/l 




Creatinine 112 mmol/l 




Glucose 6.1 mmol/l 




Amylase 2000 IU/I 


Haematology 


Haemoglobin 12.3 g/dl 




White cell count 27,000 




Platelets 235,000 


Arterial blood gas 


pH 7.21 




pC0 2 3.2 kPa 




p0 2 9.4 kPa 




HC0 3 - 17 mmol/l 




Base excess -8 



Question 2 

What are the most common causes of AMI? 

A. Renal failure. 

B. Atrial fibrillation. 

C. Multi-organ failure. 

D. Anti-phospholipid syndrome. 

E. Atherosclerotic disease 

Question 3 

Which of the following tests are of use in the acute management of a patient with 
AMI? 

A. Echocardiography. 

B. Lateral-view mesenteric angiography. 

C. Thyroid function tests (TFTs). 

D. Non-contrast computed tomography (CT) scanning. 

E. Mesenteric vessel duplex Doppler. 

At this point, the patient was taken to the high-dependency unit, where the fol- 
lowing measures were undertaken: high-flow oxygen therapy by mask (15 1/min), 
continuous ECG monitoring, central venous pressure (CVP) monitoring, urinary 
catheter inserted to monitor urinary flow hourly, and infusion of 4 litres of fluid 
resuscitation. Intravenous broad-spectrum antibiotics and an anticoagulant dose of 



Acute Mesenteric Ischaemia 223 

Table 25.2. Repeat blood gas and blood count investigations. Updated 



Investigation 


Finding 


Haematology 


Haemoglobin 10.2 g/dl 




White cell count 37,000x1071 




Platelets 235x1071 


Arterial blood gas 


pH 7.19 




pC0 2 3.1kPa 




p0 2 49.4 kPa 




HC0 3 -11mmol/l 




Base excess -15 



intravenous heparin were also given. After 2 h of resuscitation, the patient's blood 
pressure was 130/85 mm Hg, pulse 100 bpm and CVP +8 cm water. She was still in a 
lot of pain despite 10 mg of diamorphine, and she was still tachypnoeic. Repeat 
blood gas and blood count investigations were as in Table 25.2. 

Because the patient was persistently acidotic with an elevated white count and in 
severe pain, she was taken to the operating theatre for an emergency laparotomy. 
Almost the entire small bowel and most of the large bowel were found to be 
ischaemic but viable. There was a pulse in the proximal superior mesenteric artery 
(SMA) but nothing was palpable beyond the origin of the middle colic vessel. 

Question 4 

What operative options are available to achieve restoration of flow to the bowel? 

A. Full heparinisation. 

B. Catheter thrombectomy. 

C. Axillofemoral bypass. 

D. Mesenteric bypass with a vein graft. 

E. Mesenteric bypass with prosthetic graft. 

Clot was removed successfully from the SMA. However, despite the majority of the 
bowel receiving a good blood supply, several areas remained dusky in appearance. 

Question 5 

What features of the bowel's appearance determine whether it is viable? 

A. The presence of peristalsis. 

B. Lack of foul odour from the peritoneal cavity. 

C. Serosal sheen. 

D. Mesenteric pulsation. 

E. Active bleeding from the cut surface of the bowel at the time of resection. 



224 Vascular Surgery 

Question 6 

Having determined that an area of the bowel is non-viable, what action should you 
take? 

A. Revascularise the bowel, then remove that which is non-viable. 

B. Remove the non-viable bowel, then revascularise the remaining bowel. 

C. Resect all non-viable bowel and primarily anastomose ends; then close the 
abdomen. 

D. Close the abdomen and start the patient on an intravenous infusion of diamor- 
phine. 

E. Resect all non-viable bowel and exteriorise viable ends; plan relook laparotomy. 



Commentary 

The incidence of AMI is approximately 1/100,000 in the UK and USA [1, 2]. AMI is a 
life-threatening vascular emergency. It accounts for 0.1 per cent of emergency 
admissions [3], and based on several large series it has a mortality of between 60 
and 100 per cent [3-7]. Females are affected twice as often as males, and the median 
age at presentation is 70 years [6]. 

The clinical presentation is often not as clear-cut as described in the case above. 
However, some if not all of the described features will be present. One must have a 
high index of clinical suspicion in anyone aged over 55 years who presents with 
abdominal pain out of proportion to the physical signs elicited on abdominal exam- 
ination [8]. The diagnosis should also be considered in patients with known periph- 
eral arterial disease and abdominal pain. The triad of abdominal pain, a cardiac 
source of embolus and gut emptying, as described by Klass [9], make AMI the most 
likely diagnosis. In addition to this triad, the finding of a marked leucocytosis, 
metabolic acidosis and hyperamylasaemia are also suggestive of AMI. It is also not 
unusual for there to have been a history of previous embolic events [8]. Pancreatitis 
can be difficult to differentiate from AMI, and laparotomy is indicated if suspicion 
of AMI is aroused. [Q1: E] 

Defining the aetiology of AMI is important as the different causes have different 
treatments. The most common presentation, as described in our case, is of superior 
mesenteric embolus; this accounts for about 50 per cent of all cases [3, 5-7]. The 
usual source for these clots is the atria in patients in atrial fibrillation or the ventri- 
cle if the patient has recently sustained a myocardial infarction. Another potential 
source of emboli is atheroma from the aortic wall following radiological procedures 
in which catheters and guidewires have been passed up the aorta. The consequence 
of such an event can be catastrophic, as the mesenteric circulation has not had time 
to develop a collateral circulation. 

The next most frequent aetiology is SMA thrombosis, which accounts for between 
25 and 50 per cent of cases [3, 5-7]. This results from progression of atheromatous 
disease at the origin of the SMA. It is important to note that a long-standing steno- 
sis may have caused symptoms of chronic mesenteric ischaemia in the months 
before its ultimate occlusion [10]. Therefore, in a patient with pre-existing symp- 



Acute Mesenteric Ischaemia 225 

toms of mesenteric ischaemia, sudden onset of abdominal pain should be regarded 
as AMI until proven otherwise. 

Non-occlusive mesenteric ischaemia (NOMI), first described by Ende in 1958 
[11], is the next most frequently encountered condition, occurring in about 20 per 
cent of cases [3, 5, 6]. In this situation, the patient is often critically ill from another 
cause and the mesenteric ischaemia is due to vasoconstriction leading to reduced 
flow in the splanchnic circulation. This may be due to cardiogenic shock, hypo- 
volaemia or vasoconstricting inotropes. Even after reversal of shock, mesenteric 
hypoperfusion may persist for several hours [12, 13]. 

Mesenteric venous thrombosis (MVT) is the least common cause and accounts 
for about 5 per cent of AMI [14-16]. The thrombotic process is thought to start in 
the superior mesenteric vein and spreads to the portal vein; the inferior mesen- 
teric vein is usually spared. The onset of symptoms is more insidious and may 
have a history of several days. It is caused by the same provoking factors that one 
finds in any thrombotic situation: sluggish flow, clotting abnormality and vessel 
wall damage (Virchow's triad). It is associated most commonly with hypercoagu- 
lable states, abdominal trauma or intra-abdominal sepsis [17-20]. The diagnosis 
is often made at laparotomy and encompasses a spectrum of severity from seg- 
mental mesenteric venous thrombosis to the entire portal vein being thrombosed. 
[Q2: B, E] 

Diagnostic confirmation of AMI poses a dilemma. Should one delay in order to 
confirm a suspicion and risk converting a salvageable situation into a non-salvage- 
able one [21]? There is little evidence on which to base sound advice. However, if 
the patient is cardiovascularly stable with minimal symptoms, and one has prompt 
access to angiography, then this provides accurate diagnosis (Fig. 25.1). Some 
authors recommend colour-flow duplex at the bedside during the resuscitation 
phase [22, 23]; this procedure is less time-consuming than angiography, but it 
requires considerable skill that is not always available. Furthermore, good views are 
often hampered by obesity and/or overlying bowel gas. 

Transthoracic echocardiography is useful in identifying a cardiac source for 
emboli and may help in making the decision regarding postoperative anticoagula- 
tion. However, it is not as sensitive as transoesophageal echo in searching for left 
atrial emboli and may waste valuable time. 

Contrast-enhanced CT maybe of use in identifying mesenteric venous thrombo- 
sis [24]. This will not prevent laparotomy as bowel resection may well be necessary. 
If there is clear evidence of peritonism and a high index of suspicion for AMI, then 
the patient should be resuscitated rapidly and this should be followed by urgent 
laparotomy. [Q3: B, E] 

In order to answer Question 4, one has to be confident of the aetiology of 
AMI. In our case, there is embolus in the SMA. The abdomen should be 
approached through a long midline incision, which will afford excellent expo- 
sure. Having entered the abdomen, a quick survey of the viscera and extent of 
ischaemia should give some information on the aetiology of the AMI (see 
below). At this point, the main aim of surgery is to restore flow to the ischaemic 
viscera if viable. In order to do this, the transverse mesocolon is elevated and 
the ligament of Treitz identified and the fourth part of the duodenum mobilised. 
The root of the mesentery is palpated to feel for the SMA pulse. If, as in our 
case, the AMI is due to an embolic event, then a proximal SMA pulse should be 
palpable and the duodenojejunal flexure and proximal few centimetres of 
jejunum should be viable. The emboli usually lodge at a variable site 3-8 cm 



226 



Vascular Surgery 




Fig. 25.1 . Narrowed atherosclerotic aorta with no coeliac or superior mesenteric filling. 



from the origin of the SMA, usually at the point where the middle colic artery 
arises [6, 8, 21 ] (Fig. 25.2a, b). 

In order to expose the SMA, the inferior leaflet of the mesocolon is entered along 
the course of the vessel and a 5-cm section of artery is dissected out and slung with 
Silastic sloops. At this point, if the patient has not been heparinised previously, then 
they should be given an intravenous dose of 5000 units of unfractionated heparin. A 
longitudinal arteriotomy is made in the cleared SMA (on the left margin of the 
vessel, to permit easier graft placement, should a bypass be necessary) [21], and a 
size 3 or 4 Fogarty embolectomy catheter is passed up and down the vessel to 
retrieve the embolus. Once adequate backward and forward flow has been achieved, 
the vessel should be flushed with heparinised saline; the arteriotomy can then be 
closed primarily or with a patch (depending on size), using a 6/0 or 7/0 Prolene 
suture. 

At this point, the anaesthetist should be warned that you are about to reperfuse 
the viscera. Metabolites that have accumulated in the ischaemic viscera will pass 
rapidly from the mesenteric venous circulation into the systemic circulation, which 
can precipitate circulatory collapse and, over several hours, result in the develop- 
ment of systemic inflammatory response syndrome (SIRS). There is no point in 
revascularising dead bowel; indeed, this is dangerous. Irrefutably dead bowel 
should be removed before revascularisation. It is important to note that if, on 



Acute Mesenteric Ischaemia 



227 





Fig. 25.2. a, b Patchy mid-gut infarction due to atherosclerotic occlusion of superior mesenteric artery. 



opening the abdomen, the entire small bowel is black and irreversibly ischaemic, 
then the most appropriate thing to do is close the abdomen and keep the patient 
comfortable with morphine: death will usually follow within a matter of hours. 

In the event that there is no pulse palpable at the SMA origin, then the revascular- 
isation strategies are similar to those for chronic visceral ischaemia (see Chapter 
24). In this case, attempts at thrombectomy will fail as the catheter will not cross the 
occlusion. The options available are retrograde bypass from the infrarenal aorta to 
the SMA, antegrade bypass from the supracoeliac aorta (with concomitant revascu- 
larisation of the coeliac trunk if it is occluded), aortomesenteric endarterectomy, or 
side-to-side anastomosis between the SMA and aorta. Prosthetic material should be 
avoided since transmural migration of bacteria is likely to contaminate the graft. 
Also, if the lower aorta and iliac systems are heavily calcified, then it may be better 



228 Vascular Surgery 

to select the supracoeliac aorta for the inflow site of the bypass. The use of a tempo- 
rary shunt for immediate reperfusion while the bypass is being constructed seems 
sensible [21]. 

At this point in the procedure, the viscera need to be inspected again and any 
dubiously viable bowel resected. If the patient is otherwise young and fit, then it 
may be better to resect all the necrotic bowel, exteriorise the remaining ends, and 
consider long-term total parenteral nutrition or small-bowel transplantation. If 
there is no evidence of arterial compromise and a pulse is palpable in the SMA, then 
one should suspect either NOMI or MVT. 

NOMI usually results in widespread patchy ischaemia. Treatment entails resec- 
tion of obviously non-viable bowel, attempting to be as conservative as possible. 
The remaining bowel must be inspected at a second-look laparotomy 24-48 h later. 
During surgery, several strategies have been described to improve mesenteric flow: 
a combination of systemic dopamine and an opiate epidural [25], papaverine infu- 
sion (30-60 mg/h) into the SMA either via direct puncture fine bore or angiographi- 
cally placed catheter [13, 26, 27]. At relook laparotomy, the extent of the ischaemia 
should be reassessed; treatment may need to be abandoned if the gangrene is pro- 
gressing. Mortality of this condition is depressingly high at 70-80% despite the ther- 
apeutic measures outlined above [13]. 

The classic laparotomy findings in MVT are ascites and swollen omentum with 
bowel infarction. Like NOMI, MVT is managed by resection of non-viable bowel, 
which is most frequently sharply demarcated and found in the mid-small bowel. 
Again, second-look laparotomy is mandatory. Anticoagulation with heparin and 
then warfarin is mandatory in view of the high incidence of recurrent MVT. 
Investigation of any underlying prothrombotic disorder along with long-term anti- 
coagulation improves survival [28, 29]. 

Thrombolysis has been used successfully in MVT in two studies where the diag- 
nosis had been established by non-invasive means and peritoneal signs had not 
developed [30, 31]. Venous thrombectomy has also been reported to be successful 
in a handful of cases [19, 32-34]. This procedure is likely to be difficult, as the peri- 
toneum is usually oedematous. [Q4: B, D] 

The classic features of ischaemia are oedema, loss of peristalsis, loss of surface 
sheen, staining of the serosa, absent mesenteric pulsation, or frank gangrene with or 
without perforation (Fig. 25.3). The decision at surgery is, after revascularisation 
(providing it was appropriate or feasible), how much small bowel to resect. Once 
there is good flow to the viscera, then reversibly ischaemic segments should declare 
themselves viable and the rest will need to be resected. Other adjuncts to inspection 
and palpation are continuous-wave Doppler, pulse oximetry and fluorescein dye 
[35]. The next question is whether to exteriorise or anastomose the open ends of the 
bowel. The arguments against exteriorising are that many stomata maybe necessary 
and they do not guarantee that the intervening segments may not subsequently 
become ischaemic; however, this is extremely safe and avoids the complication of a 
necrotic anastomosis. Performing primary anastomoses and leaving the abdomen 
open and covering it with a see-through bag (a cut-open bag of saline) [36] allows 
direct visualisation of the viscera at all times; second-look laparotomy can then be 
planned on the appearance of the gut [36]. If all looks well by 72 h and the patient's 
condition is stabilising, then they can be returned to theatre for planned abdominal 
closure. [Q5: A, C, E], [Q6: B, E] 

The short-term management of these cases is demanding on staff and time, but if 
best results are to be achieved, then no short cuts can be taken. 



Acute Mesenteric Ischaemia 



229 




Fig. 25.3. Gut showing features of fixed staining. 

AMI is a treatable vascular emergency. It requires a high index of clinical suspi- 
cion, rapid aggressive resuscitation and diagnostic manoeuvres to determine the 
specific underlying cause. This will allow a prompt, directed revascularisation pro- 
cedure after optimisation of cardiac performance, or correction of a hypercoagula- 
ble state. This effort is directed at maximising the amount of salvageable bowel. 
These strategies are the cornerstones for a successful outcome in this life-threaten- 
ing vascular catastrophe. 



References 



1. Marston A. Diagnosis and management of intestinal ischaemia. Ann R Coll Surg Engl 1972;50:29-44. 

2. Stemmer EA, Connolly JE. Mesenteric vascular insufficiency. Identification and management. Calif 
Med 1973;118:18-29. 

3. Stoney RJ, Cunningham CG. Acute mesenteric ischemia. Surgery 1993;114:489-90. 

4. Klempnauer J, Grothues F, Bektas H, Pichlmayr R. Long-term results after surgery for acute mesen- 
teric ischemia. Surgery 1997;121:239-43. 

5. Montgomery RA, Venbrux AC, Bulkley GB. Mesenteric vascular insufficiency. Curr Probl Surg 
1997;34:941-1025. 

6. McKinsey JF, Gewertz BL. Acute mesenteric ischemia. Surg Clin North Am 1997;77:307-18. 

7. Schoots IG, Koffeman GI, Legemate DA, Levi M, van Gulik TM. Systematic review of survival after 
acute mesenteric ischaemia according to disease aetiology. Br J Surg 200;91(l):17-27. 

8. Bergan JJ. Diagnosis of acute intestinal ischaemia. Semin Vase Surg 1990;3:143-8. 

9. Klass AA. Embolectomy in acute mesenteric ischaemia. Ann Surg 1951;134:913-17. 

10. Dunphy JE. Abdominal pain of vascular origin. Am J Med Sci 1936;192:109-12. 

11. Ende N. Infarction of the bowel in cardiac failure. N Engl J Med 1958;258:879-81. 

12. Fry RE, Huber PJ, Ramsey KL, Fry WJ. Infrarenal aortic occlusion, colonic blood flow, and the effect 
of nitroglycerin afterload reduction. Surgery 1984;95:479-86. 

Boley SJ, Sprayregan S, Siegelman SS, Veith FJ. Initial results from an aggressive roentgenological 
and surgical approach to acute mesenteric ischemia. Surgery 1977;82:848-55. 
Rhee RY, Gloviczki P. Mesenteric venous thrombosis. Surg Clin North Am 1997;77:327-38. 

15. Bassiouney HS. Non-occlusive mesenteric ischaemia. Surg Clin North Am 1997;77:319-26. 

16. Krupski WC, Selzman CH, Whitehill TA. Unusual causes of mesenteric ischaemia. Surg Clin North 
Am 1997;77:471-502. 

17. Clavien PA, Durig M, Harder F. Venous mesenteric infarction: a particular entity. Br J Surg 
1988;75:252-5. 



13 



14 



230 Vascular Surgery 

18. Grewal HP, Barrie WW. Congenital antithrombin III deficiency causing mesenteric venous infarc- 
tion: a lesson to remember - a case history. Angiology 1992;43:618-20. 

19. Vates P, Cumber PM, Sanderson S, Harrison BJ. Mesenteric venous thrombosis due to protein C 
deficiency. Clin Lab Haematol 1991;13:137-9. 

20. Tossou H, Iglicki F, Casadevall N, Delamarre J, Dupas JL, Capron JP. Superior mesenteric vein 
thrombosis as a manifestation of a latent myeloproliferative disorder. J Clin Gastroenterol 
1991;13:597-8. 

21. Whitehill TA, Rutherford RB. Acute intestinal ischaemia caused by arterial occlusions: optimal man- 
agement to improve survival. Semin Vase Surg 1990;3:149-56. 

22. Nicoloff AD, Williamson WK, Moneta GL, Taylor LM, Porter JM. Duplex ultrasonography in evalua- 
tion of splanchnic artery stenosis. Surg Clin North Am 1997;77:339-55. 

23. Jager K, Bollinger A, Valli C, Ammann R. Measurement of mesenteric blood flow by duplex scan- 
ning. J Vase Surg 1986;3:462-9. 

24. Clavien PA, Huber O, Mirescu D, Rohner A. Contrast enhanced CT scan as a diagnostic procedure in 
mesenteric ischaemia due to mesenteric venous thrombosis. Br J Surg 1989;76:93-4. 

25. Lundberg J, Lundberg D, Norgren L, Ribbe E, Thorne J, Werner O. Intestinal hemodynamics during 
laparotomy: effects of thoracic epidural anesthesia and dopamine in humans. Anesth Analg 
1990;71:9-15. 

26. Aldrete JS, Han SY, Laws HL, Kirklin JW. Intestinal infarction complicating low cardiac output 
states. Surg Gynecol Obstet 1977;144:371-5. 

27. Rivers SP. Acute non-occlusive intestinal ischaemia. Semin Vase Surg 1990;3: 172-5. 

28. Jona J, Cummius GM, Head MB, Govostis MC. Recurrent primary mesenteric venous thrombosis. 
JAMA 1974;227:1033-5. 

29. Matthews JE, White RR. Primary mesenteric venous occlusive disease. Am J Surg 1971;122:579-83. 

30. Al Karawi MA, Quaiz M, Clark D, Hilali A, Mohamed AE, Jawdat M. Mesenteric vein thrombosis, 
non-invasive diagnosis and follow-up (US + MRI), and non-invasive therapy by streptokinase and 
anticoagulants. Hepatogastroenterology 1990;37:507-9. 

31. Robin P, Gruel Y, Lang M, Lagarrigue F, Scotto JM. Complete thrombolysis of mesenteric vein occlu- 
sion with recombinant tissue-type plasminogen activator. Lancet 1988;1:1391. 

32. Inahara T. Acute superior mesenteric venous thrombosis: treatment by thrombectomy. Ann Surg 
1971;174:956-61. 

33. Mergenthaler FW, Harris MN. Superior mesenteric vein thrombosis complicating pancreatoduo- 
denectomy: successful treatment by thrombectomy. Ann Surg 1968;167:106-11. 

34. Daune B, Batt M, Graglia JC, Rogopoulos A, Hassen-Khodja R, Avril G, et al. Mesenteric ischemia of 
venous origin. The value of early computed tomography. Phlebologie 1990;43:615-18. 

35. Tollefson DF, Wright DJ, Reddy DJ, Kintanar EB. Intraoperative determination of intestinal viability 
by pulse oximetry. Ann Vase Surg 1995;9:357-60. 

36. Agrawal T, Refson J, Gould S. "Telly Tubby Tummy", a novel approach to the management of 
laparostomy. Ann R Coll Surg 2001;83(6):440. 

37. Hanisch E, Schmandra TC, Encke A. Surgical strategies - anastomosis or stoma, a second look - 
when and why? Langenbecks Arch Surg 1999;384:239-42. 



26. Renovascular Hypertension 

David Bergqvist and Martin Bjorck 



A 58-year-old male smoker had had essential hypertension for about 15 years. It 
was well balanced with diuretics. He came for regular checks over the years. At 
such a check 3 months ago, his blood pressure was 195/110 mm Hg. With the 
addition of a beta-blocker and an angiotensin-converting enzyme (ACE) 
inhibitor, it was possible to keep the pressure at around 180/100 mm Hg; thus it 
was not an optimal treatment result. Creatinine had increased over the last 3 
months from 90 to 180-200 |imol/l. The patient lived an active life and felt well. 



Question 1 

Which of the following statements support your suspicion that the patient has reno- 
vascular hypertension? 

A. The patient has had essential hypertension for more than 10 years. 

B. There is difficulty in controlling the blood pressure with three different drugs. 

C. There was an increase in serum creatinine when starting ACE inhibitor 
treatment. 

D. There is an absence of an epigastric bruit. 

Question 2 

What is the best investigation to carry out to decide whether the hypertension has a 
renovascular origin? 

A. Renin in serum. 

B. Renography with captopril provocation. 

C. Magnetic resonance angiography (MRA). 

231 



232 Vascular Surgery 

D. Duplex scanning. 

E. Angiography with pressure gradient. 

In this patient, an isotope renogram showed a prolonged uptake on the left side 
after captopril provocation. Duplex investigation showed a peak systolic velocity in 
the left renal artery of 2.7 m/s with normal findings on the right side. Both kidneys 
measured about 9 cm in length, the left being perhaps 0.5 cm shorter. MR angio- 
graphy showed a left-sided ostial stenosis with a suspect post-stenotic dilatation. It 
was decided to perform an angiography with the aim to establish whether there was 
a left-sided stenosis, and if there was to give a basis for deciding the optimal treat- 
ment. The suspicion of a left-sided stenosis was verified. It was localised near the 
aortic wall. The narrowest part of the renal artery was about 1.5 mm, with a distinct 
post-stenotic dilation. A pressure gradient of 40 mm Hg was measured. Corkscrew 
collaterals were seen along the ureter. 

Question 3 

What is the first treatment option to normalise the renal artery stenosis? 

A. Aortorenal bypass. 

B. Thromboendarterectomy. 

C. Percutaneous transluminal angioplasty (PTA) with stent. 

D. PTA. 

E. Nephrectomy. 

Question 4 

What is the main complication that may occur directly after renal artery PTA? 

A. Arterial rupture. 

B. Occlusion. 

C. Microembolisation. 

The patient underwent dilation with a stent without complications and was 
sent home the day after treatment in good condition. During the next 3 weeks, 
he was able to stop taking the beta-blocker and ACE inhibitor, and his blood 
pressure was maintained at around 160/90 mm Hg. Creatinine was around 
100 jimol/1. 

Question 5 

How should the patient be followed up? 
A. Serum creatinine. 



Renovascular Hypertension 233 

B. Clinical investigation with blood pressure control. 

C. Angiography. 

D. Duplex ultrasonography. 

E. Captopril scintigraphy. 



Commentary 

Renovascular disease is responsible for hypertension in around 1 per cent of all 
patients with high blood pressure. The definition of renovascular hypertension is, 
however, complicated by the complex and sometimes unclear relation between 
morphological alterations in the renal artery and the physiological effect of the 
stenosis. Hypertension is seen in 10-15 per cent of the adult population, and a reno- 
vascular cause varies between 0.2 and 5 per cent of those with hypertension; most 
often, a figure of around 1 per cent is given [1]. The most common cause is arte- 
riosclerosis, which is the probable aetiology in our case (age, sex, smoking, previous 
hypertension). With the potent antihypertensive drugs of today, it is possible to 
obtain fairly good blood pressure control in patients with renovascular hyperten- 
sion. Patients therefore are often not evaluated until there is ischaemic nephropathy 
with increasing creatinine [2]. As every patient with hypertension cannot be 
screened for renal artery stenosis, there are some criteria that may raise suspicion: 

Rapid onset of hypertension in young people. 

Rapid deterioration of previously well-controlled essential hypertension. 

Malignant hypertension or hypertensive crises. 

Three-drug-resistant hypertension. 

Hypertension and deteriorated renal function. 

Impaired renal function when starting ACE inhibitors. 

Abdominal, flank or back bruit. 

Our patient had had essential hypertension for more than 10 years, there was 
difficulty in controlling the blood pressure with three drugs, and there was an 
increase in serum creatinine when starting ACE inhibitor treatment. The 
absence of bruit is of no value; its presence would have given further support. 
[Q1:A,B,C] 

Diagnosis is difficult and the low prevalence contributes to the problem. If there 
is a clinical suspicion of renovascular hypertension, as in our patient, then renin in 
serum is of little if any value [3]. Angiography is not indicated to decide whether 
there is renovascular hypertension: it gives a morphological picture but there are 
many patients even with advanced renal artery stenosis in whom it is not of func- 
tional importance [4-6]. Although hypertension is prevalent in patients with renal 
artery stenosis, this does not mean it is a causal relationship [7]. To further 
strengthen the diagnosis a trans-stenotic pressure gradient of >15 mm Hg indicates 
that stenosis is of functional importance. The only absolutely certain way to define 
the relationship of renal artery stenosis in hypertension or ischaemic nephropathy 



234 Vascular Surgery 

is to observe the beneficial effect of an intervention. MRA and CT angiography are 
increasingly used to verify the diagnosis. The alternatives, isotope renography, 
preferably with captopril provocation [8], and duplex ultrasonography [9], both 
give some functional information but none has sufficiently high sensitivity and 
specificity to be truly diagnostic. [Q2: C, E] 

The first treatment choice in an uncomplicated case of renal artery stenosis is 
PTA [10], which was performed in this patient. Because of an irregular lumen with 
dissection, the procedure was completed with a stent with good morphological 
result. Stent placement in renal arteries was first used to correct suboptimal balloon 
dilations (recoil or plaque resistance), and complications such as dissection or 
restenosis [11-13]. Some authors recommend primary stenting [11, 14]. [Q3: D] 

Acute serious complications are infrequent. In a recent overview, the total rate 
was 17 per cent, with 2 per cent leading to surgery [15]. The most common serious 
acute complication is acute occlusion. [Q4: B] 

PTA of the renal artery stenosis is a convenient procedure for the patient, with 
few complications and short hospitalisation time. However, restenosis is not 
uncommon [10, 16, 17], and it therefore seems reasonable to have a surveillance 
programme, although the value of such a programme has not been established in 
any scientific study. The risk for restenosis is highest during the first year [10]. 
When there has been adequate preinterventional duplex investigation, as in this 
patient, it seems logical to continue with this investigation every 3 months during 
the first year and biannually thereafter. If this is not possible, then clinical investiga- 
tion with blood pressure control will be the choice. It must be noted, however, that 
the evidence basis for follow-up routines after renal artery angioplasty has yet to be 
defined. [Q5: B, D] 



References 



1. Berglund G, Andersson O, Wilhelmsson L. Prevalence of primary and secondary hypertension. BMJ 
1976;2:554-6. 

2. Bergentz S-E, Bergqvist D, Weibull H. Changing concepts in renovascular surgery. Br J Surg 
1989;76:429-30. 

3. Pohl MA. Renal artery stenosis, renal vascular hypertension and ischemic nephropathy. In: Schrier 
RW, Gottschalk CW, editors. Diseases of the kidney, 6th edn. Boston: Little, Brown, 1997;1367-423. 

4. Dustan HP, Humphries AW, De Wolf VG, Page IH. Normal arterial pressure in patients with renal 
arterial stenosis. JAMA 1964;187:1028-9. 

5. Schwartz CJ, White TA. Stenosis of renal artery: an unselected necropsy study. BMJ 1964;2:1415-21. 

6. Missouris CG, Buckenham T, Cappucio FP, Mac Gregor GA. Renal artery stenosis: a common and 
important problem in patients with peripheral vascular disease. Am J Med 1994;96:10-14. 

7. P, Thorvinger B, Parsson H, Norgren L. Renal artery stenosis in patients with peripheral vascular 
disease and its correlation to hypertension. A retrospective study. Int Angiol 1992;11:195-9. 

8. Nolly JV, Chen C, Fire E. Diagnostic criteria of renovascular hypertension with captopril renography 
- a consensus statement. Am J Hypertens 1991;4:7495-525. 

9. Hansen KJ, Tribble RW, Reavis SW. Renal duplex sonography: evaluation of clinical utility. Vase 
Surg 1990;12:227-36. 

10. Weibull H, Bergqvist D, Bergentz S-E. Percutaneous transluminal renal angioplasty versus surgical 
reconstruction of atherosclerotic renal artery stenosis: a prospective randomized study. J Vase Surg 
1993;18:841-50. 

11. Dorros G, Prince C, Mathiak L. Stenting of a renal artery stenosis achieves better relief of the 
obstructive lesion than balloon angioplasty. Catheter Cardiovasc Diagn 1993;29:191-8. 

12. Raynaud AC, Beyssen BM, Turmel-Rodriques LE. Renal artery stent placement: immediate and mid- 
term technical and clinical results. J Vase Intervent Radiol 1994;5:849-58. 



Renovascular Hypertension 235 

13. Van de Ven PJ, Bentler J J, Kaatee R. Transluminal vascular stent for ostial atherosclerotic renal 
artery stenosis. Lancet 1995;346:672-4. 

14. Bush R, Najibi S, MacDonald J, Lin P, Chaikol E, Martin L, Lumsden A. Endovascular revasculariza- 
tion of renal artery stenosis: technical and clinical results. J Vase Surg 2001;33:1041-9. 

15. Slonim S, Dake M. Radiographic evaluation and treatment of renovascular disease. In: Rutherford R, 
editor. Vascular surgery, 5th edn. Philadelphia: WB Saunders, 2000;1611-39. 

16. Jensen G, Zackrisson BF, Delin K. Treatment of renovascular hypertension: one year results of renal 
angioplasty. Kidney Int 1995;48:1936-45. 

17. Tullis M, Zierker R, Glickerman D, Bergelin R, Cantwell-Gab K, Strandness E. Results of percuta- 
neous transluminal angioplasty for atherosclerotic renal artery stenosis: a follow-up study with 
duplex ultrasonography. J Vase Surg 1997;25:46-51. 



27. Management of Portal Hypertension 

Yolanda Y. L. Yang and }. Michael Henderson 



A 37-year-old woman with a history of hepatitis C, cirrhosis, and esophageal 
varices presented with hematemesis and melena. The patient had a history of a 
prior esophageal variceal bleeding episode 7 years ago, which required transfu- 
sion of 4 units of packed red blood cells (PRBC) and had been treated with endo- 
scopic sclerotherapy. She was placed on nadolol at that time. 



Question 1 

If the patient had been found to have varices before any bleeding episode, she would 
benefit from which of the following? 

A. Endoscopic treatment: sclerotherapy or band ligation. 

B. Transjugular intrahepatic portal systemic shunt (TIPS). 

C. Non-cardioselective beta-blocker. 

D. A surgical shunt. 

The patient re-presents one year prior to her current admission with a further 
variceal bleed documented at endoscopy, which required 5 units of PRBC. The acute 
episode of bleeding was managed with variceal banding, and the patient underwent 
a course of banding on an outpatient basis. She had no encephalopathy at that time, 
but did develop some ascites for a short period that responded to salt restriction, 
Aldactone, and Lasix. Over this past year, her liver function tests have been stable 
with her bilirubin at 1.0, albumin at 3.5, and a normal prothrombin time. 

Question 2 

An episode of acute variceal bleeding usually requires which of the following? 



239 



240 Vascular Surgery 

A. ICU admission with hemodynamic monitoring, blood, blood products, and fluid 
resuscitation. 

B. An emergency portacaval shunt. 

C. A transjugular intrahepatic portal systemic shunt. 

D. Endoscopic therapy with sclerosis and/or band ligation. 

E. Pharmacologic therapy. 

At the present admission the patient is alert and oriented with no evidence of 
encephalopathy. She has well-preserved muscle mass on examination and is not 
clinically jaundiced. Her abdomen shows minimal ascites, with no hepatomegaly, 
but evidence of splenomegaly. Her laboratory studies showed a hemoglobin of 
7 g/dl, AST 24, alkaline phosphatase 84, albumin 2.6, bilirubin 3.4, and international 
normalized ratio (INR) 1.6. She was receiving blood transfusion when examined 
and octreotide infusion at 50 |ig/h. Esophagogastroduodenoscopy showed clot over 
an esophageal varix with evidence of other non-bleeding varices in both the distal 
esophagus and gastric fundus. 

Question 3 

Which of the following studies are important in evaluation and management 
decisions? 

A. Calculation of Child's score. 

B. Calculation of MELD score. 

C. Endoscopy. 

D. Doppler ultrasound. 

E. Angiography. 

Question 4 

Which of the following statements are accurate in prevention of recurrent variceal 
bleeding? 

A. All patients require portal decompression. 

B. First-line treatment is with endoscopic band ligation and a beta-blocker. 

C. Variceal decompression can only be achieved with a surgical shunt. 

D. Liver transplant is good treatment for variceal bleeding in patients with end- 
stage liver disease. 

Question 5 

Decompression of gastroesophageal varices: 



Management of Portal Hypertension 



241 



65 YEARS 




Fig. 27.1 . Splenic artery injection. The catheter is in the splenic artery and is injected with contrast. 



A. Can be achieved equally well with surgical shunt or TIPS. 

B. Should only be used for patients who have failed endoscopic and pharmacologic 
therapy for bleeding varices. 

C. Improves survival in patients with bleeding varices when compared to endo- 
scopic therapy. 

D. Is best achieved by liver transplant for all patients with variceal bleeding. 

The patient presented in this case had recurring bleeding episodes through first- 
line treatment and was therefore a candidate for decompression. Evaluation with 
angiography and ultrasound showed patent splenic and portal veins and a normal 
left renal vein (Figs 27.1-27.4). The patient had an elective distal splenorenal shunt 
for variceal decompression. She was in hospital for 7 days, and was discharged fol- 
lowing shunt catheterization (Fig. 27.5) and documentation of patency. Follow-up 
over the next 4 years showed some progression of her hepatitis C, but no further 
episodes of variceal bleeding. 



Commentary 

The case presented illustrates several important points: 

• Prophylactic management of gastroesophageal varices, strictly speaking, is prior 
to the first bleeding episode. The risk of bleeding in a patient with cirrhosis is 



242 



Vascular Surgery 



£5 YEARS 



SPLENH 
9-SEP-9: 

il 13 4; 




LEV ELAND CL 



Fig. 27.2. Splenic vein. The contrast is followed as it flows out of the splenic vein and then cephalad in the 
portal vein. There is a significant umbilical vein (double shadow with the portal vein) and a small left gastric vein 
(off the splenic vein) filling on this study. The second, more caudal catheter is positioned within the left renal 
vein to aid preoperative determination of the spatial relationship between the splenic and left renal veins. 



LT RENAL VEI 
29-SEP-3 




Fig. 27.3. Normal left renal vein. This study has been performed via the right jugular vein, and demonstrates 
the left renal vein as it heads cephalad towards the inferior vena cava. 



Management of Portal Hypertension 



243 



CI eveland CI in Foun' 



M: 3/13 
1:9/13 

LliLI^: 






m' 




Fig. 27.4. Circumaortic left renal vein. A circumaortic left renal vein, present in 20 percent of the population, 
does not prevent construction of a distal splenorenal shunt. The superior and anterior component is always 
larger and can be used for the shunt. More problematic is a totally retroaortic vein, found in 4 percent of the 
population, which runs transversely and is more fixed in the retroperitoneum, making exposure of the anasto- 
mosis more difficult. These patients are better served with a splenocaval shunt. 




Fig. 27.5. Postoperative catheterization of the distal splenorenal shunt. The tip of the catheter lies within the 
mobilized splenic vein, and the first bend marks the splenorenal anastomosis. The skin staples mark the 
extended left subcostal incision. 



244 Vascular Surgery 

approximately 30 percent. Once they have had one bleeding episode, the risk of 
rebleeding rises to 75 percent without active therapy. Non-cardioseclective beta- 
blockade with propranolol or nadolol is the preferred treatment for true pro- 
phylaxis for medium or large size varices. 

• Acute variceal bleeding is an emergency situation with a high mortality if not 
appropriately managed. Appropriate monitoring, pharmacologic therapy, and 
endoscopic diagnosis and treatment are the mainstays of treatment of an acute 
bleeding episode. It is a very small percentage of patients who do not have their 
bleeding controlled with the above measures and come to an emergency decom- 
pression. 

• The evaluation of the patient after an acute bleeding episode should assess the 
varices (endoscopy), the vascular anatomy (ultrasound and angiography) (Figs 
27.1-27.4), and the liver disease (Child's class and MELD score). [Q3: A, B, C, D, E] 

• When a patient has had an acute bleeding episode, their risk of rebleeding is 
over 70 percent if they have no specific treatment. The initial approach to treat- 
ment is to reduce the portal hypertension with a non-cardioselective beta- 
blocker, and to deal with the bleeding varices directly with endoscopic therapy. 
The majority of patients do not need variceal decompression at this stage. If the 
patient obviously has advanced to end-stage liver disease, a transplant evalua- 
tion is in order, and appropriate candidates should move forward with that 
treatment [Q4: B, D] 

• When patients have recurrent bleeding through first-line treatment they may 
need decompression of their gastroesophageal varices. Surgical therapy will do 
this well in 95 percent of patients, while the success rates of radiologic shunts in 
the literature are not this high. Decompression of varices does not improve the 
survival of patients compared to other first-line treatment options. Liver trans- 
plant provides excellent variceal decompression, but its use is dictated by end- 
stage disease rather than variceal bleeding. [Q2: A,D, E] 

General Considerations 

The major complications of portal hypertension are variceal bleeding, ascites, and 
progressive hepatic dysfunction. Ascites and encephalopathy are signs of decom- 
pensation, and as a general guideline, are only effectively managed by liver trans- 
plant. Not all patients with these clinical endpoints maybe suitable candidates for 
transplant. In contrast, variceal bleeding can occur in patients who have well- 
preserved liver function and therefore have a wider range of treatment options 
available. 

The etiology of portal hypertension may be presinusoidal, as in portal vein 
thrombosis; sinusoidal, as in cirrhosis; and rarely, post sinusoidal, as in 
Budd-Chiari syndrome. Much the most common etiology in the USA and Europe is 
cirrhosis, with approximately 90 percent of patients having this etiology. The evalu- 
ation of the patient with suspected portal hypertension includes an endoscopy to 
assess size and extent of varices with risk factors for bleeding. Larger varices with 
red color signs are at increased risk of bleeding or of rebleeding. Laboratory tests 
should assess liver function, and overall disease status. Non-specific tests include 
bilirubin, prothrombin time, albumin, and liver enzymes. Recently documented is 
the importance of serum creatinine in assessing overall severity of disease and prog- 



Management of Portal Hypertension 245 

Table 27.1 . Child-Pugh classification 



Parameter 


1 point 


2 points 




3 points 


Serum bilirubin (mg/dl) 


<2 


2-3 




>3 


Albumin (g/dl) 


>3.5 


2.8-3.5 




<2.8 


Prothrombin time (Ts) 


1-3 


4-6 




>6 


(INR) 


<1.7 


1.71-2.24 




>2.25 


Ascites 


None 


controlled 


medically 


controlled poorly or 
uncontrolled 


Encephalopathy 


None 


1-2 




3-4 



Classification: A, 5-6 points; B, 7-9 points; C, 10-15 points. 



Table 27.2. MELD score for stratification of liver disease severity 



Score = 0.957 x log e creatinine (mg/dl) 
+ 0.378 x log e bilirubin (mg/dl) 
+ 1.120xlog e INR 



nosis. The two standard methods for assessing this are the Child-Pugh score (Table 
27.1), and the Model for Endstage Liver Disease (MELD score - Table 27.2). Other 
laboratory studies that are important relate to the etiology with hepatitis panels, 
alpha-fetoprotein as a marker for hepatocellular carcinoma, and specific markers 
for metabolic diseases such as hemochromatosis and Wilson's disease. 

Imaging studies are important in evaluation, with ultrasound used to assess the 
liver morphology, and Doppler evaluation for liver vasculature. Patency of the main 
vessels and direction of flow can be assessed well with Doppler ultrasound. 
Angiography is still indicated for patients being considered for surgery. Accurate 
assessment of the splenic, portal, and left renal veins is important for distal 
splenorenal shunt, and may further elucidate details that are not seen on ultra- 
sound. Liver biopsy is occasionally indicated in some patients for clarification of 
etiology and to delineate the activity of the liver disease process. 

Management of portal hypertension falls in to three broad groups: 

• Prophylactic treatment. 

• Management of an acute variceal bleed. 

• Prevention of recurrent variceal bleeding. 

Prophylactic treatment is indicated for moderate or large size varices to reduce 
the risk of an initial bleed. Varices are present in 30-60 percent of patients with cir- 
rhosis. Thirty percent of patients with varices will bleed from them. After an initial 
bleed, 20-50 percent will rebleed in the first week, and 75-80 percent will rebleed 
within a year. The mortality of an acute bleeding episode is approximately 
25 percent. To reduce the risk of this initial bleed, the goal is to reduce portal pres- 
sure to <12 mm Hg or by 20 percent from the baseline. This is best achieved with a 
non-cardioselective beta-blocker (propranolol, nadolol) [1]. Other treatments, 
such as endoscopic therapy, TIPS, or surgical shunt are not indicated for prophy- 
laxis. There are currently further ongoing trials looking at band ligation for 



246 Vascular Surgery 

patients with large varices where this might be an appropriate method for prophy- 
laxis [2]. [Q1:C] 

Management of an acute variceal bleed involves resuscitation, pharmacologic 
reduction of variceal pressure, and endoscopic therapy [3]. Resuscitation requires 
careful monitoring and enough blood volume and transfusion to maintain blood 
pressure, but not over-transfuse and precipitate a vicious cycle of further bleeding. 
Octreotide is the drug of choice for pharmacologic pressure reduction and is given 
as a continuous infusion of 50 |ig/h. Endoscopic therapy is combined with endo- 
scopic evaluation and it best done with banding of varices if visibility is adequate. 
Occasionally, direct sclerotherapy injection maybe required to stop acute bleeding. 
In the <10 percent of patients who do not have their acute bleeding controlled with 
such measures, or in whom early significant rebleeding occurs, early decompression 
is occasionally required. This can best be achieved with TIPS at the current time. 

Prevention of recurrent variceal bleeding has to take in to account the risk of 
rebleeding, and the underlying liver disease. First-line treatment to prevent rebleed- 
ing is with a course of endoscopic banding in conjunction with pharmacologic 
therapy to reduce portal pressure with non-cardioselective beta-blocker [3]. This 
combination will reduce the risk of rebleeding to approximately 20 percent. 
Banding has been shown to be considerably better than sclerotherapy in terms of 
bleeding control and fewer complications. However, mortality is not significantly 
different in the randomized trials that compared banding to sclerotherapy. 
Concurrent with this first-line treatment, assessment and management of the 
underlying liver disease is important. At this time, assessment as to whether the 
patient is headed for transplant now or in the foreseeable future is important. If this 
is the case, more invasive therapies are precluded, transplant evaluation should be 
completed, and the patient should be appropriately listed. In this population, trans- 
plant has significantly improved the outcome of patients with Child's class C cirrho- 
sis who have end-stage disease, and have variceal bleeding. 

For better-risk patients who have recurrent bleeding through first-line treatment, 
variceal decompression may be indicated. The current options are with a radiologic 
shunt (TIPS) [4] or with a surgical shunt such as a distal splenorenal shunt (DSRS) 
[5], or some type of portacaval shunt. The literature data indicates that the rebleed- 
ing rate with TIPS is in the 15-20 percent range. Rebleeding with surgical shunts is 
in the 5 percent range. However, TIPS can be achieved in a much less invasive 
fashion compared to the major surgery required for a surgical shunt. Two random- 
ized trials have compared TIPS to surgical shunt. Rosemurgy et al. [6] compared 
TIPS to an 8-mm H-graft interposition portacaval shunt. They showed notably 
lower rebleeding in the surgical shunt group, significantly less need for transplant, 
but no difference in mortality. They concluded that surgical shunt was preferable to 
TIPS. Henderson et al. [7] have compared TIPS to DSRS in Child's class A and B 
patients. They showed no significant difference in rebleeding between DSRS (6 
percent) and TIPS (9 percent) in this trial; however, the TIPS group had an 82 
percent reintervention rate to maintain decompression and this excellent control of 
bleeding. The encephalopathy rates were not significantly different in the two 
groups, and neither was survival. The conclusion from this trial is that bleeding can 
be equally efficaciously managed with TIPS or DSRS with no difference in survival 
or encephalopathy; however, significantly more reintervention is required in 
patients managed with TIPS. This trial is summarized in Table 27.3. 

Have covered stents improved TIPS outcome? A multicenter prospective ran- 
domized trial in Europe [8] has shown a significantly lower dysfunction with 



Management of Portal Hypertension 247 

Table 27.3. Data for DSRS versus TIPS randomized trial [7] 



Results 


DSRSa? = 73 


TIPS n = 67 


P 


Rebleeding 


4 (5.5%) 


6 (9%) 


NS 


Reintervention 


8(11%) 


55 (82%) 


<0.0001 


Encephalopathy 








Single event 


36 (50%) 


34 (50%) 


NS 


Multiple events 


18(25%) 


17(25%) 


NS 


Survival 








2-year 


81% 


88% 


NS 


5 -year 


64% 


60% 


NS 



Table 27.4. Data for covered versus uncovered TIPS - European trial [8] 



Results 


PTFE 


Uncovered 


P 


"Dysfunction" 


5(15%) 


18(44%) 


<0.001 


Bleeding 


2/19(11%) 


4/29(14%) 


NS 


Ascites 


1/20(5%) 


8/1 2 (67%) 


<0.05 


Reintervention 


6/39(15%) 


22/41 (54%) 


<0.05 


Survival 


27/39 (69%) 


22/41 (54%) 


NS 



covered stents, with a particular advantage in control of ascites in that trial. Survival 
was not significantly different with covered or uncovered TIPS stents (Table 27.4). 
The data at this time would therefore indicate the following: 

• Patients with cirrhosis and moderate to large varices should receive prophylac- 
tic therapy with non-cardioselective beta-blocker prior to the initial bleed. 

• Patients with acute variceal bleeding should be managed in an intensive care 
unit with careful monitoring, adequate transfusion, pharmacologic and endo- 
scopic therapy. 

• Patients with recurrent variceal bleeding should be managed with endoscopic 
banding and a non-cardioselective beta-blocker. Only those patients who have 
well-preserved liver function, and rebleed through first-line treatment, should 
be considered for decompression. This can be achieved with either a surgical 
shunt or TIPS. Patients with end-stage liver disease need to be evaluated for 
their suitability for transplant and transplanted if appropriate. [Q5: B] 



References 

1. Schepke M, Kleber G, Nurnberg D, et al. Ligation versus propanolol for the primary prophylaxis of 
variceal bleeding in cirrhosis. Hepatology 2004;40:65-72. 

2. Sarin SK, Lamba GS, Kumar M, Murthy NS. Comparison of endoscopic ligation and propanolol for the 
primary prevention of variceal bleeding. N Engl J Med 1999;340:988-93. 

3. Grace ND, Groszmann RJ, Garcia-Tsao G, Burroughs AK, Pagliaro L, Makuch RW, et al. Portal hyper- 
tension and variceal bleeding: an AASLD single topic symposium. Hepatology 1998;28:868-80. 

4. Boyer TD, Haskal ZJ. The role of transjugular intrahepatic portosystemic shunt in management of 
portal hypertension. AASLD Practice Guidelines. Hepatology 2005;41:386-400. 



248 Vascular Surgery 

5. Henderson JM. Distal splenorenal shunt. In: Blumgart LH, editor. Surgery of the liver, biliary tract, 
and pancreas, 4th edn; Sect XV, Chap 98 2005; in press. 

6. Rosemurgy AS, Serafini FM, Zweibel BR, et al. Transjugular intrahepatic portosystemeic shunt vs. 
small- diameter prosthetic H-graft portacaval shunt: extended follow-up of an expanded randomized 
prospective trial. J Gastrointest Surg 2000;4:589-97. 

7. Henderson JM, Boyer TD, Kutner MH et al and the DIVERT study group. DSRS vs. TIPS for refractory 
variceal bleeding: a prospective randomized controlled trial. Hepatology 2004;40:725A. 

8. Bureau C, Garcia-Pagan JC, Otal P, et al. Improved clinical outcome using PTFE coated stents for 
TIPS: results of a randomized study. Gastroenterology 2004; 126:469. 



28. Management of Patients with Carotid 
Bifurcation Disease 

Wesley S. Moore 



A 72-year-old white male was referred for evaluation and management following 
the finding of an asymptomatic carotid bruit, picked up on routine physical 
examination by his primary-care physician. The patient was asymptomatic with 
respect to ocular or hemispheric ischaemic events. His risk factors included a 30- 
year history of smoking one pack of cigarettes a day, which he quit a year ago. He 
had hypertension that was controlled well by two drugs. He had no history of 
coronary artery disease, diabetes mellitus, or symptoms of peripheral vascular 
disease. On physical examination, his temporal pulses were equal. His carotid 
pulses were full and equal, but there was a loud bruit over the right carotid bifur- 
cation. His femoral, popliteal, dorsalis paedis and posterior tibial pulses were 
normally palpable bilaterally. 



Question 1 

What should the next step in this patient's evaluation be? 

A. Counselling with respect to the nature of carotid ischaemic attacks. 

B. Start the patient on an antiplatelet drug such as aspirin. 

C. Counsel the patient with respect to the importance of refraining from cigarette 
smoking and careful control of blood pressure. 

D. Obtain bilateral carotid duplex scanning. 

E. All of the above. 

The patient underwent a bilateral carotid duplex scan. [Q1: D] The scan demon- 
strated a category 60-79% right carotid bulb stenosis. The plaque characteristic was 
one of mixed consistency, a mildly irregular surface, and minimal calcification. The 
left carotid bulb showed a category 20-59% stenosis. Both vertebral arteries were 
imaged with normal antegrade flow velocities. 

251 



252 Vascular Surgery 

Question 2 

What would be appropriate management for this patient? 

A. Elective carotid endarterectomy. 

B. Full Coumadin anticoagulation. 

C. Aspirin antiplatelet management and risk factor control. 

The patient was placed on aspirin antiplatelet therapy, counselled regarding the 
importance of good blood pressure control, and given an appointment for a 
return visit in 6 months' time for a repeat carotid duplex scan to see whether there 
was any evidence of progression. The patient was also counselled regarding the 
importance of calling the vascular service should he develop ocular or hemi- 
spheric transient ischaemic attacks within the 6-month interval before his return 
appointment. [Q2: C] 

The patient did quite well for the next 4 months; then one afternoon, he noted the 
onset of an episode of numbness and weakness of his left hand. The hand was not 
totally paralysed, but it was clearly numb, weak and uncoordinated. This cleared 
completely within a period of 10 min. The patient thought that this might have been 
related to his arm position and chose to do nothing further until the next day, when 
the same event occurred. At this point, he called his physician and was advised to 
return immediately. An emergent carotid duplex scan was ordered. The scan now 
showed progression to a category 80-99% stenosis with plaque once again of mixed 
consistency. 



Question 3 

What is the best management for this patient? 

A. Clopidogrel antiplatelet therapy. 

B. Full Coumadin anticoagulation. 

C. Schedule elective carotid endarterectomy 1 month from now. 

D. Urgent right carotid endarterectomy. 

The patient now had two clear indications for proceeding with carotid 
endarterectomy: the onset of symptoms in the territory of the carotid lesion, and 
progression of the lesion to an 80-99% stenosis. Two additional decisions also had 
to be considered: the timing of operation and whether brain imaging was indicated. 
In view of the fact that the patient had an appropriate carotid artery lesion, and the 
symptoms were typical for hemispheric transient ischaemic events in the distribu- 
tion of the carotid lesion, information gained from brain imaging such as computed 
tomography (CT) or magnetic resonance imaging (MRI) would be limited. 
Therefore, the cost/benefit ratio for brain imaging was clearly unfavourable. The 
timing of carotid endarterectomy was urgent. The patient had a new onset of tran- 
sient ischaemic attacks and evidence of plaque progression. Therefore, the patient 



Management of Patients with Carotid Bifurcation Disease 253 

was now at highest risk of a hemispheric stroke. The optimum management for this 
patient would be emergent admission to the hospital and rapid evaluation for oper- 
ation, including the patient's cardiac status. While this was taking place, it would be 
appropriate to start the patient on intravenous heparin anticoagulation. Once 
cleared from a cardiac standpoint, plans should be made to proceed with operation 
either that day or the next morning. [Q3: D] 

The patient was admitted as an emergency to the hospital and started on intra- 
venous heparin with a loading dose of 5,000 units and a continuing dose of 1,000 
units/h. He was seen in cardiology consultation, and an electrocardiogram (ECG) 
was obtained. In the absence of any symptoms of coronary disease, and with a rela- 
tively normal ECG, he was cleared for operation. 



Question 4 

What should the next step in this patient's management be? 

A. Aortic arch angiogram with selected carotid arteriograms. 

B. Magnetic resonance angiogram (MRA). 

C. CT angiogram. 

D. Proceed with operation on the basis of a duplex scan of diagnostic quality in an 
accredited laboratory. 

The patient was taken to the operating room the next morning. Before this, 
EEG electrodes were placed for intraoperative monitoring. An arterial line was 
placed for blood pressure monitoring, and general anaesthesia was administered. 
A vertical incision along the anterior border of the sternomastoid muscle was 
made. The facial vein was divided, and the common carotid, carotid bifurcation, 
internal and external carotid arteries were fully mobilised. There was a posterior 
plaque present in the common carotid artery, which was nonocclusive. The major 
plaque build-up was in the bulb of the internal carotid artery, which went a short 
distance beyond the bulb into the internal carotid artery distally. Beyond this 
point, the vessel was circumferentially soft. The distal internal carotid artery was 
somewhat collapsed, and no distal pulse was noted. Since the patient had experi- 
enced only transient symptoms and not a completed stroke, it was our plan to use 
an internal shunt only if there were electroencephalogram (EEG) changes with 
trial clamping. A bolus of 5,000 units of heparin was administered, and the inter- 
nal, external and common carotid arteries were clamped. The EEG was observed: 
there were no changes. The amplitude and frequency of the EEG wave form were 
maintained. A longitudinal arteriotomy was made in the common carotid artery 
and extended through a very tight carotid stenosis. The plaque within the carotid 
bulb showed evidence of recent intraplaque haemorrhage. As we passed through 
the plaque, we emerged into an unencumbered internal carotid artery distally. A 
bifurcation endarterectomy was then performed with clean endpoints in the 
internal, external and common carotid arteries. The intimectomised surface was 
then irrigated with heparinised saline, and small bits of medial debris were 
removed carefully. Fixation of the endpoint was carried out. Once we were 



254 Vascular Surgery 

satisfied that there was no evidence of intimal flap and all of the loose bits of 
medial debris were removed, attention was turned to closure. 



Question 5 

Closure of the arteriotomy should be: 

A. A primary, carefully placed closure with 6-0 prolene. 

B. Closure with a patch angioplasty. 

The patient's arteriotomy was closed with a patch angioplasty using a collagen- 
impregnated knitted Dacron patch that was cut to length and bevelled at each end. 
Upon completion of the closure, blood flow was begun first to the external then to 
the internal carotid artery. Excellent pulsation in all vessels was noted. We then 
carried out a completion angiogram by placing a small needle in the patch and 
injecting contrast into the carotid bifurcation using a portable cine-fluoro unit. The 
carotid bifurcation was imaged, and there was an excellent technical result with no 
evidence of residual stenosis or intimal flap. Intracranial imaging was also carried 
out, and excellent flow into the carotid siphon and the anterior and middle cerebral 
arteries was confirmed. After meticulous haemostasis was achieved, a 7.0-mm 
Jackson Pratt drain was placed in the wound and brought out through a separate 
stab wound. The platysmal layer was closed with an absorbable suture, and the skin 
was closed with a subcuticular absorbable suture. An adhesive plastic dressing was 
applied directly to the skin, and the patient was returned to the recovery room. The 
patient awoke at his neurological baseline with no evidence of cerebral or cranial 
nerve deficit. His blood pressure was monitored carefully and was noted to be stable 
at 150/80 mm Hg. 



Question 6 

After an appropriate stay in the recovery room, to where should the patient be 
transferred? 

A. An intensive care unit with continual monitoring overnight. 

B. A step-down unit with 3 : 1 nursing coverage and monitoring capability. 

C. The patient should be left in the recovery room overnight. 

D. A regular hospital room. 

Since the patient was neurologically intact and was maintaining his normal 
blood pressure, he was transferred to a regular hospital room for routine 
overnight care. The patient spent an uneventful night in a regular hospital room. 
The following morning, we removed the dressing and drain. The patient was 
ambulatory and on a regular diet and was discharged home on the first postoper- 
ative day [1]. This management is typical of the so-called "fast-track" manage- 



Management of Patients with Carotid Bifurcation Disease 255 

ment of carotid bifurcation disease. Patients are usually admitted electively on 
the morning of operation, undergo carotid endarterectomy, spend a period of 
2-3 h of observation in the recovery room, transfer to a regular hospital room, 
and are discharged the following morning. Thus, carotid endarterectomy has 
become extremely cost-effective in the overall medical economic environment. 
The patient was instructed to return for a routine visit in 3 weeks. At that time, 
we obtained a right carotid duplex scan to confirm the result of carotid 
endarterectomy and to establish a new baseline for future comparison. The next 
visit will be in 6 months, at which time a bilateral carotid duplex scan will be per- 
formed. The objective will be to look for evidence of intimal hyperplasia and 
recurrent stenosis on the side of operation as well as to document whether there 
is any progression of disease on the contralateral, nonoperative side. If that test is 
unremarkable, then the next study will be at the 1-year anniversary. We will then 
see the patient on a yearly basis and obtain a bilateral carotid duplex scan as a 
part of that visit. 



Commentary 

Many decisions concerning recommendation to perform carotid endarterectomy 
are based upon the degree of stenosis, as measured by a percentage, in the carotid 
artery. All of the randomised trials have reported their data and have established a 
baseline threshold stenosis as an appropriate indication for carotid endarterectomy. 
While this would appear to be a very tangible and straightforward method of quan- 
tifying a carotid stenosis, confusion has developed because there are at least two dif- 
ferent techniques for measuring percent of carotid stenosis: the North American 
method and the European method. The North American method was first described 
in a publication by Blaisdell et al. as part of the Extracranial Arterial Occlusive 
Disease Study of the 1960s. This method was used in the Veterans Administration 
Asymptomatic Carotid Stenosis trial and the Asymptomatic Carotid Atherosclerosis 
Study (ACAS), and was subsequently adopted by North American Carotid 
Endarterectomy Trial (NASCET) as their method of measurement. The North 
American method utilises the following formula: percentage stenosis = 1 - R/D, 
where R is the minimal residual lumen diameter in millimetres, and D is the diame- 
ter of the normal internal carotid artery, distal to the bulb, where the walls of the 
artery become parallel. In contrast, the European method, which has been used in 
European trials, including the European Carotid Surgery Trial (ECST) trial, uses the 
following formula: percentage stenosis = 1 - R/B, where R again is the minimal 
residual lumen diameter in millimetres, and B is the projected diameter of the 
carotid bulb. Since the bulb is not visualised on a carotid arteriogram of a patient 
with carotid stenosis, a theoretical line is drawn outlining the bulb, emphasising the 
atheromatous burden within the bulb. Because of these two different methods, per- 
centage stenoses as expressed in the European literature are not equal to percentage 
stenosis as measured by the North American method. For example, a 60 per cent 
stenosis European is equal to an 18 per cent stenosis North American; 70 per cent 
stenosis European equals 40 per cent stenosis North American; 80 per cent stenosis 
European equals 61 per cent stenosis North American; and 90 per cent stenosis 



256 Vascular Surgery 

European equals 80 per cent stenosis North American. Thus, when reading a 
specific article relating to carotid stenosis, it is important to determine which 
method of measurement is used in order to appropriately follow the recom- 
mendations made by the authors. 

The management of patients with asymptomatic high-grade carotid stenosis 
has been controversial. However, with the recent publication of the ACAS, the 
approach to management of patients who are asymptomatic has received more 
universal acceptance in the USA, although not necessarily so in Canada and 
Europe. The findings of this trial demonstrated that there was a 53 per cent rela- 
tive risk reduction of stroke in patients who underwent carotid endarterectomy 
for lesions producing at least a 60 per cent diameter-reducing stenosis, by angiog- 
raphy, when compared with medical management alone. It was also pointed out 
that a 60 per cent diameter-reducing stenosis by angiography is not the same as a 
60 per cent stenosis as measured by duplex scan, since the duplex scan criteria for 
stenosis are concerned with carotid bulb measurement rather than a stenosis as 
compared with the diameter of the distal internal carotid artery. It is generally 
accepted that a 60 per cent diameter-reducing stenosis of the internal carotid 
artery, by angiography, usually corresponds to a duplex scan finding of an 
80-99% stenosis [2]. 

Patients with carotid artery disease who develop symptoms of hemispheric or 
monocular transient ischaemic events, or who have had a stroke with good recov- 
ery, are clearly good candidates for carotid endarterectomy providing that they 
have a diameter-reducing stenosis of 50 per cent or greater by angiography. This is 
now accepted uniformly and has been well established by prospective randomised 
trials in both North America and the UK [1, 3, 4]. 

The work-up of patients with carotid bifurcation disease for operation usually 
involves the performance of a contrast angiogram to confirm the lesion, establish 
the degree of stenosis, and evaluate the intracranial circulation for other pathology, 
such as a stenosis of the carotid siphon or an aneurysm of the intracranial branches. 
As the quality and accuracy of carotid duplex scanning has improved in accredited 
laboratories throughout the world, the practice of using carotid duplex scan data as 
the sole imaging requirement before endarterectomy has proliferated. In our own 
unit, the accuracy of carotid duplex scanning in our laboratory is continually com- 
pared with the operative findings at the time of carotid endarterectomy. Initially, 
the carotid duplex scan data were compared with angiography. As our level of 
comfort with carotid duplex scanning has increased, contrast angiography has 
essentially been eliminated in our protocol. The only time we resort to a contrast 
angiogram is when the carotid duplex scan data and the clinical picture fail to cor- 
relate. If the patient has equal upper-extremity blood pressures, as well as good and 
equal quality pulses in the carotid artery bilaterally, then the likelihood of the 
patient harbouring a lesion at the level of the aortic arch is quite small. The only 
other pathology that might be missed in the absence of a contrast carotid 
angiogram is the rare occurrence of an intracranial lesion. It has been our practice 
to carry out completion angiography following carotid endarterectomy on the oper- 
ating table. When the completion study is performed, we always make an effort to 
examine the intracranial circulation as well. To date, after many hundreds of 
carotid endarterectomy without angiography, there have only been two instances in 
which significant intracranial arterial pathology has been found. One was a small 
intracranial aneurysm measuring less than 10 mm; the other was a siphon stenosis, 
which, had it been known preoperatively, would not have changed the indication 



Management of Patients with Carotid Bifurcation Disease 257 

for carotid endarterectomy. Based upon this experience, we routinely carry out 
carotid endarterectomy on the basis of duplex scan alone. However, this duplex 
scan must be performed in our own laboratory, as we are unwilling to accept data 
from other laboratories as the sole basis for proceeding with operation. While there 
are many excellent laboratories that provide reliable data, we routinely cross-check 
data from outside laboratories with a test in our own laboratory. Since duplex scan- 
ning is relatively inexpensive, and since it has become a substitute for expensive 
studies associated with morbidity and mortality, such as contrast angiography, it is 
our opinion that this additional cost is money well spent [5]. Contrast angiography, 
while a longstanding gold standard, is expensive, promotes patient anxiety, and is 
associated with neurological morbidity and mortality. In the ACAS, where angio- 
graphy was required before carotid endarterectomy, the risk of the angiogram with 
respect to stroke morbidity and mortality was equal to the risk of the operation 
itself [1]. MRA, while noninvasive, tends to be less accurate than a well-performed 
carotid duplex scan. MRA of the carotid bifurcation will frequently overestimate the 
percentage of stenosis and will lead to unnecessary operation in many instances. CT 
angiography, while more accurate, requires a large intravenous contrast bolus to 
perform the study. [Q4: D] 

Another controversy in the management of patients with carotid bifurcation 
disease concerns the question of whether a carotid arteriotomy should be closed pri- 
marily or with a patch angioplasty. For many years, we routinely closed arteriotomies 
primarily when the vessel appeared to be of good calibre. A retrospective review of 
our data suggested that this had been a good practice in that our incidence of 
restenosis had been quite low. Many retrospective comparisons as well as prospective 
trials have shown inconclusive data concerning the merit of patch angioplasty versus 
primary closure. However, recently, a prospective trial in patients scheduled for 
staged bilateral carotid endarterectomy in whom one side was primarily closed and 
the second side closed with patch angioplasty conclusively demonstrated that those 
sides closed with patch angioplasty were associated with a statistically lower inci- 
dence of restenosis and complication. Based upon these convincing data, it is now 
our practice to routinely close all arteriotomies with a patch angioplasty [6]. [Q5: B] 
Other surgeons have modified their surgical practice to perform the operation using 
eversion endarterectomy, thus avoiding a longitudinal arteriotomy. For those sur- 
geons who are experienced with this technique, and in properly selected patients, this 
also appears to be a satisfactory alternative. The postoperative monitoring of the 
patients is important in ensuring the best outcome for these patients. In the past, it 
had been our practice to monitor patients routinely in the intensive care unit. 
However, with a retrospective review of our experience, the likelihood of having an 
untoward event requiring intensive-care nursing in a patient who was neurologically 
intact and with a normal blood pressure was extremely low. Therefore the 
cost/benefit advantage of intensive care unit utilisation was clearly not there. We now 
routinely send patients to a regular hospital room. To date, there have been no unto- 
ward incidents that have led us to regret this policy. [Q6:D] 



References 

1. Moore WS, Barnett HJ, Beebe HG, Bernstein EF, Brener BJ, Brott T, et al. Guidelines for carotid 
endarterectomy: a multidisciplinary consensus statement from the ad hoc committee, American Heart 
Association. Stroke 1995:26:188-201. 



258 Vascular Surgery 

2. Executive Committee for the Asymptomatic Carotid Atherosclerosis Study (ACAS). Endarterectomy 
for asymptomatic carotid artery stenosis. JAMA 1995;273:1421-8. 

3. North American Symptomatic Carotid Endarterectomy Trial Collaborators. Benefit of carotid 
endarterectomy in patients with symptomatic moderate or severe stenosis. N Engl J Med 
1998;339:1415-25. 

4. Randomized trial of endarterectomy for recently symptomatic carotid stenosis: final results of the 
MRC European Carotid Surgery Trial. Lancet 1998;351:1379-87. 

5. Chervu A, Moore WS. Carotid endarterectomy without arteriography. Personal series and review of 
the literature. Ann Vase Surg 1994;8:296-302. 

6. AbuRahma AF, Robinson PA, Saiedy S, Richmond BK, Khan J. Prospective randomized trial of bilateral 
carotid endarterectomies: primary closure versus patching. Stroke 1999;30:1185-9. 



29. Carotid Endarterectomy and Cranial 
Nerve Injuries 

Christos D. Liapis and John D. Kakisis 



A 75-year-old male was admitted with a high-grade left internal carotid artery 
(ICA) stenosis. The patient had a history of a reversible ischemic neurological 
defect (RIND) causing him a right upper- and lower-extremity paralysis and 
dysarthria a month prior to his admission. Colour duplex ultrasonography 
revealed a carotid bifurcation stenosis with a 90 per cent reduction in the diame- 
ter of the left ICA. The right ICA was also stenosed with an 80 per cent diameter 
reduction. There was no history of cerebrovascular symptoms that could be 
attributed to the right ICA stenosis. 

On examination there were bilateral carotid bruits. Peripheral pulses were 
normal in the upper extremities. In the lower extremities there were no pulses 
distal to the popliteals. Full blood count, biochemical profile and a clotting 
screen were within normal limits, while an electrocardiogram (ECG) and dipyri- 
damole stress test revealed changes consistent with coronary artery disease. 
Chest X-ray and arterial blood gas were normal. Computed tomography (CT) 
scan of the brain showed a small cerebral infarct at the left parietal lobe and two 
smaller ones at the right. 



Question 1 

What other preoperative examination would you perform routinely before taking 
this patient to the operating theatre for a carotid endarterectomy? 

A. Lumbar puncture. 

B. Electroencephalography (EEG). 

C. Evaluation of the function of the IXth, Xth and Xllth cranial nerves. 

D. Ocular plethysmography. 

E. Magnetic resonance imaging (MRI) of the brain. 



259 



260 Vascular Surgery 

A preoperative examination of the IXth, Xth and Xllth cranial nerve function was 
carried out, which revealed no neurological abnormalities. 

Before the operation, the surgeon had a detailed discussion with the patient, 
including information about the possible complications of carotid endarterectomy. 

Question 2 

Which of the following is not an expected complication following carotid 
endarterectomy? 

A. Stroke. 

B. Myocardial infarction (MI). 

C. Trigeminal nerve palsy. 

D. Vocal cord paralysis. 

E. Hypoglossal palsy. 

Following informed consent, the patient underwent a left carotid endarterec- 
tomy. Postoperatively, neurological examination was normal, but the patient com- 
plained of hoarseness. 

Question 3 

Which of the following is the most likely cause of speech malfunction in our 
patient? 

A. Injury to the vocal cords during intubation. 

B. Recurrent laryngeal nerve injury. 

C. Superior laryngeal nerve injury. 

D. Injury to the larynx due to excessive retraction. 

E. Cerebral haemorrhage. 

Indirect laryngoscopy revealed left vocal cord paralysis. The postoperative course 
was otherwise uneventful. The patient was discharged on the fourth postoperative 
day. 

Question 4 

How long after the operation should the patient be re-evaluated, considering his 
vocal cord paralysis? 

A. Weekly. 

B. Monthly. 



Carotid Endarterectomy and Cranial Nerve Injuries 261 

C. At 2 and 6 months. 

D. There is no reason for re-evaluation since all of these palsies resolve within a few 
days. 

E. There is no reason for re-evaluation since no improvement is anticipated. 

Two months postoperatively, laryngoscopy and voice evaluation tests revealed 
normal function of the cranial nerves. The patient was scheduled for right carotid 
endarterectomy. 

Question 5 

What are the treatment options for a patient with recurrent laryngeal nerve injury 
after carotid endarterectomy who needs a contralateral carotid operation? 

A. Carotid stenting. 

B. Carotid endarterectomy after nerve function recovery. 

C. Immediate carotid endarterectomy. 

D. Carotid endarterectomy after two weeks. 

E. Never operate on this carotid. 



Commentary 

Carotid endarterectomy is associated with a number of serious complications, 
including stroke, death, MI, wound complications, hypertension or hypotension, 
and cranial nerve injuries. [Q2: C] Stroke has undoubtedly been the main focus of 
interest, while local injuries to the cranial nerves and their branches have received 
considerably less attention. However, cranial nerve injuries during carotid 
endarterectomy are quite frequent and potentially serious; they may even be life- 
threatening when they are bilateral. 

There is remarkable discrepancy regarding the occurrence of cranial nerve 
injuries following carotid arterial surgery in various studies. The reported incidence 
ranges from 3 per cent to more than 50 per cent [1-20] (Table 29.1). These differ- 
ences are due mainly to the different investigative methods used for the evaluation 
of the cranial nerve function. The inclusion of speech therapists and their examina- 
tion tests in the evaluation, as proposed by Liapis et al. [9], can clarify most accu- 
rately the function of the nerves in question. An additional problem is the lack of 
standardisation in reporting lesions, concerning the number of cranial nerves eval- 
uated and the different methods of reporting multiple injuries. Some authors use 
the number of damaged nerves in their reports, while others use the number of 
patients with damaged nerves. Nevertheless, the actual incidence of cranial nerve 
injuries seems to range between 8 and 16 per cent in most reports. 

The nerves that can be damaged during carotid endarterectomy, due to their 
close relation to the carotid bifurcation, are the hypoglossal, recurrent laryngeal, 



262 Vascular Surgery 

Table 29.1. Incidence and persistence of cranial nerve injuries after carotid endarterectomy (incidence/per- 
sistence). Updated 







Recurrent 


Superior 


Marginal 








Ref. 


Hypoglossal 


laryngeal 


laryngeal 


mandibular 


Glossopharyr 


igeal 


Total 


1 


13.5/0 


5.8/0 




5.8/0 






25/0 


2 


5.5/0 


4/0 


1/0 


1/0 






1 1 .5/0 


3 


4.7/0 


25.6/13.9 










30.3/13.9 


4 


11/4 


35/15 






1.5/0 




39/19 


5 


3.7* 


2.5* 




2.2* 






8.6* 


6 


10.7/0.2 


1 .2/0.2 


0.3/0 




0.3/0 




12.5/0.3 


7 


5.4/0.8 


5.8/0.8 


2.1* 


2.5* 






15.8/2.1 


8 


2.3/0 


3.9/3.1 




3.1/0.8 






9.3/3.9 


9 


20/2.5 


27.5/5 




5/0 


0/0 




52.5/7.5 


10 


4.8* 


6* 




2.4* 






13.5* 


11 


3.3/0 


2.6/0 




0.4/0 


0.7/0 




5.6/0 


12 


8.2/2.5 


5.1/0 




3.2/1.3 






16.4/3.8 


13 


1.2/0.2 


1 .2/0.7 




0.5/0.2 


0.2/0 




3/1.2 


14 


4.4/0.6 


7.7/0.6 




1.1/0 






14.2/1.1 


15 


8.3/0 


3.7/1.8 




2.8/0 






12.8/1.8 


16 


2.6/0 


1/0 




1/1 






4.7/1 


17 


8.6/2.6 


3.7/0.6 




6.2/2.8 


0.4/0 




18.9/6 


18 


4/0.5 


7/1 




12/1 






27/7 


19 


2.1/0.2 


1/0.06 




1.8/0.06 






5.1/0.5 


20 


1.1/0 


0.6/0.6 




3.9/0 






5.6/0.6 



incidence of cranial nerve injury (%). 



superior laryngeal and glossopharyngeal nerves, the marginal mandibular branch of 
the facial nerve, and the greater auricular and cervical sensory branches (Fig. 29.1). 
The hypoglossal (Xllth) nerve descends into the neck between the internal 
carotid artery and the internal jugular vein. It then crosses the internal and the 
external carotid artery approximately 2-4 cm above the carotid bifurcation to enter 
the root of the tongue. It is a purely motor nerve, innervating all of the intrinsic 
muscles of the tongue and three extrinsic muscles: the styloglossus, hyoglossus and 
genioglossus. The function of the nerve is assessed by measuring tongue muscle 
activity. Patients are asked to engage in tasks requiring tongue strength, mobility 
and range of motion, namely to stick out their tongue, to move it from side to side, 
to raise it to the nose or lower it to the chin and to lick their lips [9]. Injury to the 
hypoglossal nerve, which can happen during dissection of a high bifurcation, can be 
avoided by following the ansa hypoglossi up to its junction with the hypoglossal 
nerve and by dividing the sternocleidomastoid artery and vein separately, since the 
nerve maybe adherent to the posterior surface of these vessels [7]. Division of the 
ansa hypoglossi, which supplies motor innervation to the infrahyoid muscles, will 
not be clinically overt. However, it should be spared if possible, because it can be 
invaluable in cases of future laryngectomy/laryngoplasty. In contrast, injury to the 
hypoglossal nerve is a serious complication, causing deviation of the protruding 
tongue to the ipsilateral side, while the relaxing tongue deviates to the contralateral 
side. Inarticulate speech and clumsy mastication will also ensue, whereas bilateral 
injury may cause life-threatening airway obstruction. 



Carotid Endarterectomy and Cranial Nerve Injuries 



263 



& 



<xc\<x\ n 



Vaaus v\ 




AnscL 
CevvlcoJi 



<S\o^ofb\ayyyiae.o,l n. 



H vpo^loi-val n 



Descending bvancla. 
of hypoalo^>aJ n.. 



Fig. 29.1. Anatomy of the cranial nerves in relation to the carotid bifurcation. 



The superior laryngeal nerve arises from the vagus (Xth) nerve near the jugular 
foramen and passes diagonally behind the internal and external carotid artery 
before dividing into an internal sensory branch, supplying sensation to the mucosa 
of the larynx and an external motor branch, supplying innervation to the cricothy- 
roid muscle and the inferior pharyngeal constrictor. Assessment of the superior 
laryngeal nerve is done by asking the patient to produce tones of the upper range 
for about 15 s [9]. Injury to the superior laryngeal nerve can occur if the clamp is 
placed away from the origin of the external carotid artery and will cause voice 
fatigue and loss of high-pitch phonation. 

The recurrent laryngeal nerve arises from the vagus nerve in the mediastinum, 
passes around the subclavian artery on the right side and the aortic arch on the left, 
and ascends the neck in the groove between the oesophagus and the trachea. Only 
in cases of anomalous origin or course in the neck can the recurrent laryngeal nerve 
be injured directly during carotid endarterectomy. Dysfunction of the recurrent 
laryngeal nerve is due to direct injury of the vagus nerve during clamp placement, 
except in the very rare cases of the non-recurrent laryngeal nerve, when it can be 



264 Vascular Surgery 

mistaken for the ansa hypoglossi and severed erroneously. The recurrent laryngeal 
nerve is motor to all of the muscles of the larynx except for the cricothyroid muscle, 
and sensory to the mucosa below the vocal cords. Assessment of its function can be 
made by laryngoscopy as well as by asking the patient to produce sounds of the lower 
range [9]. In cases of recurrent laryngeal nerve injury, laryngoscopy will reveal paral- 
ysis of the ipsilateral vocal cord in a median or paramedian position, resulting in 
hoarseness and loss of effective cough mechanism. However, it should be noted that 
postoperative hoarseness does not necessarily imply damage to the recurrent laryn- 
geal nerve. Injury of the vocal cords during intubation, with subsequent oedema or 
haematoma formation, is a fairly frequent cause of postoperative hoarseness [18, 20]. 
[Q3: A] Differential diagnosis will be made by indirect laryngoscopy. 

It should also be noted that vocal cord paralysis of degenerative aetiology is 
sometimes seen in elderly people. It is therefore recommended that all people 
scheduled for carotid endarterectomy should undergo preoperative examination of 
the vocal cords by indirect laryngoscopy. [Q1: C] Such an examination is of the 
utmost importance in patients with a history of thyroidectomy or contralateral 
endarterectomy. Preoperative evaluation should be completed with examination of 
the IXth and Xllth cranial nerves in order to document possible pre-existing cranial 
nerve deficits. 

The glossopharyngeal (IXth) nerve exits the skull through the jugular foramen, 
passes between the internal jugular vein and the internal carotid artery, and then 
crosses the distal cervical segment of the internal carotid artery and the stylopha- 
ryngeal muscle to enter the pharynx. The glossopharyngeal nerve supplies (1) sen- 
sation from the posterior third of the tongue and the walls of the pharynx, being the 
afferent limb of the gag reflex, (2) taste sensation from the posterior third of the 
tongue, and (3) innervation to the stylopharyngeal muscle. The function of the glos- 
sopharyngeal nerve is assessed by administering tests requiring stylopharyngeal 
muscle activity, such as swallowing, gag reflex and clearing the throat [9]. Damage 
to the glossopharyngeal nerve can occur when using cautery during high dissection, 
especially when the posterior belly of the digastric muscle is divided [21]. Difficulty 
in swallowing is the principal manifestation of such an injury. 

The marginal mandibular branch of the facial (Vllth) nerve emerges onto the 
face from the anterior border of the parotid gland, loops down over the ramus of 
the mandible, crosses the masseter muscle and enters the perioral muscles in the 
lower lip. It supplies the risorius muscle and the depressor of the lower lip. Injury to 
the marginal mandibular branch of the facial nerve may occur during superior 
extension of the incision and upward retraction. In order to avoid such an injury, 
the upper end of the incision should be curved posteriorly toward the mastoid 
process, while the retractors should be placed away from the angle of the mandible 
and superficial to the platysma [14]. Injury to the marginal mandibular nerve will 
cause sagging of the ipsilateral corner of the lower lip, which has to be differentiated 
from stroke. 

The spinal accessory (Xlth) nerve arises from the jugular foramen, courses poste- 
riorly and passes laterally to the jugular vein, sending branches to the sternocleido- 
mastoid, trapezius and rhomboid muscles. It is not in the field of a routine carotid 
endarterectomy and can only be injured when dealing with extremely distal internal 
carotid artery lesions. Paralysis of the spinal accessory nerve results in an inability 
to shrug the shoulder and rotate the head. 

Damage to sensory nerves deriving from the cervical plexus, such as the greater 
auricular and the transverse cervical nerves, is often thought to be trivial by the sur- 



Carotid Endarterectomy and Cranial Nerve Injuries 265 

geons performing the endarterectomy, but it may cause considerable annoyance to 
the patient. Dysfunction of these nerves has been observed in as many as 70-90 per 
cent of patients in some reports [1,3]. The greater auricular and the transverse cer- 
vical nerves emerge from behind the posterior border of the sternocleidomastoid 
muscle and course anteriorly over its surface at the upper and lower part of the inci- 
sion for the carotid endarterectomy, respectively. The greater auricular nerve sup- 
plies the skin of the scalp over the mastoid process and the external ear, while the 
transverse cervical nerve supplies the anterior cervical triangle. Injury to these 
nerves during the incision or due to excessive traction will result in numbness or 
painful paraesthesia in the distribution of these nerves. 

In the great majority of cases, cranial nerve injuries are caused by blunt trauma, 
due to excessive retraction. Less common causes are injuries by forcers, electro- 
cautery or the application of arterial clamps. Therefore, most of the cranial nerve 
injuries are transient and complete recovery occurs within 6 months and usually 
within 4-6 weeks [9, 11, 14, 18]. Thus, follow-up examination is recommended at 2 
months to verify recovery and, in cases of persistent paralysis, at 6 months. [Q4: C] 
Extended follow-up will identify the small subset of patients with delayed (>6 
months) complete nerve recovery. 

The only factor that has been associated with an increased risk of cranial nerve 
injury is the duration of operation, with every 30 minutes of operative time increas- 
ing the risk of nerve injury by 50%, according to a post hoc analysis of data from the 
ECST [19]. It seems that the longer a nerve is mechanically retracted, the greater the 
risk of a traction injury. 

Since bilateral injury to the recurrent laryngeal nerves or the hypoglossal nerves 
is a life-threatening complication, endarterectomy of the contralateral carotid artery 
should be postponed until cranial nerve dysfunction has resolved (usually after 2 
months), unless there are compelling reasons to do otherwise [15]. 

Endoluminal treatment of carotid stenosis is currently evaluated against carotid 
endarterectomy. One of the advantages of carotid angioplasty is that it is free of 
local complications at the neck, including haematomas and cranial nerve injuries. 
[Q5: A, B] According to the CAVATAS trial, cranial nerve palsy occurred in 9 per cent 
of the surgical patients but not in the endovascular group [22]. Such injuries should 
therefore be assessed carefully and quoted in all cases of carotid endarterectomy to 
provide a standard to which emerging catheter-based therapies for carotid stenosis 
should be compared. 



References 

1. Aldoori MJ, Baird RN. Local neurological complications during carotid endarterectomy J Cardiovasc 
Surg 1988;29:432-6. 

2. Ballotta E, Da Giau G, Renon L, Name S, Saladini M, Abbruzzese E, Meneghetti G. Cranial and cervi- 
cal nerve injuries after carotid endarterectomy: a prospective study. Surgery 1999;125:85-91. 

3. Dehn TC, Taylor GW. Cranial and cervical nerve damage associated with carotid endarterectomy. Br 
J Surg 1983;70:365-8. 

4. Evans WE, Mendelowitz DS, Liapis CD, Wolfe V, Florance CL. Motor speech deficit following carotid 
endarterectomy. Ann Surg 1982;196:461-4. 

5. Ferguson GG, Eliasziw M, Barr HW, Clagett GP, Barnes RW, Wallace MC, et al. The North American 
Symptomatic Carotid artery Endarterectomy Trial: surgical results in 1415 patients. Stroke 
1999;30:1751-8. 

6. Forsell C, Kitzing P, Bergqvist D. Cranial nerve injuries after carotid artery surgery. A prospective 
surgery study of 663 operations. Eur J Vase Endovasc Surg 1995;10:445-9. 



266 Vascular Surgery 

7. Hertzer NR, Feldman BJ, Beven EG, Tucker HM. A prospective study of the incidence of injury to the 
cranial nerves during carotid endarterectomy. Surg Gynecol Obstet 1980;151:781-4. 

8. Knight FW, Yeager RM, Morris DM. Cranial nerve injuries during carotid endarterectomy. Am J 
Surg 1987;154:529-32. 

9. Liapis CD, Satiani B, Florance CL, Evans WE. Motor speech malfunction following carotid 
endarterectomy. Surgery 1981;81:56-9. 

10. Maniglia AJ, Han DP. Cranial nerve injuries following carotid endarterectomy: an analysis of 336 
procedures. Head Neck 1991;13:121-4. 

11. Maroulis J, Karkanevatos A, Papakostas K, Gilling-Smith GL, McCormick MS, Harris PL. Cranial 
nerve dysfunction following carotid endarterectomy. Int Angiol 2000;19:237-41. 

12. Massey EW, Heyman A, Utley C, Haynes C, Fuchs J. Cranial nerve paralysis following carotid 
endarterectomy. Stroke 1984;15:157-9. 

13. Rogers W, Root HD. Cranial nerve injuries after carotid artery endarterectomy. South Med J 
1988;81:1006-9. 

14. Schauber MD, Fontenelle LJ, Solomon JW, Hanson TL. Cranial/cervical nerve dysfunction after 
carotid endarterectomy. J Vase Surg 1997;25:481-7. 

15. Schmidt D, Zuschneid W, Kaiser M. Cranial nerve injury during carotid arterial reconstruction. J 
Neurol 1983;230:131-5. 

16. Theodotou B, Mahaley MS, Jr. Injury of the peripheral cranial nerves during carotid endarterectomy. 
Stroke 1985;16:894-5. 

17. Weiss K, Kramar R, Firt P. Cranial and cervical nerve injuries: local complications of carotid artery 
surgery. J Cardiovasc Surg 1987;28:171-5. 

18. Zannetti S, Parente B, De Rango P, Giordano G, Serafini G, Rossette M, Cao P. Role of surgical tech- 
niques and operative findings in cranial and cervical nerve injuries during carotid endarterectomy. 
Eur J Vase Endovasc Surg 1998;15:528-31. 

19. Cunningham EJ, Bond R, Mayberg MR, Warlow CP, Rothwell PM. Risk of persistent cranial nerve 
injury after carotid endarterectomy. J Neurosurg 2004;101:445-8. 

20. Assadian A, Senekowitsch C, Pfaffelmeyer N, Assadian O, Ptakovsky H, Hagmuller GW. Incidence of 
cranial nerve injuries after carotid eversion endarterectomy with a transverse skin incision under 
regional anaesthesia. Eur J Vase Endovasc Surg 2004;28:421-4. 

21. Rosenbloom M, Friedman SG, Lamparello PJ, Riles TS, Imparato AM. Glossopharyngeal nerve injury 
complicating carotid endarterectomy. J Vase Surg 1987;5:469-71. 

22. Endovascular versus surgical treatment in patients with carotid stenosis in the Carotid and Vertebral 
Artery Transluminal Angioplasty Study (CAVATAS): a randomised trial. Lancet 2001;357:1729-37. 



30. Paragangliomas of the Head and Neck 

Johanna G. H. van Nes, Sylvia C. de Jong, Marc R. H. M. 
van Sambeek and Hero van Urk 



A 47-year-old man was referred for investigation and management of a swelling 
in the neck on the right side (Fig. 30.1). For several months there had been a 
slowly enlarging and painless palpable mass. Besides problems with swallowing, 
there were no other symptoms or complaints. The previous medical history was 
unremarkable and there were no similar lesions known in family members. 

On physical examination, the patient had a non-tender mass located just ante- 
rior to the sternocleidomastoid muscle in the anterior triangle of the neck. The 
mass was mobile in a back-forwards direction, but could not be moved in the 
cranial-caudal direction. No signs of cranial nerve deficits were detected. An 
echo-duplex showed a highly vascularised structure adherent with the carotid 
artery located in the bifurcation. 



Question 1 

Which of the following is the most likely diagnosis causing this patient's swelling? 

A. Enlarged lymph node. 

B. Goitre of the right thyroid lobe. 

C. Paraganglioma. 

D. Carotid artery aneurysm. 

E. Thyroid tumour. 

F. Bronchial cleft cyst. 

Based on the history, physical examination and echo-duplex, the diagnosis of a 
carotid body tumour was made. 



267 



268 



Vascular Surgery 













Fig. 30.1. Patient with a swelling in the neck, which was diagnosed as a carotid body tumour. 

Question 2 

Which further examination is preferable to confirm the diagnosis of para- 
ganglioma? Rank them in order and explain why. 

A. MRI/MR angiography. 

B. Somatostatin receptor scintigraphy (OctreoScan). 

C. Angiography. 

D. MIBG scanning. 

E. Duplex scanning. 

F. Needle biopsy. 

G. Incision biopsy. 

The diagnosis of carotid body tumour was confirmed by angiography (Fig. 30.2) 
and somatostatin receptor scintigraphy (Fig. 30.3). This showed a tumour of 4 x 2.5 
x 1.0 cm. Because of the size and difficulties with swallowing, it was decided to start 
treating the patient. 



Question 3 

What are the treatment options? 



Paragangliomas of the Head and Neck 



269 




Fig. 30.2. Angiography of a carotid body tumour showing the typical tulip configuration at the site of the 
bifurcation. 



A. Conservative treatment. 

B. Selective embolisation. 

C. Radiation therapy. 

D. Surgical excision. 

E. Chemotherapy. 



270 



Vascular Surgery 





Fig. 30.3. a An OctreoScan of a carotid body tumour showing elevated uptake, b An OctreoScan with multiple 
tumours. 



Paragangliomas of the Head and Neck 



271 




Fig. 30.4. The macroscopic anatomy of a carotid body tumour after surgical excision. 



A surgical excision of the carotid body tumour was performed (Fig. 30.4). There 
were no complications, the patient recovered quickly and was discharged after 2 
days. Pathological anatomical examination confirmed the diagnosis. 

Question 4 

What can be the complications due to surgical excision? 



A. Deafness. 

B. Horner's syndrome. 

C. Vocal cord paralysis. 



272 Vascular Surgery 

D. Paresis of the mandibular branch of the trigeminal nerve. 

E. Ipsilateral tongue paresis. 



Commentary 

Paragangliomas are usually benign tumours derived from the paraganglia, a collec- 
tion of neuroendocrine tissues which have a close relationship to the autonomic 
nervous system [1]. The normal paraganglia play an important role in homeostasis 
either by acting directly as chemical sensors or by secreting catecholamines in 
response to stress. The paragangliomas are generally divided into two groups, those 
occurring in the head and neck region and those occurring elsewhere, with the 
adrenal medulla being the most frequent site. In the head and neck, common loca- 
tions of paragangliomas are the carotid bifurcation, the jugular foramen, the vagal 
nerve and the middle ear. Tumours in the carotid bifurcation, the carotid body 
tumours, are the most common type (60 percent) and are usually referred to the 
vascular surgeon. Another name commonly used for carotid body tumours is 
chemodectoma because carotid body tumours are tumours of the chemoreceptor 
system. 

The carotid body is a highly vascularised, reddish-brown coloured, ellipsoid 
structure measuring 3 x 5 x 1.5 mm and located in the adventitia of the bifurcation 
of the common carotid artery [2]. The carotid body has a homeostatic role and 
functions as an oxygen sensor. It stimulates the cardiopulmonary system in hypoxia 
through afferent input by way of the glossopharyngeal nerve to the medullary retic- 
ular formation [3]. 

Carotid body tumours can develop spontaneously and be induced by chronic 
hypoxia. The latter includes living at high altitudes and certain medical conditions 
(patients with chronic obstructive pulmonary disease, cyanotic heart diseases) 
[4-6]. Familial cases are reported in the literature in the percentage range 10-50 
percent [7] but the range is probably 10-15 percent. They are frequently bilateral or 
multifocal and have an earlier age of detection. The inheritance pattern of paragan- 
glioma is autosomal dominant modified by genomic imprinting. To date, four 
genetic loci have been identified [8-10]. The overwhelming majority of the tumours 
are benign but local expansion can cause cranial nerve deficits, invasion of the skull 
base, and eventually compression of the brain stem. Malignancy is mostly cited as 5 
percent of cases [11], but in practice it is rarely seen. Malignancy cannot be defined 
on histology of the tumour itself but the unique criterion of malignancy is the pres- 
ence of metastases [12], mostly in regional lymph nodes. 

Clinical Presentation 

Carotid body tumours have to be distinguished from enlarged lymph nodes, 
branchial cleft cysts, parotid tumours, thyroid tumours, neurogenic tumours and 
carotid artery aneurysms. A detailed history and physical examination will elimi- 
nate most other possibilities. Carotid body tumours can occur at any age, but they 
are typically diagnosed between the third and sixth decades of life [13]. The pres- 
ence of a slowly growing, asymptomatic palpable mass in the anterior triangle of the 
neck must raise suspicion for this diagnosis (Fig. 30.1). In addition to the painless 



Paragangliomas of the Head and Neck 273 

mass, the patient may present with pain, hoarseness, dysphagia, Horner's syn- 
drome, tongue paresis and vertigo. The tumours are generally non-functional; cate- 
cholamine secretion is present in only 5 percent of patients [11] and can cause 
hypertension. On physical examination, a carotid body tumour is noted to be pul- 
satile and can be moved laterally, but not vertically because of adherence to the 
carotid artery. A bruit may be auscultated over the mass, but this is a rare condition. 
The cranial nerves should also be evaluated. For instance, the tongue should be 
examined since paresis of the hypoglossal nerve can cause tongue dysfunction. 
Paresis of the vagal nerve can cause hoarseness as a result of vagal nerve paresis or 
paralysis. [Q1:C] 

An echo-duplex is usually performed first, to differentiate between a carotid body 
tumour and other items on the differential diagnosis list. For further investigation, 
arteriography has long been regarded as the gold standard for diagnosis, but this is 
no longer the case. Magnetic resonance imaging (MRI) (Fig. 30.5) can reveal a cir- 
cumscript mass at or above the carotid bifurcation. Use of contrast material usually 
shows a "salt and pepper" appearance caused by vessels with signal-void within the 
tumour stroma. Both MR angiography and digital subtraction angiography reveal 
marked vascularity of the tumour, and this may help to differentiate these hyper- 
vascular tumours from other tumours. If the tumour produces catecholamines, an 
MIBG (meta-iodobenzylguanidine) scan can be used or plasma levels can be 
checked. 

An MIBG scan is a nuclear scan that uses an injected radioisotope to localise cat- 
echolamines; they are mostly used to locate or confirm a phaeochromocytoma. The 
patient receives an injection into a vein and imaging will take place between 1 and 4 
days later. Somatostatin receptor scintigraphy has a much higher sensitivity for 
paraganglioma than an MIBG scan. Somatostatin receptor scintigraphy can be used 
to detect multiple paragangliomas because paragangliomas contain somatostatin- 
receptor carrying tissue (Fig. 30.3). If a carotid body tumour is suspected, a fine 
needle aspiration should not be performed and certainly an incision biopsy should 
be avoided in all cases. The diagnosis of a carotid body tumour is difficult to make 
on fine needle aspiration and this procedure can give rise to unnecessary complica- 
tions such as massive bleeding. [Q2: E, A, B, C, D] It is of interest to note that more 
patients have died from the complications of invasive diagnostics and surgical treat- 
ment than from the natural cause of the disease. So the answer to Question 2 should 
be E, A, B. Answers F and G are obviously wrong and answers C and D are subjects 
of discussion. 

Treatment 

The preferred treatment for carotid body tumours is either conservative or surgical. 
Excision is the preferred definitive treatment, although the postoperative morbidity 
rate as quoted in the literature is rather high. Morbidity includes cranial nerve dys- 
function, mostly of nerves X and XII. Tumour size is important; larger tumours 
have a higher incidence of complications [7]. [Q4: A, B,C, D, E] All the answers given 
in Question 4 are correct. 

Postoperative mortality should not exceed 2-5 percent and occurs only in large 
tumours, while mortality is negligible in small tumours. Damage to the wall of the 
carotid artery, especially in the bifurcation, which is difficult to repair because the 
vessel wall is very thin as a result of dissection in the subadventitial space, may force 



274 



Vascular Surgery 





Fig. 30.5. a, b An MRI of a carotid body tumour. 



Paragangliomas of the Head and Neck 275 

the surgeon to clamp the internal carotid artery, sometimes leading to ischaemic 
stroke and death. 

In 1972, Shamblin proposed a surgical classification for carotid body tumours 
based on their tendency to encase the carotid arteries. Shamblin group I are small 
tumours with minimal attachments to the carotid vessels; surgical excision can be 
performed without difficulty. Shamblin group II tumours are larger, and Shamblin 
group III are large tumours that encase the carotid arteries, which makes them 
more difficult to resect. Sometimes, it is even necessary to sacrifice the carotid bifur- 
cation to be replaced by a venous or synthetic interposition graft in order to recon- 
struct the carotid artery [14]. 

Embolisation of the feeding branches of the external carotid artery can be per- 
formed a few days prior to surgery with the intention to decrease blood loss during 
operation. However, this is an area of continuing controversy. Some groups claim 
that embolisation decreases blood loss [15-17]; other groups have not found the 
embolisation procedure helpful and they warn of the increased risk of stroke caused 
by emboli to the brain through collateral pathways [18, 19]. Embolisation has been 
used in the past in very high-risk patients who probably would not tolerate surgical 
excision. It now seems to be largely abandoned because tumour growth appears to 
be stimulated in the non-necrotic hypoxic areas of the tumour. Radiation therapy 
and chemotherapy are seldom used as a treatment for carotid body tumours. 
Radiotherapy might be an alternative to surgery. It may be the treatment option for 
large, fast growing tumours, which are not eligible for surgery, and it may be effec- 
tive in arresting growth. However, radiotherapy does not result in complete eradica- 
tion. There is no evidence and there are no prospective trials showing that 
chemotherapy might be effective against carotid body tumours. [Q3: A, D] 

Summary 

Paragangliomas are slowly growing benign tumours. In the head and neck region, 
the carotid body tumour is the most common type. The diagnosis is suspected from 
the patient's history and physical examination. A somatostatin receptor scintigra- 
phy is a reliable method for confirming the diagnosis and detecting multiple 
tumours at the same time. If the carotid body tumour is small and there is no docu- 
mented growth, a wait-and-see policy is justified. A fast growing or large tumour 
should be treated surgically, cranial nerve dysfunction being the most common 
postoperative complication. 



References 

1. Lack EE. Pathology of adrenal and extra-adrenal paraganglia. Philadelphia: WB Saunders, 1994. 

2. Netterville JL, Reilly KM, Robertson D, Reiber ME, Armstrong WB, Childs P. Carotid body tumors: a 
review of 30 patients with 46 tumors. Laryngoscope 1995;105(2):1 15-26. 

3. Pryse-Davies J, Dawsom IMP, Westbury G. Some morphologic, histochemical, and chemical obser- 
vations on chemodectomas and the normal carotid body, including a study of the chromaffin reac- 
tion and possible ganglion cell elements. Cancer 1964;17:185-202. 

4. Edwards C, Heath D, Harris P, Castillo Y, Kruger H, Arias-Stella J. The carotid body in animals at 
high altitude. J Pathol 1971;104(4):231-8. 

5. Lack EE, Perez- Atayde AR, Young JB. Carotid body hyperplasia in cystic fibrosis and cyanotic heart 
disease. A combined morphometric, ultrastructural, and biochemical study. Am J Pathol 
1985;119(2):301-14. 



276 Vascular Surgery 

6. Roncoroni AJ, Montiel GC, Semeniuk GB. Bilateral carotid body paraganglioma and central alveolar 
hypoventilation. Respiration 1993;60(4):243-6. 

7. McCaffrey TV, Meyer FB, Michels VV, Piepgras DG, Marion MS. Familial paragangliomas of the 
head and neck. Arch Otolaryngol Head Neck Surg 1994;120(ll):1211-6. 

8. Astuti D, Latif F, Dallol A, Dahia PL, Douglas F, George E, et al. Gene mutations in the succinate 
dehydrogenase subunit SDHB cause susceptibility to familial pheochromocytoma and to familial 
paraganglioma. Am J Hum Genet 2001;69(l):49-54. 

9. Baysal BE, Ferrell RE, Willett-Brozick JE, Lawrence EC, Myssiorek D, Bosch A, et al. Mutations in 
SDHD, a mitochondrial complex II gene, in hereditary paraganglioma. Science 2000;287(5454):848-51. 

10. Niemann S, Muller U. Mutations in SDHC cause autosomal dominant paraganglioma, type 3. Nat 
Genet 2000;26(3):268-70. 

11. Manolidis S, Shohet JA, Jackson CG, Glasscock ME, III. Malignant glomus tumors. Laryngoscope 
1999;109(l):30-4. 

12. Lee JH, Barich F, Karnell LH, Robinson RA, Zhen WK, Gantz BJ, Hoffman HT. National cancer data 
base report on malignant paragangliomas of the head and neck. Cancer 2002;94(3):730-7. 

13. Ward PH, Jenkins HA, Hanafee WN. Diagnosis and treatment of carotid body tumors. Ann Otol 
Rhinol Laryngol 1978;87(5 Pt 1):614-21. 

14. Shamblin WR, ReMine WH, Sheps SG, Harrison EG, Jr. Carotid body tumor (chemodectoma). clini- 
copathologic analysis of ninety cases. Am J Surg 1971;122(6):732-9. 

15. LaMuraglia GM, Fabian RL, Brewster DC, Pile-Spellman J, Darling RC, Cambria RP, Abbott WM. The 
current surgical management of carotid body paragangliomas. J Vase Surg 1992; 15 (6): 1038-44. 

16. Muhm M, Polterauer P, Gstottner W, Temmel A, Richling B, Undt G, et al. Diagnostic and thera- 
peutic approaches to carotid body tumors. Review of 24 patients. Arch Surg 1997;132(3):279-84. 

17. Wang SJ, Wang MB, Barauskas TM, Calcaterra TC. Surgical management of carotid body tumors. 
Otolaryngol Head Neck Surg 2000;123(3):202-6. 

18. Leonetti JP, Donzelli J J, Littooy FN, Farrell BP. Perioperative strategies in the management of carotid 
body tumors. Otolaryngol Head Neck Surg 1997;1 17(1):1 1 1-5. 

19. Litle VR, Reilly LM, Ramos TK. Preoperative embolization of carotid body tumors: when is it appro- 
priate? Ann Vase Surg 1996;10(5):464-8. 



31 . Vertebrobasilar Ischemia: Embolic and 
Low-flow Mechanisms 

Ramon Berguer 



Vertebrobasilar Ischemia: Embolic Mechanism 

A 51 -year-old male experienced over a period of 6 months a major stroke and 
several transient ischemic attacks (TIAs) of vertebrobasilar distribution. The 
original episode consisted of loss of balance, loss of coordination, and loss of the 
left visual field while driving a bus, which resulted in a road accident. Since then, 
he had experienced four additional episodes of aphasia and paraparesis lasting 
for 4-5 h. A diagnosis of vertebral artery dissection was made at the local hospi- 
tal and he was placed on Coumadin. Concomitant diagnoses were hypertension, 
non-insulin-dependent diabetes, and hypercholesterolemia. In spite of adequate 
international normalized ratio (INR) levels, his symptoms continued and he was 
referred to us. 

On admission, magnetic resonance imaging (MRI) showed right occipital and 
left cerebellar infarctions (Fig. 31.1). 



Question 1 

The work-up of this patient presenting with symptoms of vertebrobasilar ischemia 
and MR evidence of infarction in the posterior region must include: 

A. CT scan of the brain. 

B. Carotid-vertebral duplex. 

C. Electroencephalogram (EEG). 

D. Arteriogram. 

E. Echocardiogram. 



277 



278 



Vascular Surgery 




Fig. 31.1. MRI showing cerebellar and brainstem infarctions. 



Question 2 

The etiology of infarction in the posterior circulation territory is: 

A. Distal embolization of atheromatous material from vertebral or basilar artery 
lesions. 

B. Arrhythmia. 

C. Bilateral carotid disease in patients with absent vertebral arteries. 

D. Traumatic or spontaneous dissection of the vertebral artery. 

E. Transient drop in central aortic pressure in a patient with severe bilateral 
stenoses of both vertebral arteries. 



An arteriogram showed a 60 percent stenosis in the fourth portion of the right 
vertebral artery, and a tenuous, incomplete (dissected) left vertebral artery, which, 
at the level of CI, became a normal artery and, higher up, joined with the opposite 
vertebral artery (Fig. 31.2). A diagnosis of embolizing dissection of the left vertebral 
artery was made. Because the dissection was not responsive to medical therapy, the 
patient underwent a bypass from the left internal carotid to the left (suboccipital) 
vertebral artery using a saphenous vein [1]. The proximal vertebral site of the 
embolizing dissection was ligated above CI, immediately below the distal anasto- 
mosis of the carotid-vertebral bypass (Fig. 31.3). The patient did well from this 
operation and stopped having symptoms. His anticoagulation was discontinued. He 
remains asymptomatic after 5 years of follow-up. 



Vertebrobasilar Ischemia: Embolic and Low-flow Mechanisms 



279 




Fig. 31.2. Arteriogram: dissection of the left vertebral artery, which is occluded from its origin to C4 (lower 
arrow), dissected and partially occluded from C4 to C1 (between arrows), and normal distal to C1 . 

Reprinted from J Vase Surg, vol. 30, Berguer R, Suboccipital approach to the distal vertebral artery, pages 344-9, 
© 1999, with permission from The Society for Vascular Surgery. 

Question 3 



Once the objective diagnosis of vertebral artery dissection is made in a patient with 
vertebrobasilar symptoms the next step is: 

A. Anticoagulation with heparin, then Coumadin. 



280 



Vascular Surgery 




Fig. 31 .3. Postoperative carotid arteriogram showing a saphenous vein bypass from the distal cervical internal 
carotid to the vertebral artery beyond C1 . 

Reprinted from J Vase Surg, vol. 30, Berguer R, Suboccipital approach to the distal vertebral artery, pages 344-9, 
© 1999, with permission from The Society for Vascular Surgery. 



B. Stenting of the dissection followed by antiplatelet therapy. 

C. Surgical bypass of the dissected segment with ligation of the proximal vertebral 
artery. 



Vertebrobasilar Ischemia: Embolic and Low-flow Mechanisms 281 

Commentary 

Dissection of the vertebral artery may occur spontaneously or result from trauma 
[2-6]. The traumatic event is usually an exaggerated extension or rotation of the 
neck as may occur during sports and deceleration injuries. 

Clinical presentation of dissection of the vertebral artery starts with pain over the 
posterolateral aspect of the neck irradiating to the nuchal area. There may be an 
interval of several days between the initial pain, announcing the dissection, and the 
development of clinical symptoms. The latter are ischemic manifestations of the 
dissection and appear in 60-90 percent of patients after an interval of several days, 
usually 1-2 weeks. A carotid-vertebral duplex would not provide a discriminating 
datum to help in the decision on the management of our patient because: (i) it 
would be only confirmatory for a possible concomitant carotid atheroma, which has 
never been shown to be the source of infarction in the cerebellum or brain stem; (ii) 
infarction in the posterior region can only be evaluated by means of an arteriogram. 
The latter will provide in addition the information about the carotid arteries that 
you would have derived from the carotid-vertebral duplex. [Q1 : D, E] [Q2: A, D] 

The treatment of symptomatic vertebral artery dissection is empirical with sys- 
temic anticoagulation. Patients with posterior fossa symptoms should undergo MRI 
before starting anticoagulation to rule out a subarachnoid hemorrhage. The latter 
may occur following dissection and rupture of the fourth (intracranial) segment of 
the vertebral artery. 

Anticoagulation is empirically used for the treatment of symptomatic dissection 
because the ischemia that follows is the consequence of embolization from the 
double channel, not a low-flow effect. The fear of distal extension of the dissection 
with anticoagulants has prompted some leading experts to give antiplatelet therapy 
to patients with local symptoms (pain) and evidence of dissection but without 
central manifestations of ischemia (central nervous system deficits or MR evidence 
of infarction). Patients with massive infarction are not anticoagulated to avoid 
intraparenchymal bleeding. There is no indication for wire-catheter-stent manipu- 
lation of a dissected vertebral. In patients who are anticoagulated appropriately and 
continue to have intermittent symptoms, the dissected vertebral artery is consid- 
ered to be the source of emboli. [Q3: A] In these circumstances, and if technically fea- 
sible, the dissected segment is excluded and bypassed [7, 8]. 



Vertebrobasilar Ischemia: Low-flow Mechanism 

A 62-year-old woman with a healthy lifestyle presented with a history of 
dimming of the visual field and passing out when she turned her head to the 
extreme right. Three months before, she had been evaluated elsewhere with a 
history suggestive of amaurosis fugax and bouts of imbalance and vertigo when 
she turned her head to the right. A carotid endarterectomy had been performed 
at another institution. 

She continued to have severe vertebrobasilar symptoms with head turning. She 
had a myocardial revascularization 20 years ago, at which point she stopped 
smoking. 

On examination, the patient appeared healthy, with normal and equal 
(124/80 mm Hg) blood pressure in both brachial arteries. Neurological examina- 



282 Vascular Surgery 

tion under resting conditions was normal. Her neck was silent. When her head 
was turned to the right, the patient developed dimming of vision, loss of balance, 
and a sensation of passing out. The arteriogram available from the previous 
operation carried out elsewhere showed a clearly dominant large left vertebral 
artery, but we could not see clearly the distal segment of the vessel. The right ver- 
tebral artery was small and diseased severely to a preocclusive level throughout 
its second segment. There was no evidence of posterior communicating arteries. 

Because the symptoms were repetitive and induced posturally, the patient was 
scheduled for a dynamic arteriogram. First, we obtained a view with a selective 
subclavian injection of the dominant left vertebral in the neutral position, which 
was normal. Following this, the patient's head was turned to the right; when she 
became symptomatic, the contrast injection was repeated (Fig. 31.4). This 
revealed a severe compression of the vertebral artery at the level of the pars 
atlantica of CI. 

The patient underwent exploration of the suboccipital space with dissection 
and exposure of the vertebral artery where it crossed the lamina of CI. The com- 
pression mechanism was between the sharp upper edge of the lamina and the 
occipital bone. A laminectomy was carried out to provide space for the artery 
to pass from the exit of the transverse foramen of CI to the foramen magnum 
without bony compression (Fig. 31.5). The artery was examined by palpation 
and direct duplex interrogation; we could find no element of plaque or steno- 
sis in the lumen once the artery was freed and the laminectomy completed. 
The patient became asymptomatic. Full-range motion of the neck no longer 
caused syncope or vertigo. 



Question 1 

Which of the following statements regarding posturally induced symptoms is true? 

A. The mechanism for ischemia is the restriction of flow by external compression 
of the artery. 

B. The mechanism for ischemia is embolization from the damaged wall (dissection) 
or thrombus overlying the endothelial lining of the artery at the site of trauma. 

C. Both mechanisms may exist. 

Question 2 

Which of the following statements are correct? 

A. When dynamic symptomatic compression of the vertebral artery is demon- 
strated, angioplasty (with or without stent) is never indicated. 

B. Angioplasty of a stenosed or dissected vertebral artery at the suboccipital 
level is likely to result in rupture of the artery or formation of an arteriove- 
nous fistula. 

C. Angioplasty and stenting of the distal vertebral artery is successful in stenosing 
lesions caused by external compression. 



Vertebrobasilar Ischemia: Embolic and Low-flow Mechanisms 



283 




Fig. 31.4. Selective injection of a left subclavian artery while the patient is experiencing symptoms with her 
head turned to the right. The single, dominant vertebral artery is severely compressed above C1 in its pars 
atlantica. 



Commentary 

In patients with low-flow ischemia secondary to extrinsic compression of the artery, 
the clinical picture is repetitive and can be induced by manipulating the patient's 
head in the trigger position. 

Patients with symptoms occurring with head rotation or extension should have a 
dynamic arteriogram to show the anatomic lesion (extrinsic compression) at the 



284 



Vascular Surgery 




Fig. 31.5. Three-dimensional reconstruction of a CT scan of the craniocervical junction. The lamina of C1 has 
been removed. 



same time as the patient experiences symptoms. Patients with low-flow symptoms 
(repetitive) and no evidence of embolization (negative MRI) may show 
deformity/compression of one vertebral artery but a normal contralateral vertebral 
artery during head rotation or extension. If the contralateral, undisturbed artery is 
of normal size and empties normally into the basilar artery, then the role of the 
compression of one vertebral artery causing the symptoms must be doubted. 

The suboccipital approach permits access to the vertebral artery from the trans- 
verse process of C2 to the foramen magnum. The techniques used to relieve com- 
pression at the suboccipital level are laminectomy, laminectomy plus bypass, and 
bypass alone. 

Vertebrobasilar ischemia of postural origin is generally the consequence of 
mechanical compression of the vertebral artery by osteophytes (and occasionally 
ligaments) in its extracranial trajectory. The mechanism for symptoms is generally 
low flow in a dominant vertebral artery that cannot be compensated for by flow 
from a contralateral hypoplastic or absent vertebral artery. This compression is 
seen very rarely in the first segment (origin-C6) caused by compression by the 
tendon of the longus colli. It is usually observed in the second and third segments of 
the artery. In the second segment (C6-C2), the artery is usually compressed by 
osteophytes, and the symptoms generally appear with rotation of the neck. In the 
third segment (C2-C0), the compression may occur at CI or, more commonly, in 
the pars atlantica of the artery between CI and the foramen magnum. The artery is 



Vertebrobasilar Ischemia: Embolic and Low-flow Mechanisms 285 

compressed between the sharp upper edge of the lamina of CI and the occipital 
ridge. [Q1:C] 

The ischemic symptoms are usually the consequences of low flow through a dom- 
inant vertebral artery because of complete or near-complete occlusion at the latter 
by an osteophyte. Less frequently, the ischemic effects may be embolic from the 
mural thrombi that develop at the site of repetitive trauma on the artery by the 
offending osteophyte. In other cases, the artery may dissect at the point of repetitive 
traumatic compression, which results in its occlusion and/or distal embolization. 

Symptoms in patients with vertebrobasilar ischemia from the low-flow mechanism 
are repetitive and can be reproduced every time the neck is brought to the trigger posi- 
tion. Patients with vertebrobasilar ischemia of embolic origin usually present with a 
clinical stroke or TIA in different areas. MRI in the low- flow group is usually normal, 
but in the embolic group it will show cerebellar, brainstem or occipital infarctions. 

An arteriogram is needed to outline precisely the point of compression and to 
discern the possibility of a dissection and/or tandem lesions. It is also important to 
outline the entire course of the opposite vertebral artery to establish whether it is 
complete, normal or hypoplastic, and whether at the time of the provocative 
dynamic arteriogram the opposite vertebral artery fills the basilar artery normally 
while the patient has symptoms. The latter suggests that the mechanism of symp- 
toms is not low flow. 

There is no role for angioplasty, with or without stent, in the treatment of extrinsic 
compression of the vertebral artery. Balloon dilation of the thin-walled vertebral artery 
against the hard bony prominence of an osteophyte is likely to result in the rupture of 
the arterial wall and the formation of a false aneurysm or an arteriovenous fistula. 

If the compression of the vertebral artery is limited to the V2 segment (C6-C2), 
then the single or multiple elements of compression are bypassed by reconstructing 
the artery to the level of CI. This is done through an anterior approach [8]. In 
dynamic compression at the suboccipital level, the approach is posterior [1] and the 
treatment consists of a laminectomy, a bypass or both. In the case of a bypass, the 
inflow is obtained from the high cervical carotid. The latter is exposed by moving 
aside the cranial nerves that block access to the internal carotid when approached 
posteriorly. [Q2: A, B] 



References 

1. Berguer R. Suboccipital approach to the distal vertebral artery. J Vase Surg 1999;30:344-9. 

2. Mas JL, Bousse M-G, Harbourn D, Laplanc D. Extracranial vertebral artery dissection: a review of 13 
cases. Stroke 1987;18:1037-47. 

3. Mokri B, Houser OW, Sandok BA, Peipgzas DG. Spontaneous dissection of the vertebral arteries. 
Neurology 1988;38:880-5. 

4. Chiras J, Marciano S, Vega Molina J, Touboul J, Poirier B, Bories J. Spontaneous dissecting aneurysm 
of the extracranial vertebral artery (20 cases). Neuroradiology 1985;27:327-33. 

5. Ringel SP, Harrison SH, Noremberg MD, Austin JH. Fibromuscular dysplasia: multiple "spontaneous" 
dissecting aneurysms of the major cervical arteries. Ann Neurol 1977;1:301-4. 

6. Noelle B, Clavier I, Berson G, Hommel M. Cervicocephalic arterial dissections related to skiing. Stroke 
1994;24:526-7. 

7. Caplan L. Posterior circulation disease. Cambridge, MA: Blackwell Science, 1996;257. 

8. Berguer R, Morasch MD, Kline RA. A review of 100 consecutive reconstructions of the distal vertebral 
artery for embolic and hemodynamic symptoms. J Vase Surg 1998;27:852-9. 



32. Neurogenic Thoracic Outlet Syndrome 

Richard }. Sanders 



A 30-year-old woman presented with complaints of pain in her neck, right shoul- 
der, right trapezius, right anterior chest wall, right arm, elbow, and forearm; 
occipital headaches every other day; numbness and tingling in all fingers of the 
right hand, worse in the fourth and fifth fingers; aggravation of her symptoms 
when elevating her arms, especially to comb or blow dry her hair or drive a car; 
weakness of her right hand and dropping coffee cups; and coldness and color 
changes in her right hand. The symptoms had been present for one year and 
began following a rear-end collision. 

Her history began one year ago when her automobile was sitting still at a traffic 
light and another vehicle hit her from the rear. She wore a seat belt and recalled 
going forward and backward, but did not recall what happened to her neck at the 
time of the accident. She had no immediate symptoms. On the next day she 
awoke with a sore neck and pain above her shoulder blades. A few days later, she 
began noticing headaches in the back of her head that radiated forward to behind 
her eyes, and the neck soreness became progressively painful. Two or three 
weeks later, pain developed in the right shoulder area and down the right arm. 
Several weeks later, numbness and tingling developed in the fingers of the right 
hand, more noticeable in the ring and baby fingers. Because of severe, persistent 
right shoulder pain, arthroscopic repair of the right shoulder had been per- 
formed 6 months ago with partial improvement of her shoulder pain, but no 
change in any of her other symptoms. 

Her occupation was a legal secretary. Since the accident, although she had been 
able to return to work, she was now able to work only 4 hours a day. She could 
not type for more than 10 minutes because the pain and numbness in her right 
hand was too uncomfortable. At home she could do light housework only. She 
could not vacuum, wash windows or floors, or lift heavy laundry baskets. 
Diagnostic studies to date included cervical spine X-rays, which were normal, 
and an electromyography/nerve conduction velocity (EMG/NCV) study, which 
revealed very mild nonspecific changes in the ulnar nerve distribution, but was 
close to normal. 

Treatment to date included 6 months of physical therapy with the following 
modalities: heat, massage, ultrasound, neck stretching exercises, and posture 



289 



290 Vascular Surgery 

correction. She was continuing neck stretching exercises at home on a daily basis 
emphasizing doing each stretch slowly, holding each stretch for a minimum of 
15 seconds, and performing no more than three repeats at each session. In spite 
of this treatment, there was no improvement in her symptoms. 



Question 1 

What is the most common cause of neurogenic thoracic outlet syndrome (TOS)? 

A. Neck trauma. 

B. Cervical rib. 

C. Anomalous bands. 

D. Abnormal first rib. 

E. All of the above. 

On physical examination there was supraclavicular tenderness over the right 
scalene muscles but no tenderness over the left scalenes; a positive TinePs sign over 
the right brachial plexus and a negative sign over the left; and reproduction of arm 
and hand symptoms with pressure over the right scalene muscles, but no such 
symptoms with pressure over the left scalene muscles. Head rotation and head 
tilting each caused pain in the contralateral hand and arm when turning and tilting 
to the left side. This did not occur when rotating and tilting to the left side. 
Abducting the arms to 90° in external rotation (AER position) reproduced the right 
arm and hand symptoms within 15 seconds while no symptoms developed on the 
left side. 

Scalene muscle block, injecting 4 ml of 1% lidocaine into the right anterior 
scalene muscle area, resulted in significant improvement in most of her physical 
findings. 

Question 2 

The diagnostic criteria for neurogenic TOS include which of the following? 

A. History of neck trauma. 

B. Paresthesia in the hand involving all five fingers, more frequently in the fourth 
and fifth. 

C. Pain in the neck, shoulder, and upper extremity. 

D. Occipital headaches. 

E. Scalene muscle tenderness and duplication of symptoms in the 90° AER 
position. 

F. Cut-off of the radial pulse on Adson's or 90° AER positioning. 

G. Positive response to the scalene muscle block. 



Neurogenic Thoracic Outlet Syndrome 291 

Question 3 

Which of the following conditions can coexist with TOS or require differentiation 
from it? 

A. Carpal tunnel syndrome. 

B. Biceps/rotator cuff tendinitis or impingement syndrome. 

C. Cervical spine disease-disc, arthritis, spinal stenosis, cervical spine strain, 
etc. 

D. Ulnar nerve entrapment at the elbow (cubital tunnel syndrome). 

E. Fibromyalgia. 

F. Brachial plexus injury. 

G. Brain tumor. 

Question 4 

The indications for surgical decompression of the thoracic outlet areas are: 

A. Failure of conservative treatment after a trial of several months. 

B. All other associated conditions have been recognized and treated as completely 
as possible. 

C. Symptoms are interfering with work, sleep, recreation, or activities of daily 
living. 

D. All of the above. 

Because of persistent symptoms in spite of adequate conservative therapy, and 
because she was partially disabled at work and at home, a supraclavicular anterior 
and middle scalenectomy, brachial plexus neurolysis, and first rib resection were 
performed. 

Question 5 

Which surgical procedures are acceptable to decompress the thoracic outlet 
area? 

A. Transaxillary first rib resection. 

B. Supraclavicular anterior and middle scalenectomy with brachial plexus neurolysis. 

C. Supraclavicular anterior scalenectomy with or without brachial plexus neurolysis. 

D. Supraclavicular anterior and middle scalenectomy, first rib resection, and 
brachial plexus neurolysis. 

E. All of the above. 



292 Vascular Surgery 

Question 6 

What are the major complications of TOS surgery? 

A. Brachial plexus traction injury. 

B. Phrenic nerve injury. 

C. Subclavian artery injury. 

D. Subclavian vein injury. 

E. Long thoracic nerve injury. 

F. Second intercostal brachial cutaneous nerve injury (transaxillary approach 
only). 

G. Thoracic duct injury (left side, supraclavicular approach only). 
H. Supraclavicular nerve injury (supraclavicular approach only). 
I. Horner's syndrome (supraclavicular approach only). 

J. All of the above. 

She tolerated surgery well, had no postoperative complications, and was dis- 
charged from the hospital on the second postoperative day. After 4 weeks of conva- 
lescence at home, she returned to work, 4 hours a day. After one month she was able 
to resume her job on a full-time basis. While most of her symptoms had improved, 
she still noticed occasional paresthesia in her hand and pain in her right shoulder 
when working for long periods. Her headaches were completely gone. She was 
pleased with her improvement from surgery even though she was not back to 
normal. 



Question 7 

What are the long-term results of surgical decompression of the thoracic outlet 
area? 

A. 90% success. 

B. 75% success. 

C. 60% success. 

D. 40% success. 

E. None of the above. 



Commentary 

There are three types of thoracic outlet syndrome (TOS): arterial, venous, and neu- 
rogenic. Neurogenic TOS comprises more than 95 percent of all TOS cases and is 



Neurogenic Thoracic Outlet Syndrome 293 

the most difficult to diagnose and treat. The etiology of neurogenic TOS in most 
patients is either a hyperextension neck injury or repetitive stress at work. The 
mechanism of neck injury from repetitive stress is a little obscure, but it probably 
comes from the worker's hands being occupied in one place so that the worker is 
constantly rotating his/her neck back and forth to perform the job or talk to people. 
Holding a telephone between ear and shoulder while typing is also a common form 
of neck strain. While some TOS patients have cervical ribs or congenital cervical 
bands, these are regarded as predisposing conditions and seldom are the primary 
cause. These patients usually do not develop symptoms until they experience some 
form of neck trauma. 

Although first rib resection has become a standard form of therapy for neuro- 
genic TOS, the first rib is rarely the cause of the symptoms. The pathology is tight- 
ness and scarring of the scalene muscles [1]. Rib resection is successful because the 
anterior and middle scalene muscles must be divided in order to remove the rib. 
Thus, by necessity, first rib resection includes scalenotomy and it is probably the 
latter that relieves the symptoms. [Q1: A] 

The diagnosis of neurogenic TOS is by history and physical examination. This is 
not a diagnosis of exclusion. The typical history includes some type of neck trauma, 
although the patient does not always remember the incident, especially if there was 
no litigation involved. It is the job of the examiner to thoroughly ask about neck 
trauma. The symptoms usually include pain, paresthesia, and weakness in the upper 
extremity, but over 75 percent of patients also complain of neck pain and occipital 
headaches. The latter symptoms are not the result of brachial plexus compression; 
rather, they result from stretch injuries to the scalene muscles and referred pain to 
the back of the head. Most commonly paresthesia involves all five fingers of the 
hand, although it tends to involve the ulnar side of the hand and forearm more 
often than the radial side. The significant physical findings are scalene muscle ten- 
derness, a positive TinePs and positive Spurling's sign over the scalene muscles, and 
duplication of symptoms with the arms in the 90° AER position. A cut-off of the 
radial pulse in either the Adson's or 90° AER position is not a reliable sign in estab- 
lishing a diagnosis. Up to 60% of normal people cut off their pulses in these 
dynamic positions while most TOS patients do not cut off their pulses [2]. Not every 
patient will exhibit all of these criteria, but a diagnosis can be established if the 
majority of these criteria have been met [3] [Q2: A, B, C, D, E, G] 

All symptoms of TOS are nonspecific. Other conditions that also exhibit similar 
symptoms include abnormalities of the shoulder, elbow, wrist, and parascapular 
muscles. It is quite common for TOS to coexist with some of these other conditions. 
[Q3:A,B,C,D,E,F] 

In less than 1 percent of patients with neurogenic TOS, atrophy of hand muscles 
supplied by the ulnar nerve exists. In these patients, EMG studies demonstrate 
typical findings of ulnar neuropathy [4]. Otherwise, EMG and NCV studies are 
either normal or reveal nonspecific changes. Unfortunately, once atrophy develops, 
it is usually nonreversible [5]. At this stage, surgery can relieve pain and paresthe- 
sia, but not weakness. 

Conservative therapy is always indicated first and is effective in the majority of 
patients [6]. Surgery should be regarded as a last resort. There are a variety of 
modalities of therapy for TOS patients, the most important being home exercises, 
including neck stretching, abdominal breathing, and posture correction. After being 
instructed by a physical therapist the patient carries out the program on a daily 
basis at home. Hands-on therapy by a physical therapist is indicated for some of the 



294 Vascular Surgery 

associated diagnoses that coexist with TOS. Because neck traction, weights, resis- 
tance exercises, and strengthening exercises tend to make TOS symptoms worse, we 
do not recommend them for TOS patients. 

Some patients are refractory to all forms of physical therapy. If there is no 
improvement after several months of exercises, the patient's options are to either 
live with the symptoms or consider surgical decompression of the thoracic outlet. 
To be a candidate for surgery, in addition to failing conservative therapy after a trial 
of several months, the patient should have had all associated diagnoses treated and 
the symptoms should be partially or totally disabling. [Q4: D] 

That there is more than one acceptable surgical procedure from which to choose 
indicates that no one operation has proved itself to be greatly superior to any other. 
In 1972, after performing transaxillary first rib resection [7] for several years, we 
were disappointed to find the long-term success rate was under 70%. We then 
changed to supraclavicular anterior and middle scalenectomy with brachial plexus 
neurolysis but were again disappointed to discover the success rate was identical to 
transaxillary first rib resection. The next choice of procedure was supraclavicular 
anterior and middle scalenectomy plus first rib resection through the same supra- 
clavicular incision [8, 9]. With this combined operation our early results were a few 
percentage points better than the first two operations, but the difference was not 
statistically significant. Other observers who have compared scalenectomy alone to 
scalenectomy with first rib resection have also not noted statistically significant dif- 
ferences between the two [10-12]. Finally, some surgeons still perform just anterior 
scalenectomy with neurolysis and report results that are similar to the more exten- 
sive procedures. [13, 14]. [Q5: E] 

Major complications occur from all operations to decompress the thoracic outlet 
area regardless of the surgical approach. Injury to the subclavian artery and vein, 
brachial plexus, phrenic nerve, and long thoracic nerve are the most common 
serious complications. Less common are injuries to the thoracic duct and cervical 
sympathetic chain. Injuries to cutaneous nerves from either transaxillary or supra- 
clavicular approaches are common. Plexus injury occurs from excessive traction, 
which at the time may not seem excessive. Plexus injury can also occur when a 
clamp on the subclavian artery to control bleeding accidentally includes a nerve of 
the plexus. 

Plexus injury makes symptoms worse in 1 percent of patients. The incidence of 
temporary phrenic nerve injury during supraclavicular approaches is 6-10 percent 
because the phrenic nerve is often in the middle of the field and is very sensitive to 
even mild retraction [12]. [Q6: J] 

The results of surgery are about the same for all procedures. The biggest variable 
is etiology. When the etiology is an auto accident related injury, the 1-year success 
rate is 75-80 percent; when the etiology is repetitive stress at work or a work injury, 
the success rate is 15 percent lower [11, 12, 15]. [Q7: B (for auto accident etiology), 
C (for work related and repetitive stress etiology)] 



References 

1. Sanders RJ, Jackson, CGR, Banchero N, Pearce WH. Scalene muscle abnormalities in traumatic tho- 
racic outlet syndrome. Am J Surg 1990;159:231-6. 

2. Gergoudis R, Barnes R W. Thoracic outlet arterial compression: prevalence in normal persons. 
Angiology 1980;31:538-41. 



Neurogenic Thoracic Outlet Syndrome 295 

3. Sanders RJ, Haug CE: Thoracic outlet syndrome: a common sequela of neck injuries. Philadelphia: 
Lippincott, 1991;71-84. 

4. Gilliatt RW, Willison RG, Dietz V, Williams IR. Peripheral nerve conduction in patients with a cervi- 
cal rib and band. Ann Neurol 1978;4:124-9. 

5. Green RM, McNamara MS, Ouriel K: Long-term follow-up after thoracic outlet decompression: an 
analysis of factors determining outcome. J Vase Surg 1991;14:739-46. 

6. Novak CB, Collins ED, Mackinnon SE: Outcome following conservative management of thoracic 
outlet syndrome. J Hand Surg 1995;20A:542-8. 

7. Roos DB. The place for scalenectomy and first rib resection in thoracic outlet syndrome. Surgery 
1982;92:1077-85. 

8. Sanders RJ, Pearce WH: The treatment of thoracic outlet syndrome: A comparison of different oper- 
ations. J Vase Surg 1989;10:626-34. 

9. Sanders RJ, Cooper MA, Hammond SL, Weinstein ES. Neurogenic thoracic outlet syndrome. In 
Rutherford RB, editor. Vascular surgery, 5th edn. Philadelphia: Saunders, 1999; 1184-200. 

10. Cheng SWK;, Reilly LM, Nelken NA, at al. Neurogenic thoracic outlet decompression: rationale for 
sparing the first rib. Cardiovascular Surg 1995;3:617-23. 

11. Thomas GI. Diagnosis and treatment of thoracic outlet syndrome. Perspect Vase Surg 1995;8:1-28. 

12. Sanders RJ, Hammond SL. Complications and results of surgical treatment for thoracic outlet syn- 
drome. Chest Surg Clin N Am 1999;9:803-20. 

13. Razi DM, Wassel HD. Traffic accident induced thoracic outlet syndrome: Decompression without rib 
resection, correction of associated recurrent thoracic aneurysm. Int Surg 1993;78:25-7. 

14. Gockel M, Vastamaki M, Alaranta H: Long-term results of primary scalenotomy in the treatment of 
thoracic outlet syndrome. J Hand Surg 1994;19B:229-33. 

15. Ellison DW, Wood VE: Trauma-related thoracic outlet syndrome. J Hand Surg 1994;19B:424-6. 



33. Thoracoscopic Sympathectomy 

Samuel S. Ahn, Huck A. Mandel and Kyung M. Ro 



A 22-year-old female undergraduate student who had had bilateral palmar 
hyperhydrosis since the fifth grade presented with hyperhydrosis that was refrac- 
tory to noninvasive treatment. Her sweating was severe, to the point that it 
dripped and was thus incapacitating. Her symptoms become more severe pri- 
marily when she was anxious or upset, but they also increased with physical 
activity or elevated temperatures. She was unable to shake or hold hands in social 
settings. She had difficulty taking exams because the sweat made the paper wet. 
She was currently looking for a job and felt socially inhibited when being inter- 
viewed. Conservative therapy had failed, trying Drysol, Robinul, Drionic and 
roll-ons. Her hyperhydrosis had also proven refractory to hypnotherapy, 
biofeedback and iontophoresis. She denied any axillary hyperhydrosis. Her past 
medical history was negative for any major illnesses, and she denied any thyroid 
conditions, hypertension or diabetes. She had had no previous lung infections or 
prior surgeries. Her family history revealed a sister and a maternal grandfather 
also positive for hyperhydrosis. 

A review of symptoms was negative for weight loss, malaise, fatigue, chills and 
fever. The patient denied any constipation, diarrhoea, chest pain, shortness of 
breath or palpitations. She had no polydipsia, polyuria, or loss of strength or func- 
tion in the extremities. She also denied any Raynaud's syndrome-like symptoms. 

On physical examination, both hands were cool and clammy to the touch and 
exhibited severe dripping. Lung sounds were clear bilaterally. The neck was 
supple, with full carotid pulses and no thyromegaly or lymphadenopathy. No 
thoracic outlet compression was evident on examination. 

In this patient with severe bilateral hyperhydrosis refractory to medical treat- 
ment, the plan was to proceed with bilateral thoracoscopic sympathectomy. 






Question 1 

What is primary hyperhydrosis, and what is its incidence? 



Question 2 

What is the natural history of primary palmar hyperhydrosis? 

297 



298 Vascular Surgery 

Question 3 

What are the indications for thoracoscopic sympathectomy? 

Question 4 

What are the relative contraindications for the procedure? 

The patient was taken to the operating room and intubated with a double-lumen 
endotracheal tube. She was then placed in the lateral decubitus position, with the 
ipsilateral arm abducted in the arm holder at a 90-degree angle. Strong radial pulses 
were palpated. The chest wall was prepped and draped in sterile fashion. Entrance 
into the chest was through three separate 7-mm ports. Soft flexible trocars were 
used. The first port was placed in the sixth intercostal space in the posterior axillary 
line, a second port in the sixth intercostal space in the mid-axillary line, and a third 
port in the third intercostal space along the anterior axillary line. 

The ipsilateral lung was deflated mechanically and retracted anteriorly and inferi- 
orly. The superior aspect of the sympathetic chain and the extent of surgical dissec- 
tion were identified through visualisation of the subclavian artery. The azygous, 
innominate and subclavian veins, along with the phrenic and vagus nerves, were 
identified and preserved. Electrocautery was used solely to unroof the parietal pleura 
from rib segments 1-4, thereby exposing the sympathetic chain and ganglia. 
Electrocautery was not to be used in proximity to the nerve fibres. The rami commu- 
nicantes of the second and third ganglion were cut (the fourth ganglion as well if the 
axillary hyperhydrosis is significant) while taking extreme precautions to avoid injury 
to the intercostal artery and vein. The dissection proceeded in a rostral fashion and 
was carried to the lower portion of the stellate ganglion. Next, the nerve of Kuntz was 
identified (this is a large branch of the Tl ramus that runs parallel and lateral to the 
trunk of the sympathetic chain at the inferior aspect of the superior stellate ganglion). 
The sympathetic chain was resected inferiorly and caudally to the stellate ganglion at 
the level of the superior margin of the second rib. The third ganglion and the chain 
just caudal to the end of the third ganglion were subsequently divided, corresponding 
to the level of superior aspect of the fourth rib. The chain ganglion was sent to pathol- 
ogy for intraoperative histological confirmation. A 16 Fr chest tube was inserted 
through the middle port and placed under direct visualisation. The lung was then 
reinflated using a positive pressure of 30-40 mm Hg, and adequate inflation was 
confirmed by endoscope. The port was closed using 3-0 Dexon running sutures. The 
patient was then repositioned on the contralateral side and completely reprepped and 
redraped, and the procedure was repeated on the other side. 

Question 5 

What are the short-term complications associated with the procedure? 



Question 6 

What are the long-term complications associated with the procedure? 



Thoracoscopic Sympathectomy 299 

Immediately following the operation, a chest X-ray was performed to rule out 
pneumothorax or pleural effusions. No air leaks were present, and the chest tubes 
were removed in the recovery room. The patient was placed on oral analgesics and 
discharged from the hospital the same day. 

On her 1-week postoperative visit, the patient had no evidence of Horner's syn- 
drome, compensatory hyperhydrosis, neuralgia or bradycardia. The patient had 
complete resolution of her palmar hyperhydrosis and no recurrence of symptoms at 
2-years' follow-up. 



Question 7 

What are the long-term results of thoracoscopic sympathectomy? 



Question 8 

What are the results of the thoracoscopic sympathectomy procedure for palmar 
hyperhydrosis in children and adolescents? 



Commentary 



Primary hyperhydrosis is a pathological condition of overperspiration caused by 
oversecretion of the eccrine sweat glands [1]. The aetiology is unknown. An exag- 
gerated response to emotional stimuli, heat and physical effort has been implicated 
[2, 3]. It has also been suggested that hyperhydrosis is a condition of central origin, 
with resultant excessive stimulation of the sweat glands by an overactive sympa- 
thetic system; it may also be attributed to an autonomic dysfunction [4, 5]. Primary 
hyperhydrosis maybe confined to the palms in some patients; however, the axillae 
and soles are also often affected. 

The incidence of primary hyperhydrosis is poorly understood. The only epidemi- 
ological study was conducted by Adar et al. [6], who reported an incidence of 
0.6-1% with no evidence to suggest a greater male or female predominance. Over 
the past several years, various investigators have described positive family histories, 
ranging from five to 50 per cent, with 40 per cent of these patients having excessive 
sweating [7-12]. Although many ethnic predispositions have been implicated, 
primary hyperhydrosis appears to be a worldwide problem [3, 13]. [Q1] 

The natural history of primary palmar hyperhydrosis is poorly documented; 
however, the onset appears to be during childhood or early adolescence. Most 
patients present for treatment in the second or early third decades of life, with com- 
plaints of psychological, social and occupational disabilities [10, 13]. [Q2] 

Thoracoscopic sympathectomy has been used effectively to treat several neuro- 
logical and vascular conditions. The majority of procedures have been performed 
to treat primary palmar hyperhydrosis [10, 11, 14-33], Raynaud's syn- 
drome [18, 26], Buerger's disease [34], reflex sympathetic dystrophy (causalgia) 
[18, 26, 34, 35], refractory cardiac arrhythmia [18, 35], and intractable visceral 
pain [36, 37]. Noninvasive treatments may be used in those patients who have 



300 Vascular Surgery 

minor symptoms or who are reluctant to undergo surgery. Topical injection of 
botulinum has been shown to reduce the amount of local perspiration, but long- 
term evaluation is needed [38, 39]. Hypnosis, psychotherapy and biofeedback 
have been beneficial in a limited number of cases [40]. Iontophoresis has attained 
satisfactory results, but has not yet been studied long term [41-43]. Percutaneous 
computed tomography (CT)-guided phenol sympathicolysis has shown short- 
term success, but its long-term failure rate is prohibitive [44]. The topical appli- 
cation of astringents or antiperspirants is considered effective, but only 
temporarily, thus requiring persistent application; this can often lead to irritation 
[41,45].[Q3] 

Thoracoscopic sympathectomy has generally been considered to be contraindi- 
cated in patients with previous thoracic operations or pulmonary infections who 
may have dense adhesions that impede thorough visualisation of the sympathetic 
chain [46]. Nonetheless, it has been shown that fibrous adhesions complicated the 
procedure in only five per cent of cases [47]. Due to the positioning of the arms 
throughout the procedure, it is necessary to rule out thoracic outlet syndrome 
(TOS). If a patient is identified with TOS, the arm should be positioned to min- 
imise the compression of the thoracic outlet, and the upper extremity should be 
monitored for adequate peripheral pulses throughout the duration of the 
procedure. [Q4] 

Possible short-term complications include pneumothorax, haemothorax. subcu- 
taneous emphysema, pleural effusion, and segmental atelectasis [48]. Incomplete 
reinflation of the collapsed apical lobe will result in postoperative complications 
from atelectasis and pneumothorax [48]. Rare complications, including false 
aneurysm of the intercostal artery and inferior brachial plexus injury, have been 
described in case reports [49-51]. [Q5] 

Long-term complications include compensatory sweating in the trunk and/or lower 
extremities, intercostal neuralgia, and Horner's syndrome. Compensatory hyperhy- 
drosis is the most common and unpredictable side effect and is reported in 0-74.5% 
of cases, with the trunk and lower extremities primarily affected [1, 1 1, 23, 24, 48, 52]. 
Anecdotal evidence suggests that the sweating subsides over time [17, 48]. The exact 
cause is unknown, but an increased number of coagulated segments have correlated 
with the degree of compensatory sweating [17, 23, 24]. Surgeons should limit their 
resections to a minimum. Gustatory hyperhydrosis, or facial sweating triggered par- 
ticularly by spicy foods, has also been reported in up to 56 per cent of patients in 
some series [53, 54]. Phantom sweating, or a feeling of sweating in the hands without 
actual sweating, has been reported in 26-59% of patients [55-57]. 

Intercostal neuralgia may stem from thermal damage to the adjacent intercostal 
nerve or from compression injury while using large, rigid thoracoports [18]; 
however, no additional complications of neuralgia have been reported with the 
change to soft 7-mm ports. 

Horner's syndrome, which results from damage to the stellate ganglion, is of 
most concern to the patient. The condition is transient in 0-23.5% of cases [18], 
with permanent symptoms occurring in 0-6% of cases [10]. To minimise the risk of 
Horner's syndrome, the surgeon should take great care to visualise the multiple 
rami of the stellate ganglion along with their course and direction, and incise imme- 
diately caudal to it, above the level of the T2 ganglion. Traction of the stellate 
ganglion during partial resection of the sympathetic chain may result in 
transient Horner's syndrome [18]. Heat generation from electrocoagulation, 
misidentification of the second rib, and injury to the stellate ganglion from pulling 



Thoracoscopic Sympathectomy 301 

on the sympathetic chain are potential causes of permanent Horner's syndrome. 
Other possible operative complications include injury to the vagus nerve, phrenic 
nerve, subclavian artery or subclavian vein. [Q6] 

Attempts at further reducing access trauma have resulted in the development of 
ultrafine thoracoscopic instruments. The use of 2-mm ports with two ports of 
entry has been described in performing sympathectomy or sympathicotomy with 
less scarring and pain and good outcomes [52, 58]. To improve cosmesis, a two- 
port axillary and a periareolar procedure has been used in patients presenting for 
thoracoscopic sympathectomy [59]. A one-port approach has sometimes been 
used through the axilla, where simple cautery destruction of the chain is 
performed [60]. 

Failure of thoracoscopic sympathectomy has resulted primarily from inadequate 
ablation of the sympathetic nerves or regrowth of the nerve tissue; as such, many 
investigators recommend obtaining intraoperative histological confirmation of the 
resected portion of sympathetic chain [18, 22, 25]. Success rates for thoracoscopic 
sympathectomy limited only to the rami communicantes for the treatment of 
primary palmar hyperhydrosis range from 71 to 100 per cent, with recurrence rates 
ranging from zero to 1.5 per cent [15, 19, 28, 38]. For treatment of Raynaud's 
disease, success and recurrence rates have ranged from 93 to 100 per cent and from 
zero to 13 per cent, respectively. For the treatment of reflex sympathetic dystrophy, 
initial success rates have ranged from 86 to 100 per cent, with recurrence rates as 
high as 62.5 per cent [24, 25]. Results for the treatment of intractable visceral pain 
yield a success rate of 71-100% and a reported recurrence rate of 33 per cent at 
3-months' follow-up [37, 38]. [Q7] 

Cohen et al. [11, 26, 61] performed thoracoscopic sympathectomies in children 
and adolescents and concluded that the procedure was safe and effective at a 4-year 
follow-up. In another series reported by Lin [53] (mean age 14.2 years), in which the 
T2 or T2 and T3 segments of the sympathetic chain were ablated along with the 
Kuntz fibres, only one technical failure was reported, with a patient satisfaction rate 
of 93.5 per cent. The rate of compensatory hyperhydrosis was a disappointing 
86 per cent, and the recurrence rates at 1-, 2- and 3-years' follow-up were 0.6, 1.1 
and 1.7 per cent, respectively. [Q8] 



References 



1. Sato K, Kang WH, Saga K, Sato KT. Biology of sweat glands and their disorders: II. Disorder of sweat 
gland function. J Am Acad Dermatol 1989;20:713-25. 

2. Dohn DF, Zraik O. Essential hyperhidrosis - pathogenesis and treatment. Cleveland Clin Q 
1969;36:79-83. 

3. Borak J, Eller JJ, Eller WD. Roentgen therapy for hyperhidrosis. Arch Dermatol Syphil 
1949;59:644-52. 

4. Chalmers TM, Keele CA. The nervous and chemical control of sweating. Br J Dermatol 
1952;64:43-54. 

5. Momose T, Kunimoto M, Nishikawa J, Kasaka N, Ohtake T, Iio M. N-isopropyl 1-123 p-iodoamphet- 
amine brain scans with single photon emission computed tomography: mental sweating and EEG 
abnormality. Radiat Med 1986;4:46-50. 

6. Adar F, Kurchin A, Zweig A, et al. Palmar hyperhidrosis and its surgical treatment. Ann Surg 
1977;186:34. 

7. Lai YT, Yang LH, Chio CC, Chen HH. Complications in patients with palmar hyperhidrosis treated 
with transthoracic endoscopic sympathectomy. Neurosurgery 1997;41:110-15. 



302 Vascular Surgery 

8. Drott C, Claes G. Hyperhidrosis treated by thoracic sympathectomy. Cardiovasc Surg 1996;4:61-4. 

9. Rosenblum JA, Cohen LM, Lee M. Hyperhidrosis - a case history. Angiology 1994;45:61-4. 

10. Kopelman D, Hashmonai M, Ehrenreich M, Assalia A. Upper dorsal thoracoscopic sympathectomy 
for palmar hyperhidrosis: improved intermediate-term results. J Vase Surg 1996;24:194-9. 

11. Cohen Z, Shinar D, Levi I, Mares AJ. Thoracoscopic upper thoracic sympathectomy for primary 
palmar hyperhidrosis in children and adolescents. J Ped Surg 1995;30:471-3. 

12. Noppen M, Vincken W. Essential hyperhidrosis in turner syndrome. Clin Genet 1997;52:63-4. 

13. Cloward RB. Treatment of hyperhidrosis palmaris (sweaty hands): a familial disease in Japanese. 
Hawaii Med J 1957;16:381-9. 

14. Byrne J, Walsh TN, Hederman WP. Endoscopic transthoracic electrocautery of the sympathetic 
chain for palmer and axillary hyperhidrosis. Br J Surg 1990;77:1046-9. 

15. Friedel G, Linder A, Toomes H. Selective video-assisted thoracoscopic sympathectomy. Thorac 
Cardiovasc Surg 1993;41:245-8. 

16. Claes G, Drott C, Gothberg G. Endoscopic electrocautery of the thoracic sympathetic chain: a min- 
imally invasive way to treat palmar hyperhidrosis. Scand J Plast Reconstr Surg Hand Surg 
1993;27:29-33. 

17. HJ, Shih DY, Fung ST. Transthoracic endoscopic sympathectomy in the treatment of palmer hyper- 
hidrosis. Arch Surg 1994;129:630-3. 

18. Ahn SS, Machleder HI, Concepcion B, Moore WS. Thoracoscopic cervicodorsal sympathectomy: 
preliminary results. J Vase Surg 1994;20: 511-19. 

19. Herbst F, Plas EG, Fugger R, Fritsch A. Endoscopic thoracic sympathectomy for primary hyperhidro- 
sis of the upper limbs. Ann Surg 1994;220:86-90. 

20. Wu JJ, Hsu CC, Liao SY, Liu JC, Shih CJ. Contralateral temperature changes of the finger surface 
during video endoscopic sympathectomy for palmar hyperhidrosis. J Auton Nerv Syst 1996;59: 
98-102. 

21. Graham ANJ, Owens WA, McGuigan JA. Assessment of outcomes after thoracoscopic sympathec- 
tomy for hyperhidrosis in a specialized unit. J R Coll Surg Edin 1996;41:160-3. 

22. Lee KH, Hwang PYK. Video endoscopic sympathectomy for palmar hyperhidrosis. J Neursurg 
1996;84:484-6. 

23. Noppen M, Herrgodts P, D'Haese J, D'Haens J, Vincken W. A simplified T2-T3 thoracoscopic sym- 
pathicolysis technique for the treatment of essential hyperhidrosis: short-term results in 100 
patients. J Laparoendosc Surg 1996;6:151-9. 

24. Bonjer HJ, Hamming JF, du Bois NAJJ, van Urk H. Advantages of limited thoracoscopic sympathec- 
tomy. Surg Endosc 1996;10:721-3. 

25. Josephs LG, Menzoian JO. Technical considerations in endoscopic cervicothoracic sympathectomy. 
Arch Surg 1996;131:355-9. 

26. Cohen Z, Shinar D, Kurtzbart E, Finaly R, Mares AJ. Laparoscopic and thoracoscopic surgery in children 
and adolescents: a 3-year experience. Pediatr Surg Int 1997;12:356-9. 

27. Dumont P, Hamm A, Skrobala D, Robin P, Toumieux B. Bilateral thoracoscopy for sympathectomy 
in the treatment of hyperhidrosis. Eur J Cardiothorac Surg 1997;64:975-8. 

28. Gossot D, Toledo L, Fritsch S, Celerier M. Thoracoscopic sympathectomy for the upper limb hyper- 
hidrosis: looking for the right operation. Ann Thor Surg 1997;64:975-8. 

29. Lee LS, Lin CC, Ng SM, Au CF. The haemodynamic effects of thoracoscopic cardiac sympathectomy. 
Eur J Surg 1998;580 (suppl):37-8. 

30. Zacherl J, Huber ER, Imhof M, Plas EG, Herbst G, Fugger R. Long-term results of 630 thoracoscopic 
sympathicotomies for primary hyperhidrosis: the Vienna experience. Eur J Surg 19981 580 
(suppl):43-6. 

31. Hsia JY, Chen CY, Hsu CP, Shai SE, Yang SS. Outpatient thoracoscopic limited sympathectomy for 
hyperhidrosis palmaris. Ann Thorac Surg 1999;67:258-9. 

32. Ishibashi H, Hayakawa N, Yamanoto H, Nishikimi N, Yano T, Nimura Y. Thoracoscopic sympathec- 
tomy for Buerger's disease: a report on the successful treatment of four patients. Surg Today 
1995:25:180-3. 

33. Olcott C, Eltherington LG, Wilcosky BR, Shoor PM, Zimmerman JJ, Fogarty T. Reflex sympathetic 
dystrophy - the surgeon's role in management. J Vase Surg 1991;14:488-95. 

34. Honjyo K, Hamasaki Y, Kita M, Totoki T, Miyazaki S. An 11 -year old girl with reflex sym- 
pathetic dystrophy successfully treated by thoracoscopic sympathectomy. Acta Paediatr 
1997;86:903-5. 

35. Ouriel K, Moss AJ. Long QT syndrome: an indication for cervicothoracic sympathectomy. 
Cardiovasc Surg 1995;3:475-8. 

36. Lonroth L, Hyltander A, Lundell L. Unilateral left-sided thoracoscopic sympathectomy for visceral 
pain controha pilot study. Eur J Surg 1997;163:97-100. 



Thoracoscopic Sympathectomy 303 

37. Noppen M, Meysman M, D'Haese J, Vincken W. Thoracoscopic splanchnicolysis for the relief of 
chronic pancreatitis pain: experience of a group of pneumologists. Chest 1998;113:528-31. 

38. Schneider P, Binder M, Auff E, et al. Double-blind trial of botulinum A toxin for the treatment of 
focal hyperhidrosis of the palms. Br J Dermatol 1997;136:548-52. 

39. Naumann M, Hofmann U, Bergmann I, Hamm H, Toyka KV, Reiners K. Focal hyperhidrosis. 
Effective treatment with intracutaneous botulinum toxin. Arch Dermatol 1998;134:301-4. 

40. Tabet J-CM, Bay JW, Magdenic M. Essential hyperhidrosis. Current therapy. Cleveland Clin Q 
1986;36:79-83. 

41. Manusov EG, Nadeau MT. Hyperhidrosis: a management dilemma. J Fam Pract 1989;28:412-15. 

42. Midtgaard K. A new device for the treatment of hyperhidrosis by iontophoresis. Br J Dermatol 
1986;114:485-8. 

43. Reinauer S, Neusser A, Schauf G, Holzle E. Iontophoresis with alternating current and direct current 
offset (AC/DC iontophoresis): a new approach for the treatment of hyperhidrosis. Br J Dermatol 
1993;129:166-9. 

44. Haxton HA. Chemical sympathectomy. BMJ 1949;1:1026-8. 

45. Sato K. Hyperhidrosis. JAMA 1991;265:651. 

46. Rutherford RB. Role of sympathectomy in the management of vascular disease. In: Moore WS, 
editor. Vascular Surgery: a comprehensive review. Philadelphia: WB Saunders, 1993; 300-12. 

47. Yim AP, Liu HP, Hazelrigg SR, Izzat MB, Fung AL, Boley TM, Magee MJ. Thoracoscopic operations 
on reoperated chests. Ann Thorac Surg 1998;65:328-30. 

48. Ahn SS, Ro KM. Thoracoscopic sympathectomy: a case report. Surg Tech Intl VII, April 1998. 

49. Atherton WG, Morgan WE. False aneurysm of an intercostal artery after thoracoscopic sympathectomy. 
Ann R Coll Surg Engl 1991;79:229-30. 

50. Lange JF. Inferior brachial plexus injury during thoracoscopic sympathectomy. Surg Endosc 
1995;9:830. 

51. Hashmonai M, Kopelman D. Inferior brachial plexus injury during thoracoscopic sympathectomy. 
Surg Endosc 1996;10:459. 

52. Lee DY, Yoon YH, Shin HK, Kim HK, Hong YJ. Needle thoracic sympathectomy for essential hyper- 
hidrosis: intermediate-term follow-up. Ann Thorac Surg 2000;69:251-3. 

53. Lin TS. Transthoracic endoscopic sympathectomy for palmar and axillary hyperhidrosis in children 
and adolescents. Ped Surg Int 1999;15:475-8. 

54. Zacherl J, Imhof M, Huber ER, Plas EG, Herbst F, Jakesz R, Fugger R. Video assistance reduces com- 
plication rate of thoracoscopic sympathectomy for hyperhidrosis. Ann Thorac Surg 1999 68:1177-81. 

55. Kurchin A, Zweig A, Adar R, Mozes M. Upper dorsal sympathectomy for palmar primary hyper- 
hidrosis by the supraclavicular approach. World J Surg 1977;1:667-74. 

56. Hashmonai M, Kopelman D, Klein O, Schein M. Upper thoracic sympathectomy for primary palmar 
hyperhidrosis: long-term follow-up. Br J Surg 1992;79:268-71. 

57. Koikkalainen K, Luosto R, Keskitalo E, Melartin E. Upper limb sympathectomy. Ann Chir Gynaecol 
Fenn 1974;63:318-25. 

58. Yim AP, Liu HP, Lee TW, Wan S, Arifi AA. Needlescopic video-assisted thoracic surgery for palmar 
hyperhidrosis. Eur J Cardiothorac Surg 2000;17:697-701. 

59. Kesler KA, Brooks-Brunn J, Campbell RL, Brown JW. Thoracoscopic sympathectomy for hyper- 
hidrosis palmaris: a periareolar approach. Ann Thor Surg 2000;70:314-17. 

60. Weight SC, Raitt D, Barrie WW. Thoracoscopic sympathectomy: a one-port technique. Aust N Z J 
Surg 2000;70:800. 

61. Cohen Z, Levi I, Pinsk I, Mares AJ. Thoracoscopic upper sympathectomy for primary palmar hyper- 
hidrosis - the combined pediatric, adolescent and adult experience. Eur J Surg 1998;580(suppl):5-8. 



34. Acute Axillary/Subclavian Vein 
Thrombosis 

Jarlis Wesche, Torbjorn Dahl and Hans 0. Myhre 



A 34-year-old male motor mechanic was admitted with a 3-day history of severe 
swelling of the right arm. He had been undertaking physical activity, including 
weightlifting, training for about 1.5 h four times a week. There was no history of 
trauma. The patient felt discomfort, but no severe pain in the arm. The 
superficial veins were distended. The colour of the hand and forearm was slightly 
cyanotic. The pulses in the radial and ulnar arteries were palpable. No bruits 
could be heard along the brachial, supraclavicular or axillary arteries. The rest of 
the examination was unremarkable. The patient did not use any medication. 



Question 1 

What further diagnostic investigations would you recommend in this patient? 

A. Plethysmography. 

B. Phlebography. 

C. Duplex scanning. 

D. Magnetic resonance phlebography. 

E. Computed tomography (CT) scanning. 

F. X-ray of the chest and thoracic outlet. 

G. Venous pressure measurements. 

Phlebography revealed a thrombosis of the axillary/subclavian veins 
(Fig. 34.1). The brachiocephalic vein was patent. There were no signs of skeletal 
deformities. 



305 



306 



Vascular Surgery 




Fig. 34.1. Digital subtraction angiography (DSA) phlebogram showing occlusion of the right subclavian vein, 
but contrast passage to the superior caval vein via jugular/supraclavicular collateral veins (note its relation to 
the thoracic outlet). 



Question 2 

Which of the following conditions could lead to axillary/subclavian vein thrombosis? 

A. Venous-access catheters. 

B. Callus from fractured clavicle or rib. 

C. Local tumour/malignancy. 

D. Radiotherapy. 

E. Trauma to the vein caused by repeated strenuous exercise. 



Question 3 

Which therapy would you recommend in the acute (2-3 days) phase? 

A. Resection of the first rib. 

B. Balloon angioplasty of the subclavian vein. 

C. Stenting of the subclavian vein. 

D. Thrombolysis. 



Acute Axillary/Subclavian Vein Thrombosis 307 

E. Systemic heparin. 

F. Thrombectomy. 

Question 4 

Following thrombolytic therapy for axillary/subclavian vein thrombosis, what 
percentage of complete lysis can you expect provided the patient is treated within 
3 days after start of symptoms? 

A. 10%. 

B. 25%. 

C. 40%. 

D. 60%. 

E. 80%. 



Question 5 

A control phlebography revealed a stenosis of the axillary/subclavian vein at the 
thoracic outlet. There was no residual thrombotic material. At 3 months' follow-up 
the patient still had pain and discomfort in the arm when going back to his job as a 
motor mechanic. Which of the following treatment alternatives would you recom- 
mend at this stage? 

A. Repeated attempt of thrombolytic therapy. 

B. Balloon angioplasty and stenting of the subclavian artery. 

C. Continued oral anticoagulation therapy. 

D. Relief of the thoracic outlet by resection of the first rib. 

E. Direct reconstruction of the vein. 



Commentary 



In patients with acute axillary/subclavian vein thrombosis, it is important to sepa- 
rate primary from secondary thrombosis. Primary thrombosis is also known as 
Paget-Schrotter syndrome, which is induced by strenuous activity of the arm or 
venous compression at the thoracic outlet predisposing to thrombosis formation 
[1-4]. The term "effort thrombosis" is also used for this condition. Men are affected 
more often than women, and the incidence is higher in the veins of the dominant 
arm. Secondary axillary/subclavian vein thrombosis could be caused by venous- 
access catheters, pacemaker wires, malignancies, radiotherapy or compression from 
local tumour formation. Secondary thrombosis is also seen as a complication of 
thrombophilia and in patients with dialysis fistulas [5]. [Q2: A, B, C, D, E] The pre- 
ferred therapy maybe different in the two groups, and in general a more conserva- 



308 Vascular Surgery 

tive attitude is often justified in patients with secondary thrombosis. These patients 
often have a limited life expectancy due to serious co-morbidities, such as cardiac 
disease or malignancy, which would also represent a contraindication to throm- 
bolytic therapy. In addition, there is often less need for extensive activity of the 
upper extremities in this group of patients. 

Complications following axillary/subclavian vein thrombosis are swelling, pain 
and discomfort in the arm prohibiting work or daily-life activities. Furthermore, it 
has been reported that up to 10 per cent of the patients with axillary/subclavian vein 
thrombosis develop pulmonary emboli and that it is more common than usually 
appreciated [5, 6]. Phlegmasia cerulea dolens of the arm is extremely rare and is 
usually associated with hypercoagulability or malignancies. 

In patients with primary axillary/subclavian vein thrombosis, as in our patient, 
duplex scanning can be performed as a supplement to the clinical examination 
[6]. However, duplex scanning is operator dependent. If the examination is nega- 
tive, then phlebography has to be performed anyway. Thus, phlebography, prefer- 
ably by contrast injection via the basilic vein, should be the gold standard in these 
cases. Recently, three-dimensional gadolinium-enhanced magnetic resonance 
phlebography has proven excellent for imaging central veins, and series present- 
ing up to 100 per cent sensitivity and specificity in the accuracy of diagnosing 
abnormalities in the large central veins have been published [7]. Magnetic reso- 
nance phlebography may thus be the future gold standard. A chest X-ray includ- 
ing the thoracic outlet to investigate the possibility of bony deformations is also 
indicated. [Q1: B,C, D,F] 

The patient should be evaluated thoroughly for thrombophilia. Blood tests 
should include a blood count, tests for decreased levels of antithrombin (III), 
protein C and protein S deficiencies, activated protein C (APC) resistance, antiphos- 
pholipid antibodies (lupus anticoagulans) and anticardiolipin antibodies. 

As soon as the diagnosis has been established, systemic heparinisation is admin- 
istered [8]. This should be followed by local thrombolysis using urokinase or 
preferably recombinant tissue plasminogen activator (rt-PA) unless there are con- 
traindications [9-14]. At introduction of the guidewire, the resistance will indicate 
the age of the thrombosis and the possibility of obtaining lysis of the thrombotic 
occlusion. The catheter for application of the thrombolytic agent should be placed 
within the thrombosis. Usually, a dose of 5 mg rt-PA is given as a bolus, followed by 
infusion of 0.01 mg/kg body weight/hour for 24-72 h. Injection of thrombolytic 
material distal to the thrombotic occlusion is less effective since it will usually pass 
through the collateral veins. [Q3: D, E] 

Although the most favourable results are obtained in patients with less than 1 
week's duration of symptoms [10], an attempt at thrombolysis could be justified 
even if the symptoms have lasted for 1 month. [Q4: E] 

After thrombolysis, a repeat phlebography is performed to evaluate whether any 
intrinsic or extrinsic obstructions of the blood flow are present. Functional phlebog- 
raphy with the arm abducted to 90° with external rotation allows better visualisa- 
tion of compression of the vein as well as the collaterals. Often a defect is located 
close to the costoclavicular ligament. Together with hypertrophic anterior scalene 
and subclavius muscles, this ligament could cause external compression of the vein. 
The head of the clavicle could contribute to the deformation of the vein. Intrinsic 
venous stenosis is thought to be due to repetitive trauma damaging venous valves 
or the endothelium, or producing thickening of the vein wall or intraluminal 
synechiae, predisposing to thrombosis. 



Acute Axillary/Subclavian Vein Thrombosis 



309 



Acute axillary/subclavian 
vein thrombosis 
Screening forthrombophilia 



Phlebography 



Secondary thrombosis 
consider 
conservative treatment 



Haparin/oral anticoagulation 



Primary thrombosis 
Heparin (systemic) 

i 

Thrombolytic therapy 

i 

Control phlebography 



Oral anticoagulation and 
observation for 1-3 months 



Rethrombosis 



Symptomatic 



Phlebography 



i 



Asymptomatic 



Conservative 
Management 



Repeat thrombolysis 



No thrombosis 



i 



Control _ 
phlebography 



Operative relief 
of thoracic outlet 
(1 .rib resection) 



Venous obstruction 



Balloon angioplasty only 

or followed by stent placement 

or venous reconstruction 



Fig. 34.2. Algorithmic approach to treatment options for acute axillary/subclavian vein thrombosis. 
Updated 



310 Vascular Surgery 

After thrombolysis, the patient should be on oral anticoagulation for 3-6 months. 
Heparin is discontinued as soon as the INR (international normalised ratio) has 
reached therapeutic levels. Following a 3-month follow-up period, the clinical status 
of the patient should be re-evaluated. Some centres proceed with more radical 
surgery soon after thrombolysis [11, 15-16]. If the patient is asymptomatic at 
follow-up, then we do not recommend further treatment. However, this policy is 
controversial and some centres proceed with decompression of the thoracic outlet 
routinely. 

If the patient is symptomatic and there is a residual stenosis of the subclavian 
vein caused by either internal or external pathological structures, then the stenosis 
should not be treated by balloon angioplasty or stenting primarily [11-13, 15, 17]. 
Whenever these treatment modalities are applied before relief of the thoracic outlet, 
recurrence of the symptoms will inevitably occur. Furthermore, fracture of the 
stents has been described because of the "scissors effect" caused by the narrow tho- 
racic outlet [18]. Decompression of the thoracic outlet is obtained by resection of 
the first rib, including the distal part of the anterior scalenic muscle and fibrous 
structures adhering to the first rib. [Q5: D] 

The surgical approach for relieving the thoracic outlet is also controversial: some 
prefer the transaxillary approach, whereas others use the supra- or infraclavicular 
approach [3, 5, 11, 15, 16, 19, 20]. 

After thoracic outlet surgery, a venous obstruction can be treated with balloon 
angioplasty. This technique is also controversial since there are no randomised 
studies. Balloon angioplasty could be supplemented with stenting, but the experi- 
ence with this treatment modality is rather limited. Finally, some authors recom- 
mend direct reconstruction by endovenectomy and patch angioplasty for relief of 
intravenous obstructions in selected cases [3]. 

In summary, the most effective sequence to restore venous patency and reduce 
rethrombosis seems to include local thrombolytic therapy, 3-6 months of oral anti- 
coagulation, and then first-rib resection in patients who have significant symptoms 
at this stage. Occasionally percutaneous transluminal angioplasty (PTA), stent 
placement or venous reconstruction may be indicated. Following such a staged mul- 
tidisciplinary treatment (Fig. 34.2), the disability rate after acute axillary/subclavian 
thrombosis has declined from around 60 per cent to 12 per cent [19]. 



References 

1. Hughes ESR. Venous obstruction in the upper extremity (Paget-Schroetter's syndrome). A review of 
320 cases. Int Abstr Surg 1949;88:89-128. 

2. McCleery RS, Kesterson JE, Kirtley JA, Love RB. Subclavius and anterior scalene muscle compression 
as a cause of intermittent obstruction of the subclavian vein. Ann Surg 1951;133:588-602. 

3. Haug CE, Sanders RJ. Venous TOS. In: Saunders RJ, editor. Thoracic outlet syndrome. A common 
sequela of neck injuries. Philadelphia: JB Lippicott, 1991;15:233-6. 

4. Daskalakis E, Bouhoutsos J. Subclavian and axillary vein compression of musculoskeletal origin. Br J 
Surg 1980;67:573-6. 

5. Hicken GJ, Ameli M. Management of subclavian -axillary vein thrombosis: a review. Can J Surg 
1998;41:13-24. 

6. Kerr TM, Lutter KS, Moeller DM, Hasselfeld KA, Roedersheimer R, McKenna PJ, et al. Upper 
extremity venous thrombosis diagnosed by duplex scanning. Am J Surg 1990;160:202-6. 

7. Thornton MJ, Ryan R, Varghese JC, Farrell MA, Lucey B, Lee MJ. A three-dimensional gadolinium- 
enhanced MR venography technique for imaging central veins. AJR 1999;173:999-1003. 



Acute Axillary/Subclavian Vein Thrombosis 31 1 

8. Gloviczki P, Kazmier FJ, Hollier LH. Axillary- subclavian venous occlusion: the morbidity of a non- 
lethal disease. J Vase Surg 1986;4:333-7. 

9. Becker GJ, Holden RW, Rabe FE, Castaneda-Zuniga WR, Sears N, Dilley RS, Glover JL. Local throm- 
bolytic therapy for subclavian and axillary vein thrombosis. Radiology 1983;149:419-23. 

10. Beygui RE, Olcott C, Dalman RL. Subclavian vein thrombosis: outcome analysis based on etiology 
and modality of treatment. Ann Vase Surg 1997;11:247-55. 

11. Lee MC, Grassi CJ, Belkin M, Mannick J A, Whittemore AD, Donaldson MC. Early operative interven- 
tion after thrombolytic therapy for primary subclavian vein thrombosis: an effective treatment 
approach. J Vase Surg 1998;27:1101-8. 

12. Lindblad B, Tengborn L, Bergqvist D. Deep vein thrombosis of the axillary- subclavian veins: 
epidemiologic data, effects of different types of treatment and late sequelae. Eur J Vase Surg 
1988;2:161-5. 

13. Lee WA, Hill BB, Harris EJ, Jr, Semba CP, Olcott C. Surgical intervention is not required for all 
patients with subclavian vein thrombosis. J Vase Surg 2000;32:57-67. 

14. Biiller HR, Agnelli G, Hull RD, Hyers TM, Prins MH, Raskob GE. Antithrombotic therapy for venous 
thromboembolic disease. The seventh ACCP conference on antithrombotic and thrombolytic 
therapy. Chest 2004;126( suppl):401-28S. 

15. Azakie A, McElhinney DB, Thompson RW, Raven RB, Messina LM, Stoney RJ. Surgical management 
of subclavian-vein effort thrombosis as a result of thoracic outlet compression. J Vase Surg 
1998;28:777-86. 

16. Urschel HC, Jr, Razzuk MA. Paget-Schroetter syndrome: What is the best management? Ann Thorac 
Surg 2000;69:1663-9. 

17. Glanz S, Gordon DH, Lipkowitz GS, Butt KM, Hong J, Sclafani SJA. Axillary and subclavian vein 
stenosis: percutaneous angioplasty. Radiology 1988;168:371-3. 

18. Bjarnarson H, Hunter DW, Crain MR, Ferral DW, Mitz-Miller SE, Wegryn SA. Collapse of a Palmaz 
stent in the subclavian vein. Am J Radiol 1993;160:1123-4. 

19. Machleder HI. Evaluation of a new treatment strategy for Paget-Schroetter syndrome: spontaneous 
thrombosis of the axillary- subclavian vein. J Vase Surg 1993;17:305-17. 

20. Kreienberg PB, Chang BB, Darling RC III, Roddy SP, Paty PSK, Lloyd WE, et al. Long-term results in 
patients treated with thrombolysis, thoracic inlet decompression, and subclavian vein stenting for 
Paget-Schroetter syndrome. J Vase Surg 2001;33:S100-5. 



35. Raynaud's Phenomenon 



Ariane L. Herrick 



A 38-year-old female patient presented to the rheumatology clinic with a 3-week 
history of a painful fingertip ulcer. The pain was so severe that it was keeping her 
awake at night. For 20 years (since her teens) her hands had been turning white 
then purple in the cold weather, going red (with tingling) when rewarming. Her 
feet also felt cold. Her family doctor had told her that this was Raynaud's phe- 
nomenon, which was very common. However, each winter her symptoms seemed 
to be worsening, and even a slight temperature change would bring on an attack. 
The previous winter she had had some finger ulcers which had, however, been 
less painful than the current one and which had healed spontaneously. Also of 
concern to her was that for 6 months the skin of her fingers had felt tight, and she 
had recently been experiencing some difficulty swallowing, with heartburn. 
There was no past medical history of note. She had smoked five cigarettes a day 
for 2 years. There was no history of chemical exposure nor of use of vibratory 
equipment. 



Question 1 

Which symptoms suggest that this is not primary (idiopathic) Raynaud's phenome- 
non? 

A. Onset of Raynaud's phenomenon age 18 years. 

B. The feet were affected as well as the hands. 

C. Development of digital ulcers. 

D. The skin of the fingers felt tight. 

E. She was a smoker. 

On examination she had a healing ulcer at the tip of the left middle finger (Fig. 
35.1). The fingertip was extremely tender. She had mild skin thickening of the 



313 



314 Vascular Surgery 




Fig. 35.1. Fingertip ulcer in a patient with systemic sclerosis. 

fingers (sclerodactyly) but elsewhere the skin was normal. She had digital pitting of 
the right index and middle fingers. There were no other abnormal findings. 

Question 2 

What investigations would you perform? 

A. Full blood count and erythrocyte sedimentation rate. 

B. Angiography. 

C. Testing for antinuclear antibody (ANA). 

D. Testing for anticentromere antibody. 

E. Nailfold capillaroscopy. 

Full blood count and erythrocyte sedimentation rate (ESR) were normal. On 
immunological testing she was strongly antinuclear antibody (ANA) positive (titre 
1/1000) and she was anticentromere antibody positive. Chest X-ray showed no cer- 
vical rib. Hand X-rays were normal. Nailfold microscopy was abnormal, showing 
widened, dilated loops with areas of avascularity (Fig. 35.2). 

Question 3 

What is the diagnosis? 

A. Limited cutaneous systemic sclerosis ("CREST" syndrome). 

B. Hyperviscosity state, for example secondary to malignancy. 

C. Extrinsic vascular compression. 

D. Atherosclerosis. 

E. Buerger's disease. 



Raynaud's Phenomenon 315 




Fig. 35.2. Typical appearances on nailfold microscopy in systemic sclerosis - several capillary loops are dilated, 
with areas of avascularity. 



Question 4 

Which of the following are true of systemic sclerosis (also termed "scleroderma"): 

A. Digital pitting is a characteristic feature. 

B. Males are more commonly affected than females. 

C. The two subtypes - limited cutaneous and diffuse cutaneous - are separated on 
the basis of the extent of the skin involvement. 

D. Raynaud's phenomenon often precedes the diagnosis of limited cutaneous sys- 
temic sclerosis by many years. 

E. Anticentromere antibody is a risk factor for severe digital ischaemia requiring 
amputation. 

The diagnosis of limited cutaneous systemic sclerosis was explained to the 
patient. She was told that her Raynaud's phenomenon and her upper gastrointesti- 
nal symptoms were most likely related, and that some checks of her cardiorespira- 
tory function would be arranged on a routine basis. 

Question 5 

How would you have treated her Raynaud's phenomenon had you seen her 
6 months previously, when there was no digital ulceration? 

A. Avoidance of cold exposure. 

B. Low dose prednisolone. 

C. Stop smoking. 



316 Vascular Surgery 

D. Nifedipine (sustained release). 

E. Biofeedback. 

The patient was prescribed nifedipine (sustained release) and a course of 
flucloxacillin. When reviewed one week later, the fingertip had deteriorated and 
some of the tissue had become necrotic, with surrounding erythema. 

Question 6 

What would you do now? 

A. Admit to hospital for intravenous prostanoid therapy. 

B. Intravenous antibiotics. 

C. Debridement of the ulcer. 

D. Cervical sympathectomy. 

E. Anticoagulation. 

The patient was admitted for intravenous antibiotic therapy, intravenous 
prostanoid infusions, and a vascular opinion. The fingertip was debrided. The 
patient was discharged home 6 days later, with instructions to dress warmly, avoid 
cold exposure, and to seek medical advice early should any further ulcers develop. 



Commentary 

Raynaud's phenomenon - episodic digital ischaemia usually in response to cold 
exposure or stress - can be either primary (idiopathic) or secondary to a number of 
different diseases/conditions, including connective tissue disease (most characteris- 
tically systemic sclerosis), external vascular compression (as with a cervical rib), 
vibration exposure, hyperviscosity, drug treatment (for example beta-blockers, 
ergotamine) and occupational chemical exposure. The terminology is confusing: 
primary Raynaud's phenomenon was previously termed "Raynaud's disease", and 
secondary Raynaud's phenomenon "Raynaud's syndrome". However, "primary 
Raynaud's phenomenon" and "secondary Raynaud's phenomenon" are now the 
preferred terms [1]. 

The pathophysiology of Raynaud's phenomenon (either primary or secondary) is 
poorly understood. Raynaud's phenomenon can occur because of abnormalities in 
vascular structure, vascular function, or the blood itself [2]. These are interdepen- 
dent and may occur together, as in systemic sclerosis when structural vascular 
problems inevitably impair vascular function, and platelet and white blood cell 
activation, together with impaired fibrinolysis, are also thought to contribute to 
pathophysiology. It is generally accepted that primary Raynaud's phenomenon is 
mainly vasospastic and does not progress to irreversible tissue damage. In contrast, 
Raynaud's phenomenon secondary to connective tissue disease such as systemic 
sclerosis is associated with structural vascular abnormality, and patients often 
develop ulceration, scarring, and even gangrene necessitating amputation. 



Raynaud's Phenomenon 



317 



The vascular surgeon is likely to encounter patients with Raynaud's phenomenon 
for two main reasons: 

1. Diagnosis. Why does this patient have episodic digital ischaemia? 

2. Treatment of a critically ischaemic digit, or of severe Raynaud's phenomenon 
unresponsive to medical therapy. 

The onset of primary Raynaud's phenomenon is most commonly in the teens or 
twenties: onset in later years should always raise the suspicion of an underlying 
cause. Women are more commonly affected. For Raynaud's phenomenon to be 
primary, there should be no clinical features of underlying connective tissue disease 
or other disease/disorder (including absence of digital pitting or sclerodactyly), 
there should be no digital ulceration or gangrene, the ESR should be normal, testing 
for ANA negative (titre < 1/100) and the nailfold capillaries should be normal [1]. 
[Q1 : C, D] In the absence of any worrying features in the history and examination, the 
usual investigation screen therefore comprises a full blood count and ESR, testing 
for ANA, nailfold capillaroscopy and, if there is any question of a cervical rib, a 
chest or thoracic outlet X-ray. Anaemia and/or a high ESR may indicate an underly- 
ing connective tissue disease or other illness. However, a normal haemoglobin level 
and ESR (as in our patient) do not exclude a diagnosis of systemic sclerosis, in 
which the vascular abnormalities are primarily non-inflammatory [3]. In primary 
Raynaud's phenomenon, the nailfold capillaries should be fairly regular "hair-pin" 
loops as opposed to the abnormal dilated loops, with areas of loop drop-out, that 
are characteristic of systemic sclerosis [4]. 

Other investigations are indicated by the history and examination. For example, 
if there is sclerodactyly (scleroderma of the fingers) and/or digital pitting (Fig. 35.3), 
which are both characteristic of systemic sclerosis, then anticentromere antibodies 
and antibodies to topoisomerase (anti-Scl-70 antibodies) should be looked for. 
These antibodies are highly specific for systemic sclerosis [5]. If there is any 




Fig. 35.3. Digital pitting in a patient with systemic sclerosis. 



318 Vascular Surgery 

question of a proximal vascular obstruction (absent peripheral pulses) then angiog- 
raphy should be considered, but in the majority of patients with systemic sclerosis 
and digital ischaemia this is not necessary. [Q2: A, C, D, E] 

Systemic sclerosis, similarly to primary Raynaud's phenomenon, is more 
common in women than in men. There are two main subtypes of systemic sclerosis 

- limited and diffuse cutaneous - defined on the basis of the extent of the skin 
involvement. In patients with limited cutaneous disease (previously termed CREST 

- calcinosis, Raynaud's, oesophageal dysmotility, sclerodactyly, telangiectases), 
only the skin of the extremities and face is thickened, whereas in those with diffuse 
cutaneous disease there is proximal skin thickening, involving proximal limbs 
and/or trunk [6]. The patient described has clinical features typical of limited cuta- 
neous disease: Raynaud's phenomenon preceding the diagnosis of systemic sclero- 
sis by a number of years, sclerodactyly, digital pitting, and upper gastrointestinal 
problems. [Q3: A] Patients with limited cutaneous disease typically have more severe 
digital vascular disease than patients with diffuse cutaneous disease, and anticen- 
tromere antibody is predictive of severe digital ischaemia [7]. [Q4: A, C, D, E] 

Treatment of Raynaud's phenomenon is initially conservative - keeping warm, 
avoiding cold exposure, and refraining from smoking (smoking is a risk factor for 
severity of digital ischaemia in patients with systemic sclerosis [8]). If these mea- 
sures do not suffice, then a vasodilator is prescribed, usually a calcium channel 
blocker [9, 10]. There is no role for steroid therapy in most patients with systemic 
sclerosis (and steroids are relatively contraindicated in patients with diffuse cuta- 
neous disease). Biofeedback has gained considerable attention but was not found to 
be effective in a randomised trial of primary Raynaud's phenomenon [11]. [Q5: A, C, 
D] If a patient has very severe digital ischaemia, with or without digital ulceration, 
then the patient should be admitted for intravenous prostanoids [12] and, if there is 
any question of infection, then intravenous antibiotics are also indicated. 

The vascular surgeon is likely to be called to see a patient with severe Raynaud's 
(often in the context of systemic sclerosis) because of either non-healing ulceration 
or because of very severe (sometimes critical) ischaemia. The reduced blood supply 




Fig. 35.4. Digital pulp calcinosis in a patient with systemic sclerosis -- there is a risk that this deposit will 
ulcerate. 



Raynaud's Phenomenon 319 

impairs ulcer healing. Debridement often aids healing. However, a proportion of 
patients come to amputation. Some patients have calcinosis at the site of the ulcera- 
tion, and so this maybe a complicating factor (Fig. 35.4). Severe ischaemia often 
coexists with ulceration. Cervical sympathectomy is no longer advocated for upper 
limb Raynaud's phenomenon. Recently digital sympathectomy has attracted inter- 
est for the treatment of severe digital ischaemia in patients with systemic sclerosis 
[10, 13]. Digital sympathectomy is unlikely to be indicated at this stage in our 
patient, unless things do not settle with intravenous prostanoids, antibiotics and 
debridement. At present there is no evidence base for anticoagulation in patients 
with systemic sclerosis and digital ischaemia and/or ulceration although the possi- 
bility of an underlying coagulopathy, for example antiphospholipid syndrome, 
should always considered in patients presenting with digital ischaemia. [Q6: A, B, C] 

Finally, although the vascular abnormalities in systemic sclerosis are predomi- 
nantly microvascular, an increased prevalence of large vessel disease in patients 
with systemic sclerosis has recently been reported [14]. Thus the possibility of a 
proximal obstruction should always be considered in patients with systemic sclero- 
sis presenting with an ischaemic digit. 



References 

1. LeRoy EC, Medsger TA. Raynaud's phenomenon: a proposal for classification. Clin Exp Rheumatol 
1992;10:485-8. 

2. Herrick AL. Pathogenesis of Raynaud's phenomenon. Rheumatology (in press). 

3. Campbell PM, LeRoy EC. Pathogenesis of systemic sclerosis: a vascular hypothesis. Semin Arthritis 
Rheum 1975;4:351-68. 

4. Maricq HR, LeRoy EC. Patterns of finger capillary abnormalities in connective tissue disease by 
"wide-field" microscopy. Arthritis Rheum 1973:16:619-28. 

5. Harvey GR, McHugh NJ. Serologic abnormalities in systemic sclerosis. Curr Op Rheumatol 
1999;11:495-502. 

6. LeRoy EC, Black C, Fleischmajer R, et al. Scleroderma (systemic sclerosis): classification, subsets and 
pathogenesis. J Rheumatol 1988;15:202-5. 

7. Wigley FM, Wise RA, Miller R, Needleman BW, Spence RJ. Anticentromere antibody as a predictor 
of digital ischemic loss in patients with systemic sclerosis. Arthritis Rheum 1992;35:688-93. 

8. Harrison BJ, Silman AJ, Hider SL, Herrick AL. Cigarette smoking: a significant risk factor for digital 
vascular diseases in patients with systemic sclerosis. Arthritis Rheum 2002:46:3312-16. 

9. Thompson AE, Shea B, Welch V, Fenlon D, Pope JE. Calcium-channel blockers for Raynaud's phe- 
nomenon in systemic sclerosis. Arthritis Rheum 2001;44:1841-7. 

10. Herrick AL. Treatment of Raynaud's phenomenon - update, new insights and developments. Curr 
Rheumatol Rep 2003:5:168-74. 

11. Raynaud's Treatment Study Investigators: Comparison of sustained-release nifedipine and tempera- 
ture biofeedback for treatment of primary Raynaud phenomenon. Results from a randomized clini- 
cal trial with 1-year follow-up. Arch Intern Med 2000;160:1101-8. 

12. Wigley FM, Wise RA, Seibold JR, et al. Intravenous iloprost infusion in patients with Raynaud phe- 
nomenon secondary to systemic sclerosis. A multicenter, placebo-controlled, double-blind study. 
Ann Intern Med 1994;120:199-206. 

13. Tomaino MM, Goitz RJ, Medsger TA. Surgery for ischemic pain and Raynaud's phenomenon in scle- 
roderma: a description of treatment protocol and evaluation of results. Microsurgery 2001;21:75-9. 

14. Ho M, Veale D, Eastmond C, Nuki G, Belch J. Macrovascular disease and systemic sclerosis. Ann 
Rheum Dis 2000;59:39-43. 



36. Aortofemoral Graft Infection 

Christopher P. Gibbons 



A 66-year-old man, an ex-smoker with hypertension and hypercholesterolemia, 
had undergone a Dacron bifurcated aortic graft and bilateral ureteric stents for 
an inflammatory aortic aneurysm with ureteric obstruction at another hospital 
4 years previously. The left limb of the graft had been anastomosed to the 
common femoral artery and the right limb to the common iliac bifurcation. 
Postoperatively he had suffered a mild groin wound infection, which had healed 
with antibiotics. At follow-up he complained of left calf and thigh claudication. 
On examination, he appeared generally well with a midline abdominal scar and a 
left vertical groin scar. He had good right femoral pulse but an absent left 
femoral pulse. 



Question 1 

What should be the first investigation? 

A. Intra-arterial digital subtraction angiography. 

B. Duplex ultrasound scan of the aortic graft. 

C. "Technetium-labelled leucocyte scan. 

D. CT angiography of the graft. 

E. Erythrocyte sedimentation rate. 

A duplex scan showed an occluded left limb of the aortic graft with patent 
common femoral arteries. There was no evidence of any stenosis of the left common 
femoral artery but a perigraft fluid collection was noted around the intra-abdominal 
portion of the graft. 



323 



324 Vascular Surgery 




Fig. 36.1. CT scan of aortic graft showing fluid and a gas bubble around the graft. 



Question 2 

What further investigations should be performed? 

A. CT scan of the graft. 

B. Digital subtraction angiography. 

C. "Technetium-labelled leucocyte scan. 

D. Erythrocyte sedimentation rate. 

E. Aspiration of the collection. 

A CT scan confirmed the presence of fluid and gas around the intra-abdominal 
portion of the graft and the occlusion of the left limb, indicating graft infection (Fig. 
36.1). Digital subtraction angiography (Fig 36.2) confirmed the occluded left limb of 
the aortic graft and showed a stenosis at the origin of the right graft limb, presum- 
ably as a result of external compression. Aspiration of the perigraft collection would 
have allowed preoperative bacterial culture but was considered to be too difficult to 
perform safely. 

Question 3 

Having confirmed the presence of infection what is the best option for manage- 
ment? 

A. Prolonged antibiotic therapy. 

B. Drainage of the perigraft pus under anaesthesia. 

C. Insertion of gentamicin beads. 



Aortofemoral Graft Infection 325 




Fig. 36.2. Intra-arterial digital subtraction angiography (DSA) of the aortoiliac region. 

D. Excision of the graft. 

E. Excision of the graft with in situ replacement with an antibiotic bonded graft. 

F. Graft excision and extra-anatomical prosthetic bypass. 

G. Graft replacement with autologous vein. 
H. Graft replacement with an aortic allograft. 

In situ replacement with autologous vein was chosen because of the reduced risk 
of persistent infection. 

Question 4 

Which autologous veins may be used for aortoiliac or aortofemoral graft replace- 
ment? 

A. Long saphenous vein. 

B. Cephalic vein. 

C. Femoropopliteal vein. 

D. Iliac vein. 

Femoropopliteal vein was used as it is ideally suited to supra-inguinal graft 
replacement as it is relatively thick-walled, is of adequate diameter and has 
sufficient length. 



326 Vascular Surgery 

Question 5 

What further preoperative investigations should be performed? 

A. Plain abdominal X-ray. 

B. Bone scan. 

C. MRI scan of the abdomen. 

D. Duplex scan of the femoral veins. 

E. Repeat abdominal ultrasound scan. 

A duplex scan of the femoral veins confirmed that they were patent and of ade- 
quate calibre. The patient was operated on electively on the next available operating 
list. 



Question 6 

What other preoperative preparations should be undertaken? 

A. Routine full blood count. 

B. Urea and electrolyte estimation. 

C. Chest X-ray and electrocardiogram (ECG). 

D. Compression stockings. 

E. Subcutaneous heparin. 

F. Combination antibiotic therapy. 

Routine blood investigations, chest X-ray and ECG were all performed, and in 
view of the magnitude of the procedure, echocardiogram and lung function tests 
were also ordered. They were all satisfactory. Because the bacteriology of the infec- 
tion was not known preoperatively in this patient, intravenous combination 
antibacterial therapy with teicoplanin, ciprofloxacin, co-amoxiclav and metronida- 
zole was given immediately before surgery. 



Question 7 

How should the operation be performed? 

A. Laparotomy, excision of the aortic graft, harvesting of the femoral veins and 
graft replacement. 

B. Harvesting of femoral veins followed by laparotomy, excision of the infected 
graft and replacement with femoral vein. 

C. Laparotomy and exposure of the infected graft, then femoral vein harvest fol- 
lowed by graft replacement. 



Aortofemoral Graft Infection 



327 



The anaesthetised patient was catheterised, prepared and draped so that the 
abdomen and both legs were exposed. First, both superficial femoral veins were 
simultaneously dissected out by two operative teams and the branches divided 
between clips from the profunda femoris vein to the knee joint. The femoral veins 
were left in situ whilst the abdomen was opened, exposing the graft and obtaining 
control of the proximal infrarenal aorta and the right common iliac bifurcation. The 
graft was encased in fibrous tissue, making dissection difficult and hazardous. The 
underlying prosthesis showed poor tissue incorporation and there was a localised 
abscess between the graft and the duodenum, which was evacuated and cultured. 
The left groin was exposed, obtaining control of the common femoral artery, its 
branches and the profunda femoris artery. After systemic heparinisation, the vessels 
were clamped and the infected graft excised and sent for culture. The graft bed was 
washed repeatedly with povidone iodine and hydrogen peroxide. One femoral vein 
was excised, reversed and inserted end-to-end from the infrarenal aorta to the right 
common iliac artery bifurcation using 4/0 polypropylene sutures. Size discrepancy 
at the aortic anastomosis was overcome by "fishmouthing" the end of the vein to 
prevent the angulation associated with spatulation (Fig. 36.3). The other femoral 
vein was reversed and anastomosed end to side to the intra-abdominal part of the 
vein graft and to the left common femoral artery (Fig. 36.4). Both veins were led 




Fig. 36.3. "Fishmouthing" the femoral vein to equalise diameter with the aorta. 



328 



Vascular Surgery 




Fig. 36.4. Femoral vein reconstruction from the infrarenal aorta a to the right iliac b and left common femoral 
artery c. The right ureter d overlies the right limb of the graft. 

through a fresh tunnel and surrounded by greater omentum to avoid contact with 
the bed of the infected graft. The arterial anastomoses were covered by gentamicin- 
impregnated collagen foam and the wounds were closed with suction drainage. 
Antibiotic prophylaxis and low molecular weight heparin were continued postoper- 
atively. Despite the copious pus around the graft, no organisms were grown in the 
laboratory. Combination antibiotic therapy was stopped after 7 days but co-amoxi- 
clav was continued empirically for a further 5 weeks. 

Question 8 

If the patient had presented with an exposed prosthetic graft in the groin how would 
this have altered management? 

A. Prolonged antibiotic therapy. 

B. Use of vacuum dressings. 

C. Simple coverage with a muscle flap without graft replacement. 

D. Addition of a muscle flap to graft replacement with autologous vein. 

Exposed grafts present a difficult problem for achieving skin closure and the 
addition of a local muscle flap in the groin to graft replacement with autologous 
(femoropopliteal) vein is the most certain method of cure. The author's preference 
is a rectus femoris flap for this (Fig. 36.5). 



Question 9 

What complications might occur following this operation? 
A. Anastomotic haemorrhage. 



Aortofemoral Graft Infection 



329 





Fig. 36.5. Rectus femoris muscle flap to cover a femoral anastomosis, a After mobilisation of the rectus femoris 
muscle (the femoral anastomosis is obscured by a sheet of gentamicin-impregnated collagen foam), b The 
muscle now overlies the anastomosis. 



B. Graft rupture. 

C. Femoral deep vein thrombosis (DVT), 

D. Limb swelling. 

E. Seroma in the thigh. 

F. Intestinal obstruction. 

G. Wound infection. 



330 Vascular Surgery 

The patient had the most common complication of this operation, which was a 
large seroma in the left thigh and a smaller one in the right thigh despite prolonged 
suction drainage. These were aspirated repeatedly and resolved after 3 weeks, 
although in other cases reoperation and reinsertion of a drain may be required. 
Intestinal obstruction is no more common following this operation than other 
abdominal procedures. 

The patient otherwise made a good recovery and was discharged from hospital at 
14 days. The graft and pus swabs were sterile so most of the antibiotics were 
stopped at 1 week but the co-amoxiclav was continued for 6 weeks on empirical 
grounds. 



Question 10 

What routine follow-up investigations should be performed? 

A. Abdominal CT scan. 

B. Abdominal ultrasound. 

C. Graft duplex scans. 

D. Abdominal X-ray. 

E. Erythrocyte sedimentation rate (ESR). 

Routine 3-monthly duplex scans were performed over 1 year for vein graft sur- 
veillance. The patient remains well without further intervention at 5 years. 



Commentary 



Aortic graft infection is thankfully rare, occurring in 1-5 per cent of reconstructions 
[1], but is one of the most feared complications in vascular surgery because of its 
high mortality and morbidity [2]. In a UK multicentre audit of 55 graft infections 
31 per cent died, 33 per cent underwent amputation and only 45 per cent left hospi- 
tal alive without amputation [3]. If left untreated there is a high risk of graft occlu- 
sion and anastomotic haemorrhage, which may lead to aorto-enteric fistula. Prompt 
curative treatment is therefore wise in patients sufficiently fit to withstand major 
surgery. 

Graft infection may present at any time from a few days to many years after 
surgery. It can follow a wound infection, particularly in the groin where wound 
breakdown may result in exposure of the graft, or it may present later with a peri- 
graft fluid collection or sinus at the femoral anastomosis. Infection of wholly intra- 
abdominal grafts may present with backache and fever but more often remain 
undetected until anastomotic haemorrhage or graft thrombosis occurs. Duplex 
ultrasound scanning or CT angiography is wise in all cases of graft thrombosis not 
only to confirm the occlusion but also to demonstrate any perigraft fluid which 
would indicate graft infection. [Q1 : B, D] 

The most common causative organism is Staphylococcus aureus in most series 
and such infections tend to present in the early postoperative period. Methicillin- 



Aortofemoral Graft Infection 331 

resistant strains (MRSA) are said to be particularly virulent and have been associ- 
ated with a high mortality in some series [3-5]. Staphylococcus epidermidis infec- 
tions tend to be less virulent and often present many years later. They produce a 
slime or biofilm around the graft or occasionally thin pus. Isolation of Staph, epider- 
midis is more difficult and may require agitation of the extirpated graft with ultra- 
sound to release it for culture. Other infections are caused by coliforms, Salmonella, 
Serratia, Pseudomonas, enterococci, streptococci or Bacteroides [6]. Gram-negative 
organisms may be more likely to present with anastomotic haemorrhage [7]. In 
many cases no causative organism can be isolated despite obvious infection. 
Possible causes of this are previous antibiotic administration or failure to isolate 
Staphylococcus epidermidis. 

A preoperative diagnosis of graft infection is usually secured by ultrasound fol- 
lowed by CT or MRI. Aspiration of the perigraft fluid may secure a bacteriological 
diagnosis prior to surgery, although in many cases the responsible organism cannot 
be isolated. Fluid is often present in the aneurysmal sac after aortic aneurysm 
replacement and can be seen in smaller quantities around an aorto-bifemoral pros- 
thesis performed for occlusive disease on ultrasound or CT for a few weeks after 
surgery. However, persistence of fluid around an aortic prosthesis for more than 
3-6 months after surgery is highly suggestive of infection. Similarly, perigraft gas 
may be present for up to 10 days after surgery but indicates infection beyond this 
time [9, 10]. 

If a groin abscess develops in relation to an aorto-bifemoral graft, aspiration 
under aseptic conditions in the clinic will confirm the presence of graft infection 
and may provide preoperative bacteriology. Perigraft fluid or gas maybe absent in 
low-grade chronic infection or if a sinus in one or other groin allows the pus to 
escape. Exploration of a sinus under anaesthesia will demonstrate a connection with 
the infected graft and gently passing a bougie alongside the graft will determine 
whether or not the infection is confined to the anastomosis. If there is no sinus or 
perigraft fluid, a "technetium-labelled leucocyte scan may demonstrate increased 
activity over an infected graft [11]. However, this investigation has poor sensitivity 
and specificity and is only useful for chronic graft infection as increased leucocyte 
adherence is demonstrated by most prostheses for up to 6 months after insertion. 
The ultimate diagnosis of graft infection is made at operation by the lack of tissue 
incorporation into knitted Dacron or polytetrafluoroethylene (PTFE) prostheses 
and the presence of perigraft pus from which organisms may be cultured. 
Preoperative angiography is helpful for operative planning by delineating the vascu- 
lar anatomy but adds no useful information about the presence of graft infection. 
[Q2:A,B] 

There are multiple treatment options: Antibiotic therapy may buy time, but is 
rarely curative because the graft acts as a foreign body rendering the responsible 
organisms inaccessible to antibiotics. There have been occasional reports of suc- 
cessful treatment by drainage of the abscess around the graft followed by irrigation 
with antibiotic or iodine solutions [12-14] or implantation of gentamicin-impreg- 
nated beads or foam but these are anecdotal [15, 16]. Simple excision of an aortic 
graft is unwise unless it has already occluded without critical ischaemia as subse- 
quent limb loss or severe lower body ischaemia is likely. Excision of the infected 
graft with debridement and replacement with a rifampicin-bonded or silver- 
impregnated graft has been advocated [17-20] but most would reserve this for 
chronic low-grade infections because of the risk of reinfection of the new graft. 
Despite their in vitro effectiveness [21] encouraging individual series, there is no 



332 Vascular Surgery 

convincing clinical evidence that either rifampicin-bonding or silver-impregnated 
Dacron grafts are less susceptible to reinfection after replacement of infected grafts. 
Moreover, randomised clinical studies have failed to show that either rifampicin- 
bonding or silver impregnation prevents primary infection in vascular grafts, 
although these studies were somewhat underpowered [22, 23]. 

An alternative approach, which avoids direct reimplantation of prosthetic mater- 
ial, is in situ replacement with fresh or cryopreserved aortic allografts. The reported 
results have been variable but in all series there have been instances of early or late 
graft disruption or aneurysm formation particularly with fresh allografts and when 
used for aorto-enteric fistula [24-29]. 

Until the last 5 years, the mainstay of treatment has been excision of the graft 
with extra-anatomical reconstruction. For infected aortoiliac grafts reconstruction 
can be performed with an axillo-bifemoral or bilateral axillofemoral grafts. 
However, in those patients with infected aorto-bifemoral grafts, the lower anasto- 
mosis must be performed at the level of the superficial femoral or popliteal artery to 
avoid placing the new graft in an infected field. Good results can be obtained with 
this approach but there remains a 10-15 per cent risk of graft reinfection [2, 30-32]. 
If this option is used, the extra-anatomic bypass should be performed before graft 
excision to reduce the risk of irreversible limb ischaemia and amputation [2]. 

More recently, Claggett [33, 34] and Nevelsteen [35, 36] independently advocated 
aortic replacement with femoropopliteal veins for infected grafts. Femoropoliteal 
veins are much wider and thicker-walled than long saphenous or arm veins and 
have adequate length, making them ideal for aortoiliac reconstruction. Iliac veins 
are too short and their excision would result in severe limb swelling. Results were 
excellent with reduced mortality, limb loss and reinfection rates. Subsequently 
other authors have confirmed the effectiveness of this approach in eliminating rein- 
fection, with mortality and amputation rates similar to or lower than reports using 
other techniques [37, 38]. This is now recognised as the procedure of choice in most 
situations. The procedure is demanding and may take several hours to perform but 
can be made easier by the use of two or more operative teams working together. 
Femoral veins maybe harvested even after the removal of the long saphenous vein 
but it is generally advised that the profunda femoris vein should be left intact and 
that the popliteal vein should not be removed below the knee joint [39, 40]. Fears of 
venous morbidity from femoral vein harvest have not been borne out in practice 
although Valentine reported an 18 per cent incidence of prophylactic or therapeutic 
fasciotomy for compartment syndrome [41]. However, neither Nevelsteen [42] nor 
the present author has found this necessary. Femoral vein harvest should be the 
initial step in the operation to avoid prolonged abdominal exposure or aortic 
clamping. Partial graft replacement is best avoided, as the remaining graft usually 
requires later replacement [43]. [Q3: E, F, G, H] [Q4: C] [Q7: B] If femoropopliteal aortic 
reconstruction is planned, it is wise to perform a preoperative venous duplex scan 
of the legs to confirm that the femoral veins are patent and of adequate calibre 
(lcm). [Q5:D] 

Whichever technique is used, the importance of adequate debridement, antiseptic 
washouts and drainage cannot be overstressed. Combination antibiotic cover 
(beginning immediately preoperatively) to cover any cultured organism and the 
common pathogens is essential to eliminate infection and prevent catastrophic 
haemorrhage from anastomotic breakdown. Routine preoperative investigations 
such as full blood count, urea and electrolyte estimation, chest X-ray and ECG are 
indicated. Compression stockings are used by some surgeons after femoropopliteal 



Aortofemoral Graft Infection 333 

vein harvest to limit ankle swelling but cannot be used intraoperatively. 
Subcutaneous heparin is used postoperatively but not preoperatively as systemic 
heparinisation is used routinely on aortic clamping. [Q6: A, B, C, F] 

In cases where a prosthetic graft has been exposed in the groin or the tissues 
overlying a femoral anastomosis are deficient, a local muscle flap is wise to protect 
the new femoral anastomosis. [Q8: D] A sartorius rotation flap may be used provided 
the graft does not extend in front of the inguinal ligament as in femoral-femoral or 
axillofemoral grafts [44, 45]. Gracilis and rectus abdominis flaps or omental pedicles 
have also been described [46-51]. A rectus femoris flap is very quick and easy to 
prepare and will cover any femoral anastomosis with ease [38]. In occasional 
localised low-grade graft infections with exposure of the prosthesis in the groin, a 
simple muscle flap maybe successful without excision of the graft but there is a high 
rate of recurrent infection. 

Despite the magnitude of the procedure, graft replacement with femoral vein 
gives excellent results with reported mortalities in the region of 10 per cent, ampu- 
tation rates of 10 per cent and no reinfection. Possible complications include anas- 
tomotic haemorrhage, iliac venous thrombosis, limb swelling, seroma, intestinal 
obstruction and wound infection. Limb swelling usually occurs but is rarely exces- 
sive and is easily controlled by elevation provided the profunda femoris vein is pre- 
served and the popliteal vein is not harvested below the knee. Since the deep veins 
have been excised, lower limb DVT is unlikely. Wound and graft infection or anas- 
tomotic haemorrhage is similarly infrequent provided adequate antibiotic cover is 
used. [Q9: A, D, E, F, G] 

Routine duplex surveillance of the aortofemoral vein grafts is wise as late graft 
stenosis due to intimal hyperplasia is common [38]. [Q10: C] 



References 

1. Seeger JM. Management of patients with prosthetic graft infection. Am Surg 2000;66:166-7. 

2. Yeager RA, Porter JM. Arterial and prosthetic graft infection. Ann Vase Surg 1992;5:485-91. 

3. Naylor AR, Hayes PD, Darke S on behalf of the Joint Vascular Research Group. A prospective audit 
of complex wound and graft infections in Great Britain and Ireland: the emergence of MRSA. Eur J 
Vase Endovasc Surg 2001;21:289-94. 

4. Nasim A, Thompson MM, Naylor AR, et al. The impact of MRSA on vascular surgery. Eur J Vase 
Endovasc Surg 2001;22:211-4. 

5. Murphy GJ, Pararajasingam R, Nasim A, et al. Methicillin-resistant Staphylococcus aureus infection 
in vascular surgical patients. Ann R Coll Surg Engl 2001;83:158-63. 

6. Selan L, Pasariello C. Microbiological diagnosis of aortofemoral graft infections. Eur J Vase Endovasc 
Surg 1997;14(Suppl A):10-12. 

7. Hicks RJC, Greenhalgh RM. The pathogenesis of vascular graft infection. Eur J Vase Endovasc Surg 
1997;14(SupplA):5-9. 

8. Calligaro KD, Veith FJ, Schwartz ML, et al. Are Gram-negative bacteria a contraindication to selec- 
tive preservation of infected prosthetic arterial grafts? J Vase Surg 1992;136:337-45. 

9. Orton DF, LeVeen RF, Saigh JA, et al. Aortic prosthetic graft infections: radiologic manifestations 
and implications for management. RadioGraphics 2000;20:977-93. 

10. Spartera C, Morettini G, Petrassi C, et al. Role of magnetic resonance imaging in the evaluation of 
aortic graft healing, perigraft fluid collection, and graft infection. Eur J Vase Surg 1990;4:69-73. 

11. Liberatore M, Iurilli AP, Ponzo F, et al. Aortofemoral graft infection: the usefulness of 
99m Tc-HMPAO-labelled leucocyte scan. Eur J Vase Endovasc Surg 1997;14(Suppl A):27-9. 

12. Almgren B, Eriksson I. Local antibiotic irrigation in the treatment of arterial graft infections. Acta 
Chir Scand 1981;147:33-6. 

13. Morris GE, Friend PJ, Vassallo DJ, et al. Antibiotic irrigation and conservative surgery for major 
aortic graft infection. Vase Surg 1994;20:88-95. 



334 Vascular Surgery 

14. Voboril R, Weberova J, Kralove H. Successful treatment of infected vascular prosthetic grafts in the 
groin using conservative therapy with povidone-iodine solution. Ann Vase Surg. 2004;18 -372-5. 

15. Nielsen OM, Noer HH, Jorgensen LG, Lorentzen JE. Gentamicin beads in the treatment of localised 
vascular graft infection - long term results in 17 cases. Eur J Vase Surg 1991;5:283-5. 

16. Holdsworth J. Treatment of infective and potentially infective complications of vascular bypass 
grafting using gentamicin with collagen sponge. Ann R Coll Surg Engl 1999;81:166-70. 

17. Bandyk DF, Novotney ML, Johnson BL, et al. Use of rifampicin-soaked gelatin-sealed polyester grafts 
for in situ treatment of primary aortic and vascular prosthetic infections. J Surg Res 2001;95:44-9. 

18. Naylor AR. Aortic prosthetic infection. Br J Surg 1999;86:435-436. Hayes PD, Nasim A, London NJM, 
et al. In situ replacement of infected aortic grafts with rifampicin-bonded prostheses: The Leicester 
experience (1992-1998). J Vase Surg 1999;30:92-8. 

19. Zegelman M, Gunther G. Infected grafts require excision and extra-anatomic reconstruction. Against 
the motion. In: Greenhalgh RM, editor. The evidence for vascular and endovascular reconstruction. 
London: WB Saunders, 2002;252-258. 

20. Batt M, Magne JL, Alric P, et al. In situ revascularization with silver-coated polyester grafts to treat 
aortic infection: early and midterm results. J Vase Surg. 2003;38:983-9. 

21. Hardman S, Cope A, Swann A, et al. An in vitro model to compare the antimicrobial activity of 
silver-coated versus rifampicin-soaked vascular grafts. Ann Vase Surg 2004;18:308-13. 

22. Sardelic F, Ao PY, Taylor DA, Fletcher JP. Prophylaxis against Staphylococcus epidermidis vascular 
graft infection with rifampicin-soaked, gelatin-sealed Dacron. Cardiovasc Surg 1996;4:389-92. 

23. Earnshaw JJ, Whitman B, Heather BP, on behalf of the Joint Vascular Research Group. Two-year 
results of a randomized controlled trial of rifampicin-bonded extra-anatomic Dacron grafts. Br J 
Surg 2000;87:758-59. 

24. Vogt PR, Brunner-LaRocca HP, Lachat M, et al. Technical details with the use of cryopreserved 
arterial allografts for aortic infection: influence on early and midterm mortality. J Vase Surg 
2002;35:80-6. 

25. Verhelst R, Lacroix V, Vraux H, et al. Use of cryopreserved arterial homografts for management of 
infected prosthetic grafts: A multicentre study. Ann Vase Surg 2000;14:602-7. 

26. Noel AA, Gloviczki P, Cherry KJ, Jr, et al. United States Cryopreserved Aortic Allograft Registry. 
Abdominal aortic reconstruction in infected fields: early results of the United States cryopreserved 
aortic allograft registry. J Vase Surg 2002;35:847-52. 

27. Teebken OE, Pichlmaier MA, Brand S, et al. Cryopreserved arterial allografts for in situ reconstruc- 
tion of infected arterial vessels. Eur J Vase Endovasc Surg 2004;27:597-602. 

28. Gabriel M, Pukacki F, Dzieciuchowicz L, et al. Cryopreserved arterial allografts in the treatment of 
prosthetic graft infections. Eur J Vase Endovasc Surg 2004;27:590-6. 

29. Kieffer E, Gomes D, Chiche L, et al. Allograft replacement for infrarenal aortic graft infection: early 
and late results in 179 patients. J Vase Surg 2004;39:1009-17. 

30. Quinones-Baldrich WJ, Hernandez JJ, Moore WS. Long-term results following surgical management 
of aortic graft infection. Arch Surg 1991;126:507-11. 

31. Yeager RA, Taylor LM, Jr, Moneta GL, et al. Improved results with conventional management of 
infrarenal aortic infection. J Vase Surg 1999;30:76-83. 

32. Seeger JM, Pretus HA, Welborn MB, et al. Long-term outcome after treatment of aortic graft infec- 
tion with staged extra-anatomic bypass grafting and aortic graft removal. Surgery 2000;32:451-9. 

33. Clagett GP, Valentine RJ, Hagino RT. Autogenous aortoiliac/femoral reconstruction from superficial 
femoral-popliteal veins: feasibility and durability. J Vase Surg 1997;25:255-70. 

34. Gordon LL, Hagino RT, Jackson MR, et al. Complex aortofemoral prosthetic infections: the role of 
autogenous superficial femoropopliteal vein reconstruction. Arch Surg 1999;134:615-21. 

35. Nevelsteen A, Lacroix H, Suy R. Autogenous reconstruction with lower extremity deep veins: an 
alternative treatment of prosthetic infection after reconstructive surgery for aortoiliac disease. J Vase 
Surg 1995;22:129-34. 

36. Daenens K, Fourneau I, Nevelsteen A Ten-year experience in autogenous reconstruction with the 
femoral vein in the treatment of aortofemoral prosthetic infection. Eur J Vase Endovasc Surg 
2003;25:240-5. 

37. Brown PM, Kim VB, Lalikos JF, et al. Autologous superficial femoral vein for aortic reconstruction in 
infected fields. Ann Vase Surg 1999;13:32-6. 

38. Gibbons CP, Ferguson CJ, Fligelstone L J, Edwards K. Experience with femoro-popliteal veins as a 
conduit for vascular reconstruction in infected fields. Eur J Vase Endovasc Surg 2003;25:424-31. 

39. Coburn M, Ashworth C, Francis W, et al. Venous stasis complications of the use of the superficial 
femoral and popliteal veins for lower extremity bypass. J Vase Surg 1993;17:1005-9. 

40. Wells JK, Hagino RT, Bargmann KM, et al. Venous morbidity after superficial femoral-popliteal vein 
harvest. J Vase Surg 1999;29:282-91. 



Aortofemoral Graft Infection 335 

41. Modrall JG, Sadjadi J, Ali AT, et al. Deep vein harvest: predicting need for fasciotomy. J Vase Surg 
2004;39:387-94. 

42. Nevelsteen A, Baeyens I, Daenens K, et al. Regarding "Deep vein harvest: predicting need for fas- 
ciotomy". J Vase Surg 2004;40:403. 

43. Becquemin JP, Qvarfordt P, Kron J, et al. Aortic graft infection: is there a place for partial graft 
removal? Eur J Vase Endovasc Surg 1997;14(suppl A):53-8. 

44. Sladen JG, Thompson RP, Brosseuk DT, Kalman PG, Petrasek PF, Martin RD. Sartorius myoplasty in 
the treatment of exposed arterial grafts. Cardiovasc Surg 1993;1:113-7. 

45. Galland RB. Sartorius transposition in the management of synthetic graft infection. Eur J Vase 
Endovasc Surg 2002;23:175-7. Gomes MN, Spear SL. Pedicled muscle flaps in the management of 
infected aortofemoral grafts. Cardiovasc Surg 1994;2:70-7. 

46. Thomas WO, Parry SW, Powell RW, et al. Management of exposed inguinofemoral arterial conduits 
by skeletal muscular rotational flaps. Am Surg. 1994;60:872-80. 

47. Meland NB, Arnold PG, Pairolero PC, Lovich SF. Muscle-flap coverage for infected peripheral vascu- 
lar prostheses. Plast Reconstr Surg 1994;93:1005-11. 

48. Calligaro KD, Veith FJ, Sales CM, et al. Comparison of muscle flaps and delayed secondary intention 
wound healing for infected lower extremity arterial grafts. Ann Vase Surg 1994;8:31-7. 

49. Illig KA, Alkon JE, Smith A, Rhodes, et al. Rotational muscle flap closure for acute groin wound 
infections following vascular surgery. Ann Vase Surg 2004;18:661-8. 

50. Colwell AS, Donaldson MC, Belkin M, Orgill DP. Management of early groin vascular bypass graft 
infections with sartorius and rectus femoris flaps. Ann Plast Surg 2004;52:49-53. 

51. Morasch MD, Sam AD 2nd, Kibbe MR, et al. Early results with use of gracilis muscle flap coverage of 
infected groin wounds after vascular surgery. J Vase Surg 2004;39:1277-83. 



37. Aortoenteric Fistulas 



David Bergqvist 



A 63-year-old woman, who was a smoker, presented with severe intermittent 
claudication for a couple of years. Her walking distance had decreased gradually 
to about 50-100 m. She had previously been healthy and very active. At investiga- 
tion, she had no femoral pulses and a bilateral ankle brachial index of 0.6. 
Further evaluation with angiography showed an aortic occlusion at the level of 
the renal arteries, and she was reconstructed with an aorto-bi-iliac Dacron graft 
(16x8 mm) after local proximal aortic endarterectomy. The proximal anastomo- 
sis was made end to end, and the iliac end to side. Polypropylene sutures were 
used. The operation was somewhat technically difficult, with the proximal anas- 
tomosis having to be redone; the duration of surgery was 3.5 h, with a blood loss 
of about 800 ml. The immediate postoperative course was uneventful. After 3 
years, the patient had distal septic microembolisation in the left leg with an 
abscess around the left distal graft limb. This was extirpated, the wound was 
drained, and a femoral-femoral cross-over graft was inserted. She was put on 
antibiotics for 6 months. Five years after the aortic operation, she had melaena 
and a decrease in haemoglobin. 



Question 1 

What is the time interval between aortic surgery and the presentation of an aorto- 
enteric fistula? 

A. It usually occurs in the first 48 h following aortic surgery. 

B. It typically presents within the first month following the operation. 

C. It may only occur in the first 5 years following the placement of the aortic syn- 
thetic graft. 

D. It may present at any time during the lifetime of the patient after the placement 
of the synthetic aortic graft. 



337 



338 Vascular Surgery 

The patient was investigated at her primary healthcare centre with gastroscopy 
and colon enema, with negative results. After 2 months, she had melaena again; 
after further melaena 3 months later, she was referred to the hospital. On this occa- 
sion, she also had slight back pain and low-grade fewer. 

Question 2 

What is meant by herald bleeding? 

A. Bleeding where the aetiology cannot be determined. 

B. Small bleeding(s) before a large one from a major artery. 

C. A "warning" bleeding before a fatal one. 

D. A small haematemesis before melaena. 

A gastroscopy showed a very distal duodenal "ulcer" with a green-coloured (bile- 
stained) graft in the bottom (Fig. 37.1). A computed tomography (CT) scan showed 
fluid around the proximal part of the graft, with some gas bubbles. 

Question 3 

How will you rule out the presence of an aortoenteric fistula? 

A. Gastroscopy. 

B. Computed tomography. 

C. Magnetic resonance imaging. 

D. Barium enema and or barium swallow and follow-through. 

E. None of the above. 




Fig. 37.1 . Gastroduodenoscopy showing the Dacron graft in the bottom of an ulceration, the graft being bile 
stained. 



Aortoenteric Fistulas 339 

Question 4 

Which part of the bowel is involved in an aortoenteric fistula? 

A. Duodenum. 

B. Jejunum. 

C. Ileum. 

D. Appendix. 

E. Any of the above could be involved. 

Following a diagnosis of secondary aortoenteric fistula, and with the patient 
being circulatory stable, an axillo-bifemoral Dacron graft was inserted. During the 
same period of anaesthesia, the old aortic graft was extirpated. A duodenorrhaphy 
was made, and the aortic stump, which was about 2 cm below the renal arteries, was 
sutured and covered with omental tissue. 

Question 5 

Which treatment options are not to be recommended? 

A. Stent grafting the anastomosis. 

B. Wait and see if the patient starts bleeding again. 

C. Extirpation of the aortic graft and then an axillofemoral reconstruction. 

D. Axillofemoral reconstruction and then extirpation of the aortic graft. 

E. In situ reconstruction with a new graft. 

The patient recovered and she left hospital after 12 days. After 10 months, she 
had melaena again and was admitted to hospital. Based on her previous history, a 
CT scan was ordered, but she suddenly developed abdominal and back pain, large 
gastrointestinal bleeding - both haematemesis and melaena - and went into shock. 
She died before any treatment could be given. Autopsy showed a blow-out of the 
aortic stump with a fistula to the duodenum and also bleeding into the retro- 
peritoneal space. 



Commentary 



The term "aortoenteric fistula" means a communication between the aorta and 
some part of the gastrointestinal tract. It is rarely primary; most often, it is seen sec- 
ondary to reconstructive vascular surgery, that is, secondary aortoenteric fistula. In 
the majority of cases, it is seen after aortic graft insertion. It has also been reported 
after stent-grafting [1] and after simple aortic suture [2]. The majority (about 75 per 
cent) of fistulas involve the duodenum, but any part of the gastrointestinal tract 
may be involved. [Q4: E] A few patients have more than one fistula. In exceptional 



340 Vascular Surgery 

cases, it can occur after other abdominal operations or radiation treatment. It is an 
emergency situation and should always be suspected in patients with an aortic 
reconstruction presenting with gastrointestinal bleeding. It can occur at any time 
postoperatively, which means that the patient with an aortic graft is at risk for their 
entire lifetime of developing a fistula. Thus, the true incidence of this condition 
cannot be established until all patients in a risk population have died. The longest 
interval reported is 18 years. Often, there is a delay of several years. [Q1: D] During a 
period of 21 years in Sweden, there are indications that the incidence has decreased 
to around 0.5 per cent after abdominal aortic operations [3]. 

Two factors have been considered to be of major aetiological importance: 
mechanical stress from the pulsating graft, which is in continuous contact with 
the intestine, and the presence of a low-grade infection. In patients with an aor- 
toenteric fistula, there is often a history of complicated and troublesome 
primary graft operation or infectious complications in the postoperative course. 
The three most common findings at surgery are suture line contact with the 
bowel, pseudoaneurysm rupturing into the intestine, and graft body erosion of 
the intestine. To avoid complications, atraumatic surgical technique is impor- 
tant, avoiding bowel trauma and large haematomas. The surgeon should always 
try to cover the graft to avoid direct contact between the graft and the bowel. 

The main symptom is gastrointestinal haemorrhage, which can range from mild 
melaena with anaemia to a profuse, immediately fatal haematemesis. Often, this 
massive bleeding is preceded by small herald bleedings, which are an important 
warning symptom. [Q2: C] About half of the patients also have septic symptoms of 
varying severity. In some patients, septic symptoms dominate, and the bleeding 
may even be occult. 

There is often a long delay between onset of the symptoms and final diagnosis. In 
some patients with a large initial bleeding, the diagnosis is established at autopsy. 
The cardinal importance of a high degree of clinical suspicion for obtaining a 
correct diagnosis must be emphasised. Unfortunately, there is no specific diagnostic 
test. At gastroduodenoscopy, it is important to scrutinise the whole duodenum 
down to the ligament of Treitz. Observation of a bile-stained graft is obviously 
pathognomonic. Endoscopy is also important to reveal other sources of bleeding. 
CT, magnetic resonance and angiography may be helpful in showing pseudoa- 
neurysm or fluid outside the graft, sometimes with gas in it. Conventional radiolog- 
ical methods for gastrointestinal examination are rarely helpful. One great problem 
is that the absence of abnormalities does not exclude the diagnosis. Exploratory 
laparotomy is indicated in patients with massive bleeding or where diagnostic 
efforts have been negative and the patient is still bleeding. [Q3: E] 

The management is difficult. Total removal of all old graft material and revascu- 
larisation seems to give the best results [4]. Just closing the fistula locally always 
leads to recurrence and the mortality is close to 100 per cent and cannot be recom- 
mended [5]. It seems optimal to start with an extra-anatomical revascularisation of 
the extremities and thereafter removal of the graft. Some authors recommend a 
delay of a few days between the two procedures [6]; this is possible when the haem- 
orrhage is under control. In emergency situations, an abdominal exploration with 
closure of the fistula and graft removal is vital, but this may lead to delayed revascu- 
larisation with profound limb ischaemia. When the graft is removed, the problem is 
how to deal with the aortic stump, which must be closed, preferably with double 
sutures. This may, however, not be possible if the distance to the renal arteries is too 
short. The stump is preferably covered with some vascularised tissue, and most fre- 



Aortoenteric Fistulas 341 

Table 37.1 . Surgical treatment options for aortoenteric fistula 



Extra-anatomic bypass with resection of the infected prosthesis 

Staged 

Simultaneous 
Resection with in situ reconstruction 

Antibiotic (rifampicin)-soaked graft with omental wrap 

Homograft 

Autologous vein 



quently an omental pedicle has been used. Some authors advocate removal of the 
graft and an in situ reconstruction with expanded polytetrarTuoroethylene (ePTFE) 
graft or an antibiotic-bonded Dacron graft (often with rifampicin) [7, 8] or in situ 
autologous vein [2, 9] Table 37.1 summarises the treatment options for the surgical 
management of aortoenteric fistula. [Q5: B] 

The prognosis is poor, with a high postoperative mortality, often several compli- 
cations should the patient survive, and a risk of aortic stump blow-out, which very 
few patients survive. Results have improved over recent years, but aortoenteric 
fistula is still a very serious and challenging complication. The 5 -year survival rate is 
50-60 per cent [3,7]. 



References 

1. Norgren L, Jernby B, Engellau L. Aortoenteric fistula caused by a ruptured stent-graft: a case report. J 
Endovasc Surg 1998;5:269-72. 

2. Moore RD, Tittley JG. Laparoscopic aortic injury leading to delayed aortoenteric fistula: an alternative 
technique for repair. Ann Vase Surg 1999;13:586-9. 

3. Bergqvist D, Bjorkman H, Bolin T, Dalman P, Elfstrom J, Forsberg O. Secondary aortoenteric fistulae - 
changes from 1973 to 1993. Eur J Vase Endovasc Surg 1996;11:425-8. 

4. Nagy SW, Marshall JB. Aortoenteric fistulas. Recognizing a potentially catastrophic cause of gastroin- 
testinal bleeding. Postgrad Med 1993;93:211-12, 215-16, 219-22. 

5. Miiller BT, Abbara S, Hennes N, Sandmann W. Diagnosis and therapy of second aortoenteric fistulas: 
results of 16 patients. Chirurg 1999;70:415-21. 

6. Geroulakos G, Lumley JS, Wright JG. Factors influencing the long-term results of abdominal aortic 
aneurysm repair. Eur J Vase Endovasc Surg 1997;13:3-8. 

7. Hayes PD, Nasim A, London NJ, Sayers RD, Barrie WW, Bell PR, Naylor AR. In situ replacement of 
infected aortic grafts with rifampicin-bonded prostheses: the Leicester experience (1992 to 1998). J 
Vase Surg 1999;30:92-8. 

8. Young RM, Chery KJ, Jr, David PM, Gloviczki P, Bower TC, Panneton JM, Hallet JW, Jr. The results of 
in situ prosthetic replacement for infected aortic grafts. Am J Surg 1999;178:136-40. 

9. Franke S, Voit R. The superficial femoral vein as arterial substitute in infections of the aortoiliac 
region. Ann Vase Surg 1997;11:406-12. 



38. The Optimal Conduit for Hemodialysis 
Access 

Frank T. Padberg Jr, Robert W. Zickler and 
Joseph M. Caruso 



A 42-year-old type 1 diabetic of normal weight has recently progressed to chronic 
renal disease. Insulin-dependent diabetes mellitus (DM) has been managed by 
the same primary care physician for the preceding 12 years; glucose control has 
never been a problem in this cooperative and well-educated individual. The renal 
failure was initially managed with appropriate adjustments to diet and medica- 
tions; the presumptive diagnosis is diabetic nephropathy. Recent laboratory tests 
demonstrate a creatinine of 4.1, a blood urea nitrogen of 94, a potassium of 4.8, 
mild proteinuria, and a creatinine clearance of 20 ml/min. 



Question 1 

At this juncture the physician's most appropriate course of action is: 

A. Refer the individual to a surgeon for hemoaccess. 

B. Refer the individual to a nephrologist to refine diagnosis and initiate specialty 
care. It is not time to initiate dialysis. 

C. Refer the individual to a nephrologist who will refine diagnosis, and determine if 
there is a reversible cause for the renal insufficiency. 

D. Refer the individual to a nephrologist who will evaluate the etiology of the renal 
insufficiency and determine if there is a reversible cause. If not, a surgeon 
skilled in the construction of durable hemoaccess should be consulted. 

E. Refer the individual to a nephrologist to commence dialysis with a central 
venous catheter. 

Question 2 

A nephrology work-up finds no reversible cause and the patient's immune status 
precludes any further consideration of transplantation. The patient is referred for 

345 



346 Vascular Surgery 

construction of a hemoaccess. The most appropriate action is to perform a clinical 
vascular examination with specific attention to: 

A. The pedal pulses and examination of the foot; extensive arterial occlusive 
disease is common in diabetic patients and infection would complicate any 
hemoaccess procedure. 

B. The radial pulses and superficial venous anatomy. Book the operating room and 
proceed to construct an access in the upper extremity, guided by your clinical 
examination. 

C. The radial pulses and superficial venous anatomy supplemented by a duplex 
ultrasound (DU) study. Book the operating room and proceed to construct an 
access in the upper extremity guided by these findings. 

D. Immediate hemoaccess placement. Simultaneous placement of an arteriovenous 
fistula and a central venous catheter. 



Question 3 

Preoperative duplex ultrasound examination should include all except one of the 
following: 

A. Both upper extremities. 

B. Size and location of the arteries. 

C. Location of the brachial bifurcation. 

D. Assessment of the axillary and subclavian veins. 

E. At least one lower extremity. 

F. Size and location of the superficial veins. 

G. Evaluation of the superficial veins for evidence of prior scarring. 

The patient is right hand dominant. Non-invasive examination demonstrated the 
findings given in the caption to Fig. 38.1. 

Question 4 

Which procedure would be the best option for this individual? 

A. Left brachial to basilic transposition arteriovenous fistula in the arm. 

B. Right radial to basilic transposition arteriovenous fistula in the forearm. 

C. Left brachial to median antecubital vein forearm loop graft (PTFE). 

D. Left internal jugular tunneled, cuffed dual lumen hemodialysis catheter. 

E. Left radial to cephalic arteriovenous fistula. 



The Optimal Conduit for Hemodialysis Access 



347 




Fig. 38.1. Duplex ultrasound: The patient is right hand dominant. Non-invasive examination demonstrated the 
following findings. 

Right: Cephalic (diameter 3.3 mm) and basilic (diameter 3.5 mm) veins course through both the forearm and 
upper arm to their junctions with the axillary and brachial veins respectively; however, both superficial forearm 
veins demonstrate post-thrombotic changes in the forearm. The brachial artery (diameter 4.2 mm) bifurcates 
into a radial (diameter 2.8 mm) and ulnar (diameter 2.7 mm) artery 3 cm below the antecubital crease; the 
palmar arches are intact. The deep venous structures are normal from the forearm veins through visualization of 
the axillary and subclavian veins. 




/.eft/The basilic vein is post-thrombotic and thickened in the forearm; it has a normal 3.5-mm diameter lumen 
just below the elbow continuing into its junction with the brachial vein at mid-humerus. The cephalic vein 
(diameter 3.5 mm) has a normal luminal surface, is superficial, communicates with the proximal basilic at the 
antecubital junction, and remains patent into its junction with the axillary vein. The brachial artery (diameter 4.2 
mm) bifurcates into a radial (diameter 2.8 mm) and ulnar (diameter 2.7 mm) artery at mid-humerus; the palmar 
arches are intact. The deep venous structures are normal from the forearm veins through visualization of the 
axillary and subclavian veins. 



348 Vascular Surgery 

Question 5 

Which of the following best describes when this new hemoaccess is considered 
mature enough to begin puncture for hemodialysis? 

A. The wound is securely healed, the sutures have been removed, and there is a pal- 
pable thrill. 

B. The wound is securely healed, the sutures have been removed, and there is a pal- 
pable thrill. At 2 weeks, a duplex examination demonstrates unobstructed flow, 
but the walls of the conduit appear to be relatively thin. 

C. The wound is securely healed, the sutures have been removed, and there is a pal- 
pable thrill. At 8 weeks, a duplex examination demonstrated that there was 
unobstructed flow and the walls of the conduit have thickened measurably. 

D. The wound is securely healed, the sutures have been removed, and there is a pal- 
pable thrill. At 6 weeks, a duplex examination demonstrates an equal volume of 
flow through both the fistula vein and a large branch vein at the site of the thrill. 

E. Two weeks. 

Your initial hemoaccess has functioned well for 6.4 years, but the hemodialysis 
staff has noted increasing difficulty obtaining adequate flows for the external 
machine circuit; arterial pressures were low at 70 mm Hg and venous pressures ele- 
vated to 350 mm Hg. You are asked to consider revision or a new hemoaccess. 

A new duplex examination demonstrates progressive stenosis of the distal radial 
artery, and multiple sites of localized thrombosis extending into the upper arm 
cephalic vein. With the exception of the appropriate postoperative changes, the 
remainder of the examination is unchanged from that described in Fig. 38.1. 

Question 6 

Which is the best option to maintain hemodialysis? 

A. Right radial to basilic transposition arteriovenous fistula in the forearm. 

B. Right forearm loop graft (PTFE). 

C. Left forearm loop graft (PTFE). 

D. Left radial (antecubital) to basilic transposition arteriovenous fistula in the arm. 

E. Left internal jugular tunneled, cuffed dual lumen hemodialysis catheter. 

A new hemoaccess is constructed and an excellent thrill achieved. During initial 
maturation, hemodialysis is continued via the original left arm hemoaccess. 
Fortunately, the original left hemoaccess provides sufficient flow for adequate inter- 
val hemodialysis, but 6 weeks later has spontaneously thrombosed. Dialysis using 
the new hemoaccess is successful and the hemoaccess functions well for thrice 
weekly puncture. 

Two years later you are again contacted to evaluate this individual. One year pre- 
viously, an uneventful coronary bypass was performed. Subsequently, following an 



The Optimal Conduit for Hemodialysis Access 



349 





Fig. 38.2. The chest X-ray was taken at the time of the referral for arm edema. 



episode of syncope, and tachyarrhythmia, a permanent defibrillator was installed on 
the left anterior chest wall 2 months ago (Fig. 38.2). 

The patient is complaining of an obviously swollen left arm. 

Question 7 

What is wrong? 

A. The patient's heart failure has worsened from a combination of a fixed heart rate 
and the increased output demanded for the hemoaccess. 

B. Edema is a result of lymphatic disturbance from defibrillator implantation. 

C. The patient is hypercoagulable and has thrombosis of the superior vena cava 
(SVC). 

D. Unilateral swelling results from continuously increasing flow in the hemoaccess 
and enlargement of the arterial anastomosis. 

E. The transvenous electrodes have induced a stenosis or obstruction of the left 
subclavian vein. 



Question 8 



The best treatment for this condition is? 



A. Begin strong diuresis to counter the right heart failure. 

B. Place the arm in a sling and elevate it to reduce the existing edema from the 
operation. The patient is reassured that edema following pacemaker insertion of 
these devices is usually self-limited and will soon resolve. 



350 Vascular Surgery 

C. The defibrillator is removed and replaced in the right subclavian vein. 

D. A fistulagram/venogram is performed. This will determine the etiology of the 
edema and may offer an opportunity for interventional therapy. 

E. A hypercoagulable work-up is obtained. 

All of the interventions aimed at reducing the left arm edema are unsuccessful, 
and the patient is discharged home. After multiple attempts, the dialysis staff reluc- 
tantly admit they are no longer able to reliably cannulate the left arm arteriovenous 
fistula (AVF). You are again asked to consider revision or a new hemoaccess. 

A new duplex ultrasound is obtained. The appropriate postoperative findings are 
noted; otherwise, the relevant arm anatomy is unchanged from the initial survey as 
shown in Fig. 38.1. 

Question 9 

The optimal hemoaccess for this individual now is: 

A. Right forearm loop graft (PTFE). Ligation of left hemoaccess. 

B. Left internal jugular tunneled, cuffed dual lumen hemodialysis catheter. 
Ligation of left hemoaccess. 

C. Right internal jugular tunneled, cuffed dual lumen hemodialysis catheter. 
Ligation of left hemoaccess. 

D. Left femoral to femoral loop graft (PTFE). Ligation of left hemoaccess. 

E. Left femoral tunneled, cuffed dual lumen hemodialysis catheter. 

F. Right brachial-cephalic transposition. 

The left arm symptoms resolve, and the new access functions well for 2.6 addi- 
tional years. However, dilation begins to appear in two sites most commonly used 
for the tri-weekly puncture for hemodialysis. Duplex examination of the larger dis- 
cerns the presence of a large pseudoaneurysm with a 0.6-mm neck communicating 
with the lumen of hemoaccess. The individual reports several recent episodes of 
prolonged difficulty achieving hemostasis after removal of the access needles. 
During duplex interrogation, a thrombotic plug is dislodged. Pulsatile bleeding 
ensues, which is controlled with 30-45 minutes of direct compression. 

Question 10 

The best treatment option at this time is: 

A. Ligation of the hemoaccess. 

B. Revision by primary closure of the pseudoaneurysm. 

C. Revision with placement of an interposition, prosthetic segment. 

D. Removal of the hemoaccess. 



The Optimal Conduit for Hemodialysis Access 351 

E. Continued, but close, observation of the patient with treatment initiated if the 
bleeding recurs. 

After loss of the above hemoaccess, a new autogenous AVF was available for con- 
struction in the right arm, which remained functional until the patient's demise 3 
years later. 



Commentary 

Question 1 

The first order of business is to determine whether the individual has a reversible 
condition such as obstructive uropathy, drug-induced acute tubular necrosis, or 
another nephrotoxic condition. Commencement of hemoaccess would be unneces- 
sary at this time since the individual has minimal symptoms; however, the degree of 
renal insufficiency is sufficient to predict that it will likely be required within several 
months to a year. Since some access procedures require several months before they 
are usable, an experienced surgical specialist should be contacted to construct the 
hemoaccess, if the nephrologist confirms that the individual has chronic progressive 
renal insufficiency [1]. [Q1: D] 

Urgent or immediate hemodialysis is not indicated and because of the associated 
morbidity, catheter placement is contraindicated in the absence of acute renal 
failure. Likewise, it is inappropriate to refer directly to the surgical specialist 
without determining the cause of the renal insufficiency, whether it is reversible, 
and whether the individual should be placed on the eligibility list for transplanta- 
tion. Simply referring the individual to the nephrologist is not wrong, but the best 
option includes the diagnostic evaluation, management of treatable etiologies, and 
consideration for hemoaccess assuming that commencement of hemodialysis is 
imminent within several months to a year. It is clear that early nephrology consulta- 
tion is of benefit, from the perspective of directing appropriate therapy [2-5]. 
Likewise, early consideration of hemoaccess options facilitates preservation of vas- 
cular assets and reduces the incidence of catheter placement and the subsequent 
morbidities. 

Question 2 

The complete clinical vascular examination is an important adjunct to surgical 
planning and may direct the surgeon to either upper extremity; clearly the non- 
dominant upper extremity is preferred, unless a preferred access option is only 
available in the dominant extremity. While reliance on the clinical examination 
alone may be accurate in many cases, experience currently suggests that valuable 
information is contributed by the non-invasive ultrasound survey [6-9]. 

While reliable duplex ultrasound (DU) examinations may not be available in all 
practice situations, it does provide the "best" option by decreasing the likelihood of 
unsuccessful operations while increasing the options for autologous conduit. 
Additional information from the duplex examination may reveal proximal vein 
occlusion, visible superficial veins which are post-phlebitic, arterial abnormalities 



352 Vascular Surgery 

(location of the brachial bifurcation, occlusive disease, inadequate palmar arch col- 
lateralization, large branch veins, and relative size of the arteries and veins). 

An added reason for preoperative DU survey is that the diabetic population is the 
group most likely to harbor asymptomatic upper extremity arterial occlusive 
disease. It is unknown whether this is the reason, but functional patency of hemoac- 
cess is usually reduced in the diabetic population [8, 10, 11]. In obese patients the 
lack of visible superficial veins maybe countered by duplex examination; the depth 
of the veins precludes clinical identification and may mask prior thromboses. The 
depth of an otherwise acceptable vein is an important consideration and may 
mandate transposition to a more superficial site. Failure to identify superficial 
upper extremity veins on clinical examination is not an acceptable rationale for 
commencing access at a lower extremity site; even if no superficial veins are avail- 
able, a prosthetic graft can usually be constructed between the artery and one of the 
deep veins in the upper extremity. 

As before, there is still no indication for immediate dialysis, so that placement of 
a catheter at the time of the permanent access is not indicated in this clinical sce- 
nario. [Q2: C] 

Question 3 

The list includes all of the usual information needed for proper operative planning. 
The goal is to construct the most durable hemoaccess from autogenous tissue. 
When acceptable options exist only in the dominant upper extremity, it is selected; 
thus, both arms should be studied. 

Adequate arterial inflow is essential for the fistula or graft to function properly. 
Failure was universal with an arterial diameter <1.6 mm in one study [9]. Although 
upper extremity atherosclerotic occlusive disease is uncommon in the arm, diabet- 
ics are the group most likely to have diseased arteries, and thus it should be consid- 
ered for this patient. While all vascular laboratories may not subscribe to this 
position, no palpable pulses were described in the core scenario such that some 
reassurance is needed regarding adequate arterial flow. 

A key function of the preoperative duplex examination is to determine the 
acceptability of the superficial vein network of the forearm and arm. In addition to 
location and diameter, identification of large branches, occluded segments, scar- 
ring, other post-thro mbotic changes, and depth of vein below the skin are all critical 
to success. Failure was also universal when preoperative DU identified stenotic vein 
segments [9]. Vein diameters of less than 2.5-3 mm are generally considered unac- 
ceptable, but since there is little data to support this recommendation, the reporting 
standards did not incorporate a recommendation for a minimum venous diameter 
[7]. Complete evaluation of superficial upper extremity veins should include the 
forearm basilic vein [6, 12]. Transposition of the basilic vein is usually necessary 
whether in the forearm or upper arm, and any vein that is too deep (greater than 
0.5-1.0 cm) may need to be transposed before anastomosis. 

Central vein stenoses or occlusions are usually due to prior central vein catheter- 
izations, but the surgeon should also be wary of transvenous wires from implanted 
pacemakers or defibrillators [1, 13, 14]. Forty percent of patients with known sub- 
clavian vein catheterization had moderate to severe subclavian vein stenoses that 
were clinically silent [15]. 

In the absence of an autogenous option, the surgeon should still be informed 
regarding the best location for the first graft. The anatomic variant of a high 



The Optimal Conduit for Hemodialysis Access 353 

brachial bifurcation occurs in -10 percent of individuals. This anatomic variant 
may preclude placement of a prosthetic graft at a given site, but should have little 
adverse effect on an autogenous AVF. 

Routine evaluation of the lower extremity is unnecessary, but maybe considered 
when upper extremity sites have been completely exhausted. [Q3: E] 

Question 4 

This is a complex issue and the correct answer [Q4: E] is derived from a combination 
of experience and the recommendations of the United States Kidney Dialysis 
Outcomes Quality Initiative (K-DOQI). The best answer is a combination of the 
"best" choices summarized from the principles of all autogenous, most distal, non- 
dominant extremity. Thus, since almost all options are really open for this individ- 
ual, the non-dominant, radiocephalic AVF is the best first choice; a potential 
collateral benefit is communication with (and arterialization of) the proximal basilic 
vein. Recognition of problems causing failure to mature are the major impediment 
to wider utilization of this modality [1, 8-10, 16]. Failure to mature a forearm AVF 
may occur in 34-53 percent, and maybe less attractive in the elderly, the diabetic, 
and female patients [8, 9]. 

The proximal options of brachial and cephalic anastomoses and transpositions 
have experienced a higher incidence of arterial steal and ignore the basic principle 
of progression from distal to proximal [1]. The forearm loop graft was equated with 
the proximal brachial transposition by K-DOQI, but current initiatives more 
emphatically encourage autogenous fistula [5, 16]. 

Catheter access is to be avoided if at all possible, and is clearly not indicated in 
this situation; multiple autogenous options are available and commencement of 
dialysis is not emergent [4, 5, 13, 14]. 

A left radiocephalic AVF serves the dual purpose of increasing lifetime site 
options, and allows the distal AVF to develop more proximal veins for subsequent 
autogenous hemoaccess options. A forearm vein transposition has been touted as a 
better option than the radiocephalic AVF but this option was not available, even in 
the dominant extremity, because of post-thro mbotic changes [12, 17]. 

Question 5 

Although this is a common clinical question, informed decisions are difficult since 
there is a paucity of concrete data. There is little data to support any course of action, 
and the correct answer is based upon opinion [1]. [Q5: C] Clearly a healed wound, a 
palpable thrill, and unobstructed flow are essential. Two weeks is generally consid- 
ered early for an AVF, and the minimum recommended interval is 6-8 weeks. For 
prosthetic grafts, 2-3 weeks is usually satisfactory, as long as the edema has resolved 
sufficiently to identify the outline of the graft. Since the functional patency is so low, 
a prosthetic AV graft should not be inserted until dialysis is imminent [1]. 

A period of 6-8 weeks is an arbitrary interval often used in practice and sup- 
ported by K-DOQI. However, if at all possible, a longer interval is preferable, since a 
matured access is more likely to provide durable function. 

A large branch vein within 5 cm of the arteriovenous anastomosis can prevent 
maturation, by diversion and diffusion of fistula flow and should be ligated [9]. 

Thickening of the wall is one of the few indicators of arterialization in the conduit 
walls and would therefore be desirable, but there is no data to support this course of 



354 Vascular Surgery 

action, or to guide the obvious question of how thick? In reality, the progression of 
the individual's renal disease will likely be the best guide. If early referral can be 
achieved, urgent commencement of dialysis becomes a moot question and the 
hemoaccess is ready for use when the time arrives [1]. 

Question 6 

The best option again emphasizes the principles of most distal site, autogenous if 
possible, and in this instance the recruitment of additional vein collaterals from the 
long-standing prior left radiocephalic AVF [17, 18]. [Q6: D] The initial DU examina- 
tion specifically noted the communication from the cephalic feeding the basilic 
system; the proximal forearm basilic has a good size and does not exhibit post- 
thrombotic changes from this point into its termination in the brachial veins. Thus, 
the transposition of the proximal basilic vein, which may already be arterialized, is 
the preferred choice [17, 18]. The arterial inflow for this may be the proximal 
forearm radial artery if the vein length is satisfactory; if not the high bifurcation 
would still make the inflow from the radial the best choice. Even though the diame- 
ter of the artery would be less than the usual brachial diameter, this should not 
present a problem for construction of an autogenous fistula. 

The brachial arterial variant would compromise the inflow to a left forearm loop 
prosthetic graft and even a brachial to axillary prosthetic graft in the arm. 

The right forearm loop graft would be an acceptable alternative, except that there 
are good autogenous alternatives bilaterally. It moves the access to the dominant 
upper extremity and fails to take advantage of the previously arterialized proximal 
venous channels. 

Any right forearm transposition is inappropriate because both veins have evi- 
dence of prior thrombosis. 

Central catheter access is to be avoided if at all possible. Multiple autogenous 
options are available and dialysis can be maintained in the interval with the exist- 
ing, but poorly functioning left radiocephalic AVF. By mobilizing a proximal basilic 
vein segment that has already been exposed to arterialized fistula flow, the time for 
maturation maybe reduced [1]. The arm will need to be observed for possible arte- 
rial steal, in which case the failing AVF may need to be sacrificed and a hemoaccess 
catheter inserted until the new left radiobasilic transposition has matured satisfac- 
torily [13]. 

Questions 7 and 8 

The presence of subclavian vein electrodes in the subclavian vein is the inciting 
factor for subclavian vein thrombosis or stenosis whether for pacing or defibrilla- 
tion. [Q7: E] Symptoms from acute subclavian vein thrombosis are often expressed 
for only a short time in patients without an arteriovenous fistula. Edema will be 
severely and continuously exacerbated from the additional limb blood flow of the 
AVF [13, 14]. Often the edema becomes chronic, and precludes accurate puncture 
of an ipsilateral access. Untreated venous hypertension from this combination may 
produce the typical symptoms of venous stasis: edema, hyperpigmentation and 
even ulceration. Therefore any treatment must include ligation of the access. 

Although lymphatic disruption from an infraclavicular pocket incision is possi- 
ble, this would be so rare as to be remarkable. The electrodes are usually inserted 



The Optimal Conduit for Hemodialysis Access 355 

indirectly and do not require surgical exposure of the vein; this reduces the likeli- 
hood of injury to the lymphatic channels in the axillary- subclavian vascular sheath. 
Localized swelling in the pocket would be a more likely complication than arm 
swelling. 

There is no evidence for hypercoagulability. Since the electrodes pass through the 
SVC, obstruction there is theoretically possible. However, the absence of contralat- 
eral upper extremity edema or a swollen head fails to suggest SVC thrombosis. 
Replacing the device on the right complicates the issue immeasurably. In addition 
to incurring a real risk of SVC obstruction, it also places the remaining (right) 
upper extremity at risk for problems with subsequent hemoaccess. 

The combination of a high flow AVF with probable subclavian vein obstruction 
suggests a rather poor prognosis for the left arm radiobasilic AVF. Thus, investiga- 
tion of the etiology with a fistulagram and venogram is appropriate. [Q8: D] A DU 
should also be obtained, but since central vein visualization is poor it is inadequate 
to confirm the suspected diagnosis. Although unlikely, it is entirely possible that the 
device has nothing to do with the venous obstruction and that the outflow vein of 
the AVF may be stenotic from an intimal hyperplastic response in a different 
anatomic location more amenable to a salvage procedure. If this is a subclavian 
thrombosis, it may be a good opportunity to consider thrombolysis. However, even 
if the vein could be reopened, a subclavian vein angioplasty and/or stent has not 
proven a durable solution in this anatomic site [13]. Finally, thrombolysis carries a 
small but real risk of intracranial hemorrhage, which would be less acceptable 
without a real benefit. Removal of the electrodes would be complicated and risky. 
Unfortunately, if the obstruction is not well collateralized, the left arm should be 
excluded from future access options [1, 13, 14, 18]. 

Question 9 

The best choice is the right forearm loop prosthetic graft. [Q9: A] Either an internal 
jugular (IJ) or femoral catheter site has significant clinical negatives, and fails to 
offer a durable solution in the face of numerous better options. 

The right brachial to cephalic is an attractive option, which is considered equiva- 
lent to the loop graft in the DOQI guidelines; however, as presented in the question, 
ligation of the contralateral symptomatic radiobasilic AVF is not accomplished. 
More importantly, the two proximal transposition options remain available for con- 
struction of subsequent hemoaccess. 

Ligation of the left radiobasilic AVF is essential to control the venous obstructive 
symptoms. While a jugular vein turn-down would offer preservation of the left 
radiobasilic AVF, there is not likely to be sufficient length to reach a non-throm- 
bosed segment of axillary vein. A prosthetic extension to the jugular is another 
alternative. Adequate central outflow from the jugular would need to be assured by 
venography before further consideration of either option [14, 19]. 

Question 10 

Although there is very little data to provide a clear answer to this clinical problem, 
the best choice and DOQI recommendation is prosthetic interposition. [Q10: C] It 
preserves a functioning access in someone who has already lost the use of the con- 
tralateral upper extremity to venous outflow obstruction [1, 20, 21]. 



356 Vascular Surgery 

Ligation solves the bleeding problem but sacrifices the access. Removal may sub- 
sequently be required, but is not essential at this juncture. Revision with primary 
closure of the aneurysm is unattractive since the tissues and graft material are 
friable and usually destroyed by the repetitive puncture. Close observation is 
doomed to fail with a real risk of bleeding and hemorrhage. 

Although prosthetic interposition is the appropriate choice, this option is not 
without complications. Our own experience identified an increased incidence of 
infection, and good material to anastomose may require bypass of lengthy segments 
[22]. Recent introduction of the covered stent is an attractive, but unproven option. 
Percutaneous access, control of the neck of the pseudoaneurysm, and retention of 
hemoaccess function are currently offset by limited clinical data, and high expense 
[13,23]. 

Comment 

The initial use of distal sites, and judicious consumption of the available autoge- 
nous assets facilitated construction of several different hemoaccess sites during this 
patient's 14-year odyssey with hemodialysis. Problems such as these are common 
and require forethought and ingenuity for successful cumulative function and min- 
imization of major complications. 



References 

1. NKF-K/DOQI Clinical Practice Guidelines for Vascular Access: Update 2000. Am J Kidney Dis 
2001:37:S137-81. 

2. Stack AG. Impact of timing of nephrology referral and pre-ESRD care on mortality risk among new 
ESRD patients in the United States. Am J Kidney Dis 2003:41:310-18. 

3. Khan IH. Co-morbidity: the major challenge for survival and quality of life in end stage renal disease. 
Nephrol Dial Transplant 1998:13:S1 76-9. 

4. Powe NR. Early referral in chronic kidney disease: an enormous opportunity for prevention. Am J 
Kidney Dis 2003:41:505-7. 

5. Pisoni RL, Young EW, Dykstra DM, Greenwood RN, Hecking E, Gillespie B, et al. Vascular access use 
in Europe and the United States: results from the DOPPS. Kidney Int 2002:61:305-16. 

6. Silva MB, Hobson RW, Pappas PJ, Jamil Z, Araki CT, Goldberg MC, et al. A strategy for increasing 
use of autogenous hemodialysis access procedures: impact of preoperative noninvasive evaluation. J 
Vase Surg 1998;27:302-8. 

7. Sidawy AN, Gray R, Besarab A, Henry M, Ascher E, Silva M, et al. Recommended standards for 
reports dealing with arteriovenous hemodialysis accesses. J Vase Surg 2002;35:603-10. 

8. Miller PE, Tolwani A, Luscy CP, Deierhoi MH, Bailey R, Redden DT, Allon M. Predictors of adequacy 
of arteriovenous fistulas in hemodialysis patients. Kidney Int 1999;56:275-80. 

9. Wong V, Ward R, Taylor J, Selvakumar S, How TV, Bakran A. Factors associated with early failure of 
arteriovenous fistulae for haemodialysis access. Eur J Vase Endovasc Surg 1996;12:207-13. 

10. Hodges TC, Fillinger MF, Zwolek RM, Walsh DB, Bech F, Cronenwett JL. Longitudinal comparison of 
dialysis access methods: risk factors for failure. J Vase Surg 1997;26:1009-19. 

11. Kalman PG, Pope M, Bhola C, Richardson R, Sniderman KW. A practical approach to vascular access 
for hemodialysis and predictors of success. J Vase Surg 1999;30:727-33. 

12. Choi HM, Lai BK, Cerveira JJ, Padberg FT, Hobson RW, Pappas PJ. Durability and cumulative func- 
tional patency of transposed and non-transposed arterio -venous fistula. In press, J Vase Surg 2003. 

13. Cerveira JJ, Padberg FT, Pappas PJ, Lai BK. Prevention and management of Complications from 
hemoaccess. In: Pearce W, Yao J, Matsumura J, editors. Trends in vascular surgery. Chicago, IL: 
Greenwood Academic, 2004. 

14. Currier CBJ, Widder S, Ali A, Kuusisto E, Sidawy A. Surgical management of subclavian and axillary 
vein thrombosis in patients with a functioning arteriovenous fistula. Surgery 1986;100:25-8. 



The Optimal Conduit for Hemodialysis Access 357 

15. Surratt RS, Picus D, Hicks ME, Darcy MD, Kleinhoffer M, Jendrisak M. The importance of preopera- 
tive evaluation of the subclavian vein in dialysis access planning. AJR Am J Roentgenol 
1991;156:623-5. 

16. Huber TS, Carter JW, Carter RL, Seeger JM. Patency of autogenous and polytetrafluoroethylene 
upper extremity arteriovenous hemodialysis accesses: A systematic review. J Vase Surg 
2003:38:1005-11. 

17. Silva M, Hobson RW, Simonian GT, Haser PB, Jamil Z, Padberg FT, et al. Successful autogenous 
hemodialysis access placement after prosthetic failure: the impact of non-invasive assessment. 
Poster presentation at SVS/AAVS 2000, Toronto, CA. 

18. Haser PB, Padberg FT Jr. Complex solutions for hemoaccess. In: Matsumura J, Pearce W, and Yao 
JST, editors. Trends in Vascular Surgery, 2003;Ch 33. 

19. Puskas JD, Gertler JP. Internal jugular to axillary vein bypass for subclavian vein thrombosis in the 
setting of brachial arteriovenous fistula. J Vase Surg 1994;19:939-42. 

20. Raju S. PTFE grafts for hemodialysis access. Techniques for insertion and management of complica- 
tions. Ann Surg 1987;206:666-73. 

21. Ryan SV, Calligaro KD, Sharff J, Dougherty MJ. Management of infected prosthetic dialysis arteri- 
ovenous grafts. J Vase Surg 2004;39:7378. 

22. Padberg FT, Lee BC, Curl GR. Hemoaccess site infection. Surg Gynecol Obstet 1992;174:103-8. 

23. Lin PH, Johnson CK, Pullium JK, Koffron AJ, Conklin B, Terramani TT, et al. Transluminal stent 
graft repair with Wallgraft endoprosthesis in a porcine arteriovenous graft pseudoaneurysm model. 
J Vase Surg 2003;37:175-81. 



39. Acute Ischaemia of the Upper Extremity 
Following Graft Arteriovenous Fistula 

Miltos K. Lazarides and Vasilios D. Tzilalis 



A 65-year-old woman with end-stage renal disease and insulin-dependent dia- 
betes was admitted for access construction in order to start haemodialysis. There 
was a lack of suitable veins to construct an arteriovenous (AV) fistula, and the 
patient underwent placement of a 6-mm polytetrafluoroethylene (PTFE) AV 
bridge graft between the brachial artery and the axillary vein in the left arm. 



Question 1 

Which of the following is the order of preference for placement of a permanent 
angioaccess in new patients requiring chronic haemodialysis? 

A. (1) A brachio-cephalic AV fistula. (2) A wrist radial-cephalic AV fistula. (3) An 
AV PTFE bridge graft or a transposed brachial-basilic AV fistula. (4) A cuffed, 
tunnelled central venous catheter. 

B. (1) A wrist radial-cephalic AV fistula. (2) A brachio-cephalic AV fistula. (3) An 
AV PTFE bridge graft or a transposed brachial-basilic AV fistula. 

C. (1) A wrist radial-cephalic AV fistula. (2) A transposed brachial-basilic AV 
fistula. (3) A brachio-cephalic AV fistula. (4) An AV PTFE bridge graft. 

Question 2 

Which of the following statements represent advantages of the autologous AV 
fistulas over AV grafts? 

A. Excellent long-term patency once established. 

B. Lower complication rate. 

C. Short lag time from construction to maturation. 

D. Easy to correct surgically when thrombosed. 

359 



360 Vascular Surgery 

Immediately after surgery, the patient complained of numbness of the left hand 
with slight pain of the fingers. On examination the left radial pulse, which had existed 
previously, was absent, and the fingers were cold and cyanotic. Evaluation of the 
patient in the vascular laboratory with forearm Doppler pressure measurement 
revealed an index of 0.3. Interestingly the left forearm segmental pressure index was 
normalised after manual compression of the graft, while the left radial pulse reap- 
peared with this manoeuvre. The evaluation confirmed an obvious haemodynamic 
"steal". The patient's condition deteriorated within a few hours; she developed severe, 
acute, painful weakness of the hand, wrist-drop, and minimal ability to flex the wrist. 

Question 3 

Which of the following statements regarding the incidence of steal after proximal 
access construction is correct? 

A. The incidence of asymptomatic steal after proximal access construction, 
detected in the vascular laboratory, is rare. 

B. Clinically obvious mild ischaemia after the construction of a proximal AV fistula 
occurs in about 10 per cent of cases. 

C. Severe ischaemia necessitating surgical correction complicates 2-4 per cent of 
patients following a proximal AV fistula. 

Question 4 

Which of the following are indications for surgical correction of steal after proximal 
access construction? 

A. Absence of ipsilateral preoperative existed radial pulse. 

B. Severe symptoms (rest pain, paralysis, wrist-drop). 

C. Abnormal forearm segmental pressure index measurement. 

D. Abnormal ipsilateral nerve conduction studies. 

E. Reversal of flow in the distal artery in colour-flow duplex imaging. 

Urgent surgical correction was performed. Under local anaesthesia, a small 
segment of saphenous vein was harvested. The brachial artery was ligated just distal 
to the take-off of the graft. A vein bypass was constructed from the brachial artery 
4-5 cm proximally to the inflow of the graft to a point distal to ligation (Fig. 39.1). 
Complete relief of symptoms occurred immediately postoperatively. The recovery 
of the patient was uneventful. She was discharged home on the third postoperative 
day with a palpable left radial pulse and a patent AV graft. 

Question 5 

Which of the following are acceptable corrective options for limb-threatening steal 
following proximal access construction? 



Acute Ischaemia of the Upper Extremity Following Graft Arteriovenous Fistula 



361 




Fig. 39.1 . a The operation before the creation of the corrective procedure, b The corrective procedure with liga- 
tion of the artery just distal to the AV graft take-off and the venous bypass from a point proximal to the inflow to 
a point just distal to ligation (DRIL procedure). A, brachial artery; AV, axillary vein; BV, basilic vein; G, arteriove- 
nous PTFE graft; VG, vein graft. 



A. Percutaneous transluminal angioplasty. 

B. Flow reduction procedures (banding, plication or tapering of the AV fistula). 

C. AV fistula closure. 

D. The DRIL (distal revascularization interval ligation) procedure. 



Commentary 



Construction of an AV fistula provides a sufficiently superficial arterialised vein that 
can be punctured with ease while its flow is high enough to permit efficient dialysis. 
Post-dialysis compression of a matured thick- wall arterialised vein in order to stop 
bleeding can be obtained readily and reliably. 

The classic first-choice site for an AV fistula construction is between the radial 
artery and the cephalic vein at the wrist, as introduced by Brescia et al. [1] in 1966. If 
the cephalic vein at the wrist or forearm is not usable, then the next alternative is to 
move to the antecubital fossa. The AV fistula can be constructed at this site between 
the median cubital vein and the brachial artery. The superficial cephalic vein pro- 
vides enough length of vein suitable for haemodialysis venipunctures. Alternatively, 
if the cephalic vein is not usable, then the brachial artery can be anastomosed to the 
basilic vein of the upper arm. However, the latter is situated under the deep fascia in 
the arm, and mobilisation and transposition to a subcutaneous new position is 
always necessary. When an autologous AV fistula either at the wrist or the elbow 
cannot be created, then an AV graft using synthetic material bridging an artery and a 
vein in the upper extremity (either forearm or arm) is the next choice. Grafts maybe 
placed in straight, looped or curved configurations. AV grafts and fistulas are created 
in the lower extremity only rarely, as they are prone to infection at this site. 



362 Vascular Surgery 

The order of preference for placement of AV fistulas in patients requiring chronic 
haemodialysis according to dialysis outcomes quality initiative (DOQI) guidelines 
established by the United States Kidney Foundation is [2]: 

1 . A wrist radial-cephalic AV fistula. 

2. An elbow (brachial-cephalic) AV fistula. 

If it is not possible to establish either of these types of fistula, then access may be 
established using: 

3. An AV graft of synthetic material (PTFE grafts are preferred over other syn- 
thetic material). 

4. A transposed brachial-basilic vein fistula. 

Cuffed, tunnelled central venous catheters should be discouraged as permanent 
vascular access. [Q1: B] 

Recognising the superiority of the autologous AV fistulas over grafts, DOQI 
guidelines recommend an aggressive strategy increasing the number of native 
fistulas. DOQI guidelines suggest that autologous AV fistulas should be constructed 
in at least 50 per cent of all new patients electing to receive haemodialysis as their 
initial form of renal replacement therapy [2]. Bridge AV grafts should be reserved 
for those patients whose vein anatomy does not permit the construction of an autol- 
ogous AV fistula [3, 4]. Autologous fistulas, especially distal ones at the wrist, 
present a lower complication rate compared with other access options [5]. A vein 
must be matured before use for vascular access. The time required for maturation 
of an autologous fistula varies among patients. It is not correct to use a fistula 
within the first month after its construction. Premature cannulation may result in a 
higher incidence of haematoma formation, with associated compression of the still 
soft-wall vein, leading to thrombosis. Allowing the AV fistula to mature for 3-4 
months maybe ideal [2]. 

In contrast, PTFE AV grafts need a shorter maturation time and can be used 
approximately 14 days after placement. Within this period, an attempt to cannulate 
the still oedematous arm may lead to graft laceration from inaccurate needle insertion. 
An AV graft may be considered matured when swelling of the subcutaneous tunnel 
has reduced to the point that its course is easily palpable. Additionally, after the first 
two weeks, fibrous tissue formation round the graft is able to seal the holes caused by 
each needle puncture. PTFE grafts are easily thrombectomised, with a reported unas- 
sisted patency following thrombectomy at 6 months close to 50 per cent [6]. In con- 
trast, autologous fistulas when thrombosed are difficult to salvage [2]. [Q2: A, B] 

The reversal of flow after creation of an AV fistula in the distal artery beyond the 
fistula and before the point of entry of collateral vessels has been characterised as 
steal. This is caused by a pronounced pressure drop in the distal artery, while pres- 
sure increases with increasing distance away from the fistula as a consequence of 
inflow from arterial collaterals [7]. Steal occurs in more than 90 per cent of proxi- 
mal AV fistulas - when the arterial anastomosis is at the brachial artery - but in 
most patients, the collateral vasculature is adequate to maintain distal flow, and 
severe ischaemia does not develop in the hand [8, 9]. Clinically obvious mild 
ischaemia from steal occurs in about 10 per cent of patients. The presentation is 
coldness and numbness of the hand, and the symptoms resolve spontaneously 



Acute Ischaemia of the Upper Extremity Following Graft Arteriovenous Fistula 363 

within one month [8]. The term "steal" is used inappropriately for this condition in 
many reports because it means reversal of flow and not any of its potential 
ischaemic sequelae. A wide spectrum of symptoms and signs may occur, however, 
such as paraesthesias and sensory loss, weakened or absent distal pulse, muscle 
weakness and wrist-drop, rest pain usually getting worse during dialysis, muscle 
atrophy and - if left untreated - digital gangrene. The reported rate of steal-induced 
severe ischaemia necessitating immediate surgical treatment is 2.7-4.3 per cent [8, 
9]. In contrast to proximal AV fistulas, the incidence of symptomatic steal following 
distal radiocephalic AV fistulas is rare, at a rate of 0.25 per cent [10]. [Q3: B,C] 

Clinical signs and symptoms of steal syndrome do not differ from those of leg 
ischaemia. Therefore it can be classified according to Fontaine's classification: stage 
I, reduced wrist-brachial pressure index, coldness of the hand or no symptoms; 
stage II, intermitted pain during haemodialysis; stage III, continuous ischaemic rest 
pain; and stage IV, ulceration and necrosis. Stages I and II should be closely 
observed and treated conservatively (e.g. wearing gloves) [11]. 

In most reports, the indication for surgical correction of steal is based on clinical 
grounds only [8, 9]. Low segmental pressure, as measured by Doppler, distal to the 
fistula is not an indication per se for surgical correction of steal. Additionally, 
absence of a radial pulse is a common finding in approximately one-third of 
patients following proximal access creation [9]. A corrective surgical procedure is 
indicated when proven haemodynamic steal causes severe stage III and/or stage IV 
ischaemic symptoms early after access construction (rest pain, paralysis, cyanosis of 
digits, wrist-drop). Mild ischaemic symptoms that persist beyond one month from 
access creation should be observed closely. When these "mild" symptoms are 
present for a long time, there is always a threat of irreversible neurological impair- 
ment, termed "ischaemic monomelic neuropathy". This is a serious and disabling 
complication causing sensorimotor dysfunction without tissue necrosis [12]. 
Abnormal deteriorated nerve conduction studies in the presence of even mild 
ischaemia are an indication for surgical correction of steal [9]. [Q4: B, D] 

Several catheter-based and surgical techniques have been used to correct steal- 
induced ischaemia. Arterial stenoses proximal to the AV fistula are eligible for per- 
cutaneous transluminal angioplasty and may augment blood flow to the periphery 
with relief of symptoms. However, such proximal inflow stenoses contribute to steal 
syndrome in only 20 per cent of patients who have distal extremity ischaemia [13]. In 
the vast majority of cases (80 per cent) steal is caused by discordant vascular resis- 
tance and a poorly formed arterial collateral network. A variety of surgical tech- 
niques have been applied to correct limb-threatening steal including fistula closure 
with simple ligation, various flow reduction techniques (banding, placation or taper- 
ing) and the DRIL (distal revascularization interval ligation) procedure introduced 
by Schanzer et al. [14]. Ligation provides immediate improvement but requires cre- 
ation of a new access. Banding-plication techniques improve distal perfusion, but it 
is difficult to determine the required amount of stenosis to eliminate steal while 
allowing a flow sufficient to sustain patency of the graft. Flow reduction procedures 
are attractive options in high-flow AV fistulas (>1500 ml/min) [15]. In patients with 
normal flow through their AV fistulas often concomitant arteriosclerotic disease 
causes insufficient collateral perfusion. In these cases the DRIL procedure is the 
treatment of choice. With the DRIL a ligature (placed distal to the take-off of the 
graft) eliminates the reversal of flow, while the bypass (from a point proximal to the 
inflow to a point just distal to ligation) re-establishes flow to the limb (Fig. 39.1). 
Recent reports support the efficacy of this technique [16, 17]. [Q5: A, B,C, D] 



364 Vascular Surgery 

References 

1. Brescia MJ, Cimino JE, Appel K, Hurwich BJ. Chronic hemodialysis using venipuncture and a 
superficially created arteriovenous fistula. N Engl J Med 1966;275:1089-92. 

2. NKF-DOQI. Clinical practice guidelines for vascular access: update 2000. Am J Kidney Dis 
2001(Suppll);30:S137-81. 

3. Marx AB, Landerman J, Harder FH. Vascular access for hemodialysis. Curr Probl Surg 1990;27:1-48. 

4. Windus DW. Permanent vascular access: a nephrologist's view. Am J Kidney Dis 1993;21:457-71. 

5. Feldman H, Kobrin S, Wasserstein A. Hemodialysis vascular access morbidity. J Am Soc Nephrol 
1996;7:523-35. 

6. Marston WA, Criado E, Jaque PF, Mauro MA, Burnham SJ, Keagy BA. Prospective randomized com- 
parison of surgical versus endovascular management of thrombosed dialysis access grafts. J Vase 
Surg 1997;26:373-81. 

7. Gordon IL. Physiology of the arteriovenous fistula. In: Wilson SE, editor. Vascular access, principles 
and practice, 3rd edn. St Louis: Mosby, 1996;29-41. 

8. Schanzer H, Scladany M, Haimov M. Treatment of angioaccess-induced ischemia by revasculariza- 
tion. J Vase Surg 1992;16:861-66. 

9. Lazarides MK, Staramos DN, Panagopoulos GN, Tzilalis VD, Eleftheriou GJ, Dayantas JN. 
Indications for surgical treatment of angioaccess-induced arterial steal. J Am Coll Surg 
1998;187:422-6. 

10. Wilson SE. Complications of vascular access procedures. In: Wilson SE, editor. Vascular access, prin- 
ciples and practice, 3rd edn. St Louis: Mosby, 1996;212-24. 

11. Bakran A, Mickley V, Passlick-Deetjen J, editors. Management of the renal patient, clinical algo- 
rithms of vascular access for haemodialysis. Lengerich: Pabst Science Publishers, 2003;90. 

12. Hye RJ, Wolf YG. Ischemic monomelic neuropathy: an under-recognized complication of hemodial- 
ysis access. Ann Vase Surg 1994;8:578-82. 

13. Wixon CL, Mills JL. Hemodynamic basis for the diagnosis and treatment of angioaccess-induced 
steal syndrome. Adv Vase Surg 2000;8:147-59. 

14. Schanzer H, Schwartz M, Harrington E, Haimov M. Treatment of ischemia due to steal by arteriove- 
nous fistula with distal artery ligation and revascularization. J Vase Surg 1988;7:770-3. 

15. Tordoir JH, Dammers R, van der Sande FM. Upper extremity ischemia and hemodialysis vascular 
access. Eur J Vase Endovasc Surg 2004;27:1-5. 

16. Knox RC, Berman SS, Hughes JD, Gentile AT, Mills JL. Distal revascularization-interval ligation: a 
durable and effective treatment for ischemic steal syndrome after hemodialysis access. J Vase Surg 
2002;36:250-55. 

17. Lazarides MK, Staramos DN, Kopadis G, Maltezos C, Tzilalis VD, Georgiadis GS. Onset of arterial 
steal following proximal angioaccess: immediate and delayed types. Nephrol Dial Transplant 
2003;18:2387-90. 



40. Amputation in an Ischaemic Limb 

Mohideen M. Jameel and Kingsley P. Robinson 



A 74-year-old male was admitted to the hospital having sustained a crush frac- 
ture of the third lumbar vertebra while lifting a heavy box. He required opiate 
analgesics for the pain, which radiated to his right leg, and his mobility was 
severely limited. He made little progress in spite of analgesia and physiotherapy; 
hence, he was transferred to a geriatric rehabilitation unit, where he subsequently 
developed ulceration of his left heel. The ulcer was found to be suitable for com- 
munity care, and as his back pain improved he was discharged home. 

However, about a week later, the patient was readmitted to the general hos- 
pital with a swollen and inflamed left lower leg and foot. He was found to have 
a large left heel ulcer, which was 9 cm in diameter with a black necrotic base, 
with cellulitis of the leg and patchy ulceration of the lower leg. The arterial 
pulses were absent distal to the femoral pulse. No Doppler flow was detected in 
the posterior tibial artery, but a pressure of 90 mm Hg was found in the dor- 
salis paedis artery, with an ankle brachial pressure index (ABPI) of 0.4. A 
duplex scan excluded deep vein thrombosis. Treatment with penicillin and 
metronidazole resulted in some improvement. Beta Enterococcus was cultured 
from the ulcer. 

A duplex scan of the arterial system showed little disease in the lower aorta 
and the iliac vessels but an atheromatous plaque with a 30 per cent stenosis in the 
left common femoral artery. The popliteal artery was occluded, but the anterior 
tibial was patent to the foot. A digital subtraction angiogram confirmed the 
above findings and showed that a 2-cm segment of the distal popliteal artery was 
patent and in communication with the anterior tibial artery, which was patent to 
the dorsalis paedis. 

The patient's general condition was very poor. He had smoked very heavily for 
40 years and had a very high alcohol intake. He was found to be unfit for major 
vascular reconstruction. 






Question 1 

Which of the following procedures maybe applied in the treatment of the large heel 
ulcer in our patient? 

A. Chemical sympathectomy. 

B. Debridement. 

367 



368 Vascular Surgery 

C. Subtotal calcanectomy. 

D. Free tissue transfer and microvascular anastomosis. 

E. Amputation. 

Debridement of the heel ulcer was carried out under epidural anaesthesia. The 
patient's general condition deteriorated further due to septicaemia resulting from 
an acute colovesical fistula, which was confirmed by the emission of gas and faecal 
material in the urinary catheter. His serum albumin, which was 29 g/1 on admission, 
fell to 13 g/1. He was treated with antibiotics and nutritional support. He made slow 




Fig. 40.1. Angiogram showing a functioning femoropopliteal vein bypass graft and single-vessel run-off 
through the anterior tibial artery. 



Amputation in an Ischaemic Limb 369 

but steady improvement but was found to have m resistance Staphylococcus aureus 
Methicilline Resistant Staphylococcus Aureus (MRSA) infection in his ulcers. About 
a fortnight after the debridement of his heel ulcer, his left leg became cooler and he 
developed rest pain in his foot. 



Question 2 

In what possible ways can this limb be revascularised? 



Question 3 

What is the role of primary amputation in an ischaemic limb? 

By this time, the patient's general condition had improved somewhat to allow a 
left femoropopliteal bypass graft. A reversed long saphenous vein was anastomosed 
at the common femoral artery and was tunnelled under the deep fascia and anasto- 
mosed to the distal popliteal artery just above the trifurcation (Fig. 40.1). This 
resulted in good revascularisation of the foot. Following the vascular operation, a 
blind defunctioning transverse loop colostomy was formed. 

During the initial care in the intensive care unit, the patient developed a septic 
episode due to chest infection and pleural effusion, which required antibiotics and 
aspiration. During the first few weeks, his general condition improved gradually; in 
particular, the sepsis in his left leg and foot made a dramatic improvement. A 
follow-up duplex scan showed excellent patency and function of the vein graft. The 
heel ulcer remained a large cavity with exposed calcaneal bone. The ankle systolic 
pressure was 120 mm Hg in the dorsalis paedis and 90 mm Hg in the posterior tibial 
artery. A further debridement of the heel ulcer was carried out, with curettage of the 
calcaneum under antibiotic cover. 



Question 4 

What is the role of subtotal calcanectomy in the healing of heel ulcers? 

The possibility of a free tissue transfer and microvascular anastomosis was con- 
sidered. However it was felt that the preferred approach would be to allow the area 
to granulate and then to apply a meshed split skin graft.2. 

Although the sepsis in the leg had improved a great deal, the patient remained 
weak, depressed, reluctant to move, and unable to stand. There was also sloughing 
of the superficial part of the Achilles tendon. Therefore, an amputation became 
inevitable. 



Question 5 

Which of these amputations could be considered in this patient? 

A. Chopart amputation. 

B. Transtibial amputation. 



370 Vascular Surgery 

C. Syme amputation. 

D. Transfemoral amputation. 

E. Lisfranc amputation. 



Question 6 

Which of the following is/are likely to predict good healing of a transtibial 
amputation? 

A. Patent profunda femoris artery. 

B. Thigh doppler pressure greater than 70 mm Hg. 

C. Patent segment of popliteal artery. 

D. Transcutaneous P02 measurement of 20 mm Hg. 

E. Skin blood flow levels greater than 2.5 ml/100 g/min. 

A transtibial level was considered, but it was feared that amputation at this 
level would result in thrombosis in the saphenous vein graft due to the loss of 
run-off. Therefore, amputation at a more distal level was preferred, and hence a 
further angiogram was performed to assess the vascularity of the dorsal flap, 
which showed good patency of the dorsalis paedis artery. Consent was obtained 
for a transtibial amputation, but the possibility of a more distal procedure was 
included. A trial dissection was made for a dorsal flap transmalleolar amputation 
[1]. The instep skin was found to be very well vascularised, and the dorsal flap 
was raised, allowing disarticulation of the ankle joint and division of the Achilles 
tendon and the malleoli (Fig. 40.2). The articular surfaces at the lower ends of the 
tibia and fibula were removed, and the long dorsal flap was reflected posteriorly 
and sutured loosely. The lower leg ulcers were cleaned and dressed in prepara- 
tion for skin grafting. With relief from pain and sepsis, the patient made a 
progressive recovery. The presence of Pseudomonas organisms was a contra- 
indication to skin grafting, and hence priority was given to the patient's rehabili- 
tation. He was again transferred to a geriatric unit for rehabilitation. 



Question 7 

What factors may delay the healing of an amputation stump? 

The perfusion of the residual limb had remained satisfactory and the lower leg 
ulcers showed accelerated granulation and epithelialisation. The ankle stump had 
healed soundly with stable tissue on which the patient could stand (Fig. 40.3). He 
has been fitted with a prosthesis, consisting of a flexible foot, multiaxial ankle joint, 
and a socket extending to the patellar tendon and tibial condyles to take some of 
the weight-bearing load. However, the patient can now stand without the pros- 
thesis and can walk with it using a walking frame. He requires further physio- 
therapy to correct flexion contracture of his knee, which developed due to 
prolonged hospitalisation. 



Amputation in an Ischaemic Limb 



371 





Fig. 40.2. (left) Leg before amputation, showing line of incision of the dorsal flap and loss of most of the heel 
pad. 



Fig. A03.(right) Well-healed stump. 



Commentary 

Primary amputation as a treatment for the critically ischaemic limb has been made 
unnecessary by the progress of vascular surgery. The only indication for primary 
amputation in ischaemic disease occurs when the extent of irreversible gangrene is 
such that a functional extremity will not be produced by a revascularisation proce- 
dure. Such cases are occasionally encountered where the major part of the foot or 
the lower leg has become gangrenous. The assessment of the foot skin microcircula- 
tion has been shown to be useful in predicting imminent amputation [2]. Whether it 
is justified to perform a revascularisation procedure in order to obtain a more distal 
level of amputation is controversial, the problem being that failure of the revascu- 
larisation procedure will inevitably require a higher amputation level. It is, however, 
fully justified to strive for an amputation at the most distal level that can be 
expected to heal without delay or late breakdown, avoiding the need for a second 
surgical procedure, whether debridement, resuture or re-amputation. [Q3] 

It is widely reported that revascularisation procedures such as thrombolysis, 
distal bypass and balloon angioplasty with or without stent insertion, and in some 
cases a combination of these procedures, have all resulted in a measurable fall in 



372 Vascular Surgery 

the rate of major amputations [3, 4]. Although the evidence is conflicting, it seems 
that there is no compelling evidence that the failure of such a procedure will 
require a higher level of amputation than if a primary amputation had been 
carried out in the first place [5, 6]. It is possible that while a graft is functioning, in 
the time that has elapsed some collateral development may result in a better 
situation when the graft fails than at the time when it was first placed in position. 
[Q2][Q1:B,C,D,E] 

One difficulty that has emerged from this encouraging process is the decision of 
what constitutes a worthwhile residual limb if some part has to be removed at the 
time of the limb salvage procedure. There is no doubt that the amputation of one 
or all toes at the time of revascularisation is amply justified by the rapid healing 
and the satisfactory function that results. However, when larger parts of the foot 
require removal, then the functional outcome becomes less predictable, particu- 
larly so in diabetic patients who represent a different group although sharing 
some of the same problems. The loss of a digit and metatarsal in a "ray" type of 
amputation may be minimal for the fourth or fifth ray in the foot. The removal of 
central rays may result in very slow healing, and the removal of the hallux and 
first ray has a marked effect on walking, despite skilled podiatric attention. The 
transmetatarsal amputation may heal after revascularisation, and the gait that 
results with a filled shoe is equivalent to that obtained with a transtibial prosthe- 
sis, while retaining the advantages of a living limb that allows normal standing on 
barefoot. 

However, the classical Lisfranc operation is much less satisfactory. It involves dis- 
articulation of all the metatarsals at their base, and the residual cuboid and 
cuneiforms require a large area of skin to give satisfactory cover; the modified shoe 
and resulted gait are not very satisfactory. Chopart amputation, in which the cuboid 
and cuneiform are resected, is even less satisfactory, with an inherent tendency to 
plantar flex and evert the foot where gait problems are even more severe. While 
these can be corrected by tendon transplantation, this is an unsatisfactory option in 
the vascular patient. Amputation at the ankle has had a limited place in the manage- 
ment of vascular patients; this is a technically difficult operation, as the blood 
supply to the soft tissue heel flap is tenuous in the healthy patient and may be 
compromised in the patient with vascular disease. The whole heel flap depends 
on the calcaneal branches of the posterior tibial and peroneal artery. These run 
close to the malleoli and, if not damaged during surgery, are compromised by the 
anterior folding of the heel flap to complete the operation. A further limitation is 
the frequency of disease or damage of the skin of the heel itself, as occurred in 
our patient. Here, the whole of the soft tissue of the heel and some of the underly- 
ing calcaneum were necrotic and infected, which would exclude any ankle disar- 
ticulation of the traditional Syme type. However, in our patient, the arterial 
perfusion of the dorsum of the foot was adequate for healing. Hence, a myocuta- 
neous flap from the dorsum of the foot was utilised to cover the modified bone 
end of the tibia and fibula, after disarticulation and removal of the foot, as 
reported previously [1]. By resecting the malleoli and cartilaginous surface of the 
tibio-talo fibular articulation, a flat bone surface is produced. The anterior flap 
readily covers this and can be attached to the posterior skin, while the Achilles 
tendon is fixed to the periosteum. This arrangement will produce an extremity 
similar to that of Symes operation, in which the heel flap is preserved but is a few 
centimetres shorter. While still allowing ground contact, this stump permits a 
more satisfactory prosthesis to be used. While both Syme and dorsal flap trans- 



Amputation in an Ischaemic Limb 373 

malleolar amputation can take the body weight for a few steps without any cover, 
for natural walking a patella tendon and tibial condyle weight-bearing prosthesis 
is required. It seems that, psychologically, to lose a foot is less traumatic than to 
lose half a leg. [Q5: B, D] 

It is claimed that for critical ischaemia, certainly at the time before distal bypass 
was frequently used, the transtibial amputation was applicable to most situations. 
This was the best major amputation level for the patient, as it allowed a short pros- 
thesis, was a simple fixation, was lightweight and cosmetically acceptable, and, for 
elderly patients, was easily applied and removed. For the younger patient, it pro- 
vides a near-normal gait and the ability to run. To avoid skin breakdown over the 
sharp tibial border, and to make a satisfactory fit in the standard California patella 
tendon-bearing socket, the stump and the bone ends must be shaped carefully; the 
surgical technique is very important to obtain good function and satisfactory pros- 
thetic fitting. While the long posterior flap technique of Kendrick, Burgess and 
Romano may achieve these objectives, the skew sagittal flap is matched more pre- 
cisely to the requirements. 

Whatever technique is used, the criteria for wound healing and absence of pain 
depend on an adequate perfusion available at the level. A huge amount of work 
worldwide has endeavoured to define the best technique for determining this all- 
important factor. Of the many procedures that are available, the best is clinical 
guide providing that the amputation is decisively above the area of rest pain, the 
skin has normal warmth, colour and sensation, and muscle function is unim- 
paired. The transcutaneous oxygen level must exceed 40 mm Hg. The popliteal 
systolic pressure must be greater than 50 mm Hg, with an increasing risk of 
impaired healing and pain as the value diminishes, and healing unlikely to be 
achieved at all at a value of less than 20 mm Hg. Cutaneous laser Doppler studies 
are helpful, and emission thermography may give valuable information for 
planning the best type of skin flap for the individual patient. For a detailed review 
of the available methods, the reader is referred to Sarin et al. [7] and Clyne [8]. 
A similar assessment can be applied to transmalleolar or the transfemoral 
amputation. [Q6: A, B, C, E] 

The knee disarticulation has a controversial place in that the amount of skin 
required to cover the femoral condyles is only a little less than is required for a 
transtibial amputation. However, the skin around the knee has a good blood 
supply; provided this is not compromised by outside pressure or badly designed 
resulting in tight skin flaps, then the soft tissue cover of the condyles is not a 
problem, and as an amputation that requires no bone cutting, it is simple, quick, 
quiet and atraumatic. It has a good reputation for absence of postoperative pain, 
and the prosthesis is simplified as the stump is end bearing and patients can 
kneel on the stump. 

The Gritti-Stokes amputation at a supracondylar level is modified by the reten- 
tion of the patella, which is united to the bone end. With a similar reputation for 
healing and lack of trauma, this has enthusiastic support in some centres. 
Occasionally, it is painful due to non-union of the patella and femur, which limits 
prosthetic use. The transfemoral amputation, which must be 10-12 cm above the 
knee joint line, is often needed for the severe and critically ischaemic limb [9]. For 
optimum function, the muscles must be attached to the bone by myodesis, and 
opposing groups to each other by myoplasty. The bone end must be rounded care- 
fully with sufficient soft tissue to cover it comfortably. Soft tissue retraction fre- 
quently occurs and may even lead to protrusion of the bone through the skin, with a 



374 Vascular Surgery 

great deal of pain. In extreme instances of aortic or common iliac occlusion, tissue 
necrosis may demand a hip disarticulation, which can usually be performed without 
difficulty, although the inherent severity of the disease brings a high morbidity and 
mortality. In the upper limb, the level of amputation is determined more easily on 
clinical grounds, and prosthetic advice should be obtained where possible before 
the amputation to secure the optimum level and type of amputation for the avail- 
able prosthesis. 

In our patient, the conflict from the amputation point of view was between a 
transtibial amputation, which would be decisive and lead to acceptable prosthetic 
use at an early stage without the need for healing, but which might have compro- 
mised the bypass graft and required revision. It was therefore decided to proceed to 
the distal amputation, accepting that time would be required to heal the ulcerated 
skin in the lower leg following revascularisation. This has had a penalty of hospital- 
isation time and illustrates the many factors that may lead to the selection of a par- 
ticular amputation level. 

The vascular amputee, by nature of the extent and severity of their atheromatous 
disease, has been shown to have a diminished life expectation and an appreciable 
risk of a further amputation of the contralateral limb [10]. Therefore, there is con- 
siderable emphasis on utilising the patient's remaining lifespan to achieve a reason- 
able quality of life - if possible, independence - in their normal environment. This 
objective may not be achieved if there is delayed healing of the amputation stump, 
or problems with the amputation stump, which delay prosthetic fitting and use. 
There may be factors that cannot be avoided that lead to the patient needing a 
wheelchair or institutional care [11]. It is particularly important that the amputa- 
tion level selection is accurate as the principal cause of delayed healing or break- 
down of an amputation stump is inadequate blood supply. This will result in the 
stump being painful and prone to infection, which may precipitate the breakdown. 
[Q7] More general factors may contribute, in particular tobacco smoking and nutri- 
tional deficiency with a low total protein and albumin level, as in our patient [12]. 
The presence of renal insufficiency, liver dysfunction, advanced age and poorly con- 
trolled diabetes could contribute to wound breakdown in the amputation stump. 
Whatever the cause, the need for secondary surgery, in particular re-amputation, 
will contribute significantly to the morbidity and mortality of the procedure, 
delay the rehabilitation of the patient, and prejudice the degree of rehabilitation 
that the patient achieves. A further factor that is particularly difficult to assess 
but that is often significant in limiting the ability of the patients to use the pros- 
thesis is their cognitive state and ability to learn. One advantage of management 
by a multidisciplinary team is that the patient who is unable to use a prosthesis 
will be identified at an early stage and rehabilitated to wheelchair independence. 
If this selection is not made, then the provision of prosthesis and a failure to use 
it will be a disappointment to the patient at considerably unnecessary expense 
[13]. 

With this exception, a positive and enthusiastic attitude of a multidisciplinary 
team to the amputee suffering from vascular disease should make the patient feel 
that the turning point has been reached and a return to independent activity is 
now possible. The amputation is not a failure but a means to restoration of 
function. 

Amputation is now performed less frequently by the vascular surgeon [3, 4]. The 
improvement in limb salvage is diminishing the practical experience of amputation 
technique. It is more important now than ever before that the surgeon realises that 



Amputation in an Ischaemic Limb 375 

amputation is not just cutting off the diseased part, but also making an amputation 
stump that the patient will rely on for the rest of their life so that they can expect 
optimum function [14]. The vascular surgeon must therefore be fully competent to 
produce an amputation stump that will function well with the available prosthesis 
and will allow them to return to an independent way of life without unnecessary 
delay after the operation. 



References 



1. Robinson KP. Disarticulation at the ankle using an anterior flap. J Bone Joint Surg Br 
1999;81:617-20. 

2. Ubbin DT, Spincemaille GH, Reneman RS, Jacobs MJHM. Prediction of imminent amputation in 
patients with non-re-constructable leg ischaemia by means of microcirculatory investigations. J Vase 
Surg 1999;30:114-21. 

3. Karlstrom L, Bergqvist D. Effects of vascular surgery on amputation rates and mortality. Eur J Vase 
Endovasc Surg 1997;14:273-83 

4. Quigley FG, Ling J, Avramovic J. Impact of femorodistal bypass on major lower limb amputation 
rate. Aust N Z J Surg 1998;68:35-7. 

5. Eskov LB, Hindso K, Holstein P. Level of amputation following failed arterial reconstruction com- 
pared to primary Amputation. Eur J Vase Endovasc Surg 1999;17:35-40. 

6. Tsang GMK, Crowson MC, Hickey NC, Simms M. Failed femorocrural reconstruction does not prej- 
udice amputation level. Br J Surg 1991;78:1479-81. 

7. Sarin S, Shami S, Shields DA, Scurr JH, Smith PD. Selection of amputation level: a review. Eur J Vase 
Surg 1991;5:611-20. 

8. Clyne CA. Selection of level for lower limb amputations with severe peripheral vascular disease. Ann 
R Coll Surg Engl 1991;73:148-51. 

9. Greant P, Van den Brande P. Amputation in elderly and high risk vascular patients. Ann Vase Surg 
1990;4:288-90. 

10. Stewart CPU, Jain AS, Ogston SA. Lower limb amputee survival. Prosthet Orthot Int 1992; 
16:11-18. 

11. Campbell WB, St Johnston JA, Kernick VF, Rutler EA. Lower limb amputation: striking the balance. 
Ann R Coll Surg Engl 1994;76:205-9. 

12. Eneroth M, Apelqvist J, Larsson J, Persson BM. Improved wound healing in transtibial amputation 
receiving supplementary nutrition. Int Orthop 1997;21:104-8. 

13. Hanspal RS, Fisher K. Assessment of cognitive and psychomotor function and rehabilitation of 
elderly people with prosthesis. BMJ 1991;302:940. 

14. Bowker JH. Surgical technique for conserving tissue and function in lower limb amputation for 
trauma, infection and vascular disease. Instr Course Lect 1990;39:355-60. 



41 . Congenital Vascular Malformation 

Byung-Boong Lee 



A 10-year-old girl presented with a history of recurrent painful swelling of the left 
knee with mild ecchymosis. The latest episode of tender swelling of soft tissue 
along the left knee was preceded by a direct blow to the area during a ball game. 
In addition, she has had an abnormally grown left lower limb with scattered mul- 
tiple soft tissue masses throughout the limb since birth. 

Physical examination revealed diffuse swelling of the entire left limb, which 
was longer and larger than the opposite limb and more pronounced along the 
foot and lower leg. The swollen limb had slightly increased firmness on palpation 
throughout its entire length except for the soft tissue mass areas. 

Multiple soft tissue masses were easily compressible and scattered from the 
dorsum of foot to the upper thigh; their diameters varied between 2 and 8 cm. 

Similar lesions were also noticed at the left perineum, left labia, left lower 
abdomen, and left flank. Diffuse swelling along the medial side of left foot col- 
lapsed spontaneously when the foot was elevated. 

Further evaluation of the skeletal system revealed the left lower extremity to be 
5.0 cm longer - 3.0 cm longer in the tibia and 2.0 cm longer in the femur - in 
total length than the right lower extremity, accompanied by pelvic tilt and com- 
pensatory scoliosis of the lower spine. 

However, the patient had minimal limitation of her daily activities except for 
moderate limping. 

Family history and past history were unremarkable except for a vague history 
of cellulitis along the affected limb. 




Question 1 

What is the most fundamental problem on which clinician should focus in order to 
establish the proper diagnosis and treatment of this condition? 

A. Scoliosis with pelvic tilt. 

B. Abnormal long-bone growth with length discrepancy. 

377 



378 Vascular Surgery 

C. Abnormal swelling of lower limb with scattered soft tissue tumors. 

D. Mechanical problem of knee joint with symptoms. 

Question 2 

What is the most basic laboratory test required to verify the nature of the problem? 

A. Lumbosacral spine assessment. 

B. Radiologic assessment of bone length discrepancy. 

C. Duplex ultrasonography for the hemodynamic assessment. 

D. Locomotive test including gait evaluation. 

Question 3 

Which of the following non-invasive studies could be most useful in the clinical 
diagnosis of the disease complex in our patient? 

A. Volumetric assessment of limb size. 

B. Special radiologic study of epiphyseal plate of abnormally long bone. 

C. Magnetic resonance imaging (MRI) study of soft tissue masses. 

D. Transarterial lung perfusion scintigraphy. 

E. Bone scan. 

Question 4 

Which of the following non-invasive tests is not appropriate to assist in the differ- 
ential diagnosis for the extremity lesions in our patient? 

A. Whole-body blood-pool scintigraphy (WBBPS). 

B. Computed tomography (CT) scan. 

C. Radionuclide lymphoscintigraphy. 

D. Transarterial lung perfusion scintigraphy (TLPS). 

E. Lymphangiography (lymphography). 

Clinical Evaluation 

This patient underwent a thorough investigation of the nature and extent of the 
congenital vascular malformation (CVM) involved. 

A combination of various non- to minimally- invasive studies were performed to 
confirm the clinical impression of venolymphatic malformation (VLM): duplex 



Congenital Vascular Malformation 379 

ultrasonography, whole-body blood-pool scintigraphy (WBBPS), magnetic reso- 
nance image (MRI) study, transarterial lung perfusion scintigraphy (TLPS), and/or 
radionuclide lymphoscintigraphy. 

The primary hemodynamic impact and the secondary musculoskeletal impact of 
the venous malformation (VM) were assessed as the main CVM lesion in addition 
to the extent/degree of each component of the VM, truncular (T) and extratruncular 
(ET) form, involved in the extremity. 

A thorough skeletal evaluation of the long-bone growth discrepancy of the lower 
extremity and the degree of pelvic tilt with its compensatory scoliosis was also made 
with conventional bone X-rays. 

The TLPS assessment was performed substituting arteriographic investigation of 
the lower extremity for the possible hidden micro-arteriovenous malformation 
(AVM) lesion, which was marginally indicated due to an unusually increased 
venous flow by the isolated VM lesion alone on the duplex scan under the normally 
developed and functioning deep vein system. 

An ascending phlebography was also performed together with the percutaneous 
direct-puncture phlebography as a therapeutic guide; mandatory confirmation of 
the presence of a normal deep vein system of the lower extremity was made before 
starting the treatment to the infiltrating ET-form lesion of the VM. 

The final diagnosis confirmed extensive involvement of the VM as an infiltrating 
type of the ET form causing serious clinical impact directly to the venous system 
hemodynamically as well as to the skeletal system to induce abnormal long-bone 
growth of the left lower extremity. A moderate degree of venectasia as a T form of 
VM along the left femoral-popliteal vein segment was also found, by WBBPS, MRI 
and duplex scan, and subsequently confirmed by separate ascending phlebography. 

A venectasia of the femoral vein was assessed to have a limited clinical 
significance at this stage in comparison to the ET-form lesions of the VM. 

The lymphatic malformation (LM) component which is mixed with the ET form 
of VM, was confirmed as the ET form, giving minimum and limited clinical 
impact so that a conservative management/observation was instituted for this LM 
component. 

Therefore, the ET-form lesions of VM along the knee region were selected for 
active treatment as a priority; this was followed by the ankle and foot lesions. 

The primary indication to initiate the treatment immediately was that these 
lesions were potentially limb-threatening (e.g. hemarthrosis) due to their proximity 
to the joints with increased vulnerability to repeated trauma, especially as a cause of 
her knee symptoms. 

The treatment was further indicated to arrest/slow down their impact on abnor- 
mal long-bone growth. 

Multiple infiltrating ET lesions of the VM along the knee region, which is surgi- 
cally not amenable, were selected for ethanol sclerotherapy as independent 
therapy. Multisession ethanol sclerotherapy was given using 100-80 per cent 
absolute ethanol in calculated dosage - not exceeding 1.0 mg per kg of body 
weight as maximum dose per session - by direct puncture technique under 
general anesthesia. Close cardiopulmonary monitoring during the procedure was 
ensured to control and/or prevent transient pulmonary hypertension by the 
unavoidable spillage of ethanol into the systemic circulation from the lesion 
during treatment. 

The symptomatic lesions along the knee with recurrent painful swelling following 
minor injuries were controlled well without complication/morbidity and substan- 



380 Vascular Surgery 

tially reduced the risk of intra-articular bleeding and subsequent hemarthrosis. 
Subsequently, the ET-form VM lesions at the foot and ankle underwent surgical 
excision following preoperative multisession ethanol and N-butyl cyanoacrylic glue 
embolosclerotherapy with much reduced perioperative morbidity to improve foot 
function. 

Following successful control of multiple VM lesions along the knee, ankle, and 
foot with priority as a potentially limb-threatening condition, other VM lesions, 
scattered throughout the lower extremity, were also treated with absolute ethanol to 
assist further attempts to arrest the abnormal long-bone growth of the lower 
extremity. The abnormal long-bone growth is attributed to these VM lesions 
scattered within the muscular structure of the lower extremity in the extensive 
infiltrating type of ET, with significant impact on the venous circulation along the 
epiphyseal plate. 

In addition to the multisession embolosclerotherapy as independent and/or 
adjunct perioperative therapy to the VM lesions, the conservative supportive mea- 
sures to improve and/or maintain overall venous function have been supplemented 
with the use of a graded compression above-knee stocking to prevent chronic 
venous insufficiency. 

The final decision for the T-form lesion was left femoral-popliteal venectasia, but 
it was decided to defer treatment until urgent treatment of the ET form of the VM 
was finished, but to keep it under close observation. It might eventually require 
treatment (e.g. venorrhaphy, venous bypass) to prevent development of venous 
thromboembolism when significant venous flow/volume reduction should occur 
following successful control of the ET form of VM lesions. The hemodynamic con- 
sequences of the treatment of such extensive ET-form lesions directly affect total 
venous blood volume through the deep vein system. 

The LM component in this patient was treated only with complex decongestive 
therapy (CDT) in order to prevent full development of lymphedema. The infiltrating 
ET form of LM detected together with the ET form of VM has been shown to put 
extra burden on the marginally normal lymph-conducting system on lymphoscinti- 
graphic evaluation. Therefore, continuous surveillance for aggressive preventive 
measurement of local to systemic cellulitis along this ET form of LM lesions is 
mandated. 

This patient will continue to be managed by the multidisciplinary team of the 
CVM Clinic at regular intervals for her entire lifetime, through periodical follow- 
up assessment of the treatment results and the natural course of the untreated 
lesions. 



Question 5 

What is the first priority in the management of this patient? 

A. Correction of scoliosis. 

B. Correction of bone length discrepancy. 

C. Control of abnormal hemodynamic status of lower extremity by vascular 
lesions. 

D. Correction of gait with physical therapy and shoe adjustment. 

E. Biopsy of the soft tissue mass. 



Congenital Vascular Malformation 381 

Question 6 

Which of the following is not an indication for the treatment of venous malformation? 

A. Lesion located near to the limb threatening region. 

B. Life threatening lesion. 

C. Symptomatic lesion. 

D. Lesion with complication. 

E. All the lesions regardless of their condition. 

Question 7 

What is the International Society for the Study of Vascular Anomaly (ISSVA) rec- 
ommended and most popular strategy with respect to limb length discrepancy? 

A. Immediate surgical intervention to the epiphyseal plate to arrest further abnor- 
mal growth of the affected bone. 

B. Conservative treatment of limb length discrepancy only with physical therapy 
and shoe adjustment. 

C. Hemodynamic control of venous malformation as a priority whenever possible. 

D. Corrective surgery of bone for length discrepancy with the unaffected limb as a 



• • 



priority. 
E. None of the above. 

Question 8 

What is the current trend of therapeutic strategy for venous malformation lesions in 
the lower extremity? 

A. Surgical excision of the vascular lesions and related procedure only. 

B. Transarterial embolotherapy only. 

C. Transvenous sclerotherapy only. 

D. Multidisciplinary approach with surgical therapy and embolosclerotherapy. 

E. Percutaneous direct puncture sclerotherapy only. 

Question 9 

What is the general consensus on invasive investigations (e.g. arteriography; 
phlebography) for venous malformation? 

A. There is no indication for invasive investigation for the diagnosis and treatment 
of venous malformation. 



382 Vascular Surgery 

B. Invasive investigations are indicated in every suspected case of venous malfor- 
mation for the confirmation of the diagnosis. 

C. Invasive investigation can be reserved for the therapeutic regimen as a road map 
and/or occasional differential diagnosis. 

D. Invasive investigation should be used only for the follow-up assessment. 

E. None of the above. 

Question 10 

What is the most important precondition for the treatment of venous malformation 
in the lower extremity? 

A. History of deep vein thrombosis. 

B. Combined lymphatic malformation. 

C. Vascular-bone syndrome: length discrepancy of the long bone. 

D. Existence of deep vein system. 

E. Skin lesion with ulcer and necrosis. 

Question 1 1 

What has to be included in the differential diagnosis of venous malformation? 

A. Lymphatic malformation. 

B. AV shunting malformation. 

C. Infantile hemangioma. 

D. Capillary malformation. 

E. All of the above. 



Commentary 

Congenital vascular malformation (CVM) is regarded as one of the most difficult 
diagnostic and therapeutic enigmas in the practice of medicine. Vascular surgeons 
often take this vascular malformation quite casually without any specific knowl- 
edge, and this cavalier approach can end in failure. Clinical presentations of the 
CVMs are extremely variable, ranging from an asymptomatic birthmark to a life- 
threatening condition. This variability in the clinical presentation has been a major 
challenge to even the most experienced clinicians [1, 2]. Many attempts to control 
this ever-challenging problem, especially in the twentieth century, were led by sur- 
geons alone, but with mostly disastrous results because of poorly planned and over- 
aggressive surgical treatment carried out on the basis of limited knowledge [3, 4]. 
Recently, a multidisciplinary approach was introduced with a new concept based on 



Congenital Vascular Malformation 



383 



Table 41.1. Hamburg classification of congenital vascular malformation: 1988 consensus with modification 



Species 


Anatomical form 


Predominantly: 


Truncular forms: 


Arterial defects 


Aplasia or obstruction 




Dilatation 




Extratruncular forms: 




Infiltrating 




Limited 


Predominantly: 


Truncular forms: 


Venous defects 


Aplasia or obstruction 




Dilatation 




Extratruncular forms: 




Infiltrating 




Limited 


Predominantly: 


Truncular forms: 


Arteriovenous (AV) shunting defects 


Deep AV fistula 




Superficial AV fistula 




Extratruncular forms 




Infiltrating 




Limited 


Combined: 


Truncular forms: 


Vascular defects 


Arterial and venous 




Hemolymphatic 




Extratruncular forms 




Infiltrating hemolymphatic 




Limited hemolymphatic 


Predominantly: 


Truncular forms: 


Lymphatic defects 


Aplasia or obstruction 




Dilatation 




Extratruncular forms: 




Infiltrating 




Limited 



Hamburg classification [5, 6]. The Hamburg classification gives excellent clinical 
applicability with minimum confusion because the new terminology itself provides 
substantial information on the anatomico-pathophysiological status of vascular 
malformation; it has become the most fundamental rationale for the advanced 
concept of vascular malformation [7-9] (Table 41.1). It classifies complex groups of 
various vascular malformations based on the predominant type: VM, LM, AVM, 
and combined form which is mostly hemolymphatic malformation (HLM). The VM 
is the most common type of CVM together with LM and they often combine 
together to make the clinical condition quite complicated. 

When this HLM consists of only two components, that is, VM and LM, it is 
grouped separately as VLM which is almost equivalent to Klippel-Trenaunay syn- 
drome, where our patient belongs. 

The new Hamburg classification provides critical information relating to recur- 
rence based on precise information of embryonal stage when the developmental 
arrest has occurred [9, 10]. 



384 



Vascular Surgery 



When this developmental arrest occurs in an early stage of embryonal life, it 
remains with mesenchymal cell characteristics so it is grouped as ET form; when it 
occurs in the later stage of embryogenesis, it is grouped as T form with lack of mes- 
enchymal cell characteristics, which is extremely crucial for the clinical management. 

This patient presented with the most common clinical manifestation of CVM, 
with various findings related to the venous malformation (VM) as primary lesion as 
well as its secondary phenomena since birth (Fig. 41.1). Among many clinical 
findings, this patient presented with multiple, scattered, soft tissue mass lesions 
along the lower extremity, extending from the toe to flank, which provide the neces- 
sary clues to initiate proper investigation of VM as the etiology of this condition 
[11,12].[Q1:C] 

Relatively firm diffuse swelling of the entire left lower extremity, in addition to 
the abnormal long-bone growth with length discrepancy, may give further clues to 
the investigation on the combined nature of VM and LM as the cause of the vascu- 
lar-bone syndrome [13, 14]. 




Fig. 41 .1 .Clinical appearance of the patient, with extensive VM lesions scattered along the left lower extremity 
from toe to thigh, with extension to the perineum, labia, lower abdomen and flank, left. 



Congenital Vascular Malformation 



385 



The VM in particular has a significant incidence of secondary abnormal long- 
bone growth with subsequent bone length discrepancy. In addition, it is also known 
to have a relatively high incidence of combined LM, which is still called 
Klippel-Trenaunay syndrome [15, 16]. 

Of the many clinical clues this patient presented with that suggested VM among 
various CVMs, immediate collapse of the bulging soft lesion along the foot upon 
elevation was the most important. 

Therefore, hemodynamic assessment of the lower extremity along the scattered 
soft tissue tumors has to be the starting point for the work- up of proper diagnosis 
and treatment of this disease complex; duplex ultrasonographic study provides 
most of the essential hemodynamic information and an excellent guideline for 
further management (Fig. 41.2). [Q2: C] 





Fig. 41.2. a Sonographic identification of the communicating/draining vein between VM lesion and deep vein 
system, b Sonographic assessment of the VM lesion located superficially in the lower extremity. 



386 



Vascular Surgery 



Further study to assess scoliosis with pelvic tilt and/or abnormal long-bone 
growth with length discrepancy may be carried out once primary diagnosis of the 
vascular malformation has been made. In this case report, the patient presented 





Fig. 41 .3. a ET form of the VM in diffuse infiltrating status mostly confined within subcutaneous soft tissue, and T 
form of lesion along the deep vein system as femoral-popliteal venectasia. b ET form of the VM lesion, infiltrat- 
ing into foot muscle structure as well as sole soft tissue. 



Congenital Vascular Malformation 387 

recurrent episodes of tender swelling of the left knee following minor trauma. This 
was probably due to the bleeding/leaking from the VM lesion near to the knee joint 
to the surrounding soft tissue. A detailed evaluation of the knee joint itself can be 
deferred until the basic evaluation of VM, presented as soft tissue swelling along the 
knee joint, is completed with MRI, WBBPS, and duplex ultrasonography [17, 18]. 
This approach will delineate the accurate relationship of this VM lesion to the peri- 
articular structure including the joint space, and the potential risk of inducing 
hemarthrosis by repeated bleeding following trauma. 

Radiological assessment of lumbosacral spine together with long-bone length dis- 
crepancy should be made after hemodynamic assessment to identify the extent of 
VM, starting with duplex scan as the most basic laboratory test [19]. 

Although duplex ultrasonographic study can provide most of the crucial first-line 
hemodynamic information about vascular malformation, MRI of Tl and T2 images 
is the most valuable non-invasive study for clinical diagnosis, and has become the 
new gold standard for the diagnosis, especially for the VM [17] (Fig. 41.3). [Q3: C] 

MRI study of the soft tissue along the entire left lower extremity extending from 
toe to the torso can confirm the clinical diagnosis of VM already made preliminarily 
by ultrasonographic study. MRI can provide precise delineation of the anatomical 
relationship of the malformation lesion with its surrounding tissues like muscle, 
tendon, nerve, vessels, and bone from the foot to the retroperitoneal, pelvic, and 
gluteal regions. In addition to the duplex scan and MRI study in this patient, 
various non-invasive tests are needed for further differential diagnosis. 

Lymphoscintigraphic study based on radioisotope-tagged sulfur colloid is indi- 
cated to assess lymphatic function and the lymph-conducting system in general and 
rule out chronic lymphedema due to the T form of LM [20, 21]. 

The extremity involved was felt to be firmer than usual for a VM-affected leg, 
with general diffuse swelling throughout the entire length of the lower limb; this 
finding suggested primary lymphedema combined with venous stasis so that further 
evaluation of the lymphatic function is indicated with radionuclide lymphoscintig- 
raphy. The lymphatic function assessment of this patient with lymphoscintigraphy 
has shown the marginal status of the lymphatic system and its vulnerability to 
further insult by the ET form of LM. 

WBBPS based on radioisotope-tagged red blood cell pooling is also indicated as 
one of three basic tests for the diagnosis of VM. This relatively new investigation is 
very sensitive in detecting abnormal blood pooling throughout the body (Fig. 41.4). 
It can be used not only as a practical test to assess treatment results but also as a 
screening test for hidden vascular malformation. It also has a unique role in the dif- 
ferentiation between venous and lymphatic malformation [22, 23]. 

CT scanning also has practical value in providing information on the relationship 
of vascular malformation to its surrounding skeletal and soft tissue of the lower 
extremity. 

Transarterial lung perfusion scintigraphy (TLPS) can provide crucial information 
on possible involvement of a micro-, if not, macro-AVM lesion to the VM 
(Fig. 41.5). 

AVM involvement is a critical condition for the management strategy of VM; the 
VLM in particular is seldom combined with the AVM, especially in micro-AVM, 
which can be overlooked by conventional arteriography alone. Positive 
confirmation of no existence of micro-AVM is extremely important before the initi- 
ation of the treatment to the symptomatic VM lesions, especially when it is com- 
bined with LM. 



388 



Vascular Surgery 




junentat 



WfttIM M4 



MSTIBHIR 



kstcsih TM 



Fig. 41.4. Extensive abnormal blood pooling by the ET lesions and T lesion of the VM, diffusely involving entire 
lower extremity. 



tS M 




Bo sa 1 iaage Re i n j »'i- 1 u>n i nage 




Injected Jose t«Ci> t 2,35 reinject til i1os«- <nO> * 


4. SI 




Percent Hi? Shunt - 0.78 














UN? 


y 










ii.» + 


r^* ci ^%^Vuy\ Tji jv/ 










uut 


1 








FIJI 

f 


i< m i*i ■ 


k* 




■•Mail 













Fig. 41 .5. TLPS investigation of arteriovenous (AV) shunting status in lower extremity to assess potential risk of 
the AVM lesion involved to the VM lesion. Normal TLPS finding with no evidence of micro-AV shunting can rule 
out AVM without further investigation by arteriography. 



The TLPS can therefore provide necessary guidance for the further invasive study 
of arteriography [24, 25]. 

However, classical lymphangiography (or lymphography) using oil-based con- 
trast material is no longer performed for the screening lymphatic function because 
of the potential risk of further damaging the lymphatic vessel with the procedure. 
[Q4: E] 

Once the final diagnosis of a combination of VM and LM has been made, then the 
next decision should be whether treatment is indicated. In view of the abnormal 
long-bone growth involvement to this vascular malformation, immediate treatment 
of this particular VM is generally preferred. 



Congenital Vascular Malformation 389 

Treatment priority should be given to the primary etiology, i.e. vascular malfor- 
mation. Therefore, the control of abnormal hemodynamic status of the lower 
extremity secondary to the VM should have priority [26, 27]. [Q5: C] 

All the other clinical problems secondary to this primary lesion, including scolio- 
sis with pelvic tilt, abnormal long-bone growth with bone length discrepancy, and 
abnormal gait, can be deferred while treatment is aimed at the VM itself [5, 6, 26]. 
Not all the VM lesions are indicated or feasible for treatment. In general, VM 
lesions located near limb-threatening regions (e.g. proximity to the joint space) or 
potentially life/critical function-threatening regions (e.g. proximity to the airway), 
symptomatic lesions, and/or lesions with complications are generally considered for 
treatment [5, 11].[Q6:E] 

There is significant controversy over how to manage limb-length discrepancy as 
the secondary phenomenon of the VM in the lower extremity. Surgical intervention 
directly to the epiphyseal plate to arrest further abnormal growth of the affected 
long bone has brought mixed results, with further controversy on its long-term 
value [13, 14]. Therefore, general the consensus on this issue of vascular-bone syn- 
drome accepted by most ISSVA members these days is to endorse a new strategy to 
control the hemodyamic abnormality of VM first, since hemodynamic impact/stim- 
ulation by the VM lesions to the intraosseous tissue along the epiphyseal plates is 
known to be the cause of abnormal long-bone growth [14, 26]. The strategy based 
on conservative treatment only with physical therapy and shoe adjustment until the 
long-bone growth is completed is also not acceptable due to increasing morbidity in 
gait and spine, as well as the unpredictable outcome of late correction. Meanwhile, 
too aggressive an approach with early correction of long-bone discrepancy has also 
been abandoned due to significant difficulty in achieving good long-term results. 
[Q7: C] 

The traditional surgical approach of removing the entire lesion is still theoreti- 
cally acceptable if the lesion is located in a surgically accessible area and localized 
enough to be completely removable with limited or no morbidity. However, this 
condition is generally very rare and for most VM lesions there will be significant 
morbidity with a surgical approach aimed at complete removal of the lesion. 

Therefore, a multidisciplinary approach that combines traditional surgical 
therapy with newly introduced embolosclerotherapy utilizing various emboloscle- 
roagents is the treatment strategy of choice [5, 6, 8]. This can substantially reduce 
overall treatment-related morbidity with good long-term therapy results [11, 12]. 

A lesion located along the surgically inaccessible area and/or with prohibitively 
high surgical morbidity is generally treated with sclerotherapy alone. The current 
trend in the management of VM of the lower extremity involves a multidisciplinary 
approach combining surgical therapy, sclerotherapy, and/or embolotherapy, when- 
ever feasible [5, 27]. [Q8: D] 

Most of the diagnosis of VM in the lower extremity in particular can be made 
efficiently on the basis of non-invasive studies. However, classical invasive studies, 
including arteriography and phlebography, are still considered to be the gold stan- 
dard for the management of all vascular malformations, but they are generally 
reserved for use as a road map for the final therapeutic regimen (Fig. 41.6). These 
invasive imaging techniques are also used to rule out hidden micro-AVM combined 
with the VM, especially when TLPS findings indicate a high possibility of a micro- 
AV shunting condition [6, 8, 25]. [Q9: C] 

Numerous emboloscleroagents have been tested for the treatment of VM; most 
recently, absolute ethanol has been accepted as the scleroagent of choice not only 
for VM but also for AVM, with excellent long-term outcome with no recurrence 



390 



Vascular Surgery 




Fig. 41.6 Percutaneous direct puncture phlebographic findings of the ET-form lesions of VM in the thigh; it may 
become a road map for the subsequent endovascular management with embolo/sclerotherapy. 



when treated properly [11, 12, 25, 28-30]. However, this has significant side effects, 
resulting in various acute and/or chronic complications/morbidity, such as deep 
vein thrombosis, pulmonary embolism, nerve palsy, and various degrees of skin to 
soft tissue damage from bullae to full thickness necrosis. Therefore, the selection of 
ethanol as the scleroagent to treat VM has to be based on the risk involved of recur- 
rence, acute morbidity during the therapy, and long-term sequelae of the treatment 
[6]. In order to treat VM of the lower extremity safely, careful hemodynamic assess- 
ment of the deep vein system is also mandatory, including confirmation of the exis- 
tence of a normal deep vein system. This is crucial before treatment of the T-form 
lesion of VM, the marginal (lateral embryonic) vein in particular. Once the 
deep vein system is properly documented, proper treatment of VM can be initiated. 
[Q10: D] However, all the other issues raised in Question 10, including history of 
deep vein thrombosis, combined LM, and history of skin damage during previous 
sclerotherapy, will also require careful assessment to improve overall safety of the 
planned treatment. 

Differential diagnosis with other forms, T or ET forms, as well as other kinds of 
vascular malformation, VM. LM, VLM, or AVM, is mandatory, in view of their dif- 
ferent behavior with different clinical impact. This is particularly important for the 
ET form of various vascular malformations whose behavior is totally unpredictable. 
The ET form retains the original evolutional ability of mesenchymal cells, in con- 
trast to the T form, so that it can grow when the condition/stimulation should meet 
(e.g. trauma, surgery, pregnancy, hormone therapy) [10]. Regarding the VM of the 
lower extremity, precise differential diagnosis of other conditions such as LM or 
AVM is extremely important, because the treatment strategy is substantially differ- 
ent [6, 27]. Besides, initial differential diagnosis for VM, like any vascular malfor- 



Congenital Vascular Malformation 391 

mation, should start from the infantile (neonatal) hemangioma which also belongs 
to the vascular anomaly together with the vascular malformation. Hemangioma is a 
true vascular tumor and not a vascular malformation, possessing distinctively dif- 
ferent pathophysiology, anatomico-histology, and clinical behavior [1, 31]. [Q11: E] 
The clinical significance of capillary malformation is not understood properly yet, 
but it should be included in the evaluation of any vascular malformation although 
the modified Hamburg classification did not include it in the classification of 
various CVMs, due to the lack of clinical significance for the vascular surgeon [32]. 



References 



1. Mulliken JB. Cutaneous vascular anomalies. Semin Vase Surg 1993;6:204-218. 

2. Rutherford RB. Congenital vascular malformations: diagnostic evaluation. Semin Vase Surg 
1993;6:225-32. 

3. Malan E. Vascular malformations (angiodysplasias). Milan: Carlo Erba Foundation, 1974;17. 

4. Szilagyi DE, Smith RF, Elliott JP, Hageman JH. Congenital arteriovenous anomalies of the limbs. 
Arch Surg 1976;111:423-29. 

5. Lee BB, Bergan JJ. Advanced management of congenital vascular malformations: a multidisciplinary 
approach. J Cardiovasc Surg 2002;10(6):523-33. 

6. Lee BB. Critical issues on the management of congenital vascular malformation. Ann Vase Surg 
2004;18(3):380-92. 

7. Belov St. Anatomopathological classification of congenital vascular defects. Semin Vase Surg 
1993;6:219-24. 

8. Lee BB. Advanced management of congenital vascular malformation (CVM). Int Angiol 
2002;21(3):209-13. 

9. Belov St. Classification of congenital vascular defects. Int Angiol 1990;9:141-6. 

10. Bastide G, Lefebvre D. Anatomy and organogenesis and vascular malformations. In: Belov St, Loose 
DA, Weber J, editors. Vascular malformations. Reinbek: Einhorn-Presse Verlag, 1989;20-2. 

11. Lee BB, Kim DI, Huh S, Kim HH, Choo IW, Byun HS, Do YS. New experiences with absolute ethanol 
sclerotherapy in the management of a complex form of congenital venous malformation. J Vase Surg 
2001;33:764-72. 

12. Lee BB, Do YS, Byun HS, Choo IW, Kim DI, Huh SH. Advanced management of venous malforma- 
tion with ethanol sclerotherapy: mid-term results. J Vase Surg 2003;37(3):533-8. 

13. Mattassi R. Differential diagnosis in congenital vascular-bone syndromes. Semin Vase Surg 
1993;6:233-44. 

14. Belov St. Correction of lower limbs length discrepancy in congenital vascular-bone disease by vascu- 
lar surgery performed during childhood. Semin Vase Surg 1993;6:245-51. 

15. Lee BB: Klippel-Trenaunay syndrome and pregnancy. Int Angiol 2003;22(3):328. 

16. Servelle M. Klippel and Trenaunay's syndrome. Ann Surg 1985;201:365-73. 

17. Lee BB, Choe YH, Ahn JM, Do YS, Kim DI, Huh SH, Byun HS. The new role of MRI (magnetic reso- 
nance imaging) in the contemporary diagnosis of venous malformation: can it replace angiography? 
J Am Coll Surg 2004;198(4):549-58. 

18. Lee BB: Current concept of venous malformation (VM). Phlebolymphology 2003;43:197-203. 

19. Lee BB, Mattassi R, Choe YH, Vaghi M, Ahn JM, Kim DI, et al. Critical role of duplex ultrasonogra- 
phy for the advanced management of venous malformation (VM). Phlebology, March 2005 (in 
press). 

20. Lee BB, Seo JM, Hwang JH, Do YS, Kim DI, Byun HS, et al. Current concepts in lymphatic malforma- 
tion (LM). J Vase Endovasc Surg 2005;39(1) 67-81. 

21. Lee BB, Kim DI, Whang JH, Lee KW. Contemporary management of chronic lymphedema - 
personal experiences. Lymphology 2002;35(Suppl):450-5. 

22. Lee BB, Mattassi R, Kim BT, Kim DI, Ahn JM, Choi JY. Contemporary diagnosis and management of 
venous and AV shunting malformation by whole body blood pool scintigraphy (WBBPS). Int Angiol 
April 2005 (in press). 

23. Lee BB, Kim BT, Choi JY, Cazaubon M. Prise en charge des malformations vasculaires congenitales 
(MVC) en 2003: role de la scintigraphy corps entier dans las surveillance evolutive. Angeiologie 
2003;55(3):17-26. 



392 Vascular Surgery 

24. Lee BB, Mattassi R, Kim BT, Park JM. Advanced management of arteriovenous shunting malforma- 
tion with transarterial lung perfusion scintigraphy (TLPS) for follow up assessment. Int Angiol 
2005;24(2):173-84. 

25. Lee BB, Do YS, Yakes W, Kim DI, Mattassi R, Hyun WS, Byun HS. Management of arterial-venous 
shunting malformations (AVM) by surgery and embolosclerotherapy. A multidisciplinary approach. 
J Vase Surg 2004;39(3):590-600. 

26. Lee BB, Kim HH, Mattassi R, Yakes W, Loose D, Tasnadi G. A new approach to the congenital vascu- 
lar malformation with new concept - Seoul Consensus. Int J Angiol 2004;12:248-251. 

27. Lee BB, Beaujean M, Cazoubon M. Nouvelles strategies dans la prise en charge des malformations 
vasculaires congenitales (MVC): un aperc[,]u de l'experience clinique coreenne. Angeiologie 
2004;56(2):ll-25. 

28. Yakes WF, Pevsner PH, Reed MD, Donohu HJ, Ghaed N. Serial embolizations of an extremity arteri- 
ovenous malformation with alcohol via direct percutaneous puncture. AJR 1986;146:1038-40. 

29. Yakes WF, Haas DK, Parker SH, et al. Symptomatic vascular malformations: ethanol embolotherapy. 
Radiology 1989;170:1059-66. 

30. Yakes WF, Parker SH, Gibson MD, Haas DK, Pevsner PH, Carter TE. Alcohol embolotherapy of vas- 
cular malformations. Semin Intervent Radiol 1989;6:146-61. 

31. Mulliken JB, Young AE, editors. Vascular birthmarks: hemangiomas and malformations. 
Philadelphia, PA: WB Saunders; 1988. 

32. Van Der Stricht J. Classification of vascular malformations. In: Belov St, Loose DA, Weber J, editors. 
Vascular malformations. Reinbek: Einhorn-Presse Verlag, 1989;23. 



42. Deep Venous Thrombosis 



Fahad S. Alasfar, Dwayne Badgett and Anthony }. Comerota 



A 67-year-old male had a history of a right calf deep venous thrombosis (DVT) 
following a flight from California to New York. He was treated on that occasion 
with anticoagulation with unfractionated heparin then Coumadin for 3 months. 
Recently, he was diagnosed with sigmoid cancer. He is now on postoperative day 
three from exploratory laparotomy, sigmoid colectomy and extensive lysis of 
adhesions. Although he was haemodynamically stable, he required a transfusion 
of three units of blood. DVT prophylaxis for the perioperative period included 
graded knee-high compressive stockings and intermittent pneumatic compres- 
sion (IPC). 






Question 1 

What are the risk factors that predispose to DVT? 

Question 2 

What is the clinical presentation of a patient with anti-thrombin III (ATM) 
deficiency? 



Question 3 

Regarding antiphospholipid antibody (APA) syndrome, which of the following is 
not correct? 

A. Procainamide has been associated with the development of APA syndrome. 

B. Thrombotic complications associated with APA syndrome are limited to the 
venous system. 

C. Long-term anticoagulation has been recommended in managing APA syn- 
drome, maintaining the international normalised ratio (INR) at 3 or higher. 

D. Recurrent venous and arterial thrombosis is a major feature of the APA 
syndrome. 

395 



396 Vascular Surgery 

Question 4 

Regarding Factor V Leiden gene mutation, which of the following is/are correct? 

A. Factor V Leiden mutation is an important risk factor for pulmonary embolism 
and DVT during pregnancy or use of oral contraceptives. 

B. Factor V Leiden mutation is associated with an increased risk of myocardial 
infarction and angina. 

C. Hyperhomocystinaemia increases the risk of Factor V Leiden carriers having 
any Venous Thromboembolic Episodes (VTE) from two per cent to ten per cent. 

D. A single-point mutation in the gene coding for coagulation Factor V results in 
the formation of a Factor V molecule that is not inactivated properly by acti- 
vated protein C (APC). 



Question 5 

Which of the following statements are true concerning prophylaxis for DVT? 

A. There are many prospective randomised studies supporting the efficacy 
of graded compression stockings in preventing DVT in patients with 
malignancy. 

B. IPC is as effective as low-dose unfractionated heparin (LDUH) in reducing the 
risk of DVT. 

C. LDUH and low-molecular-weight heparin (LMWH) are most effective in pre- 
venting DVT. 

D. Dextran is an excellent alternative to LDUH in preventing DVT. 

On the fifth postoperative day, the patient began complaining of mild left calf 
pain and swelling. On physical examination, his lower extremities were warm with 
normal pulses. The left calf was mildly swollen with slight tenderness. A venous 
duplex of the lower extremity revealed thrombosis of the left popliteal, posterior 
tibial and peroneal veins. 



Question 6 

Which of the following statements regarding perioperative DVT is/are correct? 

A. In general surgery, the overall incidence of DVT as assessed by labelled fibrino- 
gen uptake (FUT) is 25 per cent. 

B. In surgical patients with malignant disease, the incidence of postoperative DVT 
is 60 per cent. 

C. The incidence of postoperative DVT after total hip replacement is 45-55%. 

D. Major trauma patients have a low risk for DVT. 



Deep Venous Thrombosis 397 

E. Patients undergoing elective neurosurgical procedures have a 20-25% incidence 
of DVT documented by radioisotopic scanning. 

The patient was started on a therapeutic regimen of LMWH (enoxaparin) 
1 mg/kg every 12 h and a daily dose of Coumadin .The patient's baseline coagulation 
profile was normal and his platelet count was 190,000. On day three of anticoagula- 
tion, his INR was 2.2 and his platelet count dropped to 67,000. 



Question 7 

Regarding heparin-induced thrombocytopoenia (HIT), which of the following is/are 
correct? 

A. It is caused by IgM antibodies that recognise the complex of heparin and platelet 
factor 4. 

B. The peak incidence occurs 4-14 days after initiation of heparin. 

C. It occurs more commonly with unfractionated heparin than with LMWH. 

D. It can be treated by reducing the dose of LMWH. 

E. Argatroban and hirudin are acceptable agents used for the treatment of HIT. 

LMWH was discontinued and the patient started on Argatroban. On the tenth 
postoperative day, the patient started complaining of left flank pain and his 
haemoglobin level dropped to 6 g/dl.A computed tomography (CT) scan of his 
abdomen revealed a 6x7-cm retroperitoneal haematoma. Because of the 
haematoma, anticoagulation was discontinued and an inferior vena cava (IVC) 
filter inserted. 



Question 8 

Which of the following are acceptable indications for an IVC filter? 

A. Complication or contraindication to anticoagulation in a patient diagnosed with 
a pulmonary embolism. 

B. Recurrent thromboembolism despite therapeutic anticoagulation. 

C. Acute iliofemoral DVT. 

D. Recurrent pulmonary embolism with pulmonary hypertension. 

Question 9 

Regarding thrombolysis for acute DVT, which of the following is/are correct? 

A. Studies show no difference in lysis capability between anticoagulation and lytic 
therapy. 



398 Vascular Surgery 

B. Randomised studies support lytic therapy for all lower-extremity DVT. 

C. Patients with iliofemoral DVT treated with catheter-directed thrombolysis have 
a better quality of life than patients treated with anticoagulation alone. 

D. Lytic agents are more effective when delivered by catheter-directed intrathrom- 
bus infusion rather than systemic intravenous infusion. 



Commentary 



The natural history of DVT has been described well in the literature. Complications 
of venous thromboembolism continue to be a major cause of death and morbidity 
each year. In the USA, there are approximately 50,000-200,000 deaths each year sec- 
ondary to pulmonary embolism. Fifty- two per cent of patients with DVT develop 
pulmonary embolism [1], most of which occur from the proximal venous segments 
of the lower extremities. 

Patients with proximal DVT had a pulmonary embolism incidence of 66 per 
cent, whereas tibial thrombi had a 33 per cent incidence [1]. Multiple studies have 
shown a 50 per cent reduction in fatal pulmonary embolism when prophylaxis 
with LDUH is used [2]. Moreover, natural history studies have shown that the 
long-term morbidity of post-thrombotic syndrome (PTS) is significant following 
DVT. PTS has been reported in 33-79% of patients following proximal DVT and 
2-29% of patients with calf DVT. Masuda et al. [3] reported valve reflux in 30 per 
cent of individuals with calf DVT followed for 3 years. Furthermore, they reported 
that 23 per cent of patients with calf DVT have ongoing pain and swelling of the 
affected extremity. 

Thus, proper prophylaxis, early diagnosis and appropriate therapy are of 
paramount importance in preventing the short- and long-term complications of 
DVT. 

An understanding of the risk factors for DVT is helpful for appropriate DVT pro- 
phylaxis. These risk factors include prior DVT/pulmonary embolism, prolonged 
immobilisation or paralysis, malignancy, major surgery (especially abdominal, hip 
and lower-extremity surgery), age over 40 years, and severe heart disease. There are 
also hypercoagulable states that predispose to thrombosis. Haematological abnor- 
malities include protein C and protein S deficiency, Factor V mutation, disorders of 
plasminogen activation and antiphospholipid antibodies. 

Lupus anticoagulant and HIT are also associated with DVT. Proteins C and S are 
part of the naturally occurring balance of coagulation that prevents thrombosis by 
inactivating Factors Va and Villa. Deficiency of these factors leads to an increased 
risk of thrombosis. Proteins C and S, like Factors II, VII, IX and X, depend on 
vitamin K. Because of the shorter half-life of protein C, a transient hypercoagulable 
state can be induced early in the course of treating patients with a warfarin com- 
pound due to the acute reduction in protein C level. A search for an underlying 
hypercoagulable disorder should be undertaken in patients with recurrent DVT or 
unexplained arterial or graft occlusion. 

Chronic warfarin therapy may reduce the level of proteins C and S by 30-50%; 
therefore, these levels should be measured after the patient has discontinued 
warfarin. Indefinite oral anticoagulation is indicated in patients with confirmed 
deficiency. [Q1] 



Deep Venous Thrombosis 399 

ATM is an important naturally occurring anticoagulant that inhibits the enzy- 
matic activation of thrombin and other naturally occurring clotting factors. The het- 
erozygous form of ATM deficiency is asymptomatic and may affect 1 in 2,000 
people. A chronic deficiency of ATM can occur with protein loss in nephrotic syn- 
drome, liver disease, sepsis and Disseminated Intravascular Coagulation (DIC). 
When complications occur, heparin followed by Coumadin is the treatment of 
choice. [Q2] 

APA is a heterogeneous group of circulating autoantibodies directed primarily 
against negatively charged phospholipid compounds. These antibodies interfere 
with the thromboplastin reaction against the activated platelet. Recurrent venous 
and arterial thrombosis is a major feature of APA syndrome. Thrombosis associated 
with APA syndrome has occurred in diverse anatomic locations, causing a wide 
spectrum of clinical manifestations. DVT and pulmonary embolism are common 
complications of APA [4]. Similarly, arterial thrombosis involving carotid [5], 
hepatic, splenic, mesenteric and retinal arteries causing infarction has occurred. 
APA syndrome should be suspected in young patients with stroke or arterial 
occlusion. 

APA syndrome has been associated with multiple medications. However, pro- 
cainamide has been implicated more commonly than other drugs [6]. 

The diagnosis should be suspected based on the clinical presentation or the unex- 
plained prolonged PTT. Diagnostic tests for APA syndrome include serology testing 
for APA and clotting assays. The primary treatment remains anticoagulation, main- 
taining an INR at or above 3.0 [7, 8]. [Q3: B] 

Protein C is one of the key regulatory proteins for coagulation cascade. APC 
cleaves and inactivates Factors Va and Villa. A single-point mutation in the gene 
coding for Factor V results in the formation of a Factor V molecule that is not inac- 
tivated properly by APC [9]. Factor V Leiden mutation is an important risk factor 
for pulmonary embolism and DVT, especially during pregnancy or oral contracep- 
tive use [10]. 

Hyperhomocystinaemia increases the relative risk of a Factor V leiden carrier 
having any VTE [11]. There is no increased risk of myocardial infarction or angina 
in patients with Factor V Leiden mutation [12]. [Q4: A, C, D] 

Among the available methods of DVT prophylaxis, LDUH and LMWH are the 
most effective in reducing DVT as assessed by FUT [13]. LDUH was the first anti- 
thrombotic agent evaluated in early randomised studies. LDUH, dextran, IPC and 
graded elastic stockings also significantly reduce the incidence of postoperative 
DVT [13]. 

LDUH given subcutaneously (5,000 U) every 8 or 12 h started preoperatively and 
continued postoperatively for 7 days has been shown to decrease the incidence of 
DVT from 25 to eight per cent [14]. Moreover, these studies have shown a 50 per 
cent reduction of fatal pulmonary embolism when patients are treated with LDUH. 
LMWH and LDUH have been shown to be equally effective in preventing DVT in 
general surgery patients [14]. 

Advantages of LMWH include improved bioavailability, once-daily dosing, and a 
lower incidence of HIT [15]. 

IPC is an attractive method of DVT prophylaxis since there are no observed 
complications. This device provides intermittent compression lasting 10 s/min 
with insufflation pressures of 35-40 mm Hg. In a trial comparing IPC with LDUH, 
both agents were effective in reducing lower-extremity DVT in high-risk patients 
[16]. 



400 Vascular Surgery 

Graded compression stockings decrease the risk of DVT, but data are limited 
regarding the effect on the prevention of DVT and pulmonary embolism. There 
are no randomised trials on the use of these stockings alone in high-risk patients, 
although current recommendations suggest the use of more effective methods. 
Fifteen to twenty per cent of patients will not receive benefit from elastic stock- 
ings because of their leg shape or size. Dextran has not been shown to be as effec- 
tive as either LMWH or LDUH in preventing DVT; however, it may reduce the 
incidence of pulmonary embolism. Disadvantages of dextran include its high 
price, risk of anaphylaxis, potential for volume overload, and need for intra- 
venous access. It is also contraindicated in patients with impaired renal and 
cardiac function. [Q5: B, C] 

The incidence of DVT in general surgery patients has been well established. 
Overall, the incidence of DVT was 25 per cent in general surgery patients not receiv- 
ing prophylaxis. In patients with other risk factors, i.e. malignancy, the risk of DVT 
is 29 per cent. Overall, the risk of pulmonary embolism is 1.6 per cent while the risk 
of fatal pulmonary embolism is 0.8 per cent [13]. 

Patients undergoing major orthopaedic surgery of the lower extremity are at high 
risk of postoperative DVT, despite improved techniques and early mobilisation. The 
incidence of postoperative DVT after total hip replacement is 45-57%, with the risk 
of proximal DVT being 23-36% [17]. The incidence of pulmonary embolism in this 
group is 6-30% and that of fatal pulmonary embolism is 3-6%. Because many pul- 
monary embolisms are asymptomatic, and because of the high incidence of DVT in 
the postoperative period, proper prophylaxis is mandatory [18]. 

DVT and pulmonary embolism are considered common complications 
after major trauma. A recent study using a venographic endpoint demonstrated 
that major trauma patients (injury severity score >9) have an exceptionally high 
risk of venous thromboembolism (58 per cent). This study also revealed that 
there is a greater than 50 per cent incidence of DVT in the major trauma subset 

[19]. 

Pulmonary embolism is the most frequent reason for death following spinal cord 
injury. Clinically recognised DVT and pulmonary embolism occur in only 15 and 
five per cent of cases, respectively [20]. However, the incidence of DVT in patients 
with acute spinal cord injury by venography has been reported to be between 18 and 
100 per cent, with an average of 40 per cent. The incidence of fatal pulmonary 
embolism is 4.6 per cent, with the greatest risk occurring in the first 2-3 months 
after spinal-cord injury [21]. [Q6: A, C, E] 

Approximately 2-5% of patients exposed to heparin will develop HIT. This is 
caused by IgG antibodies that recognise complexes of heparin and platelet factor 4, 
leading to platelet activation via platelet Fc gamma Ha receptors. Formation of a 
procoagulant, platelet-derived microparticles generates thrombin and makes 
patients especially vulnerable to venous thromboembolism [22, 23]. 

When examined directly, the clot appears white due to the concentration of fibrin 
and platelets. HIT should be suspected if a patient develops DVT or pulmonary 
embolism while receiving heparin, especially if the platelet count drops below 35 
per cent. HIT usually develops between the fourth and fourteenth days after initia- 
tion of heparin, although a rapid fall in platelet count can occur in response to 
heparin if the patient has had recent heparin exposure. 

HIT occurs much more commonly with unfractionated heparin than with LMWH 
[15]. Upon recognition of HIT, heparin should be discontinued; however, appropri- 
ate anticoagulation should be continued to avoid a thrombotic complication, which 



Deep Venous Thrombosis 401 

has been observed in up to 50 per cent of patients within 30 days of the diagnosis of 
HIT [15]. 

Current treatment options include lepirudin [24], argatroban and danaparoid. 
Lepirudin is recombinant hirudin (r-hirudin) and is approved for the treatment of 
patients with HIT. It is a potent direct thrombin inhibitor and is given in a bolus 
dose of 0.4 mg/kg/min followed by an infusion of 0.2 mg/kg/h, but the dosage 
should be adjusted in patient with renal dysfunction. Argatroban is a synthetic 
peptide that binds to and inhibits thrombin. It is given in doses of 0.5-4 |jLg/kg/min 
and has the advantage of normal excretion (hepatic) in patients with impaired renal 
function. Danaparoid is a mixture of heparan sulphate and dermatan sulphate, 
which inhibits thrombin generation indirectly via inhibition of Factor Xa, with 
some direct anti-thrombin activity as well. The disadvantage of danaparoid includes 
a 10-20% in vitro cross-reactivity with HIT antibodies and long half-life. [Q7: B, C, E] 

IVC filters are intended to prevent pulmonary emboli following filter insertion. 
Anticoagulation should be continued whenever possible to prevent further throm- 
bosis [25, 26]. The primary indication for the insertion of an IVC filter is the occur- 
rence of a complication of or contraindication for anticoagulation therapy. Less 
frequent indications for the insertion of an IVC filter are recurrent thromboem- 
bolism despite adequate anticoagulation therapy and chronic recurrent pulmonary 
embolism with pulmonary hypertension. 

Finally, IVC filters have been used for pulmonary embolism prophylaxis in 
patients with proximal DVT who are at high risk for bleeding and selected trauma 
patients (pelvic fracture) who are at high risk for VTE and cannot be managed with 
effective prophylaxis. [Q8: A, B, D] 

Restoring patency by eliminating the thrombus in the deep venous system is the 
ideal goal of therapy for acute DVT. Many reports have shown that lysis can be 
achieved and patency restored with thrombolysis, and that long-term sequelae 
occur less often with successful treatment. Systemic thrombolytic therapy for lower- 
extremity DVT is associated with a 40-60% success rate. While recanalisation is 
better than standard anticoagulation, the increased risk of bleeding complications 
has reduced enthusiasm for thrombolysis. 

It has been shown that patients with iliofemoral DVT treated with catheter- 
directed thrombolysis have better functioning and wellbeing than patients treated 
with anticoagulation alone [28]. Currently, it is recommended that lytic agents be 
delivered via catheter-directed technique into the thrombus. Thrombolytic 
therapy is recommended for patients with iliofemoral DVT and selected 
patients with infrainguinal DVT who are severely symptomatic due to multilevel 
thrombosis. [Q9: C, D] 



References 



1. Kistner RL, Ball JJ, Nordyke RA, Freeman GC. Incidence of pulmonary embolism in the course of 
thrombophlebitis. Am J Surg 1972;124:169-76. 

2. Collins R, Scrimgeour A, Yusuf S, Peto R. Reduction in fatal pulmonary embolism and 
venous thrombosis by perioperative administration of subcutaneous heparin: overview of results 
of randomized trials in general, orthopaedic, and urologic surgery. N Engl J Med 1988; 
318:1162-73. 

3. Masuda EM, Kessler DM, Kistner RL, Eklof B, Sato DT. The natural history of calf vein thrombosis: 
lysis of thrombi and development of reflux. J Vase Surg 1998;28:67-74. 



402 Vascular Surgery 

4. Lechner K, Pabinger-Fasching I. Lupus anticoagulant and thrombosis: a study of 25 cases and review 
of the literature. Haemostasis 1985;15:254-62. 

5. Baker WH, Potthoff WP, Biller J, McCoyd K. Carotid artery thrombosis associated with lupus antico- 
agulant. Surgery 1985;98:612-15. 

6. Li GC, Greenberg CS, Currie MS. Procainamide-induced lupus anticoagulant and thrombosis. South 
Med J 1988;81:262-4. 

7. Asherson RA, Chan JK, Harris EN, Gharavi AE, Hughes GR. Anticardiolipin antibody, recurrent 
thrombosis, and warfarin withdrawal. Ann Rheum Dis 1985;44:823-5. 

8. Khamashta MA, Cuadrado MJ, Mujic F, Taub NA, Hunt BJ, Hughes GR. The management of throm- 
bosis in the antiphospholipid-antibody syndrome. N Engl J Med 1995;332:993-7. 

9. Bertina RM, Koeleman BPC, Koster T, Rosendaal FR, Dirven RJ, de Ronde H, et al. Mutation in blood 
coagulation factor V associated with resistance to activated protein C. Nature 1994;369:64-7. 

10. Hirsh DR, Mikkola KM, Marks PW, Fox E, Dorfman D, Ewenstein B, Goldhaber S. PE and DVT 
during pregnancy or oral contraceptive use: prevalence of factor V Leiden. Am Heart J 
1996;131:1145-8. 

11. Ridker PM, Glynn RJ, Miletich JP, Goldhaber SZ, Stampfer MJ, Hennekens CH. Age-specific incidence 
rates of venous thromboembolism among heterozygous carriers of factor V mutation. Ann Intern 
Med 1997;126:528-31. 

12. Cushman M, Rosendaal FR, Psaty BM. Factor V Leiden is not a risk factor for arterial vascular 
disease in the elderly: result from the cardiovascular health study. Thromb Haemost 1998;79:912-15. 

13. Geerts WH, Heit JA, Clagett GP, Pineo GF, Colwell CW, Anderson FA, Wheeler HB. Prevention of 
venous thromboembolism. Chest 2001;119:132S-75S. 

14. Clagett GP, Reisch JS. Prevention of venous thromboembolism in general surgical patients. Results 
of meta-analysis. Ann Surg 1988;208:227-40. 

15. Warkentin TE, Levine MN, Hirsh J, Horsewood P, Roberts RS, Gent M, Kelton JG. Heparin-induced 
thrombocytopenia in patients treated with low-molecular-weight heparin or unfractionated heparin. 
N Engl J Med 1995;332:1330-35. 

16. Nicolaides AN, Miles C, Hoare M, Jury P, Helmis E, Venniker R. Intermittent sequential pneumatic 
compression of the legs and thromboembolism-deterrent stockings in the prevention of postoperative 
deep venous thrombosis. Surgery 1983;94:21-5. 

17. Hoek J A, Nurmohamed MT, Hamelynck KJ, Marti RK, Knipscheer HC, ten Cate H, et al. Prevention 
of deep vein thrombosis following total hip replacement by low molecular weight heparinoid. 
Thromb Haemost 1992;67:28-32. 

18. Turpie AG, Levine MN, Hirsh J, Carter CJ, Jay RM, Powers PJ, et al. A randomized controlled trial of 
low-molecular-weight heparin (enoxaparin) to prevent deep-vein thrombosis in patients undergoing 
elective hip surgery. N Engl J Med 1986;315:925-9. 

19. Geerts WH, Code KI, Jay RM, Chen E, Szalai JP. A prospective study of venous thromboembolism 
after major trauma. N Engl J Med 1994;331:1601-6. 

20. Waring WP, Karunas RS. Acute spinal cord injuries and the incidence of clinically occurring throm- 
boembolic disease. Paraplegia 1991;29:8-16. 

21. Myllynen P, Kammonen M, Rokkanen P, Bostman O, Lalla M, Laasonen E. Deep venous thrombosis 
and pulmonary embolism in patients with acute spinal cord injury: a comparison with nonparalysed 
patients immobilized due to spinal fractures. J Trauma 1985;25:541-3. 

22. Warkentin TE: Heparin-induced thrombocytopenia: a ten-year retrospective. Ann Rev Med 
1999;50:129. 

23. Magnani HN. Heparin induced thrombocytopenia (HIT): an overview of 230 patients treated with 
Orgaran (Org 10172). Thromb Haemost 1993;70:554. 

24. Greinacher A, Janssens U, Berg G, Bock M, Kwasny H, Kemkes-Matthes B, et al. Lepirudin (recombinant 
hirudin) for parenteral anticoagulation in patients with heparin-induced thrombocytopenia. 
Circulation 1999;100:587-93. 

25. Decousus H, Leizorovicz A, Parent F, Page Y, Tardy B, Girard P, et al. A clinical trial of vena caval 
filters in the prevention of pulmonary embolism in patients with proximal deep-vein thrombosis. 
N Engl J Med 1998;338:409-15. 

26. Becker DM, Philbrick JT, Selby JB. Inferior vena cava filters: indications, safety, effectiveness. Arch 
Intern Med 1992;152:1985-94. 

27. Duckert F, Muller G, Nyman D, Benz A, Prisender S, Madar G, et al. Treatment of deep vein thrombosis 
with streptokinase. BMJ 1975;1:479-81. 

28. Comerota AJ, Throm RC, Mathias SD, Haughton S, Mewissen M. Catheter-directed thrombolysis for 
iliofemoral deep venous thrombosis improves health-related quality of life. J Vase Surg 
2000;32:130-7. 



43. Primary Varicose Veins 



Michael Dialynas and Stephen G. E. Barker 



A 39-year-old female sales assistant presented with a lengthy history of a dull 
ache affecting her right leg. She complained also of occasional night cramps and 
mild ankle swelling. Usually, the ache began mid-morning, became progressively 
worse during the course of the day and was exacerbated further with prolonged 
standing and in warmer weather. She had noticed also that the symptoms 
became worse following strenuous exercise. The patient attributed these symp- 
toms to her rather unsightly, dilated, tortuous veins present throughout the leg. 
Otherwise, her medical history was unremarkable. She was not on any regular 
medication. 



Question 1 

At this stage, what else would you like to know from the history? 

A. How long has she been aware of her "varicose" veins? 

B. Are there any other members of the family with varicose veins? 

C. Is there any history of superficial, or deep venous thrombosis? 

D. Is she taking an oral contraceptives? 

The patient had first noticed her varicose veins in her late twenties, but she was 
certain that they had become far more prominent following her second pregnancy. 
During this pregnancy, she was told that she may have had "a clot in the right leg", 
but she could not remember the exact diagnosis or whether she had had any treat- 
ment for it. On current inspection, there were prominent varicose veins affecting 
her right leg, mainly below the knee, on the anteromedial aspect, but with varices 
sited posteriorly, in addition. Multiple cutaneous telangiectases were present. The 
left leg appeared quite normal. Both her mother and eldest sister suffered from vari- 
cose veins. 



403 



404 Vascular Surgery 

Question 2 

Identify which of the following statements are true or false. 

A. Premenopausal women have a higher prevalence of varicose veins. 

B. Age, gender, parity, race and occupation are factors contributing to the develop- 
ment of varicose veins. 

C. Leg oedema is a prominent feature of varicose veins. 

D. In the presence of a varicose long saphenous vein (LSV), the saphenofemoral 
junction is always incompetent. 

E. Cutaneous telangiectases are only ever associated with varicose veins. 

F. Patients with varicose veins are more likely to complain about unsightliness, 
rather than any other specific symptom. 



Question 3 

At this stage how would you assess the patient? 

A. Examine clinically using a tourniquet test. 

B. Examine clinically using a hand-held Doppler. 

C. Request a venogram. 

D. Measure the ankle brachial pressure index (ABPI). 

E. Request a venous duplex scan of both legs. 

On examination, with the patient lying flat and having raised the limb to drain 
the blood within the veins, a tourniquet was placed around the upper thigh (the 
Trendelenburg test). On standing the patient, varices below the applied tourniquet 
filled very quickly. No further distension was noticed, however, when the tourniquet 
was released. A second application of the tourniquet, just above the knee, controlled 
those varices sited anteromedially on the leg, but did not control those sited more 
posteriorly. Examination using a hand-held, 8-mHz, directional Doppler system 
suggested reflux in the lower LSV and in the short saphenous vein (SSV), behind the 
knee. However, the Doppler findings were not suggestive of reflux at the saphe- 
nofemoral junction, in the groin. 

Later, a formal venous duplex ultrasound scan was requested. This confirmed 
incompetence and reflux within the LSV, but in the presence of a competent saphe- 
nofemoral junction. An incompetent perforating vein was demonstrated just above 
the knee (a Hunterian perforator) and its position was recorded. Other perforating 
veins lower down the leg appeared normal, with no evidence of reflux. Varices asso- 
ciated with the SSV were identified, together with an incompetent saphenopopliteal 
junction, sited some 5 cm above the popliteal skin crease. The deep venous system 
was patent and competent, with no signs of other abnormality. Hence, the varicose 
veins present were classified as "primary". 



Primary Varicose Veins 405 

Question 4 

Identify which of the following statements are true or false. 

A. The Trendelenburg test is a reliable method for assessing valvular incompetence 
and reflux. 

B. Incompetent perforating veins are detected accurately with clinical examination 
alone. 

C. Duplex ultrasound scanning constitutes the "gold standard" for investigating 
venous anatomy, incompetence and reflux. 

D. Descending phlebography is a more accurate method of assessing the deep 
venous system than duplex ultrasound scanning. 

E. The Hunterian perforating vein is the only common "communicating" vein in 
the lower limb. 



Qustion 5 

How would you treat this patient? 

A. There is no clear indication to offer any treatment. 

B. Offer injection sclerotherapy. 

C. Offer below-knee compression stockings. 

D. Offer surgery. 

The patient was advised that the "disease" progression is very slow. However, the 
varicosities are likely to "deteriorate" with age and therefore it is likely that some 
active measures would need to be taken. Patient compliance and some change in 
lifestyle, along with conservative measures such as compression stockings, seem to 
provide symptomatic relief in the majority of cases (often reducing the need for 
surgery). 

The patient opted for surgical treatment, under general anaesthesia. On admis- 
sion and after obtaining patient consent, the operating surgeon marked the position 
of the varicose veins, with an indelible ink pen, with her standing. The previous 
duplex ultrasound scan report was reviewed, with attention paid in particular to the 
position of the incompetent perforating vein and the saphenopopliteal junction. 

An appropriate consent was obtained from the patient. On transfer to the operat- 
ing theatre, the patient was placed prone. The skin was prepared with a non- 
alcoholic, iodine-based solution. Sterile drapes were placed as appropriate first, for 
exploration of the saphenopopliteal junction. A transverse incision was made above 
the popliteal skin crease, where the saphenopopliteal junction had been identified 
previously by the duplex ultrasound scan. The saphenopopliteal junction was 
identified formally, with the artery adjacent. The SSV was ligated and divided 
almost flush to the junction. A short segment of SSV was excised (although the SSV 
was not formally stripped). Multiple avulsions of the previously marked varicosities 
were performed (using an Oesch hook), through 1-2-mm skin incisions. After 



406 Vascular Surgery 

closure of wounds, the patient was placed supine, with the legs abducted and with 
head-down tilt (Trendelenburg position), to help reduce any intraoperative blood 
loss. The skin was re-prepared and new drapes were placed appropriately. A short, 
oblique incision was made in the groin crease, near to the saphenofemoral junction, 
and by careful dissection, the saphenofemoral junction was identified formally. 
Subsequently, all tributaries to the LSV were ligated (using absorbable sutures) and 
divided and in particular, the deep pudendal vein(s). Formal exposure of a small 
length of the femoral vein enabled ligation of a small tributary. Following this, the 
saphenofemoral junction was ligated almost flush to the femoral vein and the LSV 
divided. A disposable vein stripper was inserted intraluminally and advanced down 
the LSV, to just below the knee. A small skin incision (in Langer's lines) was made 
down onto the palpable tip of the stripper, to exit it. The LSV was stripped down- 
wards. Varicose veins lower in the leg were avulsed via multiple, 1-2-mm skin inci- 
sions, using an Oesch hook. The Hunterian perforator noted and marked previously 
was ligated individually and subfascially through a small incision placed directly 
over it. The groin wound was closed (inclusive of a subcuticular suture) and Steri- 
Strips alone were applied to all the small stab incisions. A local anaesthetic agent 
was injected in the wounds to facilitate postoperative pain control. To finish and 
with all wounds closed, the whole leg was bandaged firmly, including the foot. Later, 
the bandage was exchanged for a full-length, class II graduated compression stock- 
ing that the patient was advised to wear for 6 weeks. The patient was reviewed 6 
weeks postoperatively. All the wounds had healed and there were no residual 
varicosities. 

The patient re-presented 5 years following the initial surgery with recurrent vari- 
cose veins in the same leg. She described symptoms similar to what she had prior to 
the initial operation, though to a lesser degree. The patient was concerned and 
requested treatment. 



Question 6 

How would you assess the patient? 

A. Examine clinically using a tourniquet test. 

B. Examine clinically using a hand-held Doppler. 

C. Request a venogram. 

D. Request photoplethysmography (PPG). 

E. Request a duplex scan. 

The patient was clinically examined using the tourniquet test and also a hand- 
held Doppler. A long varicose vein in the anteromedial aspect of the thigh and also a 
few varicose veins in the anteromedial aspect of the lower leg were noted. Some 
reflux was evident in the groin but not in the popliteal fossa when examined with 
the hand-held Doppler. 

A duplex scan was requested to assess the anatomy of the varicosities further. 
This demonstrated a small anterior, incompetent, thigh vein originating from the 
groin (communicating with the femoral vein) that was feeding the lower varicosi- 
ties. No perforators or saphenopopliteal incompetence were noted. 



Primary Varicose Veins 407 

Question 7 

How would you treat the patient given these findings? 

A. Offer conservative treatment with compression hosiery. 

B. Proceed to surgery. 

C. Review in 6 months. 

D. Offer sclerotherapy. 

The options of treatment were again discussed with the patient as in the initial 
consultation. It was explained that surgery is thought to provide a more definitive 
treatment than other methods, albeit with an increased risk of complications for 
recurrence. The patient was not keen on symptomatic control with compression 
stockings. She wished to have more definitive treatment than that with sclerother- 
apy or local ablation of the visible varicosities and opted for surgery under general 
anaesthesia. 

The patient was appropriately consented and informed about the increased risks 
of recurrent groin dissection, in particular the risk of damaging the femoral vein. 
Prior to the operation the recurrent groin vein was marked over the skin of the 
patient with a permanent marker pen, using duplex scan, to facilitate dissection. 

The patient was then placed in the operating theatre, the skin prepared and draped 
as described previously for the initial operation. An incision was made over the previ- 
ous scar and careful groin dissection was undertaken starting more medially where 
there was more virgin territory towards the previously marked vein. The recurrent 
anterior thigh vein was identified and ligated flush with the carefully exposed femoral 
vein. No further tributaries were noted. The recurrent vein was then stripped to as low 
as possible as described before. Multiple avulsions of the rest of the varicosities were 
also undertaken to complete the procedure. The leg was then bandaged as described 
in the initial operation. The bandages were later exchanged to a full-length class II 
graduated compression stocking that the patient was advised to wear for 6 weeks. 



Commentary 

This case is representative of the very many patients seen in varicose vein clinics. 
Patients with varicose veins are very likely to complain about unsightliness, but in 
addition, may focus on symptoms such as aching (less commonly pain), night 
cramps, "itching", mild ankle swelling and, perhaps, bleeding from varicosities 
(usually following trauma). [Q2: F = true] Symptoms often become worse towards the 
end of the day and in warmer weather. In women, varicose veins may become more 
prominent during menstruation and commonly during pregnancy [1,2]. 

Varicose veins associated with the LSV present usually, in the anteromedial and 
posterior aspects of the thigh and in the suprapatellar region. In the lower leg, LSV- 
associated varicosities are commonly seen antero- and posteromedially. SSV vari- 
cosities are most commonly situated along the posterolateral aspect of the lower leg. 
The presence of significant lower limb ankle oedema is not a common feature of 
varicose veins and more general causes such as cardiac failure, nephrotic syndrome, 
or other fluid retention syndromes should be considered [1,2]. [Q2: C= false] 



408 Vascular Surgery 

Superficial thrombophlebitis of varicose veins is not uncommon. The patient may 
present with tenderness along the line of any inflamed superficial vein. Rarely, 
superficial thrombophlebitis may extend to the femoral vein, resulting in a deep 
venous thrombosis (DVT). 

Cutaneous telangiectases (dilated, intradermal venules) may be present. They are 
more common in women and often develop during pregnancy or at the menopause. 
They may extend to cover extensive regions of the leg, causing blue-purple discol- 
oration of the skin. They are not true varicose veins, but represent a separate and 
distinct entity which may, or may not, be directly associated with varicose veins. 
Paradoxically, they can appear more profoundly following varicose vein surgery [1, 
2].[Q2:E = false] 

Prolonged "venous hypertension" within the varicosities can cause skin changes 
later on, mainly around the lower third of the leg (the "gaiter area"), which can 
range from mild erythematous pigmentation (with haemosedirin deposition), to 
full-blown lipodermatosclerosis (although this is more commonly associated with a 
post-phlebitic limb), or even ulceration. Occasionally, lipodermatosclerosis may 
appear in an acute form as a painful, thickened and sometimes raised tender area 
over the lower leg, with no associated pyrexia, or lymphadenopathy. It can resolve 
spontaneously, or progress to a more chronic form [3]. 

A number of epidemiological studies from both Europe and North America have 
shown the prevalence rate of varicose veins within the general population to be 
approximately 2 per cent. Women seem to be affected more than men. [Q2: A = true] 
However, the incidence of varicose veins seems to increase with age, with a peak 
between 50 and 60 years [4]. Being multiparous, having an occupation involving 
prolonged standing and being of excessive height and weight are all thought to con- 
tribute to the development of varicose veins. [Q2: B = true] Post-thrombotic damage 
to deep veins, pelvic tumours and arteriovenous fistulas are all implicated in the 
development of (secondary) varicose veins. All patients presenting with primary 
varicose veins should have a careful history taken and be examined to exclude the 
(rare) presence of congenital syndromes (such as Klippel-Trenaunay and 
Parkes-Weber syndromes) and more importantly, any history of previous DVT. 
Even though the genetic predisposition for varicose veins seems uncertain, there 
does appear to be a strong association within families. [Q1: A, B, C] 

Primary varicose veins are not always associated with an incompetent saphe- 
nofemoral junction. A recent survey of patients attending a major teaching hospital 
in London demonstrated (on duplex ultrasound scanning) that 42 per cent of cases 
presenting with primary varicose veins had in fact an intact saphenofemoral junc- 
tion [5]. Varicosities can appear in isolation, or might be fed from incompetent per- 
forating veins (including Hunterian and Boyd's perforators and those found at 
various intervals above the medial malleolus), in addition to those found because of 
saphenofemoral incompetence. [Q4: E = false] 

The distribution of varicosities down the leg is unlikely to define with certainty 
their origin from either the LSV, or SSV. When examining patients, the tourniquet 
test (Trendelenburg test), being operator dependent, can not always provide accu- 
rate information as to the presence and location of incompetent perforating veins 
[6] . [Q4: A, B = false] The use of a hand-held, directional Doppler ultrasound system 
is a quick and easy way to assess incompetence of the LSV when clinical signs are 
equivocal, but it is not reliable when assessing for perforating veins, or the SSV. 
There are a number of investigations available to assess venous insufficiency 
further. The use of invasive diagnostic tests such as phlebography, to assess incom- 



Primary Varicose Veins 409 

petence of perforating veins and the deep venous system, even though commonly 
used in the past, is probably not justified nowadays. Although phlebography is con- 
sidered specific, its specificity is poor when compared to duplex ultrasound scan- 
ning [7]. [Q4: D = false] The optimum, non-invasive method to investigate venous 
anatomy, reflux and incompetence is duplex ultrasound scanning. Its value in also 
assessing recurrent varicose veins, short saphenous incompetence and perforating 
vein incompetence is well established and its use should be regarded as the new 
"gold standard" for venous examination [8]. [Q4: C = true] [Q3: A, B, D] 

When treating varicose veins, the clinical objectives should be satisfactory cosme- 
sis, relief of symptoms and prevention of complications and recurrence. Treatments 
should be discussed on an individual basis and may be non-surgical or surgical. 
Simple reassurance maybe all that is required for some patients. For others, appli- 
cation of fitted, elasticated, graduated compression stockings may provide enough 
relief to avoid surgical or other intervention. Injection sclerotherapy aims to oblit- 
erate varicose veins by placing an irritant solution directly within the vein lumen, 
causing a local chemical reaction, promoting thrombosis. There are a number of 
commercially available sclerosing agents, with more or less similar properties. 
Many advocates for the method use it to treat all degrees of varicosity, with the 
"French school" injecting sclerosant directly into the uppermost portion of the LSV 
to obliterate its termination. However, the results are highly variable and operator- 
dependent and there is always the risk of DVT if the agent disperses into the deep 
venous system. Extravasation of the sclerosant can cause local irritation and pain 
and on occasion, pigmentation and ulceration of the overlying skin. Recurrences 
with this method of treatment are frequent [9, 10]. [Q5: B,C, D] 

Surgical treatment for long and short saphenous varicose veins is performed 
(usually) under general anaesthesia. The procedure can be undertaken on a day case 
basis for those patients who are suitable (often only single leg). Various techniques 
such as ligating the saphenofemoral junction only, obliterating the LSV near to the 
junction using either chemical agents, or even electrical current, have been used as a 
method of treatment with varying success. The most acceptable surgical procedure, 
however, is near-flush ligation and division of the saphenofemoral junction, ligation 
of all the tributaries and stripping of the LSV to just below the knee. The procedure 
is completed with ligation (or avulsion) of incompetent perforating veins and avul- 
sion of varicosities via stab incisions [11]. 

Preoperative identification and positional marking of incompetent perforating 
veins, with duplex ultrasound, is an imperative step to help prevent recurrence and 
facilitates the ligation, avoiding unnecessarily long scars [12]. 

SSV reflux and saphenopopliteal incompetence is treated using similar principles. 
Near-flush ligation of the saphenopopliteal junction is important. However, as the 
anatomical position of the junction can vary considerably, preoperative marking 
using duplex ultrasound scanning is essential. The junction may be "behind the 
knee", in the skin crease, but often, it can be many centimetres higher. Definitive 
exposure of the main popliteal vein, adjacent to the artery, should be attempted. 
Stripping of the SSV is considered unnecessary by many surgeons, but excision of a 
2-5-cm length of vein helps prevent the possibility of recurrence. 

Significant postoperative complications following primary varicose vein surgery 
include: haematomas from slipped ligatures, unligated vessels, injury to the femoral 
vein, injury to the femoral artery and development of a lymphocele. During 
exploration of the saphenopopliteal junction, injury to the popliteal vein and artery 
can occur. Furthermore, damage to the sural, the saphenous and lateral popliteal 



410 Vascular Surgery 

nerves can occur while stripping veins and avulsing varicosities. The development 
of a DVT is a potential, but fortunately rare, complication of varicose vein surgery. 
More general surgical complications such as wound infection and dehiscence can 
occur also. Damage to major vessels should be repaired by direct suturing, patch- 
ing, or even grafting where necessary. Lymphoceles, in general, regress with time. 
Sural or more commonly saphenous nerve damage was a frequent complication 
when the LSV used to be stripped upwards from the ankle. However, stripping 
downwards to just below the knee is safer, as the nerve is quite distant from the vein 
at this level. Care should be taken when avulsing varicosities as it is possible to 
damage adjacent nerves [13]. 

Recurrence of varicose veins constitutes approximately 20 per cent of the total 
operations undertaken for varicose veins overall. Various reports have demon- 
strated recurrence rates of anywhere between 5 and 80 per cent, within 5 years [14, 
15]. Failure to ligate and divide the LSV and all its tributaries has been shown to be 
associated with the highest recurrence rates. Recurrences occur also when incompe- 
tent perforating veins go unrecognised. An alternative mechanism to explain recur- 
rence, "neovascularisation", was suggested by Glass; new veins could develop 
through growth of newly formed vessels, or through widening of small, pre-existing 
collaterals. Re-canalisation of ligated, but undivided, vessels has also been sug- 
gested as a possible mechanism. Clinical examination follows the same principles as 
for primary varicose veins. [Q6: A, B, E] The management of recurrent varicose veins 
follows the same pathway as with primary varicose veins, with surgery being the 
most definitive treatment. Many patients seek simply reassurance about the poten- 
tial risks of ulceration and deep venous thrombosis and decide to have only symp- 
tomatic treatment with graduated compression stockings. 

The indications for sclerotherapy for primary and recurrent varicose veins 
depend upon the size, site and extent of the varicose veins, the presence of reflux 
points as well as the functional state of the main vein trunks. If the main vein trunks 
are incompetent, sclerotherapy on its own is unlikely to provide a long-term satis- 
factory result. Here, it should be considered as a useful adjuvant therapy to surgery, 
but not a complete treatment on its own. [Q7: A, B, D] 

With recurrent varicose veins, a preoperative assessment using duplex ultra- 
sound scanning is essential. Surgical treatment, if indicated, should be performed 
by an experienced vascular surgeon, as injury to the major vessels in the groin is far 
more likely [16, 17]. 

The most effective and long-lasting treatment for primary and recurrent varicose 
veins remains surgery, even though it can become a compromise between the 
attempt to remove the diseased veins and the need for an acceptable cosmetic 
result. It is imperative that the operation is performed properly in the first attempt 
by an appropriately qualified surgeon following all the principles outlined previ- 
ously. Further research into the causes of recurrence may provide information on 
how to manage the condition more effectively. 



References 

1. Browse NL. Diseases of the veins, 2nd edn. London: Arnold, 1999. 

2. O'Leary DP, Chester JF, Jones SM. Management of varicose veins according to reason for presenta- 
tion. Ann R coll Surg Engl 1996;78:214-16. 

3. Browse NL. Venous ulceration. BMJ 1983;286:1920-3. 



Primary Varicose Veins 411 

4. Callam MJ. Epidemiology of varicose veins. Br J Surg 1994;81:167-73. 

5. Hollingsworth SJ, Tang CB, Barker SGE. Phlebology 2000 

6. Browse NL. Can we diagnose long saphenous incompetence correctly? In: Negus D, Jantet G, editors. 
Phlebology '85. London: John Libbey, 1985;101-5. 

7. Baker SR, Burnand KG, Sommerville KM, Lea Thomas M, Wilson NM, Browse NL. Comparison of 
venous reflux assessed by duplex scanning and descending phlebography in chronic venous disease. 
Lancet 1993;341:400-3. 

8. Coleridge-Smith PD, Scurr JH. Duplex scanning for venous disease. Curr Pract Surg 1995;7:182-8. 

9. Davy A, Ouvry P. Possible explanation for recurrence of varicose veins. Phlebology 1986;1:15. 

10. MacGowan WAL. Sclerotherapy: prevention of accidents. A review. J R Soc Med 1985;78:136-7. 

11. Sarin S, Scurr JH, Coleridge-Smith PD. Stripping of the long saphenous vein in the treatment of 
primary varicose veins. Br J Surg 1994;81:1455-8. 

12. 

13. Tennant WG, Ruckley CV. Medicolegal action following treatment for varicose veins. Br J Surg 
1996;86:291-2. 

14. Juhan C, Haupert S, Miltgen G, Barthelemy P, Eklof B. Recurrent varicose veins Phlebology 
1990;5:201-11. 

15. Negus D. Recurrent varicose veins: a national problem. Br J Surg 1993;80:823-4. 

16. Campbell WB. Varicose veins. BMJ 1990;300:763-4. 

17. Jones L, Braithwaite BD, Selwyn D, Cooke S, Earnshaw JJ. Neovascularisation is the principal cause 
of varicose vein recurrence: results of a randomised trial of stripping the long saphenous vein. Eur J 
Vase Endovasc Surg 1996;12:442-5. 



44. Venous Ulcers Associated with Deep 
Venous Insufficiency 

Seshadri Raju 



A 46-year-old female schoolteacher and non-smoker presented with an ulcer on 
the medial side of the ankle. The ulcer had persisted for the past year despite 
compressive dressings at a hospital wound care center. Ulcers in the same 
general area had occurred intermittently in the past but had healed with local 
wound care and dressings. The ulcer was very painful, particularly with depend- 
ency of the leg (7/10 over a visual analogue scale) and frequently at night. The 
patient had made a habit of elevating her legs during the day whenever feasible, 
and to sleep with her legs elevated on a pillow at night. She had been using a non- 
steroidal anti-inflammatory drug once or twice a day at work for pain relief, but 
lately a narcotic prescribed by her physician was required for sleep at night. Even 
so, on some nights, she had to "walk off the pain for twenty to thirty minutes 
before she could fall asleep. 

Past medical history: She had been hospitalized on two occasions during the 
past year for cellulitis of the leg, which required intravenous antibiotics. Her 
saphenous vein was stripped 15 years ago when the ulcer initially appeared. This 
resulted in healing of the ulcer but it recurred 2 years later. During adolescence, 
she sustained a closed tibial fracture of the same extremity during a ski accident 
and was in a plaster cast and crutches for several weeks. 

Family history: No one in the family had varicose veins or deep venous 
thrombosis. 

Examination: The patient was found to be healthy except for the affected 
extremity, which had a large 5 x 10-cm indolent ulcer on the medial aspect of the 
lower third of the leg. The ulcer bed had clean granulation tissue with serous 
drainage. The ulcer was surrounded by a broader area of hyperpigmentation in 
the "gaiter" area. No obvious varicosities or "blow outs" were noted. Good pedal 
pulses were present. 



Question 1 

Which of the following is least likely in this patient? 
A. "Primary" deep vein valve reflux. 

413 



414 Vascular Surgery 

B. Post-thrombotic syndrome. 

C. Popliteal artery entrapment. 

D. Recurrent saphenous reflux from neovascularization. 

E. Perforator incompetence. 

The patient was referred to the vascular laboratory, where a detailed duplex venous 
examination was performed. Extensive reflux throughout the deep venous system in 
the affected extremity was found. Both the femoral and popliteal valves were 
refluxive, with valve closure times of 7 s and 6 s, respectively. The great saphenous 
was confirmed absent with no evidence of tributary or collateral reflux around the 
short sapheno-femoral stump. Neovascularization was not detectable. No significant 
perforator reflux was found, and the short saphenous vein was not refluxive. The 
deep venous system was widely patent without evidence of prior thromboses. 

Air-plethysmography (APG) results were as follows: venous filling index 
(VFI 90 ) 7 ml/s; venous volume (VV) 135 ml; ejection fraction (EF) 60 percent; 
residual volume fraction (RVF) 48 percent. 

Based on the above findings and the clear failure of conservative therapy to heal 
the ulcer, surgical intervention was discussed with the patient. She consented to this 
approach. Other preoperative work-up included a hypercoagulation profile and 
ascending and descending venography. 

Question 2 

Which of the following statements is true? 

A. Duplex is more specific than descending venography in assessing reflux. 

B. Valve closure time (VCT) is a reliable quantitative measure of reflux. 

C. Venous filling index (VFI 90 ) with APG correlates best with ambulatory 
venous pressure. 

D. Absence of varicosities or "blow outs" on physical examination rules out neo- 
vascularization or perforator reflux as a significant source of reflux. 

E. Palpable pedal pulses rule out arterial insufficiency as the etiology in patients 
with painful leg ulcer. 

The patient underwent internal valvuloplasty (Kistner technique) of the femoral 
vein valve under general anesthesia. Postoperative recovery was uneventful. DVT 
prophylaxis included low-molecular-weight heparin (LMWH) started preopera- 
tive^ and continued until discharge, intraoperative intravenous heparin (5000 
units), and daily warfarin sodium. Pneumatic compression was started during 
surgery and continued postoperatively when not ambulatory. She was discharged 
on 5 mg warfarin with instructions to the local physician to maintain the interna- 
tional normalized ratio (INR) at or above 2.5 for 6 weeks, after which the dosage 
could be lowered for a target INR of 1.7-2.0. The patient was instructed to wear 
elastic stockings for at least 6 weeks on a daily basis, after which she could adjust 
the usage as desired. 



Venous Ulcers Associated with Deep Venous Insufficiency 41 5 

The patient was seen on follow-up at 6 weeks, at which time the surgical incision 
was well healed and the ulcer had become epithelialized to 90 percent of the original 
surface area. She requested and was granted permission to go back to full-time 
work. When seen in follow-up at 4 months, the patient reported that the ulcer had 
healed completely 2 weeks after the first clinic visit and had remained healed since. 
She was free of pain and had abandoned regular use of her stockings. She found it 
necessary to use them only occasionally when she expected her day to be more 
strenuous than usual. Physical examination revealed good-quality skin coverage 
over the previous ulcer, and the limb was free of edema. Interval follow-up duplex 
examination showed competence of the repaired femoral valve with valve closure 
time of 0.4 seconds. Popliteal valve reflux was unchanged. Postoperative APG 
showed that the VFI 90 had been nearly normalized at 2.3 ml/s. Other values were 
essentially unchanged from preoperative levels. 

Question 3 

Which of the following is not true? 

A. Postoperative DVT (30 day) is relatively rare after valve reconstruction proce- 
dures for correction of "primary" valve reflux. 

B. Arm swelling occurs infrequently after axillary vein harvest for valve 
reconstruction. 

C. Valve reconstruction is contraindicated in post-thrombotic veins. 

D. Saphenous vein ablation can be safely undertaken in chronic deep venous 
obstruction (secondary saphenous varix). 

E. In combined obstruction/reflux, stent placement to correct the obstruction 
alone often results in healing of stasis ulceration. 



Commentary 

The differential diagnosis of venous ulcers includes ischemic ulcers, diabetic foot 
ulcers, ulcers related to vasculitis from hypertension or other causes, ulcers 
related to connective tissue disorders (rheumatoid arthritis, scleroderma, etc.), 
neuropathic ulcers, Marjolin's ulcer, and numerous other conditions that are clin- 
ically quite rare. Popliteal vein (not artery) entrapment is a rare cause of venous 
ulcers [1]. The clinical features of venous ulcers are so characteristic and obvious 
that a positive diagnosis can be made on the basis of clinical examination alone in 
all but a few cases. When doubt exists, or when combined pathologies are sus- 
pected, a punch biopsy of the skin should be performed without hesitation to 
clarify the situation. Relevant testing for specific connective tissue, immunological 
or hematological conditions maybe required in some cases. Venous ulcers are dif- 
ferentiated quite easily from arterial (ischemic) ulcers in most instances. The 
former are indolent and recurring with episodes of healing and breakdown and 
are generally confined to the gaiter area of the leg. In contrast, the arterial ulcer is 
progressive without periods of remission and has a wider distribution in the leg 



416 Vascular Surgery 

with characteristic gangrenous or ischemic appearance devoid of granulation 
tissue and covered with necrotic tissue. There is seldom the surrounding hyper- 
pigmentation or dermatitis that occurs so commonly with venous ulcers. Palpable 
pedal pulses virtually rule out ulcers of ischemic origin, with the notable exception 
of diabetic foot ulcers and less common entities in which vasculitis or small-vessel 
disease is often implicated (e.g. collagen disorders such as scleroderma and 
rheumatoid arthritis). It is usually possible, however, to narrow down the possibil- 
ities by a combination of clinical features (history, appearance and location of the 
ulcer), skin biopsy, and specific testing directed toward suspected non-venous 
pathology. Ankle/arm arterial index and toe pressure measurements may be 
required in some cases to clarify the issue. Because of their wide prevalence, 
venous ulcers can and do occur in combination with the other pathologies listed 
above. To establish the presence of venous ulcers in concert with other non- 
venous pathology, it is necessary to confirm that significant reflux is present based 
on venous duplex examination and venous hemodynamic tests such as ambula- 
tory venous pressure measurement and/or air plethysmography. In combined 
arterial/venous ulcers, treatment should be directed initially towards improving 
arterial perfusion. 

However obvious the diagnosis, patients with venous ulcers should be evaluated 
through a detailed assessment protocol to assess severity and form a base for later 
outcome assessment. Use of CEAP classification [2] and venous clinical severity 
scoring [3] provides a standardized format to accomplish this. Quality-of-life 
assessment methodologies [4] in venous disease have been validated and provide a 
way for outcome assessment from the patient's perspective. [Q1: C] 

Many patients with chronic venous insufficiency will not volunteer information 
such as relief of leg pain with leg elevation and stocking use, night leg cramps and 
restless legs, or their developed habit of sleeping with the leg elevated at night, 
unless specifically asked. Perhaps because of the chronicity of the condition, these 
details have become an integral part of their daily lives and may not be mentioned 
as complaints without direct questioning. Even potentially important information, 
such as previous attacks of cellulitis or "phlebitis" that occurred years or decades 
ago and required hospitalization and a period of anticoagulant treatment may not 
be forthcoming unless specifically inquired, because the patient has forgotten the 
episode or does not consider it relevant to their current condition. Besides solidify- 
ing the diagnosis of venous ulcer, such information maybe important in narrowing 
down the differential diagnosis in doubtful cases or combined pathologies. For 
example, ischemic rest pain at night is often relieved by hanging the leg over the 
side of the bed at night, whereas patients with venous pain seldom resort to this 
practice. Pain of claudication (arterial or venous) worsens with ambulation, 
whereas patients with limb pain from venous reflux have often learned to "walk off 
their nocturnal pain. Venous claudication is estimated to occur in about 15 percent 
of patients with chronic venous insufficiency. Climbing up stairs is particularly 
difficult for these patients. Pain out of proportion to clinical signs is a characteristic 
of deep venous pathology. Pain, nocturnal leg cramps or restless legs may be the 
only clinical feature(s) in some patients. Recording the level of pain preoperatively 
by a visual analogue scale [5] is a simple reliable tool in severity assessment. The 
type and frequency of analgesic use (narcotic, non-narcotic, non-steroidal) is also 
useful. Past and current list of medications, particularly estrogen-type hormones 
and anticoagulants/platelet inhibitors, are relevant parts of the history and useful 
information in future management. 



Venous Ulcers Associated with Deep Venous Insufficiency 41 7 

Limb swelling is a frequent manifestation of venous disease. It is hard to quantify 
by examination except in very gross terms. Plethysmographic techniques including 
the commonly used limb circumference measurement are unreliable as swelling is 
quite variable during the day with the extent of orthostasis. Patients' own percep- 
tion of limb swelling is strongly influenced by the degree of accompanying pain. 
Patients themselves may not be aware of swelling obvious to the examiner if pain- 
less; conversely, even mild swelling when painful, may be rated as severe by the 
patient. For these reasons, quantification of swelling either by history or by 
examination is subject to considerable variance and error. Although some clinical 
features are described as unique to lymphedema in texts, differentiation of venous 
from lymphatic swellings on clinical grounds alone is generally not possible. 
Furthermore, the two pathologies frequently coexist. Lymphatic dysfunction 
appears to be secondary to venous obstruction in many cases; relief of venous 
obstruction can reverse the lymphatic dysfunction [6]. A thorough venous investi- 
gation is essential even when lymphoscintigraphy is abnormal. 

The investigation of venous ulcers is directed toward positive establishment of 
venous etiology, identification of regional pathology, and assessment of hemody- 
namic severity. Hypercoagulability work-up provides guidance to the institution of 
anticoagulation, its duration and intensity. Duplex examination has replaced 
venography as the primary investigation for both screening and definitive assess- 
ment of chronic venous insufficiency. Overall accuracy of duplex ultrasound is 
superior to that of descending venography in the assessment of reflux [7, 8]. Duplex 
examination in the erect position yields more accurate results than does examina- 
tion in the sitting or recumbent position [9]. Quick inflation/deflation cuffs with 
pressures set for various levels provide for standardized compression maneuvers 
and allow measurement of valve closure times; reflux is present when these exceed 
threshold values for the various valve stations. Disappointingly, valve closure times 
do not correlate with clinical or hemodynamic severity of reflux [10] and cannot be 
used in a quantitative way as originally hoped. The size and location of perforators 
can be assessed by duplex and is superior to physical examination. Patency of 
venous structures can be confirmed positively and post-thrombotic changes can be 
identified. Despite evolving refinement, duplex remains a largely qualitative mor- 
phologic technique. 

Descending venography can document reflux through valve stations. The best 
results are obtained when the test is performed in the near-erect position with stan- 
dardized Valsalva maneuver [11]. Comparison with duplex has led to the realization 
that the test, though sensitive, is not very specific. Descending venography is easily 
combined with transfemoral ascending venogram for assessment of the iliac veins, 
which may not visualize adequately by pedal injections of contrast. Even trans- 
femoral venogram is only about 50 percent sensitive for detection of iliac vein 
obstructions [12]. Intravascular ultrasound (IVUS) is the gold standard for assess- 
ment of iliac veins for stent placement [13]. 

Ambulatory venous pressure is a global test of venous function. About 
25 percent of patients with venous stasis ulceration have normal ambulatory 
venous pressure measurement parameters. Factors other than venous reflux, 
such as compliance, ejection fraction and arterial inflow, affect ambulatory 
venous pressure [14]. The latter factors are often abnormal in patients with 
chronic deep venous insufficiency. Consequently, ambulatory venous pressure 
often improves after valve reconstruction surgery but complete normalization is 
less frequent [15]. Measurement of ambulatory venous pressure via the dorsal 



418 



Vascular Surgery 



foot vein has been believed to accurately reflect deep venous pressure changes 
with calf exercise. Recent data throw considerable doubt on this long-held 
assumption [16]. 

Air-plethysmography is a non-invasive test of calf venous pump and can be used 
to assess surgical outcome [17]. Residual volume correlates with ambulatory venous 
pressure. However, venous filling index (VFI 90 ) has been a more consistent index of 
reflux with normalization after corrective surgery [18, 19]. 

Venous endothelial injury that occurs with deep venous surgery takes about 6 
weeks to heal [20]. Patients should be anticoagulated adequately during this vulner- 
able period. With proper management, the thromboembolic complication rate is 
surprisingly low [21]. Patients who have suffered from previous bouts of throm- 
boembolism and those with known hypercoagulable abnormalities are under 
increased risk of recurrent thrombosis and are candidates for long-term or even 
permanent anticoagulation. [Q2: A] 

Once thought a rarity, primary deep venous reflux comprises about 30-40 
percent of all deep venous reflux in centers active in deep venous reconstruction. 
Differentiating "primary" deep venous reflux from secondary or post-thrombotic 
reflux is problematic. The presentation and clinical features may be similar. 
Negative history for prior DVT may be unreliable as some thromboses are silent; 
and others might have been overlooked ascribing limb pain to trauma or orthope- 
dic surgery that initiated it. Preoperative venography is a poor guide, and surgical 
exploration of the valve station is often the final arbiter [22]. Some patients with 
primary reflux develop actual distal thrombosis that can be recurrent. Correction of 
proximal reflux in this group of patients may alleviate these recurrent symptoms 




Fig. 44.1. Technique of transcommissural valvuloplasty. Transluminal sutures are placed from outside without 
opening the vein; they traverse the valve attachment lines, taking a bite of the free edge of each redundant 
valve cusp. When tied down, the valve attachment lines are brought together while tightening the valve cusps. 

Reprinted from J Vase Surg, vol. 32, Raju S, Berry MA, Neglen P, Transcommissural valvuloplasty: technique and 
results, pages 969-76, © 2000, with permission from The Society for Vascular Surgery. 



Venous Ulcers Associated with Deep Venous Insufficiency 



419 



[23]. Conversely, deep venous thrombosis initiates by unknown mechanisms even- 
tual development of reflux in adjacent and remote valve stations [24]. 

Correction of primary deep venous reflux by internal valvuloplasty was first 
described by Kistner in 1964. Subsequently, he described an external technique as 
well. A variety of open and closed techniques (Fig. 44.1) for correction of primary 
and post-thrombotic deep venous reflux are currently in use [25, 26]. The internal 
valvuloplasty technique has provided excellent results [21, 23, 27-29] and 
remains the standard (Fig. 44.2). The newer techniques provide a wider choice 
that may be more appropriate in certain circumstances, and yield clinical results 
similar to the original internal technique [21, 30]. Direct valvuloplasty maybe 
feasible in some cases of post-thrombotic reflux where the valves have escaped 
destruction [22, 31]. Axillary vein valve transfer is the standard commonly used for 
correction of post-thrombotic reflux. It can be used with some modifications even 
in trabeculated veins with surprisingly good long-term patency and clinical success 
[32]. Arm swelling after axillary vein harvest is rare. [Q3: C] 

The recent introduction of vein stent technology has decreased the number of 
valve reconstructions in our institution. Post-thrombotic syndrome is known to be 
due to a combination of obstruction and reflux in the majority of patients [33]. Use 
of intravascular ultrasound (IVUS) has shown that Cockett's syndrome (alias 
May-Thurner syndrome) is surprisingly frequent in "primary" reflux as well [34]. 
Correction of the obstructive component with stent placement in thrombotic and 
non-thrombotic cases results in excellent relief of pain and swelling and the patency 
rate is quite good [35]. Even totally occluded iliac veins can be successfully recanal- 
ized and stented [36]. Venous stasis ulcers are generally believed to be the result of 
reflux, not obstruction. Yet this relatively simple percutaneous outpatient stent 
technique results in healing of about 60 percent of venous stasis ulceration [37], 
even when the associated reflux remains uncorrected. Iliac vein stent placement is 



Before 




After 


L X 


A 


L^J, 


ri 




ri 



Plicate 1/5 of valve length 
at each end 




i * 




Side View Side View 
(1800) (goo) 



View after valvuloplasty 



Fig. 44.2. Internal valvuloplasty technique. 



420 Vascular Surgery 

currently the first choice in combined obstruction/reflux, and does not preclude 
later open surgery such as valve reconstruction or venous bypass if the stent were to 
fail. It is often combined with percutaneous laser ablation of the saphenous vein 
when reflexive. Saphenous ablation can be carried out safely, even in the presence of 
chronic deep venous obstruction (secondary varix) [38]. 



References 

1. Raju S, Neglen P. Popliteal vein entrapment: a benign venographic feature or a pathologic entity? 
J Vase Surg 2000;31(4):631-41. 

2. Beebe HG, Bergan JJ, Bergqvist D, et al. Classification and grading of chronic venous disease in the 
lower limbs. A consensus statement. Eur J Vase Endovasc Surg 1996;12(4):487-91; discussion 491-2. 

3. Rutherford RB, Padberg FT, Jr, Comerota AJ, Kistner RL, Meissner MH, Moneta GL. Venous severity 
scoring: an adjunct to venous outcome assessment. J Vase Surg 2000;31(6):1307-12. 

4. Launois R, Rebpi-Marty J, Henry B. Construction and validation of a quality of life questionnaire in 
chronic lower limb venous insufficiency (CIVIQ). Qual Life Res 1996;5:539-54. 

5. Scott J, Huskisson EC. Graphic representation of pain. Pain 1976;2(2):175-84. 

6. Raju S, Owen S, Jr, Neglen P. Reversal of abnormal lymphoscintigraphy after placement of venous 
stents for correction of associated venous obstruction. J Vase Surg 2001;34(5):779-84. 

7. Neglen P, Raju S. A comparison between descending phlebography and duplex Doppler investiga- 
tion in the evaluation of reflux in chronic venous insufficiency: a challenge to phlebography as the 
"gold standard". J Vase Surg 1992;16(5):687-93. 

8. Masuda EM, Kistner RL. Prospective comparison of duplex scanning and descending venography in 
the assessment of venous insufficiency. Am J Surg 1992;164(3):254-9. 

9. Masuda EM, Kistner RL, Eklof B. Prospective study of duplex scanning for venous reflux: compari- 
son of Valsalva and pneumatic cuff techniques in the reverse Trendelenburg and standing positions. 
J Vase Surg 1994;20(5):711-20. 

10. Neglen P, Egger III JF, Raju S. Hemodynamic and clinical impact of venous reflux parameters. J Vase 
Surg 2004;40:303-19. 

11. Morano JU, Raju S. Chronic venous insufficiency: assessment with descending venography. 
Radiology 1990;174(2):441-4. 

12. Negus D, Fletcher EW, Cockett FB, Thomas ML. Compression and band formation at the mouth of 
the left common iliac vein. Br J Surg 1968;55(5):369-74. 

13. Neglen P, Raju S. Intravascular ultrasound scan evaluation of the obstructed vein. J Vase Surg 
2002;35(4):694-700. 

14. Raju S, Neglen P, Carr-White PA, Fredericks RK, Devidas M. Ambulatory venous hypertension: com- 
ponent analysis in 373 limbs. Vase Surg 1999;33:257-67. 

15. Kistner RL, Eklof B, Masuda EM. Deep venous valve reconstruction. Cardiovasc Surg 1995;3:129-40. 

16. Neglen P, Raju S. Ambulatory venous pressure revisited. J Vase Surg 2000;31(6):1206-13. 

17. Christopoulos D, Nicolaides AN, Galloway JM, Wilkinson A. Objective noninvasive evaluation of 
venous surgical results. J Vase Surg 1988;8(6):683-7. 

18. Sakuda H, Nakaema M, Matsubara S, et al. Air plethysmographic assessment of external valvulo- 
plasty in patients with valvular incompetence of the saphenous and deep veins. J Vase Surg 
2002;36(5):922-7. 

19. Criado E, Farber MA, Marston WA, Daniel PF, Burnham CB, Keagy BA. The role of air plethysmog- 
raphy in the diagnosis of chronic venous insufficiency. J Vase Surg 1998;27(4):660-70. 

20. Raju S, Perry JT. The response of venous valvular endothelium to autotransplantation and in vitro 
preservation. Surgery 1983;94(5):770-5. 

21. Raju S, Fredericks RK, Neglen PN, Bass JD. Durability of venous valve reconstruction techniques for 
"primary" and postthrombotic reflux. J Vase Surg 1996;23(2):357-66; discussion 366-7. 

22. Raju S, Fredericks RK, Hudson CA, Fountain T, Neglen PN, Devidas M. Venous valve station 
changes in "primary" and postthrombotic reflux: an analysis of 149 cases. Ann Vase Surg 
2000;14(3):193-9. 

23. Masuda EM, Kistner RL. Long-term results of venous valve reconstruction: a four- to twenty-one- 
year follow-up. J Vase Surg 1 994; 19(3):39 1-403. 

24. Killewich LA, Bedford GR, Beach KW, Strandness DE, Jr. Spontaneous lysis of deep venous thrombi: 
rate and outcome. J Vase Surg 1989;9(l):89-97. 



Venous Ulcers Associated with Deep Venous Insufficiency 421 

25. Raju S, Berry MA, Neglen P. Transcommissural valvuloplasty: technique and results. J Vase Surg 
2000;32(5):969-76. 

26. Raju S, Hardy JD. Technical options in venous valve reconstruction. Am J Surg 1997;173(4):301-7. 

27. Perrin M. Reconstructive surgery for deep venous reflux: a report on 144 cases. Cardiovasc Surg 
2000;8(4):246-55;2000;8:246-55. 

28. Eriksson I. Reconstructive venous surgery. Acta Chir Scand Suppl 1988;544:69-74. 

29. Sottiurai VS. Surgical correction of recurrent venous ulcer. J Cardiovasc Surg (Torino) 
1991;32(l):104-9. 

30. Camilli S, Guarnera G. External banding valvuloplasty of the superficial femoral vein in the treat- 
ment of primary deep valvular incompetence. Int Angiol 1994;13(3):218-22. 

31. Raju S, Fountain T, Neglen P, Devidas M. Axial transformation of the profunda femoris vein. J Vase 
Surg 1998;27(4):651-9. 

32. Raju S, Neglen P, Doolittle J, Meydrech EF. Axillary vein transfer in trabeculated postthrombotic 
veins. J Vase Surg 1999;29(6):1050-62; discussion 1062-4. 

33. Johnson BF, Manzo RA, Bergelin RO, Strandness DE, Jr. Relationship between changes in the deep 
venous system and the development of the postthrombotic syndrome after an acute episode of lower 
limb deep vein thrombosis: a one- to six-year follow-up. J Vase Surg 1995;21(2):307-12; discussion 
313. 

34. Neglen P, Berry MA, Raju S. Endovascular surgery in the treatment of chronic primary and post- 
thrombotic iliac vein obstruction. Eur J Vase Endovasc Surg 2000;20(6):560-71. 

35. Raju S, Owen S, Jr, Neglen P. The clinical impact of iliac venous stents in the management of chronic 
venous insufficiency. J Vase Surg 2002;35(1):8-15. 

36. Raju S, McAllister S, Neglen P. Recanalization of totally occluded iliac and adjacent venous seg- 
ments. J Vase Surg 2002;36(5):903-ll. 

37. Neglen P, Thrasher TL, Raju S. Venous outflow obstruction: an underestimated contributor to 
chronic venous disease. J Vase Surg 2003;38(5):879-85. 

38. Raju S, Easterwood L, Fountain T, Fredericks RK, Neglen PN, Devidas M. Saphenectomy in the pres- 
ence of chronic venous obstruction. Surgery 1998;123(6):637-44. 



45. Venous Ulcers Associated with 
Superficial Venous Insufficiency 

Gudmundur Danielsson and Bo Eklof 



A 59-year-old female secretary was referred for evaluation and treatment of a 
non-healing painful ulcer on the medial aspect of her right lower leg. The ulcer 
had been recurrent almost every year for the past 9 years, often healing during 
the winter season. She had since early childhood been overweight (currently 
87 kg, 170 cm, body mass index 30) and had difficulty in using compression 
stocking. She was otherwise healthy. She had two children, the first child born 
when she was 32 year of age and her second child 2 years later. After the birth of 
her second child she began to notice varicose veins on the lower leg on both sides 
and she often felt tiredness and heaviness in the leg in the afternoon. There was 
no history of deep venous thrombosis. She had been on birth control pills for 
10 years and was currently on hormone replacement therapy because of severe 
postmenopausal symptoms. She had been treated at a local dermatological clinic 
for the past 2 years and was now being evaluated by a vascular surgeon. Clinical 
evaluation showed that she had 5x5 cm well-granulated ulceration above the 
right median malleolus which was surrounded by brownish leathery skin. She 
had slight swelling of the right leg with large varicosities below the knee. The left 
leg had large varicosities below the knee but no swelling or skin changes. Doppler 
examination revealed clear reflux in the groin that could be followed over both 
great saphenous veins (GSV) down the thigh. A possible minimal reflux was also 
noted in the popliteal fossa on the right side, although it was difficult to confirm 
this when the Doppler examination was repeated. Foot arteries were palpable on 
the dorsum of the foot on both sides. 



Question 1 

What should be the next step in this patient evaluation? 

A. Measurement of ankle-brachial index. 

B. Duplex ultrasound scanning of the venous system. 

C. Plethysmography. 

423 



424 Vascular Surgery 

D. Ascending phlebography. 

E. Biopsy of the ulcer. 

Doppler measurement revealed a normal ankle/brachial index with systolic blood 
pressure 130 in both legs and right arm. Duplex ultrasound scanning of the venous 
system performed with the patient in 60° reversed-Trendelenburg position, using 
pneumatic cuff with automatic inflation/release on the lower leg to evaluate the 
reflux, showed bilateral reflux in the GSV, from the common femoral vein down to 
below knee, as well as two incompetent perforator veins on the medial aspect of the 
right calf with a diameter of 4 mm. The diameter of the GSV at the groin was 12 mm 
on the right side and 9 mm on the left side. The reflux time exceeded 4 s in both GSV, 
with peak reverse flow velocity more than 30 cm/s. Reflux less than 0.5 s was noted in 
the lesser saphenous vein on right side. No reflux was present in the deep veins 
except for minimal reflux in the common femoral vein with reflux duration of 
approximately 1 s on the right side. There were no signs of post-thro mbotic changes. 

Question 2 

How should this patient be classified? 

A. Leg ulcer. 

B. Varicose ulcer. 

C. C6,S,Ep,As,p,d,Pr. 

D. C2,3,4b,5,6,S,Ep,As,p,d,Pr2,3,ll,18. 

The patient was classified according to the CEAP (clinical, (a)etiological, anatomical, 
pathophysiological) classification based on history and results of duplex ultrasound. 

Question 3 

Which of the following is not regarded as a risk factor for venous ulcer? 

A. Diabetes. 

B. Essential hypertension. 

C. Smoking. 

D. Overweight. 

E. Resistance to activated protein C. 

Question 4 

What would be appropriate management for the right leg in this patient? 

A. Conservative treatment with below-knee compression bandage, rest and leg 
elevation. 



Venous Ulcers Associated with Superficial Venous Insufficiency 425 

B. High ligation and stripping of GSV to below knee, with local extirpation of vari- 
cose veins. 

C. High ligation of GSV with extirpation of varicose veins. 

D. Obliteration of GSV using laser or radiofrequency heating with local extirpation 
of varicose veins. 

E. Sclerotherapy with or without foam. 



Question 5 

How should the incompetent perforator veins be managed? 

A. Subfascial endoscopic perforator surgery (SEPS). 

B. Ligation through Linton-Cockett incisions. 

C. Disregard them. 

D. Ligation through small skin incisions. 

E. Duplex-guided sclerotherapy. 

Question 6 

How should the left leg be managed? 

A. Observation. 

B. Sclerotherapy. 

C. High ligation and stripping of GSV and local extirpation of varicose veins. 

D. Obliteration of GSV using laser or radiofrequency heating and local extirpation 
of varicose veins. 

The patient was treated with four-layer compression therapy until the operation 
day, which was postponed for 4 months. The ulcer and the swelling both decreased 
during this period; the ulcer measured 2x2 cm the day before operation. Both the 
right and the left leg GSV were treated with the closure method using radiofre- 
quency derived heating, and varicose veins on the lower leg were extirpated through 
multiple small incisions. Intraoperative duplex ultrasound scanning revealed that 
both GSV were occluded with no sign of reflux and the deep veins were patent with 
no sign of deep venous thrombosis. No specific treatment was performed for the 
incompetent perforator veins. The patient was discharged the same day after 
uneventful postoperative recovery and was scheduled for new duplex ultrasound 
scanning after 2 and 7 days. The postoperative duplex ultrasound scanning was 
normal, with no sign of deep venous thrombosis, and the remnant of GSV was 
occluded. The patient continued with four-layer bandaging and went back to work 
on the fifth day after operation. The ulcer was healed at the last visit, which was 
4 weeks later. Treatment with compression stocking during the daytime was 
planned for another 6 months. 



426 Vascular Surgery 

Commentary 

Investigation of both the arterial and the venous system is mandatory in cases of 
non-healing ulcer on the leg. Although Doppler examination had only revealed a 
clear reflux in GSV it is worthwhile to continue with duplex ultrasound scanning as 
deep venous incompetence and post-thrombotic changes can otherwise be over- 
looked. This is especially important when reflux is noted at the back of the knee 
where it is difficult with certainty to differentiate between deep venous reflux in the 
popliteal vein and reflux in the lesser saphenous vein. Although the history (no 
claudication or rest pain, no diabetes) and the location of the ulcer (medial aspect of 
lower leg) strongly suggest a venous ulcer, sometimes an arterial component is also 
present that might reduce the ability of the ulcer to heal. Palpable pulse on the 
dorsum of the foot (dorsalis pedis) or behind the medial malleolus (posterior tibial 
artery), as was evident in this case, almost rules out an arterial component. 
Although plethysmography can estimate the overall venous function it is not 
mandatory as a first line of investigation. Obtaining an ascending phlebography is 
also not necessary as it does not add any information that duplex ultrasound scan- 
ning does not provide and it is also an invasive method with the risk of complica- 
tions. Non-healing ulcer with unusual appearance should be considered for other 
aetiology and investigated with biopsy in the early stage of evaluation. [Q1: A, B] 

The old concept that the majority of venous ulcers are due to previous deep 
venous thrombosis [1, 2] has been altered during the last 20 years when duplex 
ultrasound studies have shown the importance of primary reflux in all venous seg- 
ments [3-7]. Superficial venous incompetence is often noted to be the sole pathol- 
ogy in patients presenting with non-healing venous ulcer [8]. Formerly the venous 
ulcer was often judged as being related to a post-thrombotic condition without any 
objective diagnosis. Because of the benign course of varicose veins in the majority 
of patients with superficial venous incompetence, the need for thorough evaluation 
is often neglected. Formerly used classifications of chronic venous disease used the 
term varicose ulcer if varicose veins were present, or post-thrombotic ulcer if they 
were less evident or if there was a previous history of deep venous thrombosis. The 
importance of classification, based on findings from duplex ultrasound scanning, 
has become more evident during the last decades as treatment and prognosis is 
largely dependent on the background history and the results of clinical investiga- 
tion. CEAP (clinical, (a)etiological, anatomical, pathophysiological) classification 
has gained more acceptance as the "gold standard" for classifying all aspects of 
venous pathology such as clinical class, aetiological background, anatomical distri- 
bution and pathophysiological findings (Table 45.1). There is a clear correlation 
between the CEAP clinical class and the venous function as measured by plethys- 
mography (foot volumetry), indicating that the clinical classification has a realistic 
meaning concerning the functional evaluation of venous disease. The duration of 
reflux in venous segments, on the other hand, does not correlate with clinical class, 
but the peak reverse flow velocity is significantly higher in patients with skin 
changes/ulcer (C4-C6) [9]. The basic part of CEAP indicates the highest clinical 
class (C6, active venous ulcer) and the anatomical distribution in superficial, perfo- 
rator or deep system (As, p, d) with reflux (Pr). S is added behind clinical class to 
indicate that the patient is symptomatic. The basic classification is sufficient for 
most clinical doctors. [Q2: C, D] The detailed version of CEAP is used when more 
information is needed as in longitudinal studies comparing treatment alternatives 



Venous Ulcers Associated with Superficial Venous Insufficiency 427 

Table 45.1 . CEAP classification 

Clinical classification 

CO: no visible or palpable signs of venous disease 

C1 : telangiectases or reticular veins 

C2: varicose veins 

C3: oedema 

C4a: pigmentation and/or eczema 

C4b: lipodermatosclerosis and/or atrophie blanche 

C5: healed venous ulcer 

C6: active venous ulcer 

S: symptoms including ache, pain, tightness, skin irritation, heaviness, muscle cramps, as well as other 

complaints attributable to venous dysfunction 

A: asymptomatic 

Aetio logical classification 

Ec: congenital 

Ep: primary 

Es: secondary (post-thrombotic) 

En: no venous aetiology identified 

Anatomic classification 

As: superficial veins 

Ap: perforator veins 

Ad: deep veins 

An: no venous location identified 

Pa thophysiological classifica tion 

Pr: reflux 

Po: obstruction 

Pr,o: reflux and obstruction 

Pn: no venous pathophysiology identifiable 



(Table 45.2). For more detailed information regarding the disease and its effect on 
daily life it is possible to use a venous severity scoring system [11]. Venous severity 
scoring is used as a complement to the CEAP classification (Fig. 45.1). Some 
medical conditions are clearly a risk factor for venous ulcer while others are less 
important. Venous ulcers are overrepresented in patients with diabetes although it 
is not clear if it is the venous pathology or if it is the diabetic microangiopathy that 
is the reason for this. Neither essential hypertension nor smoking is a proven risk 
factor for venous ulcer. The prevalence of varicose veins is increased in overweight 
individuals but the role of obesity is less clear when it comes to the risk of develop- 
ing skin changes or ulcer. The apparent association between overweight and vari- 
cose veins in women suggests that it is a risk factor even in the more severe form of 
chronic venous disease [12-14]. In a consecutive series of 272 patients with chronic 
venous disease investigated with duplex ultrasound scanning, 58 per cent of 
patients with healed or open ulcer (C5-C6) had body mass index >30 kg/m 2 (obese) 
as compared to 15 per cent of those with varicose veins but without skin changes or 
ulcer [15]. [Q3: A, B, C] Most thrombophilic conditions are risk factors for deep 
venous thrombosis and venous ulceration, as is resistance to activated protein C 
[16]. The prevalence of thrombophilia is high in patients with venous ulceration 
despite no history or duplex ultrasound findings of deep venous thrombosis [17]. 



428 Vascular Surgery 

Table 45.2. Advanced CEAP 

Same as basic CEAP with the addition that any of 18 named venous segments can be utilised as locators for 
venous pathology. 

Superficial veins: 

1. telangiectases/reticular veins 

2. great saphenous vein above knee 

3. great saphenous vein below knee 

4. small saphenous vein 

5. non-saphenous veins 

Deep veins: 

6. inferior vena cava 

7. common iliac vein 

8. internal iliac vein 

9. external iliac vein 

1 0. pelvic: gonadal, broad ligament veins, other 

11. common femoral vein 

12. deep femoral vein 

13. femoral vein 

14. popliteal vein 

1 5. crural: anterior tibial, posterior tibial, peroneal veins (all paired) 

1 6. muscular: gastrocnemial, soleal veins, other 

17. perforating veins, thigh 

18. perforating veins, calf 



Surgical treatment is mandatory in cases of isolated superficial incompetence as 
the likelihood of ulcer recurrence otherwise will remain high. Conservative treat- 
ment alone with below-knee compression had not been successful in keeping the 
ulcer healed, but it is important to continue with compression therapy while the 
ulcer is open and for some time after operation. Four-layer bandage is effective in 
healing venous ulcer [18]. High ligation with stripping of the GSV down to below 
the knee, with local extirpation of varicose veins, is the method of choice. It 
decreases the risk of ulcer recurrence and has a low incidence of nerve damage to 
the saphenous nerve. Stripping of the vein from the groin to the ankle increases the 
risk of damage to the saphenous nerve (5 per cent versus 29 per cent), although the 
recurrence rate is still the same [19]. Just doing high ligation without stripping 
the vein is less feasible as the recurrence rate is significantly higher [20]. Other 
promising methods for ablation of the refluxing GSV have emerged recently and 
might become the methods of choice in the future. As the diameter of the GSV was 
less than 15 mm it was possible to use the radiofrequency closure method to obliter- 
ate the vein. The main advantage of using less invasive methods is increased patient 
satisfaction, as the recovery time after operation has been reported to be shorter. 
Follow-up time up to 5 years with the radiofrequency method indicates that the 
method is durable. The long-term results after ablation of GSV using laser tech- 
nique or foam sclerotherapy are still unknown. [Q4: B, D] 

The varicose veins on the lower leg are dealt with by using multiple stab incisions 
and bringing them out using hooks. The cosmetic results are better and the risk of 
nerve damage is less. Care should be taken not to operate close to the ulcer area as 
healing problems and infection are more common if the incisions are made in 
damaged skin. 



Venous Ulcers Associated with Superficial Venous Insufficiency 



429 



First Name 



Sara 



Last Name Anderson 



Leg included 
Visit day 



O Left ® Right 
12/12/2004 



mpiom ana sign 



CEAP 



C linical class 
E tiologic class 

A natomic class 

* athophysiologic class Reflux 




Venous Segmental Disease Score 



CO 





CI 





C2 


1 


C3 


1 


C4 


1 


C5 


1 


C6 


El 


Clinical class 


C2,3,4t 



Qualifying 
comments 



Reflux 

LSV ®0 O0.5 

GSV 00 ®1 

Perforating Thigh ) 0.5 

Perforating Calf 00 ®1 
Calf veins, multiple ®0 02 
Popliteal ) 2 

Femoral 

Deep femoral 
Common femoral 



and above 



C2,3,4b,5,6,S-Ep-As,d,p-Pr2,3,11,18 

Venous segmental disease score 





Obstruction 
Obstruction GSV 

® O 1 
Obstruction calf vein multiple 

® O 1 
Obstruction popliteal vein 

® 02 

Obstruction FV 
® O 1 
Obstruction PFV 
® O 1 
Obstruction CFV 
® 02 

Obstruction iliac vein 

® O 1 

Obstruction IVC 

» O 1 0.0 

diameter of perforator 

4.0 < mm ) 

adjacent to ulcer 

® Yes O No 



Histor 






CEAP 



Score 



Suraer 



Clinicalal score 

Pain 3 

Varico seveins* 2 

Venous** edema 1 

Skin pigmentation 1 

Inflammation 1 

Indu ration 1 

Tolal no. ulcer*** 1 

Active ulceration, duration 1 

Active ulcer size 3 

Compressive therapy 1 



0=Absent 

1=Mild 

2= Mo derate 

3=Severe 



Qualifying 
comments 



Total clinical score 



15 



Disability score 



Disability score 2 



0=A symptomatic 

1 =Sy m pt o mat ic, work without support 

2=Can carry out usual* activitie only with 

compression and/or limbelevation 

3=Unable to carry out us ual* activities 

even with compression and/or limb 

elevation 

*Usual activities=patients activities before onset of disbility from venous disease 

Severity score (C#A#D) 
20 

Severity score mean 20 



Fig. 45.1. Venous severity scoring is used as a complement to the CEAP classification. 



Even though the role of surgery in venous ulcer disease has been unclear [21], a 
recently reported randomised controlled study comparing surgery with compres- 
sion therapy, to compression therapy alone, could clearly show a significantly lower 
recurrence rate in the surgically treated group [22]. Altogether 500 patients with 
open or recently healed ulcer (6 months) were included in the study. The healing 



430 Vascular Surgery 

rate was similar during the study period, but 12-month ulcer recurrence rates were 
significantly reduced in the surgically treated group or 12 per cent, compared to the 
compression-only group where the ulcer recurrence rate was 28 per cent. 

The pathophysiology behind venous ulcer is mainly reflux as opposed to obstruc- 
tion or occlusion. In a study on a consecutive series of 98 legs with an open venous 
ulcer, 85 per cent of the extremities had some form of superficial venous incompe- 
tence that might be treated with a simple operation on the superficial venous 
system. Axial reflux in the superficial (great saphenous vein) or the deep veins 
(femoral down to popliteal level) was present in 79 per cent of the legs [23]. 
Incompetent perforator veins and their role in chronic venous disease have been 
debated for years [24]. Incompetent perforator veins have been implicated as an 
important factor in the formation and recurrence of venous ulcers. This view is 
mainly based on clinical reports of excellent ulcer healing following the interruption 
of incompetent perforators. There is substantial evidence that subfascial endoscopic 
perforator surgery (SEPS) is effective in interrupting perforator veins, and it can be 
done without major wound complications that were often seen after the open sub- 
fascial Linton procedure [25-28]. Also, the ulcer healing rate after venous proce- 
dures that included SEPS has been satisfying [25, 26]. Patients undergoing surgery 
for incompetent perforator veins almost always have surgery simultaneously on the 
superficial venous system and therefore it is difficult to judge the actual contribu- 
tion of the incompetent perforator to the venous dysfunction. There is also evidence 
that reflux-eliminating surgery on one part of the venous system can abolish reflux 
in another part [29-31]. Operations on superficial veins have been shown to elimi- 
nate concomitant reflux in perforators [32]. Disregarding the incompetent perfora- 
tor veins in patients with superficial venous incompetence seems therefore to be 
appropriate. The low incidence of isolated perforator incompetence in patients with 
active venous ulcer does indicate that they are less important than previously 
thought [8]. The main indication (although not proven yet) for treating them is in 
patients with primary venous incompetence with recurrent ulceration despite 
optimal treatment of the superficial venous incompetence. The method of choice for 
treatment is then SEPS, mainly because of the low risk of wound complication. The 
use of sclerotherapy for the purpose of obliterating perforators is still under evalua- 
tion although the technique seems to be promising. [Q5: A, C] 

The indication for treating varicose veins in legs without skin changes or ulcer is 
less clear. The decision of recommending treatment for asymptomatic legs with vari- 
cose veins has to be judged individually; often it is the patient's preference that will 
decide. The cosmetic results of sclerotherapy on local varicose veins are poor if the 
refluxing GSV is left in place. The risk of future problems with skin changes or ulcer 
is increased when axial reflux is present in the GSV, as was the case with this patient, 
and that might be a sufficient reason to recommend even surgery for the asympto- 
matic left leg. A simultaneous operation on both legs in an otherwise healthy person 
does not seem to add any risk to the operation. If a catheter-based ablation is used to 
obliterate the GSV it is feasible to treat both legs at the same time as one catheter can 
then be used to treat both legs as the catheter is expensive. [Q6: A, C, D] 



References 

1. Homans J. The etiology and treatment of varicose ulcer of the leg. Surg Gynecol Obstet 
1917:24:300-11. 



Venous Ulcers Associated with Superficial Venous Insufficiency 431 

2. Bauer G. A roentgenological and clinical study of the sequels of thrombosis. Acta Chir Scand 
1942;86. 

3. Lees TA, Lambert D. Patterns of venous reflux in limbs with skin changes associated with chronic 
venous insufficiency. Br J Surg 1993;80:725-8. 

4. Hoare MC, Nicolaides A, Miles C. The role of primary varicose veins in venous ulceration. Surgery 
1983;82:450. 

5. Sethia KK, Darke SG. Long saphenous incompetence as a cause of venous ulceration. Br J Surg 
1984;71:754-5. 

6. Labropoulos N, Landon P, Jay T. The impact of duplex scanning in phlebology. Dermatol Surg 
2002;28:1-5. 

7. Wong JK, Duncan JL, Nichols DM. Whole-leg duplex mapping for varicose veins: observations on 
patterns of reflux in recurrent and primary legs, with clinical correlation. Eur J Vase Endovasc Surg 
2003;25:267-75. 

8. Danielsson G, Eklof B, Grandinetti A, Lurie F, Kistner RL. Deep axial reflux, an important contribu- 
tor to skin changes or ulcer in chronic venous disease. J Vase Surg 2003;38:1336-41. 

9. Danielsson G, Norgren L, Jungbeck C, Peterson K. Global venous function correlates better than duplex 
derived reflux to clinical class in the evaluation of chronic venous disease. Int Angiol 2003;22:177-81. 

10. Eklof B, Rutherford RB, Bergan JJ, Carpentier PH, Gloviczki P, et al. Revision of the CEAP 
classification for chronic venous disorders. A consensus statement. J Vase Surg 2004;40:1248-52. 

11. Rutherford RB, Padberg FT, Jr, Comerota AJ, Kistner RL, Meissner MH, Moneta GL. Venous severity 
scoring: An adjunct to venous outcome assessment. J Vase Surg 2000;31:1307-12. 

12. Iannuzzi A, Panico S, Ciardullo AV, Bellati C, Cioffi V, Iannuzzo G, et al. Varicose veins of the lower 
limbs and venous capacitance in postmenopausal women: relationship with obesity. J Vase Surg 
2002;36:965-8. 

13. Brand F, Dannenberg A, Abbott R, Kannel W. The epidemiology of varicose veins: The Framingham 
study. Am J Prev Med 1988;4:96-101. 

14. Sadick NS. Predisposing factors of varicose and telangiectatic leg veins. J Dermatol Surg Oncol 
1992;18:883-6. 

15. Danielsson G, Eklof B, Grandinetti A, Kistner RL. The influence of obesity on chronic venous 
disease. Vase Endovasc Surg 2002;36:271-6. 

16. Munkvad S, Jorgensen M. Resistance to activated protein C: a common anticoagulant deficiency in 
patients with venous leg ulceration. Br J Dermatol 1996;134:296-8. 

17. Bradbury AW, MacKenzie RK, Burns P, Fegan C. Thrombophilia and chronic venous ulceration. Eur 
J Vase Endovasc Surg 2002;24:97-104. 

18. Nelson EA, Iglesias CP, Cullum N, Torgerson DJ. Randomized clinical trial of four-layer and short- 
stretch compression bandages for venous leg ulcers (VenUS I). Br J Surg 2004;91:1292-9. 

19. Holme K, Matzen M, Bomberg AJ, Outzen SL, Holme JB. Partial or total stripping of the great saphe- 
nous vein. 5-year recurrence frequency and 3-year frequency of neural complications after partial 
and total stripping of the great saphenous vein. Ugeskr Laeger 1996;158:405-8. 

20. Dwerryhouse S, Davies B, Harradine K, Earnshaw JJ. Stripping the long saphenous vein reduces the 
rate of reoperation for recurrent varicose veins: five-year results of a randomized trial. J Vase Surg 
1999;29:589-92. 

21. Clinical evidence, Option: Vein surgery. BMJ 2001;1510. 

22. Barwell JR, Davies CE, Deacon J, Harvey K, Minor J, Sassano A, et al. Comparison of surgery and 
compression with compression alone in chronic venous ulceration (ESCHAR study): randomised 
controlled trial. Lancet 2004;363:1854-9. 

23. Danielsson G, Arfvidsson B, Eklof B, Kistner RL, Masuda EM, Sato DT. Reflux from thigh to calf, the 
major pathology in chronic venous ulcer disease: surgery indicated in the majority of patients. Vase 
Endovascular Surg 2004;38:209-19. 

24. Danielsson G, Eklof B, Kistner RL. What is the role of incompetent perforator veins in chronic 
venous disease? J Phlebol 2001;1:67-71. 

25. Nelzen O. Prospective study of safety, patient satisfaction and leg ulcer healing following saphenous 
and subfascial endoscopic perforator surgery. Br J Surg 2000;87:86-91. 

26. Gloviczki P. Subfascial endoscopic perforator vein surgery: indications and results. Vase Med 
1999;4:173-80. 

27. Wittens CH, Bollen EC, Kool DR, van Urk H, Mul T, van Houtte HJ. Good results of subfascial 
endoscopy as treatment of communicating vein insufficiency. Ned Tijdschr Geneeskd 1993;137:1200-4. 

28. Quiros RS, Kitainik E, Swiatlo MR, Breyter E. Cutaneous complications of the subaponeurotic 
surgery of the communicating venous system. J Cardiovasc Surg 1967;8:206-8. 

29. Walsh JC, Bergan JJ, Beeman S, Comer TP. Femoral venous reflux abolished by greater saphenous 
vein stripping. Ann Vase Surg 1994;8:566-70. 



432 Vascular Surgery 

30. Stuart WP, Adam DJ, Allan PL, Ruckley CV, Bradbury AW. Saphenous surgery does not correct per- 
forator incompetence in the presence of deep venous reflux. J Vase Surg 1998;28:834-8. 

31. Sales CM, Bilof ML, Petrillo KA, Luka NL. Correction of lower extremity deep venous incompetence 
by ablation of superficial venous reflux. Ann Vase Surg 1996;10:186-9. 

32. Gohel MS, Barwell JR, Wakely C, Minor J, Harvey K, Earnshaw JJ, et al. The influence of superficial 
venous surgery and compression on incompetent calf perforators in chronic venous leg ulceration. 
Eur J Vase Endovasc Surg 2005;29:78-82. 



46a. Iliofemoral Venous Thrombosis 



William P. Paaske 



A 72-year-old man was admitted in the late evening because of a turgid, white, 
painful left leg. Over the course of 4 months, he had lost 8 kg of weight (from 82 
to 74 kg); his height was 175 cm. There were general symptoms, such as tired- 
ness, slight nausea, lack of appetite and increasing apathy. Over the last 12 h, he 
had been increasingly confused and aggressive. He had been bedridden for 3 
weeks but had refused to see a doctor. There was no history of psychiatric 
disease, focal cerebrovascular events, ischaemic heart disease, hypertension, 
intermittent claudication, or venous insufficiency. He had been smoking about 
20 cigarettes a day since he was 14 years old, and for many years he had had 
slight functional dyspnoea, but otherwise no pulmonary symptoms. Stools had 
been light yellow to grey/white for the last week. His renal function had never 
been examined, and it had not been noticed whether he had passed urine in the 
last 24 h. Diazepam was the only medication. The history was provided by his 
wife, who had called the ambulance. Medical records were not available. 
The patient was confused, with delusions; he was intermittently agitated and pos- 
sibly psychotic, but he could be calmed down. He looked chronically ill, slightly 
emaciated, possibly anaemic and dehydrated. Temperature and blood pressure 
(arms) were normal. There was tachycardia with a regular rhythm. The abdomen 
was slightly distended, but there was neither a palpable mass nor peritoneal reac- 
tions. Digital rectal exploration was unremarkable. The right leg was normal with 
distal pulses. 

The left leg had diffuse swelling from the groin to the toes; there was moderate 
pallor, and no visible varicosities when standing. There was no evidence of 
superficial thrombophlebitis. Minor venous collaterals were noticed in the groin 
and just above the inguinal ligament. During palpation over the deep femoral 
veins, the patient groaned and became increasingly aggressive. The consistency 
of the calf muscle groups was increased with tenderness but not woody. There 
was less floppiness of the left leg muscles compared with those of the right. 
Spontaneous dorsiflexion of the foot was noticed, but sensory function could not 
be assessed due to lack of patient cooperation. 

The quality of the pulses in the groin and knee was good, and the pulse in the 
dorsal pedal artery was possibly present. Capillary filling in the pulp of the toes 



433 



434 Vascular Surgery 

could not be assessed. The plantar pallor did not increase during elevation, but 

discrete colour change was noticed at the back of the foot during post-elevation 

dependency. 

Bladder catheterisation did not produce urine. Electrocardiogram (ECG) was 

normal apart from a rate of 114 bpm. 



Question 1 

What is the most likely diagnosis? 

A. Thrombosis of the crural veins. 

B. Thrombosis of the femoral veins. 

C. Thrombosis of the iliac and femoral veins. 

D. Thrombosis of the superficial femoral artery. 

E. Thrombosis of the external iliac and superficial femoral arteries. 

Blood samples were taken, and the patient was admitted. 



Question 2 

Which investigation should be ordered and carried out at once? 

A. Intravenous arteriography. 

B. Intra-arterial arteriography. 

C. Ascending phlebography. 

D. Colour duplex sonography. 

E. Plethysmography. 

Due to an unusually large number of emergency admissions, the patient had to 
wait several hours before he could be examined by colour duplex sonography and 
have his chest X-ray taken. An hour before the scheduled time for these examina- 
tions (8 h after admission), and before the results of the blood tests were available, 
the patient deteriorated and the pulse rose further. His temperature was now 
38.9°C. He had become increasingly agitated and complained of severe pain in the 
left leg. 

Haemorrhagic bullae developed on the back of the foot and around the medial 
ankle, and the skin of the rest of the foot and the distal calf showed numerous 
petechiae. There was increased swelling, and the colour of the leg turned deeply 
cyanotic, even during elevation. The tips of all the toes were black. A weak pulse 
could be felt in the femoral artery in the groin, but distal pulses were absent. The 
consistency of the muscle groups of the thigh as well as the lower leg was clearly 
increased, and the patient suffered severe pain when femoral muscles were assessed 



Iliofemoral Venous Thrombosis 435 

by compression. He did not react to pain induced by pinching the skin from the 
knee and distally. At this point, the right leg exhibited slight but definite swelling, 
and the skin was beginning to become cyanotic. Pulses could still be felt in the right 
groin and popliteal artery, but pedal pulses had disappeared. 

Question 3 

What is/are the common name(s) for this clinical presentation? 

A. Iliofemoral venous thrombosis. 

B. Iliofemoral phlebothrombosis. 

C. Phlegmasia alba dolens. 

D. Phlegmasia cerulea dolens. 

E. Venous gangrene. 

Question 4 

What ideally should have been done, and what should be done at this stage at 
4 a.m. on the basis of this clinical presentation and with the additional information 
provided above? 

At this point, the results of the blood tests taken in the emergency room 
became available: they showed anaemia with haemoconcentration, thrombocy- 
topenia and electrolyte derangements; S-creatinine was 410 mmol/1, and the leu- 
cocyte count was 14 times above the upper normal limit. The large amount of 
fluid sequestered in the gangrenous left leg may account for part of the haemo- 
concentration. 



Question 5 

Would you consider a surgical thrombectomy at this stage? If so, how would you 
perform it? 

The situation was deemed hopeless and beyond medical therapy. The patient was 
given intravenous morphine to relieve the pain, and he died 13 h after admission. 



Commentary 

The tentative diagnosis at admission was acute left-sided iliofemoral venous throm- 
bosis with the clinical picture of phlegmasia alba dolens. [Q1: C] It was highly proba- 
ble - but not proven - that this bedridden patient suffered from active malignant 
disease with secondary venous thrombosis. His general appearance in connection 
with the specific signs and symptoms, including apparent lack of urine production, 
indicated a disaster in progression. The association of cancer and deep venous 
thrombosis is well established [1]. If a malignancy is definitely diagnosed, or 



436 Vascular Surgery 

suspected with a high degree of certainty, disseminated and/or in an advanced stage 
where expected residual lifespan is very short, then ultrasonically or phlebographi- 
cally verified iliofemoral venous thrombosis with venous gangrene (ischaemic 
venous thrombosis) must be interpreted as one of the signs indicating imminent 
termination of life, and treatment (medical as well as operative) is generally con- 
traindicated, including on compassionate grounds. 

With unreliable, rudimentary or uncertain information, it is essential that diag- 
nosis is established not merely as soon as possible but at once; it is not acceptable to 
wait several hours for the diagnostic test, colour duplex sonography. [Q2: D] The 
patient should be taken immediately to the ultrasound examination room and, if 
necessary, scanned by the surgeon. [Q4] 

In our case, the colour duplex sonography after a phase of phlegmasia cerulea 
dolens progressing to manifest venous gangrene on the left side and phlegmasia 
cerulea dolens in development on the right, showed bilateral thrombosis of both 
femoral and iliac veins in addition to thrombosis of the inferior vena cava up to 
and above the renal veins, which explained the lack of urine production. [Q3: E] 

Certain assessment algorithms have been devised for the management of this 
condition [2], but the remotest suspicion of deep venous thrombosis should result 
either in colour duplex sonography or ascending phlebography (with digital sub- 
traction technique), or in magnetic resonance venography with gadolinium 
enhancement plus Tl images (bull's eye sign) [3], if available, in patients with renal 
impairment or allergy to angiographic contrast media. Some centres have the 
option of computed tomographic (CT) venography (possibly with spiral/slip-ring 
technique), which has the additional advantage of being able to visualise extravas- 
cular morphology. Plethysmography (strain gauge, impedance, air, etc.) must be 
considered obsolete for precise diagnosis; isotope uptake tests have generally been 
disappointing and should be avoided. Both legs, rather than only the symptomatic 
leg, should be examined in all patients. In patients with coexisting arterial 
insufficiency of the lower extremities, the diagnosis can be even more difficult, so 
investigations should be performed at a lower level of clinical suspicion. Where 
phlegmasia cerulea dolens is surmised, or where venous gangrene is apparent, then 
one of these examinations must be performed without delay. If pulmonary 
embolism is suspected, then lung scintigraphy, pulmonary angiography or magnetic 
resonance or CT scanning of the pulmonary arteries should be performed. 

Treatment aims to prevent or decrease further thrombus formation or propagation, 
to reduce or stop acute (pulmonary embolism) and chronic (post-thrombotic 
syndrome) complications, and to reduce pain. 

In principle, the thrombus can be reduced, or removed, by chemical or mechani- 
cal means. A fresh thrombus is generally less adherent than an old thrombus. The 
preferred treatment of iliofemoral venous thrombosis is heparin administered for 
3-4 days as a continued intravenous infusion (high-dose heparin) concomitant with 
oral phenprocoumon, dicoumarol or warfarin, which should be given for a further 
3-6 months [4]. This regime probably has no influence on the development of 
chronic post-thrombotic syndrome. 

Thrombolysis with streptokinase, urokinase or recombinant tissue plasminogen 
activator (rt-PA) can be attempted for thrombi less than about 10 days old under 
close monitoring with repeated colour duplex sonography examinations [5-9]. 
Although many series have been published, the effect on pulmonary embolism is 
dubious, and the long-term clinical results of properly conducted studies are still 
poorly documented. 



Iliofemoral Venous Thrombosis 437 

Interruption of the venous system between the thrombus and the heart prevents 
pulmonary embolism and may be considered in highly selected cases; it is per- 
formed by partial or complete occlusion of the inferior caval vein by either open 
surgery [10] or deployment of temporary or permanent caval filters [11], of which 
several types are commercially available. The long-term outcomes of both tech- 
niques are not clear. The incidence of filter complications - early as well as late - is 
not negligible. 

Once again, as in many other aspects in the treatment of venous disease, there are 
widely diverging opinions as to the place of surgical thrombectomy of iliofemoral 
venous thrombosis with or without construction of an arteriovenous fistula. In 
pregnancy or during puerperium, surgical thrombectomy should not be attempted 
[12]. A balanced view, based on the available literature, would be that it may be a 
possibility that could be considered in limb-threatening phlegmasia cerulea dolens 
13-17]. 

The operation is performed with the supine patient in the reversed 
Trendelenburg position (legs down), and under general anaesthesia with continu- 
ous positive airway pressure. The femoral veins, which may bulge with thrombus, 
and arteries are exposed in the groin by a longitudinal incision. After slings have 
been applied, a longitudinal phlebotomy is made, and a venous Fogarty catheter is 
advanced towards the heart. The balloon is inflated, and the catheter is withdrawn 
together with the thrombus. The procedure is repeated until no more thrombus is 
delivered. The leg is now elevated, and by manual compression (possibly followed 
by compressing bandage, e.g. Esmarch's), one aims to remove the thrombus within 
the leg. The phlebotomy is then closed. The patient should have the leg elevated 
until mobilisation after a few days. Thrombectomy often results in incomplete clot 
removal and recurrence [18]. 

In certain centres, the contralateral groin vessels are also routinely exposed, a 
Fogarty catheter is introduced into the common femoral vein, and the tip is 
positioned in the upper part of the inferior caval vein. The balloon of this 
catheter is insufflated during the manoeuvres on the contralateral side, and it is 
retracted with inflated balloon after each of the Fogarty thrombectomy proce- 
dures. The aim is to retract fragments of thrombus and avoid (additional) pul- 
monary embolism. 

Some surgeons advocate construction of an arteriovenous fistula in addition to 
the surgical thrombectomy. The great saphenous vein is transected as appropriate 
below the saphenofemoral junction, and the distal part of the proximal segment is 
anastomosed to an arteriotomy in the common femoral artery. The aim is to 
increase blood flow, thereby reducing the risk of recurrent thrombus formation, in 
the proximal part of the femoral vein and veins proximal to that. [Q5] 

Although the extremity with phlegmasia cerulea dolens may look very bad 
indeed, a conservative approach is warranted (careful monitoring, elevation of the 
leg, heparin, fluid replacement). If systemic symptoms or signs occur, or if the 
situation deteriorates into manifest venous gangrene, then amputation must be 
performed without delay. 

Operative treatment of chronic iliofemoral venous thrombosis and its sequelae, 
notably post-thrombotic syndrome, remains controversial. Reports with various 
reconstructions, e.g. with polytetrafluoroethylene (PTFE), remain anecdotal [19]. 

Endovascular treatment options are emergent, some in combination with open 
surgery (hybrid procedures). Combined application of transcutaneous thrombec- 
tomy devices, balloon angioplasty, stenting etc. with thrombolysis may lead to a 



438 Vascular Surgery 

new level of therapeutic aggressiveness [20], but proper scientific documentation is 
so far not available, and these new developments must be considered experimental. 



References 

1. Prandoni P, Lensing AW, Buller HR, Cogo A, Prins MH, Cattelan AM, et al. Deep-vein thrombosis 
and the incidence of subsequent symptomatic cancer. N Engl J Med 1992;327:1128-33. 

2. Wells PS, Hirsh J, Anderson DR, Lensing AW, Foster G, Kearon C, et al. Accuracy of clinical assess- 
ment of deep-vein thrombosis. Lancet 1995;345:1326-30. 

3. Froehlich JB, Prince MR, Greenfield LJ, Downing LJ, Shah NL, Wakefield TW. "Bull's-eye" sign on 
gadolinium-enhanced magnetic resonance venography determines thrombus presence and age: a 
preliminary study. J Vase Surg 1997;26:809-16. 

4. Gallus A, Jackaman J, Tillett J, Mills W, Wycherley A. Safety and efficacy of warfarin started early 
after submassive venous thrombosis or pulmonary embolism. Lancet 1986;2:1293-6. 

5. Semba CP, Dake MD. Catheter-directed thrombolysis for iliofemoral venous thrombosis. Semin Vase 
Surg 1996;9:26-33. 

6. Verhaeghe R, Stockx L, Lacroix H, Vermylen J, Baert AL. Catheter-directed lysis of iliofemoral vein 
thrombosis with use of rt-PA. Eur Radiol 1997;7:996-1001. 

7. Patel NH, Plorde JJ, Meissner M. Catheter-directed thrombolysis in the treatment of phlegmasia 
cerulea dolens. Ann Vase Surg 1998;12:471-5. 

8. Mewissen MW, Seabrook GR, Meissner MH, Cynamon J, Labropoulos N, Haughton SH. Catheter- 
directed thrombolysis for lower extremity deep venous thrombosis: report of a national multicenter 
registry. Radiology 1999;211:39-49. 

9. Grossman C, McPherson S. Safety and efficacy of catheter-directed thrombolysis for iliofemoral 
venous thrombosis. Am J Roentgenol 1999;172:667-72. 

10. Silver D, Sabiston DC, Jr. The role of vena caval interruption in the management of pulmonary 
embolism. Surgery 1975;77:3-10. 

11. Magnant JG, Walsh DB, Juravsky LI, Cronenwett JL. Current use of inferior vena cava filters. J Vase 
Surg 1992;16:701-6. 

12. Torngren S, Hjertberg R, Rosfors S, Bremme K, Eriksson M, Swedenborg J. The long-term outcome 
of proximal vein thrombosis during pregnancy is not improved by the addition of surgical 
thrombectomy to anticoagulant treatment. Eur J Vase Endovasc Surg 1996;12:31-6. 

13. Roder OC, Lorentzen JE, Hansen HJB. Venous thrombectomy for iliofemoral thrombosis. Early and 
long-term results in 46 consecutive cases. Acta Chir Scand 1984;150:31-4. 

14. Hood DB, Weaver FA, Modrall JG, Yellin AE. Advances in the treatment of phlegmasia cerulea 
dolens. Am J Surg 1993;166:206-10. 

15. Perkins JM, Magee TR, Galland RB. Phlegmasia caerulea dolens and venous gangrene. Br J Surg 
1996;83:19-23. 

16. Eklof B, Kistner RL. Is there a role for thrombectomy in iliofemoral venous thrombosis? Semin Vase 
Surg 1996;9:34-45. 

17. Plate G, Eklof B, Norgren L, Ohlin P, Dahlstrom J A. Venous thrombectomy for iliofemoral vein 
thrombosis - 10-year results of a prospective randomised study. Eur J Vase Endovasc Surg 
1997;14:367-74. 

18. Patel KR, Paidas CN. Phlegmasia cerulea dolens: the role of non-operative therapy. Cardiovasc Surg 
1993;1:518-23. 

19. Alimi YS, Dimauro P, Fabre D, Juhan C. Iliac vein reconstructions to treat acute and chronic venous 
occlusive disease. J Vase Surg 1997;25:673-81. 

20. Hood DB, Alexander JQ. Endovascular management of iliofemoral venous occlusive disease. Surg 
Clin North Am 2004:84:1381-96,viii. 



46b. Iliofemoral Deep Venous Thrombosis 
(During Pregnancy) 

Anthony }. Comerota 



A 24-year-old female who was 32 weeks pregnant presented to the emergency 
department at 7 p.m. with a swollen, painful left lower extremity. Her left leg had 
become progressively more symptomatic during the past 48 hours. During the 
past 24 hours, she began feeling lethargic, had slight shortness of breath, and 
began to experience right chest discomfort with deep breathing. 

Upon physical examination, her heart rate was 106/min, respiratory rate was 
18/min, and blood pressure was 112/70. Her lungs were clear, and her abdomen 
was appropriate for her gestational age. 

She had a swollen left leg from the foot to the inguinal ligament, which had a 
bluish hue. She had pain upon palpation of the left femoral vein. Her arterial 
examination was normal. 

A venous duplex was ordered and scheduled to be performed in approximately 
3 hours. 







Question 1 

At this point, what would be your next course of action? 

A. Obtain an immediate ventilation/perfusion lung scan. 

B. Perform a venogram. 

C. Start intravenous heparin at 75 mg/kg bolus followed by a continuous infusion 
at 15 mg/kg/hour; or, an injection of subcutaneous enoxaparin at 1 mg/kg. 

D. Maintain the patient at bed rest until the duplex is completed. If the duplex 
confirms deep vein thrombosis (DVT), begin treatment with heparin. 

E. Perform an echocardiogram. 

The patient had an intravenous line started and a bolus of unfractionated heparin 
was given, followed by a continuous infusion. Four hours later, the venous duplex 
examination demonstrated venous thrombosis in the posterior tibial vein, popliteal 

439 



440 Vascular Surgery 

vein, femoral vein, proximal great saphenous vein, common femoral vein, and 
external iliac vein to the visible limit of the examination. The veins of the right 
lower extremity were normal. The patient asks, "What can I expect if treated with 
continued anticoagulation?" 

Question 2 

You tell the patient that she has iliofemoral and infrainguinal deep vein thrombosis, 
and that with continued anticoagulation: 

A. She will do much better following delivery if she remains anticoagulated for 1 year. 

B. She faces a 15-40 percent likelihood of venous claudication at 5 years. 

C. She faces a 90 percent likelihood of venous insufficiency and 15 percent like- 
lihood of venous ulceration. 

D. It is difficult to predict the natural consequences of her disease. 

Question 3 

This patient's father has long suffered with post-thrombotic chronic venous 
insufficiency, and she expresses a strong desire to avoid post-thrombotic complica- 
tions. However, she does not want to accept the risks of bleeding associated with 
thrombolytic therapy; therefore, she asks for your treatment recommendation. 
Your best recommendation to this patient would be: 

A. Intravenous heparin for 5 days, followed by oral anticoagulation with a warfarin 
compound. 

B. Heparin (unfractionated or low-molecular-weight) until the delivery, followed 
by warfarin anticoagulation. 

C. Rheolytic thrombectomy. 

D. Catheter-directed thrombolysis. 

E. Operative venous thrombectomy. 

Because of her painful lower extremity and her concern for post-thrombotic com- 
plications, the patient requested that the thrombus be removed. She was reluctant 
to accept the potential bleeding complications of catheter-directed thrombolysis, 
and the attending radiologist was reluctant to treat with catheter-directed lysis. 
Therefore, venous thrombectomy was planned. 

Question 4 

The next appropriate step is: 

A. Obtain a ventilation/perfusion scan or spiral CT scan of the chest to evaluate for 
suspected pulmonary embolism. 



Iliofemoral Deep Venous Thrombosis (During Pregnancy) 



441 




Fig. 46b.1. A contralateral iliocavagram demonstrates a large volume of nonocclusive thrombus in the vena 
cava. Note fetal skeleton in normal position. 



B. Obtain a contralateral iliocavagram prior to taking the patient to the operating 
room. 

C. Take the patient directly to the operating room and perform the procedure in 
order to avoid progressive deterioration. 

D. Anticoagulate overnight and proceed with operative thrombectomy the next 
day. 

The patient was anticoagulated with intravenous heparin overnight. The next 
morning a contralateral iliocavagram was performed (Fig 46b. 1) prior to taking the 
patient to the operating room. A large volume of nonocclusive thrombus was found 
throughout the infrarenal vena cava. 



Question 5 

In light of the findings on the cavagram, what is the best next step? 

A. Abandon operative venous thrombectomy and anticoagulate. 

B. Perform an Angiojet mechanical thrombectomy of the vena cava and ilio- 
femoral venous system. 

C. Perform a pulmonary arteriogram to confirm/exclude pulmonary embolism. 

D. Obtain an echocardiogram. 

E. Insert a suprarenal vena caval filter and proceed with venous thrombectomy 
under fluoroscopic guidance. 

The patient was presumed to have had a pulmonary embolism. A echocardio- 
gram failed to show right ventricular dysfunction, an enlarged right ventricle, tri- 
cuspid insufficiency, or elevated pulmonary artery pressures. Because of the 



442 



Vascular Surgery 




Fig. 46b.2. X-ray demonstrates suprarenal vena caval filter in proper position. 

potential risk of dislodging nonocclusive thrombus during the venous thrombec- 
tomy, a removable suprarenal vena caval filter was inserted (Fig. 46b.2). 



Question 6 

Important considerations during thrombectomy include: 

A. Shield the fetus from all X-ray exposure. 

B. Perform the venous thrombectomy under fluoroscopic guidance. 

C. Monitor the fetus throughout the procedure. 

D. Let the nonocclusive thrombus in the vena cava remain undisturbed and 
perform a thrombectomy of the iliofemoral venous system only. 



Iliofemoral Deep Venous Thrombosis (During Pregnancy) 443 

The patient was taken to the operating room for a venous thrombectomy with 
fluoroscopic guidance and fetal monitoring. A cut-down was performed on the left 
common femoral and femoral veins, with exposure of the saphenofemoral junction. 
A longitudinal venotomy was performed at the level of the saphenofemoral junc- 
tion, followed by protrusion of a large amount of acute thrombus. The leg was 
raised and a tight rubber bandage applied with minimal extrusion of the infrain- 
guinal thrombus. Attempts to pass a catheter from the inguinal ligament distally 
into the femoral vein and attempts to pass a guidewire distally were unsuccessful. 



Question 7 

The next appropriate step would be: 

A. Perform iliofemoral and caval thrombectomy with AV fistula, leaving the 
infrainguinal thrombus. 

B. Abandon thrombectomy and anticoagulate. 

C. Perform an infrainguinal venous thrombectomy aided by a cut-down on the left 
posterior tibial vein. 

A cut-down on the posterior tibial vein was performed. Following a posterior 
tibial venotomy, a no. 3 Fogarty catheter was passed upwards through the throm- 
bosed venous system, exiting the common femoral venotomy. This catheter was 
used to guide a no. 4 Fogarty catheter distally through the venous valves by placing 
both catheter tips within a 14-gauge Silastic intravenous catheter sheath after the 
hub was amputated. Following a mechanical balloon catheter thrombectomy, the 
leg was flushed using a bulb syringe with a large volume of heparin/saline solution, 
which flushed additional thrombus from the common femoral venotomy. After 
clamping the femoral vein, the deep venous system was then filled with 300 ml of a 
dilute recombinant tissue plasminogen solution (6 mg rt-PA in 300 ml). 

The iliofemoral and vena caval thrombectomy was performed under fluoroscopic 
guidance, filling the balloon with contrast to ensure that the suprarenal caval filter 
was not dislodged. After completing the thrombectomy, an operative iliocavagram 
was performed to assess the adequacy of thrombectomy and to ensure unobstructed 
venous drainage into the vena cava. An iliac vein stenosis was observed. 



Question 8 

The appropriate next step is: 

A. Close the venotomy and anticoagulate, since a common iliac vein stenosis is fre- 
quently observed due to normal vascular anatomy. 

B. Close the venotomy and perform an AV fistula. 

C. Perform angioplasty and insert a self-expanding stent if recoil occurs. 

D. Operatively expose the common iliac vein and perform an endovenectomy and 
transpose the vein above the right common iliac artery. 



444 



Vascular Surgery 





Fig. 46b.3. a A completion phlebogram following iliofemoral thrombectomy shows stenosis of the left common 
iliac vein, b Balloon dilation corrects the lesion without evidence of recoil, providing unobstructed venous 
drainage into the vena cava. 



Iliofemoral Deep Venous Thrombosis (During Pregnancy) 445 

A balloon angioplasty catheter was placed into the lesion and an angioplasty 
performed. The iliac vein was dilated to 14 mm without evidence of recoil 
(Fig. 46b.3). 



Question 9 

Now that patency has been restored to the infrainguinal and iliofemoral venous 
systems, are there any additional techniques that can be performed to reduce risk of 
rethrombus? 

A. An AV fistula, using the end of the proximal saphenous vein sewn to the side of 
the superficial femoral artery. 

B. The saphenous vein should not be used for AV fistula, since it represents collat- 
eral drainage from the leg in the event of recurrent thrombosis. 

C. Placement of a catheter into the posterior tibial vein for anticoagulation with 
unfractionated heparin. 

D. Elevate the legs and avoid ambulation for the next 4-5 days. 

E. Therapeutic anticoagulation. 

An arteriovenous fistula (AVF) using the proximal saphenous vein anasto- 
mosed to the superficial femoral artery increases flow velocity through the 
iliofemoral venous system, reducing the risk of rethrombosis. A thrombectomy 
of the proximal great saphenous vein was required in this patient, as is often the 
case. Since the goal of the AVF is to increase venous blood flow velocity, the size 
of the anastomosis is limited to 3.5-4 mm in order to avoid a steal and avoid 
venous hypertension. A small piece of PTFE is wrapped around the saphenous 
AVF and looped with a 2-cm piece of O-Prolene, which is left in the subcutaneous 
tissue (Fig. 46b. 4). This will serve as a guide should the AVF require closure. 
However, since the AVF is small, it is considered permanent and closure is not 
anticipated. 

To further reduce the risk of rethrombosis, a heparin infusion catheter (pediatric 
feeding tube) is placed into the proximal posterior tibial vein and brought out 
through a separate stab wound adjacent to the lower leg incision. Infusing unfrac- 
tionated heparin through this catheter to achieve a therapeutic PTT ensures a high 
concentration of heparin in the target vein, a concentration much higher than 
would be achieved if the patient was treated with standard intravenous anticoagula- 
tion through an arm vein. A monofilament suture is looped around the catheter in 
the posterior tibial vein and brought out through the skin and secured with a sterile 
button. This is used to occlude the vein after 5-6 days when the catheter is removed 
following full oral anticoagulation with warfarin. In the case of this pregnant 
patient, intravenous anticoagulation through the leg veins was maintained for 4 
days, after which she was converted to subcutaneous enoxaparin at 1 mg/kg every 
12 hours. The catheter was removed and the patient discharged. The patient was 
maintained on subcutaneous enoxaparin 1 mg/kg twice a day until she delivered a 
healthy baby 6 weeks later. 



446 



Vascular Surgery 




Fig. 46b.4. The construction of the arteriovenous fistula (AVF) using a large side branch of the great saphenous 
vein sutured end-side to the superficial femoral artery. Note sleeve of PTFE wrapped around the AVF and looped 
with a 2-cm piece of O-monofilament suture. The purpose of this is to assist in operative closure should obliter- 
ation of the AVF become necessary. 



Question 10 

The patient does not wish to breastfeed her baby. What is your best recommenda- 
tion for ongoing therapy? 

A. Six more weeks of Lovenox. 

B. Oral anticoagulation for 6-12 months. 

C. Patients' risk for recurrence is determined by the amount of residual thrombus. 
If there is no residual thrombus on venous duplex, additional anticoagulation is 
unnecessary. 

D. Stop anticoagulation and start aspirin. 



Iliofemoral Deep Venous Thrombosis (During Pregnancy) 447 

Question 1 1 

What is your recommendation regarding a thrombophilia evaluation? 

A. It is not necessary to perform an expensive thrombophilia evaluation since this 
was a DVT of pregnancy. 

B. Defer the thrombophilia evaluation until after the patient discontinues anti- 
coagulation. 

C. Since this patient will be on indefinite anticoagulation, a thrombophilia evalua- 
tion is not necessary. 

D. Perform tests not affected by anticoagulation and complete the evaluation after 
anticoagulation has been discontinued. 

An abbreviated thrombophilia evaluation of: lupus anticoagulant, antiphospho- 
lipid/anticardiolipin antibody, factor V Leiden, prothrombin gene mutation, and 
homocysteine was negative. The remainder of the thrombophilia evaluation will be 
completed in 1-2 years, at which time it is anticipated that the patient's Coumadin 
will be discontinued. 



Commentary 



Although not recognized by the recent ACCP guidelines on antithrombotic therapy 
for venous thromboembolism [1], iliofemoral deep venous thrombosis represents a 
condition with a uniquely high incidence of post-thro mbotic morbidity [2-4]. 

This patient's presentation was clinically consistent with iliofemoral deep venous 
thrombosis associated with a pulmonary embolism. The adventitia of the femoral 
vein is innervated with sensory nerves; therefore, pain on palpation of the femoral 
vein as a result of its distension is a frequent physical finding. The femoral vein dis- 
tends as a result of the associated venous hypertension and thrombosis. 

Patients presenting during off hours to the emergency department who are at 
high clinical risk of a venous thromboembolic condition should be anticoagulated 
[Q1: C] until a definitive diagnosis is made [1].A ventilation/perfusion (V/Q) lung 
scan is not performed in this patient because she is pregnant and the clinical proba- 
bility of a pulmonary embolism is high. The likelihood of the venous duplex 
demonstrating acute DVT is also high; therefore, this patient's treatment is unlikely 
to be altered by the V/Q scan findings. There is also reluctance to expose the preg- 
nant patient to a radioisotope. Standard ascending phlebography is not necessary, 
since the clinical presentation and venous duplex will establish the diagnosis with a 
high degree of accuracy. Once anticoagulation is established, it is not necessary and 
actually counterproductive to maintain the patient at bed rest [5]. An echocardio- 
gram is advisable in all patients who have the diagnosis of pulmonary embolism to 
evaluate its impact on right ventricular function; however, it is not necessary in this 
patient to perform an "off hours" echocardiogram since the patient can be ade- 
quately treated until the next business day. 

This patient's thrombus extends from the posterior tibial vein to the external iliac 
vein, as documented on venous duplex. The natural history of these patients is one 



448 Vascular Surgery 

of significant post-thrombotic morbidity. [Q2: B, C] Akesson and colleagues [3] 
demonstrated that within 5 years of anticoagulation for iliofemoral deep venous 
thrombosis, 95 percent of patients had documented venous insufficiency, 15 percent 
had venous ulceration, and 15 percent suffered with venous claudication. Delis et al. 
[4] studied in greater detail a similar but larger cohort of patients with iliofemoral 
deep venous thrombosis and performed exercise testing. They demonstrated that 40 
percent developed symptoms of venous claudication. While pregnancy is an 
induced hypercoagulable state, delivery of the present patient's child is not known 
to alter the natural history of the patient's acute venous thrombosis. 

In order to reduce the high risk of post-thrombotic sequelae, a strategy of throm- 
bus removal should be considered. Operative venous thrombectomy [Q3: E] is the 
best recommendation in light of the fact that the patient does not wish to face the 
additional risk of bleeding with thrombolytic therapy, and the radiologist is reluc- 
tant to treat the patient with catheter-directed thrombolysis. Rheolytic thrombec- 
tomy is in its early stages, and to date has not been shown to be effective by itself in 
the absence of incorporating a plasminogen activator [6]. Oral anticoagulation 
during pregnancy is not recommended. Although this patient is in her third 
trimester and warfarin embryotrophy is not a concern, the potential coagulopathy 
of the fetus due to its immature liver and potential fetal bleeding complications 
during delivery as a result of passage through the birth canal make oral anticoagula- 
tion inadvisable. Heparin anticoagulation until delivery followed by oral anticoagu- 
lation is commonly offered to these patients; however, their post-thrombotic 
morbidity is exceptionally high. 

A decision was made to proceed with venous thrombectomy. Patients can be anti- 
coagulated overnight and the operation performed the next business day. Venous 
thrombectomy does not need to be performed as an "emergency operation." [Q4: B, 
D] In all patients in whom a venous thrombectomy is performed, it is important to 
know the proximal extent of thrombus, particularly whether there is thrombus in 
the inferior vena cava. Therefore, a contralateral iliocavagram is performed prior to 
the iliofemoral venous thrombectomy. As mentioned earlier, it is assumed that this 
patient has had a pulmonary embolism and the radiation exposure of a CT scan or a 
V/Q scan is unnecessary, since their results are unlikely to change this patient's 
management. However, in the non-pregnant patient, a spiral CT scan of the chest, 
abdomen, and pelvis would be performed. The rationale for CT scanning is that 
approximately 50 percent of patients with proximal DVT will have an asymptomatic 
pulmonary embolism. Up to 25 percent of these patients will develop subsequent 
pulmonary symptoms [7]. When the symptoms surface during anticoagulation, the 
symptoms are often misinterpreted as "failure" of anticoagulation, when in reality it 
is the natural evolution of the patient's initially asymptomatic (undiagnosed) pul- 
monary embolism. The proximal extent of thrombus in the vena cava or iliac veins 
often can be identified, as well as screening for associated intra-abdominal, 
retroperitoneal, or pelvic pathology. 

The patient was treated with anticoagulation overnight. Before going to the oper- 
ating room a contralateral iliocavagram was performed. Information regarding the 
proximal extent of thrombus is particularly important, since the details of thrombus 
extension may alter the procedure. Nonocclusive thrombus in the vena cava is con- 
cerning because of its potential for fragmentation and embolization. This author 
believes that these patients should be protected against potential embolization 
during the procedure. This can be accomplished either with a suprarenal vena caval 
filter, as was inserted in this patient, since it was presumed that she already had suf- 



Iliofemoral Deep Venous Thrombosis (During Pregnancy) 449 

fered a symptomatic pulmonary embolism. Alternatively, suprathrombus balloon 
occlusion during the caval thrombectomy can be performed. This patient also 
underwent a preoperative echocardiogram to evaluate the impact of her presumed 
pulmonary embolism on right ventricular function. Echocardiography should be 
performed in all patients with pulmonary embolism, since it is a predictor of 
chronic thromboembolic pulmonary hypertension, and patients who have right- 
sided abnormalities should be considered for thrombolytic therapy or mechanical 
thromboembolectomy. [Q5: D, E] 

During the operative procedure, fluoroscopy is used to guide the placement of the 
balloon catheter so as not to dislodge the vena caval filter. Fluoroscopy is also used 
to assess the success of thrombectomy and to evaluate for underlying venous lesions 
and their correction (Fig. 46b. 3). Since the fetus is very well developed, the risk to the 
fetus from modest X-ray exposure is low. Fetal monitoring is routinely performed 
throughout the procedure. The monitoring devices must be checked so as not to 
interfere with appropriate imaging of the venous system during the procedure. 
Shielding of the fetus would obscure the iliac veins and distal vena cava. [Q6: B, C] 

Previous descriptions of iliofemoral venous thrombectomy focus only on the 
iliofemoral venous system. An occluded infrainguinal venous system reduces 
venous return through the thrombectomized iliofemoral veins, and leaves substan- 
tial thrombus burden infrainguinally with its resultant post-thrombotic sequelae. 
Current techniques of infrainguinal venous thrombectomy allow the procedure to 
be performed successfully following a cut-down on the posterior tibial vein. [Q7: C] 
Therefore, contemporary venous thrombectomy should be viewed much the same 
as arterial thrombectomy, that is, removing as much thrombus from the venous cir- 
culation as is physically and pharmacologically possible, correcting any underlying 
lesion, and perform mechanical and pharmacological maneuvers to avoid recurrent 
thrombosis. 

An iliac venous stenosis observed on completion phlebography is common. 
Correcting the underlying iliac vein stenosis is considered an important part of the 
procedure (Fig. 46b. 3). This is performed under fluoroscopic guidance and if recoil 
occurs, a self-expanding stent is used to maintain unobstructed venous drainage 
from the iliac venous system into the vena cava. [Q8: C] Direct endophlebectomy of 
the iliac vein lesion and transposition above the right common iliac artery is a large 
operation, which has been replaced by the relatively simple balloon dilation and 
stenting. 

Following successful thrombectomy of the infrainguinal and iliofemoral venous 
systems and correction of any underlying iliac vein stenosis, prevention of recurrent 
thrombosis is paramount. There are mechanical and pharmacologic measures 
which, if used, minimize recurrence. These include the construction of a femoral 
AV fistula using the end of the transected proximal saphenous vein (or a large side 
branch) anastomosed to the side of the proximal superficial femoral artery (Fig. 
46b. 4). Frequently, the proximal saphenous vein must undergo a thrombectomy to 
restore its patency. The saphenous vein is not a collateral pathway of venous 
drainage for patients with iliofemoral venous thrombosis. On occasion, it maybe a 
collateral drainage pathway for patients with infrainguinal DVT. Since the infrain- 
guinal venous system had patency restored, that is not an issue in this patient. The 
AV fistula is constructed to increase venous velocity in the iliofemoral veins; 
however, it should not increase venous pressure. Limiting the size of the anastomo- 
sis to approximately 4 mm usually accomplishes this goal. Pressure monitoring of 
the common femoral vein before and after flow is initiated through the AVF is 



450 Vascular Surgery 

important. If the venous pressure increases, one must suspect a proximal (iliac 
vein) stenosis or excessive flow through the AVF, either (or both) of which should 
be corrected. 

An additional, effective adjunctive technique is the placement of a catheter into 
the posterior tibial vein, which is used to anticoagulate the patient with unfraction- 
ated heparin postoperatively. A pediatric feeding tube is inserted into the posterior 
tibial vein and brought out through a separate stab wound in the skin adjacent to 
the lower leg incision. This small catheter is used for postoperative anticoagulation 
with unfractionated heparin. Targeting a therapeutic PTT ensures a high concentra- 
tion of heparin in the diseased vein, which should substantially reduce the risk of 
recurrence. In the author's experience, when these adjunctive techniques have been 
used, no patient has experienced rethrombosis. [Q9: A, C, E] 

Following delivery, women can be anticoagulated with Coumadin, assuming they 
do not wish to breastfeed. Warfarin is excreted in the breast milk of women; there- 
fore, those who breastfeed should not be taking warfarin compounds. Among the 
options, oral anticoagulation for 6-12 months is the most appropriate. [Q10: B] 
While it is true that residual thrombus increases the risk of recurrent thrombosis 
[8], it would be inappropriate to treat this patient with less than a full course of anti- 
coagulation. Since this patient had extensive venous thrombosis and a positive 
family history, an underlying thrombophilia is suspected and the author would 
extend the duration of anticoagulation to 1 year or more. 

A thrombophilia evaluation is appropriate in this patient. A complete throm- 
bophilia evaluation cannot be performed while the patient is on anticoagulation, 
since antithrombin III, proteins C and S, and factor VIII will be affected. However, 
lupus anticoagulant, antiphospholipid antibody, factor V Leiden, prothrombin gene 
mutation, and homocysteine levels can be obtained during anticoagulation and, if 
positive, may play a role in the subsequent management of this patient. [Q11: D] 



References 

1. Buller HR, Agnelli G, Hull RD, Hyers TM, Prins MH, Raskob GE. Antithrombotic therapy for venous 
thromboembolic disease: the Seventh ACCP Conference on Antithrombotic and Thrombolytic 
Therapy. Chest 2004;126:401S-28S. 

2. O'Donnell TF, Jr, Browse NL, Burnand KG, Thomas ML. The socioeconomic effects of an iliofemoral 
venous thrombosis. J Surg Res 1977;22:483-8. 

3. Akesson H, Brudin L, Dahlstrom JA, Eklof B, Ohlin P, Plate G. Venous function assessed during a 
5 year period after acute ilio-femoral venous thrombosis treated with anticoagulation. Eur J Vase Surg 
1990;4:43-8. 

4. Delis KT, Bountouroglou D, Mansfield AO. Venous claudication in iliofemoral thrombosis: long-term 
effects on venous hemodynamics, clinical status, and quality of life. Ann Surg 2004;239:118-26. 

5. Partsch H, Kaulich M, Mayer W. Immediate mobilisation in acute vein thrombosis reduces post- 
thrombotic syndrome. Int Angiol 2004;23:206-12. 

6. Kasirajan K, Gray B, Ouriel K. Percutaneous Angiojet thrombectomy in the management of extensive 
deep venous thrombosis. J Vase Interv Radiol 2001;12:179-85. 

7. Monreal M, Rey-Joly BC, Ruiz MJ, Salvador TR, Lafoz NE, Viver ME. Asymptomatic pulmonary 
embolism in patients with deep vein thrombosis. Is it useful to take a lung scan to rule out this condi- 
tion? J Cardiovasc Surg (Torino) 1989;30:104-7. 

8. Prandoni P. Risk factors of recurrent venous thromboembolism: the role of residual vein thrombosis. 
Pathophysiol Haemost Thromb 2003;33:351-3. 



47a. Management of Upper Extremity 
Lymphoedema with Microsurgical 
Lymphovenous Anastomosis 

Corradino Campisi and Francesco Boccardo 



A 59-year-old woman presented with an 8-year history of oedema of the left arm. 
Initially, the oedema appeared in the upper arm. The patient was treated with 
combined decongestive physiotherapy (manual and mechanical lymphatic 
drainage), bandaging and exercises three to four times over a 12-month period. 
Despite these measures, the oedema extended as far as the forearm and hand 
(Fig. 47a. 1), and she had several episodes of erysipeloid lymphangitis. In the 
months preceding her admission, she also complained of episodes of lymphangi- 
tis and pain. There were no warts or wounds on the skin. Her past medical 
history included lumpectomy with axillary lymphadenectomy and radiotherapy 
for left breast cancer, although routine follow-up for breast cancer did not 
suggest any local recurrence. 

Initially, the oedema had a rhizomelic location. It was hard to the touch 
without pitting oedema. There were no dystrophic or dyschromic skin lesions, 
except for signs of acute reticular erysipeloid lymphangitic attacks caused by 
Gram-positive Staphylocci infections promoted by lymph stasis. A lymphan- 
gioscintigram was performed, which showed features compatible with lymphatic 
circulatory impairment in the left arm (Fig. 47a. 2). This was followed by lym- 
phangio-magnetic resonance imaging (MRI) of the left arm and hemithorax, 
which showed no signs of locoregional relapse of breast cancer but confirmed 
lymph stasis, predominantly in the epifascial compartment. In addition, it 
showed dilated medial arm lymphatic collectors interrupted at the proximal 
third of the arm. Finally, echo-Doppler of the left subclavian and axillary venous 
axis was performed. This did not demonstrate any venous dysfunction. A diag- 
nosis of chronic secondary lymphoedema of the left arm following breast cancer 
treatment was made. 



Question 1 

How do you classify lymphoedema? 

453 



454 



Vascular Surgery 




Fig. 47a.1 . Patient before treatment. 



Question 2 

Which of the following statements regarding the diagnosis of lymphoedema are 
correct? 



A. Lymphangiography is currently the best diagnostic investigation for all kinds of 
lymphoedema. 

B. The echo-Doppler investigation has an important role in determining the 
correct treatment for the patient. 

C. Lymphangioscintigraphy is the most popular noninvasive first-line investigation 
for lymphoedema. 

D. It is difficult to diagnose lymphoedema at an early stage. 

E. Lymphangio-MRI offers precise morphological data on oedema distribution 
and topography of dilated lymphatic pathways, without requiring contrast. 



Lymphoedema 



455 




Fig. 47a.2. Lymphangioscintigram before microsurgery. 

The patient underwent microsurgical lymphatic-venous anastomoses in the 
proximal third of the volar surface of the left arm using an 8/0 nylon suture material 
(Fig. 47a.3). 




Fig. 47a.3. Lymphatic-venous anastomoses seen through the operating microscope (30¥). 



456 



Vascular Surgery 



Question 3 

Which of the following statements regarding the management of lymphoedema are 
correct? 

A. Microsurgery can reduce oedema in all patients, but the best outcome is seen in 
patients operated on in the second and third stages. 

B. Proper elastic compression garments are an important adjunct to optimise long- 
term results. 

C. Surgical intervention is not indicated in the advanced stages of lymphoedema. 

D. Microsurgical lymphatic-venous anastomoses are used more frequently than 
reconstructive microsurgical methods. 

E. Microsurgery cannot be applied in primary lymphoedema. 

The postoperative recovery was uneventful. The patient was discharged home on the 
fifth postoperative day. The incidence of lymphangitic attacks decreased significantly. 




Fig. 47a.4. Long-term clinical outcome after microsurgery. 



Lymphoedema 



457 



A reduction in arm volume was seen within 3 days of the operation, and further 
improvements were observed at medium- and long-term follow-up, particularly 
between the first and the fifth years after surgery. From the fifth year onwards, the clin- 
ical condition of the arm remained stable with time, even more than 10 years after the 
operation (Fig. 47a.4). Lymphangioscintigraphy at this point demonstrated that the 
lymphatic-venous anastomoses were still patent (Fig. 47a.5). 



Question 4 

What are the long-term results of derivative and reconstructive microsurgery for 
lymphoedema? 



Question 5 

In what ways can secondary lymphoedema be prevented? 



Commentary 

Lymphoedema is a significant worldwide problem. It can be divided into primary 
and secondary forms. Primary lymphoedemas do not have any recognisable cause 



POST 

* 


W^ 

£vm 




* ^ 

*• 

A 

* ** 
1 uSfe Br ■ 


>, 

t a 


^d pm j? 





Fig. 47a.5. Lymphangioscintigram performed after microsurgery shows the patency of the lymphatic-venous 
anastomoses more than 1 years after the operation. 



458 Vascular Surgery 

(so-called idiopathic), although triggering aetiological factors can often be found. 
Lymphoedemas that present at birth (congenital) are included in this category. 
These can be hereditary-familial (Nonne-Milroy's disease), and are often associated 
with chromosomal abnormalities. Other primary lymphoedemas may have an early 
or late onset, which can be triggered by minor trauma, infection or surgery. In 
females, the predisposing factors are often thought to be alterations in neurohormonal 
status (neuroendocrine lymphoedema). 

Primary lymphoedemas can also be due to lymphatic and/or lymphnodal dys- 
plasia, hypoplasia or even hyperplasia with associated increased lymph produc- 
tion. Lymph nodes and/or lymphatics can be involved in abnormal lymph flow. In 
most cases of hypoplasia, lymph node involvement is demonstrated and leads to 
the progressive secondary alteration of lymphatic vessels. From pathophysiologi- 
cal and diagnostic points of view, this picture is practically the same as that seen 
with secondary lymphoedemas resulting from lymphadenectomy and/or 
radiotherapy [1]. 

Approximately 90 per cent of all primary lymphoedemas are characterised by 
hypodysplastic alterations involving lymph nodes and/or lymphatics. This is 
characterised by a diminished ability to form and activate a proper collateral 
circulation in response to trauma, infection and surgery. In a further 8-10% 
of primary lymphoedemas, an increased number and size of lymphatic col- 
lectors can be found, associated with structural lymphatic and lymph nodal 
dysplasia [2]. 

Disorders in lymphogenesis also play an important role from a lymphodynamic 
point of view. Hyperlymphogenesis may derive from pre-existing regional arterial- 
venous hyperstomies, arterial-venous fistulae (i.e. in Klippel-Trenaunay's disease) 
or related angiodysplasia. In contrast, reduced or absent production of lymph, age- 
nesis, hypoplasia, or impaired permeability of the initial lymphatics is very rare, if 
not exceptional. 

Finally, among lymphodynamic abnormalities, apart from insufficient lymph 
drainage along anatomically pre-established pathways, gravitational lymph 
and/or chylous reflux pathologies should also be mentioned. This top-to-bottom 
lymph backflow is caused by insufficient antigravitational structures, normally 
represented by valves, the reticular myoelastic layer of the lymphatic walls, and 
lymph node architecture (lymphoedemas and chyloedemas due to gravitational 
reflux). 

The aetiology of secondary lymphoedemas can generally be identified in the 
patient's history or physical examination. This can be secondary to trauma, infec- 
tion, inflammation, radiotherapy, surgery, paralysis or even neoplasia. Indeed, lym- 
phatic filariasis is endemic in some tropical and subtropical areas of Asia, Africa 
and Latin America. However, secondary lymphoedemas often have some congenital 
predisposition. For instance, congenital wall-valve dysplasia of the lymphatics is 
always found in lymphatic filariasis. Similarly, arm lymphoedema secondary to 
breast carcinoma treatment occurs in 5-35% of cases, depending on whether axil- 
lary surgery is associated with radiotherapy [3]. However, it is more likely to occur 
when there is no deltoid pathway [4]. This lymphatic way drains the lymph coming 
from the arm directly into the supraclavicular lymph nodes, thus bypassing the axil- 
lary stations. With preventive lymphoscintigraphic studies, comparing the arm ipsi- 
lateral to the breast cancer site with the contralateral one, patients with a higher risk 
of developing secondary lymphoedema could be identified and could, theoretically, 
receive preventive therapeutic treatment. Based on these observations, Tosatti's 



Lymphoedema 



459 



r 



E 
u 



< 



Congenital 

(manifest 
early or late) 



r 



< 



Upper limb 

(extremely 
rarely not 
preceded by 
mastectomy) 



V 



r 



Lower limb < 



V 



Stasis in the 
interstitial 
space and 
collectors 



V 



Acquired (a congenital complaint 
is not always a prerequisite for its 
manifestation) 



r 



< 



v 



Gravitational 

reflux, from 

hypoplasia or 

dysplasia, 

obstacles, 

increased 

lymphogenesis 



r 



< 



v 



Mixed: associated with phlebopathy and, much more rarely, arteriopathy 



Insufficiency 
of the 
collectors 
(valvular and 
parietal) and 
lymph nodes 



<< 



V. 



♦ Inherited, early 

♦ Neuroendocrine 
(at puberty, 
pregnancy, 
menopause) 

♦ Post-traumatic 

♦ Post-inflammatory 
(post-lymphangitic, 
post-phlebitic, etc.) 

♦ From X-rays 

♦ Neoplastic 
(primitive or 
secondary) 



Fig. 47a.6. Classification of chronic lymphoedema of the limb according to Tosatti. 



classification of lymphoedemas (Fig. 47a. 6), proposed more than 30 years ago [5], 
still seems to be valid. [Q1] 

Apart from some exceptional cases of acute post-lymphangitis and/or post-trau- 
matic types, lymphoedema is normally a chronic, progressing and disabling condi- 
tion characterised by a progressive volume increase of the limb(s) involved. This 
disease, which evolves in phases, is characterised by frequent lymphangitic, 
erysipeloid exacerbations with subsequent lipodermatosclerotic indurated cellulitis, 
chronic fibrosclerotic lymphadenitis (in primary lymphoedemas) and lymphostatic 
warts. The worst outcome is generally elephantiasis, with severe functional impair- 
ment or systemic sepsis. However, degeneration into lymphangiosarcoma 
(Stewart-Treves syndrome) is a rare sequel, most likely to occur in post-mastec- 
tomy lymphoedemas. It is important not to confuse this with cutaneous local recur- 
rence of breast cancer. Lymphoedema of the leg can also occasionally be associated 
with Kaposi's sarcoma. This is not necessarily human immunodeficiency virus 
(HlV)-related. 

The differentiation of lymphoedema from phleboedema can be based on a thor- 
ough medical history and clinical examination, paying attention to the time and 
conditions of onset, location, evolution, extent and volume of the oedema. 
Lymphoedema is hard to the touch, while venous oedema is soft and pits under 
finger compression. This difference reflects the underlying pathophysiology, in that 
stagnant lymph in the subcutaneous connective tissue is an excellent culture 
medium for fibroblasts. In this environment, they mature rapidly into fibrocytes, 
thus forming fibrosclerotic connective tissue. Lymphoedema typically begins proxi- 
mally, whereas venous oedema initially affects the distal part of the lower limbs with 
the notable exception of phlegmasia dolens, caused by acute deep thrombophlebitis 
of the iliofemoral veins. Unlike phleboedema, lymphoedema does not usually 
evolve into dystrophic-dyschromic skin lesions or ulcers. It is more likely to be 
complicated by acute reticular erysipeloid lymphangitis, caused by Gram-positive 
cocci infection in the presence of static lymph. Phleboedema is often associated with 
varices and varicophlebitis, and unlike lymphoedema, it is subject to rapid postural 
changes and is characterised by abnormal Doppler venous flow rates with 



460 Vascular Surgery 

significant increase in venous pressure when the patient is standing up. However, 
mixed types of lymphophleboedema also exist (as in stage III postphlebitic syn- 
drome), with predominance of either the venous or lymphatic component. These 
include the complex conditions of angiodysplasia with arterial-venous hyperstomy, 
as seen in MayalPs syndrome [6], or congenital arteriovenous macro- and 
microfistulas, as seen in Klippel-Trenaunay's disease. (The latter condition is recog- 
nised by gigantism with elongation of the affected limb, varying degrees of foot dys- 
morphism, flat or map-like "port-wine" angioma, and hyperhydrosis of the sole of 
the foot.) There are also some spurious forms of lymphophleboedema, which are 
masked by prevailing lymphoedema and therefore more difficult to recognise. In 
these cases, if angiodysplasia is suspected, then routine investigations such as 
Doppler venous pressure measurements maybe insufficient, and further investiga- 
tions, including phleboscintigraphy, phlebography or digital arteriography, may be 
required. For the time being, lymphangioscintigraphy and conventional oil-contrast 
lymphography are the most suitable investigations of lymphatic and chylous 
oedemas. Lymphangioscintigraphy is the most popular method used in the rapid 
screening of lymphoedemas [7, 8] as it is a noninvasive way of imaging both 
superficial and deep lymphatic circulations. Since it is noninvasive, it can be 
repeated easily in patient follow-up, especially after microsurgery. A small tracer 
dose of "technetium adsorbed in colloid spherules (colloid sulphide, rhenium, 
dextran) is used. The lymphotropic nature of these substances permits display of the 
preferential lymphatic pathways with a gamma camera, and allows measurement of 
the flow rate and lymph node uptake. A tracer clearance measurement is a useful 
parameter from a lymphodynamic viewpoint. However, lymphoscintigraphy is most 
useful in the study of lymphoedemas at early stages [9]. Direct lymphangiography 
[10] is preferred in the study of gravitational reflux lymphatic and chylous oedema of 
the lower limb and external genitalia before surgical intervention [11, 12]. In this 
examination, ultrafluid "Lipiodol" is injected into a lymphatic collector, isolated with 
microsurgical technique, of the dorsum of both feet. This type of investigation is 
minimally invasive and, if performed according to well-established standards, has 
minimal complications. However, rare adverse reactions have been reported. These 
include general complications such as pulmonary microembolism in the presence of 
peripheral lymphovenous fistulas or allergy to contrast medium. Local complications 
may also occur in the form of infection on the site of the skin incision, acute lym- 
phangitis or lymphorrhoea. Direct lymphangiography can also be performed in chil- 
dren. It enables a morphofunctional study of the superficial and, with the use of 
proper technical support, the deep circulation [12]. 

Computed tomography (CT), ultrasonography and lymphangio-MRI may also 
provide important preoperative data on lymphatic and chylous dysfunction. 
Indirect lymphangiography [13] performed with dermo-hypodermic injection of 
a water-soluble contrast medium ("Iotasul") is useful to clarify aetiopathological 
aspects of primary lymphoedemas, and fluorescent microlymphography [14] can 
be helpful in assessing the status of the superficial dermis lymphatic web, which 
reflects the functional condition of the peripheral lymphatic circulation. The con- 
ventional Houdack-McMaster dye test with the injection of highly lymphotropic 
vital stain (Patent Blue V) is used today as a preliminary investigation in direct 
lymphangiography and microsurgery for a better and faster assessment of lym- 
phatics. Recent studies by Olszewski [15] and Campisi et al. [16] have developed 
a system to measure endolymphatic pressure and lymphatic flow rate. These 
parameters, together with venous pressure assessment, help to measure the 



Lymphoedema 461 

lymph-venous pressure gradient, which is essential for a correct approach to 
microsurgical treatment of lymphoedemas. With this method, a lymphatic vessel 
is isolated and cannulated at the lower third of the leg's medial surface. Any 
change in the flow-pressure rate can also be recorded during microsurgery, 
in clino- and orthostatic positions, at rest and under dynamic conditions. 
These studies have shown that a valuable lymphatic-venous pressure gradient is 
essential to obtain medium- and long-term results by derivative microsurgery. 
[Q2: B, C, E] 

Manual lymphatic drainage has been shown to be a highly effective treatment in 
the conservative management of lymphoedema [17-19]. This is followed by the 
application of bandaging and eventually graded compression stockings. The use of 
intermittent compression pneumatic devices is usually complementary to manual 
lymphatic drainage and may contribute to further reduction of the lymphoedema. 
Pharmacotherapy includes the use of antibiotics, particularly penicillin [20], anti- 
inflammatory drugs and benzopyrones [21]. The positive effect of benzopyrones 
was described by Casley-Smith et al. [21], but their role in the treatment of lym- 
phoedema has yet to be clarified. 

Thirty years ago, there were relatively few therapeutic solutions in the treatment 
of lymphoedema. Only the most severe and advanced cases of elephantiasis were 
treated surgically, mainly in order to reduce the volume of lymphoedematous limbs. 
The most popular surgical methods were those proposed by Charles [22] (total 
resection of skin-lipid layers), Thompson [23] (drainage with scarred subfascial 
skin flap), and Servelle [24] (total surface lymphangectomy). Being highly destruc- 
tive and invasive operations, they could not be recommended in less advanced or 
initial stages or in childhood disease [25]. More recently, microsurgical lymphatic- 
venous and lymphnodal-venous anastomoses were introduced for the management 
of lymphoedema resistant to conservative treatment [26, 27]. These techniques are 
beneficial in not only secondary but also primary lymphoedemas [28], since early 
intervention is possible even in young children with some minor modifications of 
the technique, such as lymphatic-capsule-venous anastomosis [29]. 

Lymphostatic disease maybe associated with venous impairment such as varices, 
superficial thrombophlebitis, deep venous thrombosis and postphlebitic sequelae. 
These conditions are a contraindication to traditional lymphatic-venous anastomosis. 
Therefore, novel reconstructive lymphatic surgery techniques are used [30]. These 
include segmental autotransplantation of lymphatic collectors [31] for the treatment 
of monolateral lymphoedema or the personally described method of interposition 
autologous venous grafting or lymphatic-venous-lymphatic plasty [32]. 

The use of free microvascular lymphatic or lymph nodal flaps [33, 34] is still in 
clinical trials. However, it opens up interesting options in the treatment of lym- 
phoedema that fails to respond to conservative therapy and that, for congenital 
(aplasia or hypoplasia) or acquired (elephantiasis with diffuse obstructive lymphan- 
gitis) reasons, cannot benefit from the above-mentioned derivative or reconstruc- 
tive microsurgical techniques. 

Elastic stockings are worn for an average period of 1-5 years after microsurgery 
according to the stage of the pathology at the time of operation. These stockings 
aim to prevent the closure of anastomoses in the early postoperative period, follow- 
ing a rapid reduction of oedema and consequent decrease in lymphatic pressure 
and flow as a result of the microsurgical drainage [35]. [Q3: A, B, D] 

Through a properly planned follow-up at 1, 3, 6 and 12 months, and then annu- 
ally for at least the first 5 years after surgery, lymphatic microsurgery results are 



462 Vascular Surgery 



Stage I 

a. Absence of oedema in the presence of impairment of the lymphatic vessels 

(mastectomy with axillary lymphadenectomy with coincidental limbs 

with respect to volume and consistency) 

b. Slight reversible oedema on adopting a sloping position or nocturnal rest 

Stage II 

Persistent oedema that regresses only partially 
in a sloping position or nocturnal rest 

Stage III 

Persistent oedema (not regressing spontaneously in a sloping position) 
and aggravated (acute erysipeloid lymphangitis) 

Stage IV 

Fibrolymphoedema (initial lymphostatic verrucosis) with the "columnar" limb 

Stage V 

Elephantiasis with severe limb deformation, scleroindurative pachydermitis, 
and marked, extensive lymphostatic verrucosis 



Fig. 47a.7. Clinical instrumental diagnostic staging of lymphoedema. 

positive in more than 80 per cent of cases, with an even better outcome in patients 
operated upon precociously (at stages II and III; see Fig. 47a. 7). The incidence of 
lymphangitic attacks decreases significantly after microsurgery. The reduction in 
oedema volume obtained by microsurgery is seen immediately after operation 
(within the first three postoperative days), and a further decrease in lym- 
phoedema is also observed at medium- and long-term follow-up, particularly 
between the first and fifth years after operation. From the fifth year onwards, the 
clinical condition of the limb tends to remain stable with time, even more than 10 
years after surgery. Lymphangioscintigraphy can document objectively that the 
flow through the venous graft parallels the clinical improvement over the 10-year 
period [36]. [Q4] 

Turning to the prevention of secondary lymphoedema, early identification of 
high-risk patients (such as those undergoing oncological lymphadenectomies, par- 
ticularly in combination with radiotherapy) and early diagnostic lymphan- 
gioscintigraphy [37] has been suggested [38, 39]. In these cases, early microsurgery 
is a reasonable option in order to fight, from their very onset, lymphoedemas that, 
based on a reasonable statistical probability, are expected to show unrelenting 
progression [40]. [Q5] 



References 

1. Badini A, Fulcheri E, Campisi C, Boccardo F. A new approach in histopathological diagnosis of 
lymphoedema: pathophysiological and therapeutic implications. Lymphology 1996;29S: 190-8. 



Lymphoedema 463 

2. Papendieck CM. Temas de Angiologia Pediatrica. Buenos Aires: Editorial Medica Panamericana, 
1992. 

3. Farrar WB, Lavalle G, Kim JA. Breast cancer. In: McKenna RJ, Murphy GP, editors. Cancer surgery. 
Philadelphia: Lippincott, 1994; 209-59. 

4. Witte CL. Breast cancer - an overview. Lymphology 1994;27S:397-400. 

5. Tosatti E. Lymphatique profonds et lymphoedemes chroniques des membres. Paris: Masson, 1974. 

6. Mayall JC, Mayall ACDG. Standardization of methods of treatment of lymphoedema. Progress in 
Lymphology XI - Excerpta Med 1988, 517. 

7. Mariani G, Campisi C, Taddei G, Boccardo F. The current role of lymphoscintigraphy in the diagnostic 
evaluation of patients with peripheral lymphoedema. Lymphology 1998;31S:316-19. 

8. Witte C, McNeill G, Witte M, et al. Whole-body lymphangioscintigraphy: making the invisible easily 
visible. Progress in Lymphology XII, Elsevier Science Publishers B.V. 1989;123. 

9. Bourgeois P, Leduc O, Leduc A. Imaging techniques in the management and prevention of postther- 
apeutic upper limb oedemas. Cancer 1998;83 (12 Suppl American):2805-13. 

10. Kinmonth JB. The lymphatics. Surgery, lymphography and diseases of the chyle and lymph systems. 
London: Edward Arnold, 1982. 

11. Bruna J. Indication for lymphography in the era of new imaging methods. Lymphology 
1994;27S:319-20. 

12. Campisi C, Boccardo F, Zilli A, Borrelli V. Chylous reflux pathologies: diagnosis and microsurgical 
treatment. Int Angiol 1999;18:10-13. 

13. Partsch H. Indirect lymphography in different kinds of leg oedema. In: Lymphology: advances in 
Europe. Genoa: Ecig, 1989; 95-9. 

14. Bollinger A, Jager K, Sgier F, Seglias J. Fluorescence microlymphography. Circulation 1981;64:195-200. 

15. Olszewski W. Lymph and tissue pressures in patients with lymphoedema during massage and 
walking with elastic support. Lymphology 1994;27S:512-16. 

16. Campisi C, Olszewski W, Boccardo F. II gradiente pressorio linfo-venoso in microchirurgia linfatica. 
Minerva Angiologica, 1994; 19. 

17. Vodder E. La methode Vodder - le drainage lymphatique manuel. Bagsvaer: Institute for 
Lymphdrainage, 1969. 

18. Foldi M. The therapy of lymphoedema. EJLRP 1993-1994;14:43-9. 

19. Leduc A. Le drainage lymphatique. Theorie et pratique. Paris: Masson, 1980. 

20. Olszewski WL. Recurrent bacterial dermatolymphangioadenitis (DLA) is responsible for progression 
of lymphoedema. Lymphology 1996;29S:331. 

21. Casley-Smith JR, Casley-Smith Judith R. High-protein oedemas and the benzo-pyrones. Sydney: 
Lippincott, 1986. 

22. Charles RH. A system of treatment. In: Latham A and English TC, editors. London: Churchill, 1912. 

23. Thompson N. The surgical treatment of chronic lymphoedema of the extremities. Surg Clin North 
Am 1967;47:2. 

24. Servelle M. Pathologie vasculaire. Paris: Masson, 1975. 

25. O'Brien B. Microlymphatic-venous and resectional surgery in obstructive lymphoedemas. World J Surg 
1979;3:3. 

26. Degni M. New techniques of lymphatic-venous anastomosis for the treatment of lymphoedema. 
Cardiovasc Riv Bras 1974;10:175. 

27. Campisi C. Rational approach in the management of lymphoedema. Lymphology 1991;24:48-53. 

28. Campisi C. Lymphatic microsurgery: legend or reality? Phlebolymphology 1994;7:11-15. 

29. Campisi C. Lymphatic microsurgery: a potent weapon in the war on lymphoedema. Lymphology 
1995;28:110-12. 

30. Campisi C, Boccardo F. Frontiers in lymphatic microsurgery. Microsurgery 1998;18:462-71. 

31. Baumeister RGH. Clinical results of autogenous lymphatic grafts in the treatment of lymphoedemas. 
In: Partsch H, editor. Progress in Lymphology XI, Elsevier Science Publishers BV, 1988, 419-420. 

32. Campisi C. Use of autologous interposition vein graft in management of lymphoedema. Lymphology 
1991;24:71-6. 

33. Becker C, Hidden G, Godart S, Maurage H, Pecking A. Free lymphatic transplant. EJLRP 1991;2:75-7. 

34. Trevidic P, Marzelle J, Cormier JM. Apport de la microchirurgie au traitement des lymphoedemes. 
Editions Techniques -Encycl. Med. Chir. (Paris-France), Techniques chirurgicales - Chirurgie 
vasculaire, 1994;F.a. 43-225, 3. 

35. Campisi C. Lymphoedema: modern diagnostic and therapeutic aspects. Int Angiol 1999;18:14-24. 

36. Campisi C, Boccardo F. Role of microsurgery in the management of lymphoedema. Int Angiol 
1999;18:47-51. 

37. Pecking AP, et al. Upper limb lymphedema's frequency in patients treated by conservative therapy in 
breast cancer. Lymphology 1996;29S:293-6. 



464 Vascular Surgery 

38. Campisi C, Boccardo F, Padula P, Tacchella M. Prevention of lymphoedema: Utopia or possible 
reality? Lymphology 1994, 27 (Suppl);676-82. 

39. Pissas A. Prevention of secondary lymphoedema. Proceedings of the International Congress of 
Phlebology, Corfu, Greece, 113, September 4-8, 1996. 

40. Casley-Smith JR. Alterations of untreated lymphedema and its grades over time. Lymphology 
1995:28:174-85. 



47b. Management of Upper Extremity 
Lymphoedema with Liposuction 

Hakan Brorson 



A 53-year-old woman presented with an 8-year history of oedema of the right 
arm. Twelve years ago she underwent treatment for right breast cancer. She had 
mastectomy, removal of lymph glands, and postoperative irradiation. No 
chemotherapy was given. The arm swelling started gradually without obvious 
reason. Since the appearance of arm swelling she had experienced three bouts of 
erysipelas that were treated with penicillin. Two years ago she was treated else- 
where with combined decongestive therapy (CDT) including manual lymphatic 
drainage, bandaging and a set of compression garments. She had initially gained 
some reduction of the excess volume, but no follow-up was done. 

Now her main complain was pain and problems with clothing interfering with 
her everyday activities. She stated that her arm was cosmetically unappealing and 
that she did not want to see people in public. On examination she had a swollen 
arm and limited limb movement. 









Question 1 

What would you do? 

A. Perform the pitting test. 

B. Perform an MRI. 

C. Measure arm volumes. 

D. Perform an indirect lymphoscintigraphy. 

E. Start controlled compression therapy (CCT) or complete decongestive therapy 
(CDT) with the help of a trained lymph therapist. 

The pitting test showed pitting of 2 cm (Fig. 47b. 1). Arm volume measurements 
using the formula of the truncated cone showed an excess volume of 2700 ml. She 
was referred to a lymph therapist for conservative treatment (CDT) to reduce the 
excess volume. After 2 months the excess volume was reduced from 2700 ml to 

465 



466 Vascular Surgery 




Fig. 47b.1 . Marked arm lymphoedema after breast cancer treatment with deep pitting of several centimetres. 
The arm swelling is dominated by fluid, i.e. accumulation of lymph. 

2100 ml. Further reduction was not possible in spite of a further 4 weeks' treatment. 
[Q1:A,C] 

Question 2 

What would be the next step in the management of this patient? 

A. Perform the pitting test. 

B. Perform an MRI. 

C. Measure arm volumes. 

D. Perform an indirect lymphoscintigraphy. 

A new pitting test showed minimal pitting (4-5 mm) (Fig. 47b.2). 

An MRI confirmed your suspicion of excess adipose tissue in the arm (Fig. 47b.3). 

Arm volumes were calculated using the formula of the truncated cone and 
showed an excess volume of 2045 ml (Fig. 47b. 4). The patient now wanted further 
reduction of the excess volume. [Q2: A, B, D] 

Question 3 

What would you do next? 

A. Start conservative therapy again with the help of a trained lymph therapist. 

B. Perform lymphatic-venous anastomoses. 

C. Perform lymphatic-venous-lymphatic plasties (interposition autologous venous 
grafting). 

D. Perform transplantation of lymph vessels using vessels from the thigh. 

E. Refer the patient for liposuction. 



Lymphoedema 



467 




Fig. 47b.2. Pronounced arm lymphoedema after breast cancer treatment. There is almost no pitting is spite of 
hard pressure by the index finger for one minute. A slight reddening is seen at the two spots where pressure has 
been exerted. The "oedema" is completely dominated by adipose tissue. The term "oedema" is at this stage 
improper as the swelling is dominated by hypertrophied adipose tissue and not lymph. At this stage the aspirate 
contains no or minimal amount of lymph (Fig. 47b.6). 

















.'■■ • . 














• 














' 














■ 










































^r 




<■ 










Wr 




























■ £ ■ 














^F 




^m. 










j 


1 ^1 






V 


A 
































. . 


■ 








i ■ 




■ 














. 








B 


| 




' 










































. 














■ . ' , 














■ ' 




























; 














• 

■ 










J 




■■ i 














■ 














. ■ ■ ■ " - 












■ - 


















Fig. 47b.3. a MRI (elbow region) showing a right-sided, secondary arm lymphoedema after breast cancer treatment 
in the elbow region. Note the honeycomb pattern, b The healthy left side in the same patient for comparison. 



468 



Vascular Surgery 




Fig. 47b.4. 53-year-old woman with a preoperative oedema volume of 2050 ml in the right arm for 8 years. 







Fig. 47b.5. Peroperative pictures from the beginning a, during b, c, and at the end d of surgery. 



Lymphoedema 



469 



The patient was operated on with liposuction under bloodless conditions using a 
tourniquet (Fig. 47b. 5). 

After 2 hours the aspirate removed during bloodlessness measured 2100 ml. A 
standard compression garment was fitted on the arm before the tourniquet was 
released. The size of the garment was determined from preoperative measurements 
on the healthy arm. In order to reduce bleeding, tumescent technique was used on 
the most proximal part of the upped arm that was covered by the tourniquet. 
Tumescent technique includes infiltration with a saline-adrenaline solution (1 mg 
adrenaline/1000 ml saline) followed by liposuction. The next day the aspirate had 
sedimented and showed 92 per cent adipose tissue and 8 per cent fluid (lymph/ 
interstitial fluid) (Fig. 47b.6). 

Following surgery, controlled compression therapy (CCT) was instituted: Two 
days after surgery measurements were taken for custom-made compression gar- 
ments, that is, sleeve and glove, two of each. The patient was discharged after 
5 days, and was seen after 2 weeks. During the first 2 weeks the patient alternated 
two standard sets of sleeve-and-glove garments, i.e. two sleeves and two gloves. At 
the 2-week visit plethysmographic measurement of both arms showed an excess 
volume of 185 ml (91 per cent reduction). She was fitted with the custom-made com- 
pression sleeves and gloves ordered on measurements taken 2 days postoperatively. 




Fig. 47b.6. The aspirate achieved under bloodless conditions usually contains 90-100 per cent adipose tissue. 
This picture shows a typical aspirate of 2000 ml with an adipose fraction of 90 per cent and a fluid fraction of 
10 per cent. 



470 Vascular Surgery 




Fig. 47b.7. Clinical result 1 years after liposuction. 

During the consecutive follow-up the patient was seen at 4 weeks, 3 months, 
6 months and 1 year. At the 3-month visit complete reduction was achieved and has 
lasted ever since. Then, as complete reduction had been achieved, she was seen once 
a year (see detailed information later in the text regarding CCT). At the last control, 
10 years after surgery, an excess volume of -340 ml was registered; thus the treated 
arm was somewhat smaller than the normal one (Fig. 47b. 7). 

Question 4 

How often, after the first year, is the patient checked when complete reduction has 
been achieved? 

A. Every month. 

B. Every 3 months. 

C. Twice a year. 

D. Once a year. 

E. Not necessary. 

Question 5 

At least how many compression garments (sleeve-and-glove) a year should this 
patient be provided with? 

A. One garment. 

B. Two garments. 

C. Three garments. 

D. Four garments. 

E. Garments are not needed. 



Lymphoedema 471 

Question 6 

How long must the patient wear the compression garments? 

A. Only daytime. 

B. Only at night. 

C. 24 hours a day. 

D. 24 hours a day, lifelong. 



Question 7 

Who can perform the liposuction? 

A. A plastic surgeon. 

B. A vascular surgeon. 

C. A general surgeon. 

D. A dermatologist. 

E. A surgeon trained especially for this kind of liposuction. 



Question 8 

If you have been trained to perform the surgery, but are not sure if funding for com- 
pression garments can be provided - what do you do? 

A. Go ahead with the surgery. 

B. Do not perform the surgery. 



Question 9 

In order to optimize the outcome of surgery, how do you want to arrange for the 
follow-up? 

A. Refer the patient to a lymph therapist after surgery and tell the patient to see 
you in a year. 

B. Set up a team with a physiotherapist and/or an occupational therapist at the 
clinic where you work in order to see the patient at all visits. 

C. No follow-up is needed. 



472 Vascular Surgery 

Question 10 

Does liposuction decrease the lymph transport capacity in a patient with a chronic 
arm lymphoedema? 

A. Yes. 

B. No. 



Question 1 1 

Does liposuction with complete reduction of the lymphoedema decrease the inci- 
dence of erysipelas? 

A. Yes. 

B. No. 



Commentary 



Lymphoedema is a chronic disease with increased volume causing considerable 
dysfunction. Patients typically experience decreased mobility, heaviness and sus- 
ceptibility to infections, and are affected by psychological problems that are sec- 
ondary to the cosmetic appearance of their extremity. 

Lymphoedema can be divided into various stages according to the tissue changes 
[1]. It can also be classified as primary and secondary. The later in life a lym- 
phoedema appears, the more important it is to exclude other diseases, especially 
cancer, as the cause. 

There is, so far, no cure for lymphoedema. The basis for all lymphoedema treat- 
ment is adequate compression therapy. If conservative therapy fails, liposuction can 
give complete reduction of the excess limb volume. To maintain this outcome, it is 
an absolute necessity to provide the patient with ample amounts of compression 
garments. It is important to measure the excess volume, as changes can be a sign of 
progression of the underlying disease. 

Up to 38 per cent of women with breast cancer may develop lymphoedema of the 
arm following mastectomy, standard axillary node dissection and postoperative 
irradiation [2]. Limb volume reductions have been reported using various conserv- 
ative therapies such as manual lymphatic drainage and compression therapy. Some 
patients with long-standing pronounced lymphoedema do not respond to these 
conservative treatments, because slow or absent lymph flow causes the formation of 
excess subcutaneous adipose tissue. 

Liposuction removes the hypertrophied adipose tissue and is a prerequisite to 
achieve complete reduction. The new equilibrium is maintained through constant 
(24-hour) use of compression garments postoperatively. Long-term follow-up does 
not show any recurrence of the oedema [3]. 

Pitting means that a depression is formed after pressure with the fingertip on 
oedematous tissue, resulting in displacement of lymph into the surrounding tissue 



Lymphoedema 473 

(Fig. 47b. 1). In order to standardise the pitting-test, one presses as hard as possible 
with the index finger, for 1 minute, on the region to be investigated. The amount of 
depression is estimated in millimetres. 

Arm volumes can easily be measured by circumferential measurements every 
4 cm along the arm according to Kuhnke [4]. In our clinic we use plethysmography 
(i.e. water displacement technique), which is considered the golden standard . Both 
arms are always measured at each visit, and the difference in arm volumes is desig- 
nated as the oedema volume [5, 6]. The decrease in the oedema volume is calculated 
as a percentage, thus: 

(OA -HA )-(OA -HA t ) 

v pre pre' x post post 7 - _. _. 

— x 100, 

OA -HA 

pre pre 

where 

OA pre = oedematous arm before treatment 
HA pre = healthy arm brfore treatment 
OA post = oedematous arm after treatment 
HA post = healthy arm after treatment 

Arm volume measurements for calculating the oedema volume are taken at each 
visit on both arms. 

Oedema dominated by hypertrophied adipose tissue and/or fibrosis shows little 
or no pitting (Fig. 47b. 2). Stemmer's sign implies that one with difficulty, or not at 
all, can pinch the skin at the base of the toes or fingers. This is due to increased 
fibrosis and is characteristic of lymphoedema. 

Indirect lymphoscintigraphy is not necessary in secondary lymphoedema, but is 
recommended in patients with an excess volume where the diagnosis is unclear. 

In healthy subjects the rate of blood flow and lymph flow through adipose tissue 
is inversely related to its growth, and a slow flow rate is considered to be a factor 
promoting lipogenesis and further deposition of fat. This process is enhanced by the 
transformation of macrophages into adipocytes [7]. This may explain the marked 
hypertrophy of the adipose tissue seen in patients with chronic lymphoedema 
(Fig. 47b. 6) [8]. In late stages subcutaneous lymphoedema becomes firm and denser 
and is dominated by adipose tissue hypertrophy, and pitting is usually less pro- 
nounced or sometimes absent (Fig. 47b.2). An additional factor that may play a role 
in increasing the amount of adipose tissue is chronic inflammation present in long- 
standing lymphoedema. This phenomenon is also seen in patients with Crohn's 
disease where the inflamed intestine is surrounded with excess adipose tissue ("fat 
wrapping") [9]. Probably pinocytosis of white blood cells, in combination with acti- 
vation of fibrocytes, increases the connective tissue component of the primordial 
loose subcutaneous fat [10]. 

Despite prophylaxis, the lymphoedema will often progress slowly but steadily, 
producing a variety of symptoms as described in our patient. Surgical treatment is 
indicated in patients who fail to respond to conservative treatment [7, 8, 10]. 

Various surgical procedures have therefore been proposed to reduce lym- 
phoedema, including debulking procedures with skin grafting and omental trans- 
position [11-19]. None of these methods gave satisfactory or long-lasting results. 

The breakthrough in reconstructive microsurgery has stimulated renewed inter- 
est in the management of lymphoedema. During the last decades, anastomoses have 



474 Vascular Surgery 

been established between lymph nodes [20] or lymph collectors [21, 22] and the 
venous system. Promising results have recently been reported after transplantation 
of lymph collectors [23, 24], as well as after the creation of various forms of lym- 
phatic venous anastomoses [25, 26]. 

Even if the microsurgical methods are attractive from a physiological point of 
view, they do not give consistently satisfactory results. Patients need to wear com- 
pression garments after surgery, indicating that normal lymph transport has not 
been achieved. Complete reduction cannot be achieved in patients with a long- 
standing non-pitting lymphoedema because the hypertrophied adipose tissue 
remains unaffected by the microsurgery. A surgical approach, with the intention to 
remove the hypertrophied adipose tissue, seems logical when conservative treat- 
ment has not yielded satisfactory oedema reduction and the patient has not experi- 
enced symptomatic relief. [Q3: E] This condition is especially seen in chronic, large 
arm lymphoedemas around one litre in volume, or when the volume ratio (oedema- 
tous arm/healthy arm) is approximately 1.3. There is no upper age limit in order to 
be considered for liposuction, but active tumour disease and ulcerations are con- 
traindications. The detailed surgical technique has been reported [3, 6, 27, 28]. 

Postoperatively the controlled compression therapy (CCT) is crucial, and its 
application is therefore thoroughly described and discussed at the first clinical eval- 
uation. If the patient has any doubts about continued CCT, she is not accepted for 
treatment. After institution of the compression therapy, the custom-made garments 
(Jobst® Elvarex BSN medical, compression class 2, rarely class 3) are taken in at 
each visit, using a sewing machine, to compensate for reduced elasticity and 
reduced arm volume. This is most important during the first 3 months when the 
most notable changes in volume occur. At the 1 -month visit two new custom-made 
compression garments are measured for. This is repeated at 3, 6, (9), and 12 
months. It is important, however, to take in the garment repeatedly to compensate 
for wear and tear. This requires additional visits in some instances, although the 
patient can often make such adjustments herself. When the oedema volume has 
decreased as much as possible and a steady state is achieved, new garments can be 
prescribed, using the latest measurements. In this way, the garments are renewed at 
least four times during the first year. Two sets of sleeve-and-glove garments are 
always at the patient's disposal; one being worn while the other is washed. Thus, a 
garment is worn permanently, and treatment is interrupted only briefly when show- 
ering and, possibly, for formal social occasions. [Q6: D] The patient is informed 
about the importance of hygienic measures and skin care [3, 6, 27, 28]. 

The life span of two garments worn alternately, is usually 4 to 6 months. After 
complete reduction has been achieved, the patient is seen once a year, when new 
garments are prescribed for the coming year, usually four garments and four gloves 
(or four gauntlets). [Q5: D] Patients without hand oedema can usually discontinue 
the glove or gauntlet after the first year. In very active patients three to four pairs of 
sleeve-and-glove garments a year may be needed. In very young, active patients 
sometimes two sleeve-and-glove garments every month must be prescribed. It is 
like when prescribing insulin. During the first postoperative year the correct 
amount is estimated. If too few, the volume will increase. One must remember that 
a compression garment is not a static tool and should be considered as perishable 
goods. 

Liposuction is preferably performed by plastic surgeons. The technique used for 
lymphoedema is quite different from that used in cosmetic surgery, so even a plastic 
surgeon must learn this approach for lymphoedema. Any interested surgeons, 



Lymphoedema 475 

under careful observation, can therefore learn the technique in order to treat 
lymphoedema. [Q7: E] 

If funding cannot be achieved for ordering the proper amount of garments, then 
surgery - or for that matter any kind of lymphoedema treatment - should not be 
performed. [Q8: B] It is a waste of time and money. It is like sitting in a leaking boat, 
scooping out water all the time instead of putting a plug in the leak. The compres- 
sion garment is the safe plug and the weakest link in lymphoedema treatment. So, a 
prerequisite to maintain the effect of liposuction is the continuous use of a com- 
pression garment. 

The already decreased lymph transport capacity is not further impaired by lipo- 
suction [29]. [Q10: B] Liposuction decreases the incidence of erysipelas. [Q11: A] The 
point of bacterial entry may be a minor injury to the oedematous skin, and 
impaired skin blood flow may respond inadequately to counteract impending infec- 
tion. Reducing the oedema volume by liposuction increases skin blood flow, and 
probably decreases the reservoir of proteinaceous fluid and adipose tissue, which 
may enhance bacterial overgrowth [30]. Through the combination of liposuction 
and CCT the lymphoedema can be completely removed. Long-term follow-up 
(7 years) does not show any recurrence of the oedema [3, 27, 31]. 

In our unit a team that consists of a plastic surgeon, an occupational therapist 
and a physiotherapist assesses patients with lymphoedema. [Q9: B] A 60-minute 
period is reserved for each scheduled visit to the team, when arm volumes are mea- 
sured, garments are adjusted or renewed, the social circumstances are assessed, and 
other matters of concern are discussed. The patient is also encouraged to contact 
the team whenever any unexpected problems arise, so that these can be tackled 
without delay. A team approach such as the one described seems to be ideal in pro- 
viding the patient with a fully informed consent for an intervention, and for suc- 
cessful maintenance of immediate postoperative results. The team also monitors the 
long-term outcome, and our experiences so far indicate that a visit once a year is 
necessary to maintain a good functional and cosmetic result in most cases after 
complete reduction. [Q4: D] 



References 

1. International Society of Lymphology. Summary of the 10th International Congress of Lymphology 
Working Group Discussions and Recommendations. Lymphology 1985;18:175-80. 

2. Kissin MW, Querci della Rovere G, Easton D, Westbury G. Risk of lymphoedema following the treat- 
ment of breast cancer. Br J Surg 1986;73:580-4. 

3. Brorson H. Liposuction in arm lymphedema treatment. Scand J Surg 2003;92:287-95. 

4. Kuhnke E. Die Volumenbestimmung entrundeter Extremitaten aus Umfangsmessungen. 
Lymphologie 1978;2:35-44. 

5. Bernas M, Witte M, Witte C, Belch D, Summers P. Limb volume measurements in lymphedema: 
issues and standards. Lymphology 1996;29(suppl): 199-202. 

6. Brorson H, Svensson H. Complete reduction of lymphoedema of the arm by liposuction after breast 
cancer. Scand J Plast Reconstr Surg Hand Surg 1997;31:137-43. 

7. Ryan TJ. Lymphatics and adipose tissue. Clin Dermatol 1995; 13, 493-8. 

8. Brorson H, A[o]berg M, Svensson H. High content of adipose tissue in chronic arm lymphedema - 
an important factor limiting treatment outcome. Lymphology 1999;32(Suppl):52-4. 

9. Sheehan AL, Warren BF, Gear MW, Shepherd NA. Fat-wrapping in Crohn's disease: pathological 
basis and relevance to surgical practice. Br J Surg 1992;79:955-8. 

10. Gaffney RM, Casley-Smith JR. Excess plasma proteins as a cause of chronic inflammation and lym- 
phoedema: biochemical estimations. J Pathol 1981;133:229-42. 

11. Sistrunk WE. Contribution to plastic surgery. Ann Surg 1927;85:185-93. 



476 Vascular Surgery 

12. Ghormly RK, Overton LN. The surgical treatment of severe forms of lymphedema (elephantiasis) of 
the extremities. A study of end-results. Surg Gynecol Obstet 1935;61:83-9. 

13. Thompson N. Surgical treatment of chronic lymphoedema of the lower limb. With preliminary 
report of new operation. BMJ 1962;ii: 1566-73. 

14. Clodius L, Smith PJ, Bruna J, Serafin D. The lymphatics of the groin flap. Ann Plast Surg 
1982;9:447-58. 

15. Standard S. Lymphedema of the arm following radical mastectomy for carcinoma of the breast; new 
operation for its control. Ann Surg 1942;1 16:816. 

16. Goldsmith SH, De Los Santos R. Omental transposition in primary lymphedema. Surg Gynecol 
Obstet 1967;125:607-10. 

17. Tanaka Y, Tajima S, Imai K, Tsujiguchi K, Ueda K, Yabu K. Experience of a new surgical procedure 
for the treatment of unilateral obstructive lymphedema of the lower extremity: adipo-lymphatico 
venous transfer. Microsurgery 1996;17:209-16. 

18. Charles H. Elephantiasis of the leg. In: Latham A, English TC, editors. A system of treatment, vol 3. 
London: Churchill, 1912;516. 

19. Poth EJ, Barnes SR, Ross GT. A new operative treatment for elephantiasis. Surg Gynecol Obstet 
1947;84:642-4. 

20. Olszewski W, Nielubowicz J. Surgical lymphatico -venous communication in the treatment of lymph 
stasis. Proceedings of the 43rd Congress of Polish Surgeons, 1966; Lodz, Poland. 

21. Laine JB, Howard JM. Experimental lymphatico -venous anastomosis. Surg Forum 1963;14:111-12. 

22. O'Brien BM, Mellow CG, Khazanchi RK, Dvir E, Kumar V, Pederson WC. Long-term results after 
microlymphaticovenous anastomoses for the treatment of obstructive lymphedema. Plast Reconstr 
Surg 1990;85:562-72. 

23. Baumeister RG, Siuda S, Bohmert H, Moser E. A microsurgical method for reconstruction of inter- 
rupted lymphatic pathways: autologous lymph-vessel transplantation for treatment of lymphedemas. 
Scand J Plast Reconstr Surg 1986;20:141-6. 

24. Baumeister RG, Siuda S. Treatment of lymphoedemas by microsurgical lymphatic grafting: what is 
proved? Plast Reconstr Surg 1990;85:64-74. 

25. Campisi C, Boccardo F, Tacchella M. Reconstructive microsurgery of lymph vessels: the personal 
method of lymphatic-venous-lymphatic (LVL) interpositioned grafted shunt. Microsurgery 
1995;16:161-6. 

26. Campisi C, Boccardo F, Alitta P, Tacchella M. Derivate lymphatic microsurgery: indications, tech- 
niques, and results. Microsurgery 1995;16:463-8. 

27. Brorson H, Svensson H. Liposuction combined with controlled compression therapy reduces arm 
lymphedema more effectively than controlled compression therapy alone. Plast Reconstr Surg 
1998;102:1058-67. 

28. Brorson, H. Liposuction and controlled compression therapy in the treatment of arm lymphedema 
following breast cancer. Lund University. Thesis, 1998. 

29. Brorson H, Svensson H, Norrgren K, Thorsson O. Liposuction reduces arm lymphedema without 
significantly altering the already impaired lymph transport. Lymphology 1998;31:156-72. 

30. Brorson H, Svensson H. Skin blood flow of the lymphedematous arm before and after liposuction. 
Lymphology 1997;30:165-72. 

31. Brorson H. Liposuction gives complete reduction of chronic large arm lymphedema after breast 
cancer. Acta Oncol 2000;39:407-20. 



Index 



AAA. See Abdominal aortic aneurysm 
Abdominal aortic aneurysm (AAA) 
claudication and, 17 
demographics of, 29 
endoluminal repair of, 26 
endovascular repair of, 26 
infrarenal, 23-34 
management of, 19, 158 
population screening for, 17 
postoperative mortality and, 15 
preoperative cardiac risk assessment 

and, 3-21 
renal artery stenoses and, 17-18 
ruptured (rAAA), 35-43 
complications after, 38, 42 
endovascular approach to, 38 
endovascular contraindications and, 

38 
open repair of, 41 
preoperative measures for, 40-41 
symptoms of, 39-40 
rupture risk for, 15, 23, 30 
surveillance of, 19-20 
treatment of, 30 
unknown history of, 36 
women and, 29 
Abdominal pain, 39 
generalized, 215 
postprandial, 217 
Abdominal ultrasound, 110 
ACAS. See Asymptomatic Carotid 

Atherosclerosis Study 
Acquired AVF, 128 

Acute axillary/subclavian vein thrombosis, 
305-311 
therapy for, 306 
treatment for, 309 
Acute embolism, 98 



Acute ischaemia of upper extremity, 

following graft arteriovenous fistula, 
359-364 
Acute mesenteric ischaemia (AMI), 221-230 

aetiology of, 224 

causes of, 222 

clinical presentation of, 224 

diagnostic confirmation of, 225 

incidence of, 224 

management of, 222 

surgical aims for, 225-228 

treatability of, 229 
Acute post-lymphangitis, 459 
Acute thrombosis, 71, 97-105 

long-term outcome of, 100 

treatment options for, 99 
Acute tubular necrosis, 351 
ADAM. See Aneurysm Detection and 

Management 
Adipose tissue oedema, 467 

liposuction and, 469 
Advanced trauma life support (ATLS), 116 

less obvious injuries and, 119 
Adventitial cystic disease 

cross-section of, 187 

location of, 188 

of popliteal artery, 181-189 
aetiology of, 181 
diagnosis of, 183 
reported cases of, 187 
treatment options for, 184 

reported cases of, 187 

statement of, 183 

treatment options for, 188 
Age, 3, 7 

AAA and, 29-30 
Air-plethysmography (APG), 414 

surgical outcome assessment by, 418 
Ambulatory venous pressure, 417-418 



477 



478 



Index 



AMI. See Acute mesenteric ischaemia 
Amputation, 117, 118, 365-392 

at ankle, 372 

Chopart amputation, 372 

diabetic foot and, 203 

Gritti-Stokes amputation, 373-374 

leg before, 371 

level of, 374 

limb ischaemia and, 170, 367-375 

limb salvage and, 120 

Lisfranc operation, 372-373 

performance of, 374-375 

popliteal artery bypass and, 165 

stump healing, 370 

transtibial, 370 
Anastomotic aneurysms, 79-85 

aetiology of, 82 

from aorto-bifemoral graft, 80 

complications of, 80 

incidence of, 81 

infection of, 84 

intra-abdominal, 84 

local infection and, 82 

management of, 81 

non-operative treatments of, 80 

occurrence of, 84 

perioperative antibiotics and, 84 

surgical intervention indications for, 83 

surgical outcome for, 84 
Anastomotic false aneurysm aetiology, 79 
Aneurysm 

anastomotic, 79-85 

anatomically similar, 26 

angulation of, 25 

aortic bifurcation of, 25 

arterial, 128 

dilation of, 47 

distal extent of, 51 

endoluminal repair and, 26, 29 

endovascular repair of, 30, 32-34 

erectile dysfunction sudden onset, 157 

iliac angulation of, 33 

natural history of, 18 

neck of, 24, 33 

peripheral, 70 

preoperative assessment method for, 23 

rupture of, 37, 76 

sack of, 24 

underestimation of, 20 
Aneurysmal aorta dilation, 47 
Aneurysmal disease types, 13-14 
Aneurysm Detection and Management 

(ADAM), 17 
Aneurysm size, 17 

complications due to, 83 



Angina pectoris, 3-5, 7, 13 
Angioaccess, 359 
Angiogram, 27 

contralateral femoral approach to, 98-99 

coronary, 150 

of deployed graft, 29 

of femoropopliteal vein bypass graft, 368 

on-table, 28 

ofPAE, 175 

percutaneous, 168 
Angiography, 28 

acute thrombosis and, 104 

arterial embolism and, 108 

blast injury and, 116 

of carotid body tumour, 268, 269 

of diabetic foot, 208, 210 

intra-arterial renal artery, 74 

intraoperative monitoring by, 210 

of left lower limb, 204 

lesions showing on, 143 

magnetic resonance, 50 

ofOFB, 194 

of popliteal artery, 185 

renovascular hypertension and, 232 
Angioplasty 

chronic visceral ischaemia and, 218 

for critical arterial stenosis, 141-146 

patch, 164-165 

path, 254 

postoperative management for, 153-154 

of superior mesenteric artery, 219 

vertebrobasilar ischaemia and, 285 
Angulation of neck, 34 
Ankle amputation, 372 
Ankle brachial pressure index (ABPI), 142 

intermittent claudication and, 154 
Anticoagulants 

ATIII as, 398 

bypass and, 170 

heparin and, 310 

iliofemoral DVT and, 440 

overnight administration of, 448-449 

symptomatic dissection and, 281 

venous thromboembolism and, 447 
Antigrade revascularization, 217 
Antiphospholipid antibody syndrome 

(APA), 395, 399 
Antiplatelet agents, 137 
Anti-thrombin III (ATIII) 

clinical presentation of deficiency of, 395 

DVT and, 398 
Antithrombotic therapy, 447 
Aortic aneurysm, 14 

rupture of, 36 

thoracoabdominal, 45-55 



Index 



479 



Aortic aneurysmal wall, 19 
Aortic angiogram, 27 

neck angulation and, 34 
Aortic disease, 157-159 
Aortic dissection, 57-65 

acute 

consideration of, 63 
diagnostic methods for, 60 
operative intervention for, 63 
prognosis of, 62 

beta-blockers and, 63 

blood flow restoration following, 61 

classification of, 51 

complications with, 62 

start of, 62 

thrombosed aortic lumen and, 61 

vascular complications, 63 
Aortic graft 

aortofemoral graft infection and, 323 

appearance of, 324 

infection occurrence in, 330 

replacement, 332 
Aortic lumen, 61 
Aortic surgery, 63-64 

aortoenteric fistula and, 337 
Aorto-bifemoral graft, 16 

anastomotic aneurysm and, 79 

groin abscess and, 331 

lower limb claudication and, 151 
Aorto-bi-iliac graft, 16 

Dacron, 337 
Aortoenteric fistulas, 337-341 

aortic surgery and, 337 

bowel and, 339 

definition of, 339 

diagnosis of, 340 

management of, 340-341 

postoperative mortality of, 341 

presence of, 338 

surgical treatment for, 341 
Aortofemoral graft infection, 323-335 

Dacron grafts and, 332 

management of, 324 

operation performance for, 326 

preoperative investigations of, 326 
Aortofemoral graft replacement, 325 
Aortofemoral prosthesis removal, 195-196 
Aortogram 

mural thrombosis and, 47 

of neck angulation, 33 

ofTAA,46 
Aortography of right iliac thrombosis, 98 
Aortoiliac graft replacement, 325 
Aortoiliac lesions, 153 
Aorto-superior mesenteric artery bypass, 219 



APBI. See Ankle brachial pressure index 
APG. See Air-plethysmography 
Argatroban, 397, 401 
Arm 

arteries in, 347 

swelling of, 305, 349 

volume measurement of, 466, 473 

volume reduction of, 457 
Arterial aneurysms, 1-94, 128 
Arterial blood pressure, 11 
Arterial bypass construction, 170 
Arterial compression, 283 
Arterial defects, 383 

ET form of, 383 
Arterial degeneration, 82 
Arterial duplex scan, 148 
Arterial embolectomy, 109 
Arterial embolism, 107-113 

acute ischaemia and, 110 

etiology of, 107 

fasciotomy and, 109 

heparin and, 108 

preferred treatment of, 108 
Arterial graft complex weakening, 82 
Arterial hypertension, 73, 11 
Arterial inflow, 352 
Arterial injury, 129 
Arterial ischemia, 107-108 
Arterialization, 353-354 
Arterial occlusion, diagnosis of, 103-104 
Arterial system duplex scan, 367 
Arterial thrombosis dissolution, 105 
Arterial ulcer, 415-416 
Arterial vascular surgery complications, 

321-341 
Arteriogram 

of carotid artery, 280 

performance of, 14-15 

of vertebral artery, 279 

of vertebrobasilar ischaemia, 278 
Arteriography, 14, 67 

aneurysm size and, 17 

of carotid body tumor, 273 

ofCFA,99 

PAEand, 174 

preoperative, 121 
Arteriotomy 

closure of, 254 

longitudinal, 253-254 
Arteriovenous fistula (AVF) 

acute ischaemia and, 359-364 

vs. AV grafts, 359 

cause of, 125 

complications of, 125-126, 129 

creation of, 361-363 



480 



Index 



Arteriovenous fistula-conf. 

diagnosis of, 127, 129 

endoluminal treatment of, 125-130 

first-choice site for, 361 

hemodialysis access and, 350 

iliofemoral venous thrombosis and, 437 
pregnancy and, 445 

images of, 126-127 

location of, 128 

proximal saphenous vein and, 446 

treatment of, 128, 129 
Arteriovenous graft 

vs. AVF, 359 

maturation time of, 362 
Arteriovenous shunting defects, 383 
Ascending phlebography, 379 
Aspirin antiplatelet therapy, carotid 
bifurcation disease and, 252 
Asymptomatic Carotid Atherosclerosis 

Study (ACAS), 255 
Asymptomatic high-grade carotid stenosis, 

256 
Atherosclerotic aorta, 226 
Atherosclerotic diabetic changes, 209 
Atherosclerotic occlusive disease, 97 
ATIII. See Anti-thrombin III 
ATLS. See Advanced trauma life support 
Atrial fibrillation, 221 
Autologous vein, popliteal artery bypass 

and, 163 
Autotransfusion, 19 
AVF. See Arteriovenous fistula 
Axillary artery, AVF and, 128 
Axillary/subclavian vein thrombosis 

acute, 305-311 
therapy for, 306 

complications of, 308 

conditions for, 306 

diagnostic investigations for, 305 

stenosis of, 307 

thrombolytic therapy for, 307 
Axillo-(bi)-femoral graft, 64 

aortoenteric fistula and, 339 



B 

Back pain, 35 

Bacterial culture, 198 

Baker cyst, 184 

Basilic vein, 347 

Benign tumours, 272, 275 

Beta-adrenergic antagonists, 8-9 

Beta-blocker 

acute aortic dissection and, 60 
aortic dissection and, 63 



associations of, 5 

cardiac death and, 8 

cardiac events and, 18 

hypertension and, 147 

myocardial infarction and, 8 

Stanford A dissection and, 58 

statins and, 6 

therapy with 
dosage in, 6 
duration of, 6 

vascular surgery and, 5-6 
Bifurcation endarterectomy, 253-254 
Bilateral carotid duplex scan, 251 
Bilateral palmar hyperhydrosis, 297 
Bilateral popliteal occlusion, 176 
Birth control pills, 423 
Bisoprolol, 8 
Blast injury, 115-124 

angiography and, 116 

care for, 115 

fractures and, 122 

surgery and, 116-117 
Bleeding, 355 
Blood count, 222-223 
Blood flow following aortic dissection, 61 
Blood gas, 222-223 
Blood pooling, 388 
Blood pressure 

PAD and, 137-138 

popliteal artery bypass and, 161 
Blunt trauma, 265 
Bowel 

aortoenteric fistula and, 339 

appearance of, 223 

flow restoration operative options for, 223 

viability of, 224 
Brachial anastomoses, 353 
Brachial artery, ligation of, 360-361 
Brachial artery variant, 354 
Brainstem infarction, 278 
Breast cancer, lymphoedema and, 465, 472 

pitting and, 466 
Breastfeeding, Coumadin and, 450 
Bypass 

of aorto-superior mesenteric artery, 219 

arterial, 170 

coronary, 348 

withePTFE, 168 

graft occlusions, 164 

graft patency and, 169 

graft salvage of, 164 

mortality and, 169 

obturator foramen, 191-200 

of popliteal artery, 161-166 

saphenous vein and, 280 



Index 



481 



CABG. See Coronary artery bypass 

grafting 
Calcanectomy, 369 
Capillary loop dilation, 315 
Capillary malformation clinical 

significance, 391 
Cardiac complications 

beta-blockers and, 8 

perioperative morbidity and, 3, 7-8 
Cardiac echography, 15 
Cardiac events 

beta-blockers and, 18 

perioperative, 3 
Cardiac risk assessment, 15 
Cardiac scintigraphy, 15 
Cardiac stress test, 48 
Cardiovascular events, 9 
Cardiovascular risk factors, 133-140 
Carotid artery 

arteriogram of, 280 

external embolisation of, 275 

stenosis of, 255 

management of, 256 
Carotid artery disease, 256 
Carotid bifurcation disease 

anatomy of, 263 

appropriate management of, 252-253 

controversy in, 257 

Dacron patch and, 254 

evaluation of, 215 

fast-track management of, 254-255 

intravenous heparin and, 253 

management of, 251-258 

patch angioplasty and, 254 

surgery for, 253-254 

symptoms of, 252 

work-up of, 256-257 
Carotid body tumour, 267 

anatomy of, 271 

angiography of, 268-269 

clinical presentation of, 272-273 

demographics of, 272-273 

development of, 272 

OctreoScan of, 270 

postoperative mortality and, 273-275 

surgical classification for, 275 

surgical excision of, 271 

swelling of, 268 

treatment of, 268-269, 273-274 
Carotid bruit, 251 
Carotid duplex scan data, 256 
Carotid endarterectomy, 252, 255 

complications of, 260-261 



cranial nerve injuries, 259-266 

indications for, 256-257 

nerve damage during, 261-262 

preoperative examination before, 259 

reevaluation of, 260-261 

speech malfunction and, 260 
Carotid lesion, 252 
Carotid stenosis, 265 
CARP. See Coronary artery 

revascularization prophylaxis 
CASS. See Coronary Artery Surgery Study 
Catheterization 

central access for, 354 

false aneurysm and, 91 
CCT. See Controlled compression therapy 
CDT. See Complex decongestive therapy 
CEAP classification, 426 

advanced, 428 

detailed version of, 426-427 
Cell rests, 187 

Cell-saver autotransfusion (CSA), 16, 19 
Cellulitis, 413 

Central catheter access, 354 
Central vein stenoses, 352 
Central venous pressure (CVP), 222 
Cephalic anastomoses, 353 
Cerebellar infarction, 278 
Cerebrospinal fluid (CSF), 52 
CFA. See Common femoral artery 
Chest pain, 45 

Child-Pugh classification, 245 
Cholesterol levels 

popliteal artery bypass and, 161 

statins and, 7 
Chopart amputation, 372 
Chronic critical limb ischemia, 167-171 

preoperative medications/lifestyle 
changes for, 168 
Chronic deep venous obstruction, 420 
Chronic obstructive pulmonary disease 
(COPD),20 

TAA and, 51-52 
Chronic visceral ischaemia, 215-220 

colonoscopic view of, 216 

diagnosis of, 218 

graft material for, 219 

symptoms of, 218 
Chronic warfarin therapy, 398 
Chylous dysfunction preoperative data, 

460-461 
Circumaortic left renal vein, 243 
Cirrhosis, 239 

bleeding and, 241-244 

varices and, 247 



482 



Index 



Claudication 
AAA and, 17 

intermittent, 133, 191, 337 
ABPI and, 154 

adventitial cystic disease and, 181, 183 
clinical presentation of, 185-186 
demographics of, 144 
erectile dysfunction and, 150 
exercise and, 151 
surgery and, 163 
symptoms of, 153 
of lower limb 

due to iliac artery occlusive disease, 

147-155 
erectile dysfunction and, 150 
imaging modality for, 148 
interventions for, 148 
non-disabling symptoms of, 144 
pain of, 416 

quality of life and, 151-152 
recurrent, popliteal artery bypass and, 

162 
relief for, 147 
surgery and, 177 
tolerable level of, 142 
venous, 416 
Clindamycin treatment for diabetic foot, 

202 
CNS. See Coagulase-negative staphylococci 
Coagulase-negative staphylococci (CNS), 

192 
Coagulopathy, 55 
Cockett's syndrome, 419 
Coeliac artery occlusion, 216 
Coliform graft infection, 331 
Colonoscopy, 215-216 
Colour Doppler 

during muscle contraction, 174 
PAEand, 173, 178 
Colour duplex sonography, 434, 436 
Combined decongestive therapy (CDT), 

465 
Common femoral artery (CFA), 87 
aortofemoral graft infection and, 323 
arteriography of, 99 
false aneurysm and, 88 
Common iliac artery, 59 
aneurysm of, 69 
right, 68 
Common iliac bifurcation, 327 
Common iliac vein phlebogram, 444 
Compartment syndrome, 101 
Complex decongestive therapy (CDT), 380 
Complications of arterial vascular surgery, 
321-341 



Compression garments 

liposuction and, 470, 475 

wear- time of, 471 
Concept of OFB, 196 

Congenital vascular malformation (CVM), 
377-392 

classification of, 383 

clinical evaluation of, 378-380 

clinical presentation of, 382-384 

diagnosis of, 377-378 

embolosclerotherapy for, 380 

hemodynamic assessment and, 386 

multidisciplinary approach to, 389 

symptomatic lesions of, 379-380 

treatment of, 377-378 
Conservative therapy 

for hyperhydrosis, 297 

TOS and, 293-294 
Contralateral groin vessels, 437 
Controlled compression therapy (CCT), 
474 

liposuction and, 469 
COPD. See Chronic obstructive pulmonary 

disease 
Coronarography, 15 
Coronary angiography, 18, 87 
Coronary angioplasty, 87-94 
Coronary artery bypass grafting (CABG), 
10-11,23 

percutaneous revascularization and, 1 1 
Coronary artery disease, 8 
Coronary artery revascularization 

prophylaxis (CARP), 10 
Coronary Artery Surgery Study (CASS), 1 1 
Coronary bypass, 348 
Coronary heart disease (CHD) 

equivalents of, 136 

PAD and, 136 
Coumadin 

anticoagulation with, 450 

breastfeeding and, 450 

DVT and, 395 
Cranial nerve anatomy, 263 
Cranial nerve injuries 

blunt trauma and, 265 

carotid arterial surgery and, 261 

carotid endarterectomy and, 259-266 

incidence of, 262 

increased risk of, 265 

persistence of, 262 
Craniocervical junction, 284 
Crawford classification system 

ofTAA, 50 

TAA and, 46 
Creatinine, 39 



Index 



483 



Critical arterial stenosis 

adjunctive treatment for, 144 

angioplasty for, 141-146 

PTA and, 145 
Critical ischaemia, 373 
Critical stenosis, 144 
Cross-clamp time, 52 
CSA. See Cell-saver autotransfusion 
CSF. See Cerebrospinal fluid 
CT scan 

aneurysmal dilation and, 47 

benefits from, 40 

rAAA and, 37 

ofTAA,46 
Cutaneous disease, 318 
Cutaneous telangiectases, 403 

varicose veins and, 408 
CVM. See Congenital vascular 

malformation 
CVP. See Central venous pressure 
Cyst 

surrounding popliteal artery, 186 

wall of, 186 



Dacron aorto-bifemoral bypass, 191 
Dacron graft, 61 

anastomotic aneurysm and, 80-82 

aorto-bi-iliac, 337 

aortoenteric fistula and, 338-339 

aortofemoral graft infection and, 332 

bifurcated aortic, 323 

carotid bifurcation disease and, 254 

chronic visceral ischaemia and, 217 

extra-anatomical reconstruction with, 61 

femoral arteries and, 192 
Danaparoid, 401 
de Bakey I dissection, 57 
de Bakey II dissection, 57 
de Bakey III dissection, 57 
Debridement, 368 
Deep vein system, 385 

varicose veins and, 404 
Deep venous insufficiency 

chronic, 416 

venous ulcers associated with, 413-421 
Deep venous obstruction, 420 
Deep venous reflux, 418-419 
Deep venous thrombosis (DVT), 395-402 

acute thrombolysis for, 397-398 

birth control pills and, 423 

Coumadin and, 395 

heparin and, 395 

HIT and, 398 



iliofemoral, 401 

incidence of, 400 

natural history of, 398 

perioperative, 396-397 

prophylaxis for, 396, 399 

proximal, 398 

risk factors for, 395, 398, 400 

sigmoid cancer and, 395 

venous ulcers and, 426-427 
Defibrillator, 349 
Descending venography, 417 
Diabetes mellitus, 7, 134. See also Type 2 
diabetes 

angiographic findings of, 205 

arterial embolism and, 107 

atherosclerotic changes in, 209 

limb ischemia and, 167 

renal disease and, 345 

venous ulcers and, 427 
Diabetic foot, 201-211, 203 

angiopathy assessment of, 202 

circulation examination for, 203 

classification of lesions of, 209 

clindamycin treatment of, 202 

Doppler-derived pressures of, 209 

intraoperative control of, 206 

ischaemia and, 210 

neuropathy assessment of, 202 

primary diagnostic work-up of, 209 

revascularization and, 210 

symptoms of neuropathy for, 209 

treatment strategy for, 205 

vascular reconstruction of, 205 

venous ulcer and, 415 

yearly follow-up of, 206 

control angiography of, 208 
Dialysis outcomes quality initiative 

(DOQI), 362 
Diarrhea, 191,221 
Diazepam, 433 
Digital pitting, 317 
Digital pulp calcinosis, 318 
Digital subtraction angiography (DSA), 

306 
Digital sympathectomy, 319 
Discrete semilunar deformity, 185 
Dissection of superficial femoral vein, 327 
Distal embolization, 71 
Distal endoleak, 33 
Distal extent of aneurysm, 51 
Distal revascularization interval ligation 

(DRIL), 363 
Distal splenorenal shunt (DSRS), 246 

vs. TIPS, 247 
Distended abdomen, 215 



484 



Index 



Dobutamine, 4 

cardiac echography and, 15 
Doppler evaluation, 245 

GSV and, 426 
Doppler pressure measurement, 360 
DOQI. See Dialysis outcomes quality 

initiative 
Dorsalis pedis artery (DPA), 142 
DPA. See Dorsalis pedis artery 
DRIL. See Distal revascularization interval 

ligation 
Drionic, 297 
Drysol, 297 

DSA. See Digital subtraction angiography 
DSRS. See Distal splenorenal shunt 
DU. See Duplex ultrasound examination 
Duodenum, 339 
Duplex scan surveillance, 16 

of arterial system, 367 

of carotid artery, 251, 256 

of femoral veins, 326 

primary axillary/ subclavian vein 
thrombosis and, 308 
Duplex ultrasonographic study, 387 
Duplex ultrasound examination (DU), 347 

information from, 351-352 

preoperative, 346 
function of, 352 
reasons for, 352 
DVT. See Deep venous thrombosis 
Dyslipidaemia, 136 



Ecchymosis, 377 
Eccrine sweat glands, 299 
Echocardiogram 

normal, 5 

pulmonary embolism and, 441-442 

transthoracic, 225 
Echocardiography 

aortic dissection and, 62 

normal stress protocol of, 4 

stress test and, 6 
Echo-duplex, 273 

ECST. See European Carotid Surgery Trial 
EIA. See External iliac artery 
Electrocardiogram, 35 
Embolic ischaemia, 277-281 
Embolic occlusion, 97 

of left popliteal artery, 112 
Embolic phenomena, 102 
Embolism 

acute, 98 

arterial, 107-113 



external carotid artery and, 275 

peripheral MI, 110 

pulmonary, 400 

echocardiogram of, 441-442 
venous system interruption and, 437 

recurrent, 112-113 
Emboloscleroagents, 389-390 
Embolosclerotherapy, 380 
Embryolic developmental arrest, 384 
Emerging risk factors, 138 
Encaphalopathy, 240 
Endarterectomy 

bifurcation, 253 

carotid, 252 
Endoleak, 42 

distal, 33 
Endoluminal graft, 26-28 
Endoluminal repair 

AAA complications of, 31 

of aneurysm, 29 

complications after, 31 
Endoluminal treatment 

of carotid stenosis, 265 

infrarenal AAA and, 23-34 

of traumatic AVF, 125-130 
Endoscopic retrograde 

cholecystopangreatography (ERCP), 
215-216 
Endovascular repair, 26 

ofAAA(EVAR), 31-32, 41 
morbidity rate of, 42 
mortality rate of, 42 

of AVF, 129 

of common iliac artery aneurysm, 69 

devices for, 31 

favorable anatomy for, 32 
Endovascular stent, 38 
Endovascular treatments 

false aneurysms and, 92 

for iliofemoral venous thrombosis, 
437-438 
Episodic digital ischaemia, 316 
ePTFE. See Expanded 

polytetrafluoroethylene 
ERCP. See Endoscopic retrograde 

cholecystopangreatography 
Erectile dysfunction 

aetiology of, 150 

claudication and, 154 

due to aortic disease, 157-159 

lower limb claudication and, 150 

postoperative treatment for, 159 

sudden onset of, 157 
Erectile function, 158 
Erysipelas, 472 



Index 



485 



Erysipeloid lymphangitic attacks, 453 

Esophageal varices, 239 

Essential hypertension, 233 

Ethanol, 390 

European Carotid Surgery Trial (ECST), 

255 
EVAR. See Endovascular repair, of AAA 
Exercise, 151 
Expanded polytetrafluoroethylene (ePTFE), 

168 
Exposed prosthetic graft 

complications of, 328-329 

management of, 328 

surgery for, 329-330 
External iliac artery (EIA) 

hydrophilic guidewire and, 152 

lower limb claudication and, 148 

stent placement in, 152 
Extracranial cerebrovascular disease, 

249-285 
Extratruncular form (ET), 379-380, 
383-386 



Facial nerve marginal mandibular branch, 

264 
Factor V Leiden gene mutation, 396, 399 
False aneurysm, 83 

cavity of, 90 

common femoral artery (CFA), 87 

complications of, 91 

endovascular treatments for, 92 

in groin, 87-94 

postcatheterization and, 88, 91 

thrombin injection and, 92 
Fasciotomy 

arterial embolism and, 109, 112 

formal, 118 

healing from, 119 
Fasting glucose values, 133-134 

impaired, 134 
Fasting HDL-C level, 135 
Fasting triglyceride cycle, 134 
Fecal fat measurement, 215 
Femoral anastomosis 

local muscle flap and, 333 

rectus femoris muscle flap and, 329 
Femoral artery pulse, 434 
Femoral bifurcation 

arterial embolism and, 108 

reconstitution of, 99 
Femoral false aneurysm catheterization, 91 
Femoral-femoral crossover bypass, 64 

intermittent claudication and, 154 



Femoral pulses, 142 
Femoral vein 

fishmouthing of, 327 

graft replacement with, 333 

reconstruction of, 328 

varicose veins and, 406 

venectasia of, 379 
Femoral vein duplex scan, 326 
Femoral vein ligation, 123 
Femoropopliteal artery exposure, 197-198 
Femoropopliteal atherosclerotic disease, 

164 
Femoropopliteal bypass, 162 
Femoropopliteal vein 

angiogram of, 368 

aortofemoral graft infection and, 325 

graft infection and, 332 
Fetus, 449 

FFP. See Fresh frozen plasma 
Fibrinolytic factors, 138 
Fingertip ulcer, 313 

healing of, 313-314 

systemic sclerosis and, 314 
First rib resection, 293 
Fishmouthing, 327 
Fistula 

aortoenteric, 337-341 

arteriovenous, 125-130 

diagnosis of, 340 

exclusion of, 126-127 

main symptom of, 340 

management of, 340-341 

postoperative mortality of, 341 
Flucloxacillin, 316 
Fluoroscopy, 449 

Fogarty catheter embolectomy, 111 
Four-layer compression therapy, 425 
Fresh frozen plasma (FFP), 49 



Gastrocnemius muscle, 179 
Gastroduodenoscopy, 338 
Gastroesophageal varices 

decompression of, 240-241 

prophylactic management of, 241-244 
Gastrointestinal haemorrhage, 340 
Gastroscopy, 215 
Glossopharyngeal nerve, 264 
Glucose status, 138 
Graft 

angiogram of, 29 

aortofemoral infection, 323-335 

axillo-(bi) -femoral, 64 

Dacron, 337 



486 



Index 



Graft- co nt. 

deployment of, 28 
excision of, 332 
exposed, 328 
failure 

before complete occlusion, 165 

early, 165 

options for, 162 
infection of, 192-193, 330 

aortic replacement and, 332 

diagnosis of, 331 

distal part of, 195 

extent of, 195 

femoropopliteal veins and, 332 

proximal limit of, 193 

treatment options for, 331-332 
malfunction of, 42 
material for, 219 
occlusion of, 195 
patency of, 163 

bypass and, 169 
patent aortorenal venous, 76 
prosthetic 

groin exposure of, 333 

patency rates for, 164 
pulsating, mechanical stress on, 340 
replacement of, 333 
salvage of, 164 
size of, 3 1 
of vein, 207 
Great saphenous vein (GSV), 423 

Doppler examination of, 426 
Gritti-Stokes amputation, 373-374 
Groin 

pulse in, 433-434 
vessels in, 437 
Groin abscess aorto-bifemoral graft, 331 
Groin wounds, 42 

bacterial culture of, 198 
infection of, 192 
GSV. See Great saphenous vein 
Gunshot wound, 125 



H 

Haematemesis, 339 
Haematuria, 76 
Haemofiltration, 58 
Haemorrhage 

inspection of, 119 

obvious, 120 

vascular surgery and, 121 
Haemorrhagic bullae, 434-435 
Haemostatic factors, 138 
Hamburg classification, 383, 391 



Hand muscle atrophy, TOS and, 293 

Hand numbness, 360 

Hard signs, 120 

HDL-C levels, 136 

Head 

paragangliomas of, 267-276 

rotation of, 283-284 
Heel ulcer, 367-368 

debridement of, 368 

subtotal calcnectomy and, 369 
Hemangioma, 391 
Hemispheric stroke, 253 
Hemoaccess, 346 

distal sites of, 356 

hemodialysis and, 348 

loss of, 351 

optimal, 350 
Hemodialysis, 39 

AVF and, 362 

hemoaccess and, 348 

immediate, 351 

thrombosis and, 348 
Hemodialysis access 

AVF and, 350 

nephrology work-up for, 345-346 

optimal conduit for, 345-357 
options for, 351-356 
Hemodynamic assessment 

CVM and, 386 

by duplex ultrasonography, 387 
Hemodynamic impact, 379 
Hemolymphatic malformation (HLM), 383 
Hemothorax, 125 
Heparin 

arterial embolism and, 108 

carotid bifurcation disease and, 253 

DVT and, 395 

false aneurysm and, 89 

HIT and, 400 

hypertension and, 87 

iliofemoral DVT and, 439-440 

low-molecular-weight, 414 

pregnancy and, 441 

thrombolysis and, 310 

thromboses and, 93 
Heparin-induced thrombocytopoenia 
(HIT), 397 

development of, 400 

DVT and, 398 

occurrence of, 400-401 
Heparinisation, 308 
Hepatitis C, 239 
Herald bleeding, 338 
High-flow AVF, 355 
Hirudin, 401 



Index 



487 



HIT. See Heparin-induced 
thrombocytopoenia 
HLM. See Hemolymphatic malformation 
Homocysteine, 138 
Honeycomb pattern, 467 
Horner's syndrome, 299, 300-301 
Hourglass stenosis, 181 
Hydrophilic wire, 100 

EI A and, 152 

lower limb claudication and, 151 
Hypercholesterolaemia, 135 

aortofemoral graft infection and, 323 

limb ischemia and, 167 

popliteal artery bypass and, 161 
Hypercoagulability, 355 
Hyperhomocystinaemia, 399 
Hyperhydrosis, 297 

bilateral palmar, 297 

compensatory, 299 

primary, 297, 299 

primary palmar natural history, 297, 299 
Hypertension, 13 

aortofemoral graft infection and, 323 

beta-blockers and, 147 

essential, 233 

false aneurysm and, 87 

limb ischemia and, 167 

popliteal artery bypass and, 161 

portal, 237-248 

renovascular, 231-235 
origin of, 231, 233 

smoking history and, 157 

TAA and, 45 

venous, 408 
Hypertriglyceridaemia, 135 
Hypogastric aneurysm, 14-16, 18 
Hypoglossal nerve 

bilateral injury to, 265 

function of, 262 
Hypotension, 39 
Hypothermia, 53-55 



I 

Iatrogenic injury, 128 

IFG. See Impaired fasting glucose 

Iliac bifurcation, 149 

Iliac occlusion, 104 

Iliac stenosis, 102 

Iliac stenting, 97 

Iliac system oblique projection, 149 

Iliac thrombosis, 98 

hydrophilic wire and, 100 
Iliac vein stenosis, 443, 449 
Iliac vein stent placement, 419-420 



Iliofemoral DVT, 401 

anticoagulation and, 440 

heparin and, 439-440 

during pregnancy, 439-450 
anticoagulants and, 441 
overnight anticoagulation for, 448-449 
pulmonary embolism and, 448 
venous thrombectomy and, 440 
Iliofemoral system 

patency restoration to, 445 

thrombectomy and, 449-450 
Iliofemoral thrombectomy, 443, 444 
Iliofemoral venous thrombectomy, 449 
Iliofemoral venous thrombosis, 433-438 

AVF and, 437 

common name for, 435 

diagnosis of, 435-436 

endovascular treatment options for, 
437-438 

management of, 436 

thrombectomy and, 435 

treatment aims for, 436 
Impaired fasting glucose (IFG), 134 
Impotence 

definition of, 159 

due to aortic disease, 157-159 
Incompetent perforator vein management, 

425 
Indirect laryngoscopy, 260, 264 
Infantile hemangioma, 391 
Infarction etiology, 278 
Infection 

anastomotic aneurysm and, 82, 84 

presence of, 324 
Inferior vena cava (IVC) filter indications, 

397,401 
Inflammation markers, 138 
Infradiaphragmatic aorta surgery, 213-235 
Infrainguinal DVT, 440 
Infrainguinal system 

patency restoration to, 445 

thrombectomy and, 449-450 
Infrarenal aorta, 75, 149 
Infrarenal aorto-iliac membrane resection, 58 
INR. See International normalized ratio 
Intercostal neuralgia, 300 
Intercostal vessel reimplantation, 53 
Internal carotid artery (ICA) stenosis, 259 
Internal iliac artery, 68 
Internal valvuloplasty, 418 

technique for, 419 

venous ulcer and, 414 
International normalized ratio (INR) 

levels of, 277 

venous ulcer and, 414 



488 



Index 



International Society for Study of Vascular 

Anomaly (ISSVA), 381 
Interposition vein graft, 69 
Intimal hyperplasia, 164 

carotid bifurcation disease and, 255 
Intra-abdominal graft portion, 324 
Intra-arterial digital subtraction 

angiography, 325 
Intra-arterial renal artery angiography, 74 
Intraluminal shunt, 118, 123 

use of, 122 
Ipsilateral lung deflation, 298 
Ischaemia 
acute 

arterial embolism and, 110 
AVF and, 359-364 
diagnosis of, 111 
mesenteric, 221-230 
acute arterial, 107-108 
blood flow restoration and, 61 
chronic visceral, 215-220 

diagnosis of, 218 
classic features of, 228 
critical, 191,373 
diabetic foot and, 202, 210 
episodic digital, 316 
of leg, signs of, 363 
of limb, 97-98, 116, 167-171 
acute, 104 

amputation and, 367-375 
critical, 371 
etiology of, 102-103 
primary amputation of, 369 
treatment for, 104 
of lower extremity, 103 
low-flow symptoms and, 284 
patchy, 228 
posturally induced, 282 

head rotation and, 283-284 
recurrent visceral, 219 
steal-induced, 363 
symptoms of, 285 
transient attacks of, 252, 256, 277 
ulcers and, 415 
of viscera, 226 
Ischaemic colitis, 216 
Isotope renogram, 232 
ISSVA. See International Society for Study 

of Vascular Anomaly 
IVC. See Inferior vena cava 



K 

K-DOQI. See United States Kidney Dialysis 

Outcomes Quality Initiative 
Kistner technique, 414 
Klippel-Trenaunay's disease, 460 
Knee 

disarticulation of, 373 

pulse in, 433-434 



J 

Javid shunt, 117-118 



Laminectomy, 282 

Laparotomy, 41 

Large-vessel disease, 159 

Laryngeal nerve injury treatment options, 

261 
Laryngoscopy, 260, 264 
LCX. See Left circumflex artery 
LDUH treatment, 399 
Left circumflex artery (LCX), 4 
Left common iliac aneurysm, 14 
Left common iliac artery, 59 
Left external iliac artery (EIA), 142 
Left hypogastric aneurysm, 14 
Left leg ischaemia, 58 
Left renal artery, 28 

saphenous vein bypass by, 59 
Leg pain, 141 
Lepirudin, 401 
Lesions 

along knee, 379-380 

angiography and, 143 

aortoiliac, 153 

diabetic foot and, 209 

type A, 153 

type C, 154 

ofVM, 380 
Life expectation of vascular amputee, 374 
Ligation 

bleeding and, 356 

of brachial artery, 360, 361 
Limb 

length discrepancy of, 389 

residual, 372 

revascularization of, 369 
Limb injury postoperative management, 

123 
Limb ischemia, 167-171 

amputation and, 367-375 
Limb salvage, 120, 122 
Limb swelling, 111-112 

venous disease and, 417 
Limited cutaneous disease, 318 
Lipids, 138 



Index 



489 



Lipoprotein, 138 
Liposuction 

CCT and, 469 

compression garments and, 470 

erysipelas and, 472 

hypertrophied tissue and, 472 

lymphoedema and, 469 

lymph transport capacity and, 472 

outcome of, 471 

performance of, 471, 474-475 

10 years postoperative, 470 

upper extremity lymphoedema 

management through, 465-476 
Lisfranc operation, 372-373 
Liver disease stratification, 245 
LM. See Lymphatic malformation 
LMWH. See Low-molecular-weight heparin 
Local infection, 82 
Local muscle flap, 333 
Long saphenous vein (LSV), 404 

varicose veins and, 407 
surgical treatment for, 409 
Lovenox, 446 
Lower limb blast injury, 115-124 

angiography and, 116 

assessment of, 122 

care for, 115 

fractures and, 122 

placement for, 121 

surgery and, 116-117 
Lower limb claudication 

iliac artery occlusive disease, 147-155 

imaging modality for, 148 

interventions for, 148 

revascularization and, 151 
Lower limb ulcer, 423 
Low-flow ischaemia, 281-285 

extrinsic artery compression and, 283 
Low-molecular-weight heparin (LMWH), 

414 
LSV. See Long saphenous vein 
Lupus anticoagulant, 398 
Lymph accumulation pitting, 466 
Lymphangiography, 454 

CVM and, 388 
Lymphangioscintigram, 455, 457 
Lymphatic defects, 383 
Lymphatic disruption, 354-355 
Lymphatic drainage, 461 
Lymphatic dysfunction, 460-461 
Lymphatic malformation (LM), 379 

VM and, 388 
Lymphatic-venous anastomoses 

microsurgery, 455 
Lymphodynamic abnormalities, 458 



Lymphoedema, 453-464 

breast cancer and, 467 
management of, 466 

classification of, 453-454, 459 

clinical appearance of, 454 

complete reduction of, 470 

congenital, 458 

derivative surgery for, 457 

description of, 459, 472 

diagnosis of, 454 

forms of, 457-458 

long-term microsurgery outcome for, 
456 

lymph transport capacity and, 472 

management of, 456 

microsurgery and, 473-474 

vs. phleboedema, 459-460 

pitting and, 465-466 

postoperative recovery for, 456 

primary, 457-458 

prophylaxis and, 473 

reconstructive surgery for, 457 

secondary, 457, 467 
aetiology of, 458-459 
prevention of, 462 

staging of, 462, 472 

treatment advances for, 461 
Lymphogenesis disorders, 458 
Lymphography, 388 
Lymphoscintigraphic study, 387 
Lymphostatic disease, 461 
Lymph transport capacity, 472, 475 



M 

Magnetic resonance angiography (MRA), 

50,67 
of carotid body tumour, 273 
PAEand, 174 
of popliteal occlusion, 176 
Magnetic resonance imaging (MRI), 

273-274, 274 
TAA and, 50 
Management of carotid bifurcation disease, 

251-258 
Management of chronic ischaemia of lower 

extremities, 131-211 
Management of complications of arterial 

vascular surgery, 321-341 
Management of extracranial 

cerebrovascular disease, 249-285 
Management of portal hypertension, 

237-248 
Management of venous disorders, 393-450 
Manual lymphatic drainage, 461 



490 



Index 



Marginal manidublar branch of facial 

nerve, 264 
Mastectomy, 472 
MayalPs syndrome, 460 
May-Thurner syndrome, 419 
Medial gastrocnemius muscle, 179 
Melaena, 338, 339 

Mesenteric venous thrombosis (MVT), 
225 

thrombolysis and, 228 
Metabolic syndrome features, 134 
MI. See Myocardial infarctions 
Microsurgery 

elastic stockings after, 461 

lymphangioscitigram after, 457 

lymphangioscitigram before, 455 

lymphoedema and, 473-474 

lymphoedema long-term outcome from, 
456 

methods of, 474 
Mid-gut infarction, 227 
Midline laparotomy, 41 
Minor venous collaterals, 433 
Mortality predictive factors, 18 
MRA. See Magnetic resonance angiography 
MRI. See Magnetic resonance imaging 
Mural thrombosis, 47 
Muscle flap, 329 

MVT. See Mesenteric venous thrombosis 
Myocardial infarctions (MI), 13 

anastomotic aneurysm and, 79 

arterial embolism and, 107 

beta-blockers and, 8 

PAD and, 133 

perioperative, 3 

Stanford B dissection, 59 

TAA and, 45 
Myocardial ischemia, 7 

prolonged, 8 
Myopathic statin use, 7 



N 

Nailfold microscopy, 315 

NASCET. See North American Carotid 

Endarterectomy Trial 
Neck paragangliomas, 267-276 
Neonatal hemangioma, 391 
Nephrology work-up, 345-346 
Neurogenic thoracic outlet syndrome, 
289-295 

aetiology of, 293 

atrophy of hand muscles and, 293 

coexisting conditions and, 291 

diagnosis of, 292-293 



diagnostic criteria of, 290 
therapy for, 293 
treatment of, 289-290, 292-293 
Neurovascular conditions of upper 

extremity, 287-319 
Nifedipine, 316 
NOMI. See Non-occlusive mesenteric 

ischaemia 
Noninvasive vascular evaluation of 

popliteal artery, 173 
Nonne-Milroy's disease, 458 
Non-occlusive mesenteric ischaemia 
(NOMI), 225 
patchy ischaemia and, 228 
North American Carotid Endarterectomy 
Trial (NASCET), 255 





Obesity, 13 

Obturator foramen bypass (OFB), 191-200 

angiography of, 194 

complications of, 193-194 

concept of, 196 

indications for, 193, 196 

performance of, 193 

perioperative complications of, 198 

postoperative antibiotic treatment of, 
198 

preoperative measures for, 195 

principle of, 197 

retroperitoneal access for, 196 

technique of, 196 
Occlusive disease, 97 

classification of acute, 103 

diagnosis of arterial, 103-104 

embolic, 103 

hourglass-shaped, 182 

of iliac, 104 

lower limb claudication and, 147-155 

of penis, 158 

popliteal artery bypass and, 161 

profunda branch and, 144-145 

thrombotic, 103 
OctreoScan of carotid body tumour, 270 
Oedema 

adipose tissue and, 467, 473 

lymphoedema and, 453 

preoperative, 468 

volume of, 462 
OFB. See Obturator foramen bypass 
Optimal conduit for hemodialysis access, 
345-357 

options for, 351-356 

treatment options for, 346 



Index 



491 



Osteomyelitis 

detection of, 201-202 
presence of, 202 



PAE. See Popliteal artery entrapment 
Paget-Schrotter syndrome, 307 
PAOD. See Peripheral arterial occlusive 

disease 
Para-aortic tissue, 60-61 
Paraesthsia, 59, 173 
Paragangliomas 

benign tumors and, 272, 275 

diagnosis of, 268 

of the head and neck, 267-276 

types of, 272 
Paraplegia 

cross-clamp time and, 52 

development of, 53 

postoperative, 52 

TAA repair and, 52-53 
Patchy ischaemia, 228 
Patent aortorenal venous graft, 76 
Pedal pulse, 416 
Pelvic organs, 197 
Penile brachial occlusion, 158 
Percutaneous angiogram, 168 
Percutaneous aspiration thrombolectomy, 

112 
Percutaneous revascularization, 10-11 

CABGand, 11 
Percutaneous transluminal angioplasty 
(PTA), 142, 188 

complications of, 145, 234 

follow up after, 145 

lower limb claudication and, 148 

renal artery stenosis and, 234 

renovascular hypertension and, 232 

SFA and, 144 
Percutaneous transluminal coronary 

angioplasty (PTCA), 3, 10 
Perforator vein management, 425 
Perioperative morbidity, 3 
Peripheral aneurysm, 70 
Peripheral arterial disease (PAD) 

cardiovascular risk factors and, 133-140 

long-term management of, 169 

perioperative management of, 169 

renal function and, 139 

risk factors for, 136-138 
Peripheral arterial occlusive disease 

(PAOD), 144 
Peripheral embolism, 110-111 
Peripheral vascular disease, 151 



Peritoneum, 197 

Permissive hypotensive strategy, 40 

Phleboedema, 459-460 

Phlebography 

ascending, 379 

of common iliac vein, 444 

control, 307 

thrombolysis, 308 

thrombosis and, 305 
Physical therapy, 294 
Pitting, 317,472-473 

test, 465-466 
Plexus injury, 294 
Pneumothorax, 125 
Popliteal artery 

adventitial cystic disease of, 181-189 
segmental resection of, 184 
treatment options for, 184 

angiography of, 185 

cyst surrounding, 186 

disruption of, 117 

embolic occlusion of, 112 

hourglass stenosis of, 182 

left occlusion in, 175 

normal, 175 

occlusion of, 367 

percutaneous clot lysis of, 189 

resected, 187 

sever stenosis of, 175 
Popliteal artery aneurysm, 67-72 

bilateral, 69 

demographics of, 70 

diagnosis of, 70 

diagnostic test for, 67 

operation indications of, 71 

operative treatment for, 68 

rupture of, 71 

surgical treatment for, 69, 71 

symptoms of, 70 

treatment of, 70 

treatment of acute thrombosis of, 70 
Popliteal artery bypass, 161-166 

indications for, 161 

patency for, 162 
long-term, 163 
Popliteal artery entrapment (PAE), 173-179 

adventitial cystic disease and, 188 

angiogram of, 175 

classification of, 178 

demography of, 178 

diagnosis of, 174, 178 

incidence of, 177 

presentation of, 173 

surgical treatment of, 178-179 

treatment of, 176 



492 



Index 



Popliteal cyst 

percutaneous aspiration of, 188-189 

spontaneous resolution of, 189 
Popliteal fossa operative exploration, 69 
Popliteal pulse, 207 
Popliteal venous ulcer, 415 
Popliteopedal reconstruction, 205 
Population screening, 30 

AAA and, 17 
Portal hypertension 

aetiology of, 244-245 

complications of, 244 

management of, 237-248 
Posterior circulation territory, etiology of 

infarction in, 278 
Postoperative retriperitoneal bleeding, 62 
Postprandial abdominal pain, 217 
Post-thrombotic chronic venous 

insufficiency, 440 
Post-thrombotic sequelae, 448 
Post-thrombotic syndrome, 419 
Postural vertebrobasilar ischaemia, 

283-285 
Pregnancy 

iliofemoral DVT and, 439-450 

varicose veins and, 403 
Preoperative cardiac risk assessment for 
abdominal aortic aneurysm, 3-12 
Preoperative coronary revascularization, 7 
Preoperative duplex ultrasound 

examination, 346 
Preoperative measures for OFB, 195 
Prevention of complications of arterial 

vascular surgery, 321-341 
Primary amputation, 371 

of ischaemic limb, 369 
Primary axillary/subclavian thrombosis, 

308 
Primary deep venous reflux, 418-419 

correction of, 419 
Primary hyperhydrosis, 299 
Primary lymphoedema, 457-458 
Primary palmar hyperhydrosis natural 

history, 299 
Primary varicose veins, 403-41 1 

associations with, 408 

distribution of, 408-409 

indications for treatment of, 430 

LSV and, 407 

pregnancy and, 403 

progression of, 405 

recurrent, 410 
surgery for, 410 
treatment options for, 407 

sclerotherapy for, 410 



SSV and, 407 

surgical treatment for, 405 

postoperative complications of, 409-410 

symptoms of, 407 

treatment of, 409, 428 
Profunda femoral artery exposure, 197-198 
Prophylactic management 

ofDVT,396,399 

of gastroesophageal varices, 241-244 

of lymphoedema, 473 

of moderate to large varices, 245-246 
Prosthetic graft patency rates, 164 
Prosthetic interposition, 356 
Prosthetic vascular reconstructions, 81 
Protein C, 399 
Proximal access construction, steal 

incidence and, 360 
Pseudoaneurysm, 81 
PTA. See Percutaneous transluminal 

angioplasty 
PTCA. See Percutaneous transluminal 

coronary angioplasty 
Pulmonary dysfunction, 52 
Pulmonary embolism 

iliofemoral DVT and, 441, 448 

spinal cord injury and, 400 

venous system interruption and, 437 
Pulmonary microvascular injury, 52 
Pulsatile bleeding, 350 
Pulsatile mass, 39 

bilateral groin and, 79 



Quality of life, 151-152 



RAA. See Renal artery aneurysm 
rAAA. See Abdominal aortic aneurysm 

(AAA), ruptured 
Radiobasilic AVF, 355 
Radiocephalic AVF, 353 
Radiofrequency derived heating, 425 
RAR. See Renal artery repair 
Raynaud's phenomenon, 313-319 

onset of primary, 317 

pathophysiology of, 316 

symptoms of, 313 

treatment of, 318 

types of, 316 

vascular surgery for, 318 
Raynaud's syndrome, 297 
RCA. See Right coronary artery 



Index 



493 



Recombinant hirudin, 401 
Rectus femoris muscle flap, 329 
Recurrent laryngeal nerve, 263-264 

bilateral injury to, 265 
Recurrent visceral ischaemia, 219 
Renal artery aneurysm (RAA), 73-78, 149 

aetiology of, 73-74 

arterial hypertension and, 11 

complications of, 11 

management of, 75 

rare causes of, 16 

rupture of, 76 

spontaneous course of, 74 

symptoms of, 75-76 
Renal artery PTA complications, 232 
Renal artery repair (RAR) 

indications of, 75 

method of, 11 
Renal artery stenoses 

AAA and, 17-18 

treatment options for, 232, 234 
Renal disease, 345 
Renal function 

deterioration of, 58 

PAD and, 139 

statins and, 139 
Renal insufficiency, 18 
Renal vein, 242 
Renovascular hypertension, 231-235 

angiography of, 232 

complications with, 234 

diagnosis of, 233 

symptoms of, 231 
Reperfusion syndrome, 109 

risk of, 111 
Residual limb, 372 
Rethrombosis risk, 446 
Retroperitoneal access, 196 
Retroperitoneal bleeding, 62 
Retrosternal pain, 57 
Revascularization 

antigrade, 217 

chronic visceral ischaemia and, 218-219 

diabetic foot and, 210 

of infected groin graft, 194, 198 

of limb, 369 

limb swelling and, 111-112 

lower limb claudication and, 151 

percutaneous, 10-11 

popliteal artery bypass and, 163 

preoperative coronary, 7 

procedures for, 371-372 

of tibial vessels, 169 
Reversible Ischaemic neurological defect 
(RIND), 259 



Right coronary artery (RCA), 4 
Right hand dominance, 347 
Robinul, 297 

Ruptured abdominal aortic aneurysm 
(rAAA), 35-43 

common complications of, 42 

complications after repair of, 38 

CT scan and, 3 

endovascular repair approach for, 38 

intraoperative risk factors of, 39 

open repair of, 41 

optimal treatment for, 39 

perioperative management of, 37-38 

preoperative measures for, 40-41 

prognosis factors of, 36 

symptoms of, 35, 39, 40 

ultrasound and, 37 
Rupture risk, 30 



Saphenopopliteal incompetence, 409 
Saphenous vein 

AVF and, 446 

bypass with, 280 

harvest of, 118 

PAEand, 179 
Scalene muscles, 290 
Sciatic nerve disruption, 117 
Scimitar sign, 183 

discrete semilunar deformity and, 185 

hourglass stenosis and, 181 

popliteal artery and, 182 
Sclerodactyly, 313-314, 317-318 
Scleroderma, 315 

of the fingers, 317-318 
Sclerotherapy, 410 
Scoliosis 

assessment of, 386 

CVM and, 380 

treatment of, 389 
Secondary abnormal long-bone growth, 

385 
Secondary lymphoedema, 457-459 

aetiology of, 458-459 
Sensory nerve damage, 264-265 
Seroma, 330 

SFA. See Superficial femoral artery 
Short saphenous vein (SSV), 404 

reflux of, 409 

varicose veins and, 407 
Shunt 

intraluminal, 118 
use of, 122 

Javid, 117-118 



494 



Index 



Sigmoid cancer, 395 

Sinus rhythm, 36 

SIRS. See Systemic inflammatory response 

syndrome 
Small-vessel bypass, 159 
Smoking history, 13 

carotid bifurcation disease and, 251 

cessation and, 137 

critical arterial stenosis and, 141 

hypertension and, 157 

lower limb claudication and, 147 

peripheral vascular disease and, 151 

popliteal artery bypass and, 161 
Society for Vascular Surgery (SVS), 177 
Sodium, 414 
Soft signs, 120 
Spinal accessory nerve, 264 
Spinal cord dysfunction 

pathophysiology of, 54 

after TAA repair, 48 
Spinal cord injury, 400 
Splenic artery injection, 241 
Splenic vein, 242 
Splenorenal shunt, 243 
Sports injuries, 281 
SSV. See Short saphenous vein 
Stanford A dissection, 57 

complications with, 58 
Stanford B dissection, 57 
Staphylococcus aureus graft infection, 

330-331 
Staphylococcus epidermidis graft infection, 

331 
Statins, 9-10 

benefits of perioperative use of, 9 

beta-blockers and, 6 

cholesterol levels and, 7 

limb ischemia and, 168 

myopathy and, 7 

pleiotropic effects of, 9 

renoprotective actions of, 139 

stroke risk and, 136 

therapy with, 7 

use of, 6 

vascular surgery and, 5-6 
Steal 

clinical signs of, 363 

correction of, 360 

induced ischaemia, 363 

proximal access construction and, 360 

surgical correction of, 363 
Stemmer's sign, 473 
Stenosis 

of axillary/subclavian vein, 307 

of carotid artery, 255 



central vein, 352 

endoluminal treatment for carotid, 265 

hourglass, 181 

ofICA,259 

iliac vein, 443 

of subclavian vein, 310 

of vein graft, 207 
Stenotic lesions on left internal iliac artery, 

149 
Stent graft, 16 

critical arterial stenosis and, 145 

EI A and, 152 

endovascular, 38 

intermittent claudication and, 154 
Sterilization, 332-333 
Sternocleidomastoid muscle, 267 
Stress test 

cardiac, 48 

echocardiography and, 6 

stages of, 6 
Stroke 

hemispheric, 253 

statins and, 136 
Stump 

healing of, 370 

well-healed, 371 
Subclavian artery selective injection, 283 
Subclavian vein 

occlusion of, 306 

residual stenosis of, 310 
Subclavian vein electrodes, 354 
Superficial femoral artery (SFA), 102 

critical stenosis of, 141 

disease of, 167-168 

popliteal artery bypass and, 161 

PTA and, 144 

stenosis of, 143 
Superficial femoral vein dissection, 327 
Superficial thrombophlebitis, 408 
Superficial ulcer, 201 
Superficial vein operations, 430 
Superficial venous insufficiency, 423-432 
Superior laryngeal nerve location, 263 
Superior mesenteric artery (SMA) 

angioplasty of, 219 

atherosclerotic occlusion of, 227 

exposure of, 226 

pulse at origin of, 227 

pulse in proximal, 223 

thrombosis of, 224-225 
Supraceliac aortic clamping, 53 
Supraceliac balloon occlusion, 41 
Supraceliac clamp, 38 
Supraclavicular tenderness, 290 
Suprarenal vena cava filter, 442 



Index 



495 



Surgery 

blast injury and, 116-117 

carotid body tumour classification of, 275 

of carotid body tumour complications, 
271 

claudication and, 177 

for decompression of thoracic outlet 
areas, 291 

deep venous, 418 

for lymphoedema, 457 

of major branches of infradiaphragmatic 
aorta, 213-235 

PAE and, 178-179 

steal correction and, 363 

thrombectomy and, 435 

thrombolysis and, 105 

for ulcers, 428 

for varicose veins, 409 

vascular techniques and, 169-170 
SVS. See Society for Vascular Surgery 
Symptomatic knee lesions, 379-380 
Symptoms of r AAA, 35 
Synovial mesothelium cyst wall, 185-187 
Systemic heparinisation, 308 
Systemic inflammatory response syndrome 

(SIRS), 226-227 
Systemic sclerosis, 315 

digital pulp calcinosis and, 318 

fingertip ulcer and, 314 

limited cutaneous, 315 

pitting and, 317 

primary Raynaud's phenomenon, 318 

vascular abnormalities and, 319 



TAA. See Thoracoabdominal aortic 

aneurysm 
Tachycardia, 433 
TASC. See TransAtlantic Inter-Society 

Consensus 
Telangiectases, 403 

varicose veins and, 408 
Temporal pulse, 251 
Thoracic aorta aneurysmal dilation, 47 
Thoracic outlet areas 
decompression of 
indications for, 291 
major complications of, 294 
surgical decompression of, 310 
long-term results of, 292 
physical therapy and, 294 
surgical procedures for, 291 
Thoracic outlet syndrome (TOS) 
coexisting conditions with, 291 



complications of, 292 

neurogenic, 289-295 
aetiology of, 193 
cause of, 290 

conservative therapy for, 293-294 
diagnosis of, 292-293 
diagnostic criteria of, 290 

plexus injury and, 294 

types of, 292 
Thoracoabdominal aortic aneurysm (TAA), 
45-55 

chest pain and, 45 

common complications after, 48 

Crawford classification system for, 46, 50 

demographics of, 49 

infrarenal abdominal aortic aneurysm 
and, 49 

management schemes for, 48 

natural history of, 50-51 

pulmonary microvascular injury and, 52 

repair of, 51, 53 

complications with, 52 

spinal cord dysfunction following, 48 

spinal cord dysfunction and, 54 

stages of, 51 

studies of, 46 

work-up for, 49-50 

x-ray of, 46 
Thoracoscopic sympathectomy, 297-303 

in children, 301 

failure of, 301 

indications for, 298 

long-term complications of, 298, 300 

short-term complications of, 298, 300 

use of, 299-300 
Thoracotomy, 51 
3-Hydroxy-3-Methylglutaryl coenzyme A 

reductase inhibitors. See Statins 
Thrombectomy, 205 

considerations during, 442-443 

iliofemoral, 443 

successful, 449 

vena caval, 443 
Thrombin injection, 89-91 

aneurysm thrombosis and, 90 

topical use of, 92 

ultrasound-guided, 92 
Thromboemboli, 110 
Thrombolysis, 100-101 

for acute DVT, 397-398 

acute thrombosis and, 104-105 

bypass graft occlusions and, 164 

complete, 102 

dissolution from, 101 

iliofemoral venous thrombosis and, 436 



496 



Index 



Thrombolysis-conf. 

MVT and, 228 

oral anticoagulation and, 310 

phlebegraphy and, 308 

revascularization and, 371-372 

surgery and, 105 
Thrombophilia, 308 

evaluation for, 447, 450 

prevalence of, 427 
Thrombosis 

of axillary/subclavian vein, 305-3 1 1 

deep vein, 119,395-402 

graft, 97 

hemodialysis and, 348 

phlebography and, 305 

primary and, 307 

recurrent, 446 

of right iliac, 98 

secondary, 307-308 

treatments for acute, 99 
Thrombotic events, 102 
Thrombotic occlusion, 101 
Thrombus 

natural history of, 447-448 

nonocclusive, 441 

reduction of, 436 

removal of, 448 
Thyroid function, 135-136 
TIA. See Transient ischaemic attacks 
Tibial vessel revascularization, 169 
Tibioperoneal artery, 168 
TIPS. See Transjugular intrahepatic portal 

systemic shunt 
Tissue perfusion, 115 

inadequate, 121 
TOS. See Thoracic outlet syndrome 
TransAtlantic Inter-Society Consensus 
(TASC), 153 

type A lesions, 153 

type C lesion, 154 
Transcommissural valvuloplasty, 418 
Transient ischaemic attacks (TIA), 256, 277 

carotid bifurcation disease and, 252 
Transjugular intrahepatic portal systemic 
shunt (TIPS), 239 

vs. DSRS, 247 

outcome of, 246-247 
Transthoracic echocardiography, 225 
Transtibial amputation, 370 

critical ischaemia and, 373 
Traumatic arteriovenous fistula, 

endoluminal treatment of, 125-130 
Trendelenburg position, 406 

reversed, 424, 437 

varicose veins and, 404 



Triglyceride 

fasting cycle for, 134-135 

HDL-C levels and, 136 

levels of, 135 
Troponin elevation, 10 
Tubular necrosis, 351 
Tulip configuration of carotid body 

tumour, 269 
Type 2 diabetes, 201 



u 

UGCR. See Ultrasound-guided compression 

repair 
Ulcer 

of ankle, 413 

examination of, 207 

of fingertip, 313 

on heel 

debridement of, 368 
treatment of, 367-378 

radiofrequency derived heating of, 425 

superficial, 201 

surgical treatment of, 428 
Ultrasonography, 30, 67 

graft failure and, 165 
Ultrasound 

abdominal, 110 

AVF and, 126-127 

false aneurysm and, 88 

intravascular, 128 

liver disease and, 245 

preoperative duplex, 346 

rAAA and, 37 
Ultrasound-guided compression repair 
(UGCR), 89 

disadvantages of, 89, 92 

method of, 91 

thrombin and, 89, 93 
United States Kidney Dialysis Outcomes 
Quality Initiative (K-DOQI), 353 
Upper extremity 

lymphoedema of, breast cancer and, 
465-476 

neurovascular conditions of, 287-319 
Urokinase dosage, 105 



V 

Vanectasia, 379 
Variceal bleeding 
acute, 239-240 

management of, 246-247 

mortality of, 244 



Index 



497 



recurrent 

management of, 247 
prevention of, 240, 246 
Varicose veins, 403-411 

distribution of, 408-409 

indications for treatment of, 430 

LSV and, 407 

progression of, 405 

recurrent, 410 
surgery for, 410 
treatment options for, 407 

sclerotherapy for, 410 

SSV and, 407 

superficial thrombophlebitis of, 408 

surgical treatment for, 405 

postoperative complications of, 409-410 

symptoms of, 407 

treatment of, 409, 428 
Varicosity's. See Varicose veins 
Vascular abnormalities, 319 
Vascular access, 343-364 
Vascular amputee life expectation, 374 
Vascular defects, 383 

ET form of, 383 
Vascular disease, 13, 97 
Vascular examination, 351 
Vascular graft 

infection of, 192 
management of, 198 
treatment of, 192 

routing of, 196 
Vascular injury 

hard signs for, 115-116 

signs of, 120 
Vascular malformation 

clinical evaluation of, 378-380 

congenital, 377-392 
classification of, 383 
clinical presentation of, 382-383 
multidisciplinary approach to, 389 

diagnosis of, 377-378 

treatment of, 377-378 
Vascular reconstruction of diabetic foot, 205 
Vascular surgery 

aims of, 121 

amputation and, 374-375 

beta-blockers and, 5-6 

cardiovascular events and, 9 

perioperative statin and, 7 

postoperative outcome of, 3, 5 

for Raynaud's phenomenon, 318 

statins and, 5-6 
Vein grafts 

occlusion of, 150 

stenosis of, 207 



Vein repair, 117, 123 

Vein stent technology, 419 

Vena cava filter, 442 

Vena caval thrombectomy, 443 

Venography, 417 

Venolymphatic malformation (VLM), 

378-379 
Venous claudication, 416 
Venous defects, 383 
Venous disease 

limb swelling and, 417 

treatment of, 437 
Venous disorder management, 393-450 
Venous endothelial injury, 418 
Venous function, 417-418 
Venous hypertension, 408 
Venous injuries, 122-123 
Venous malformation (VM), 379 

diagnosis of, 382 

differential diagnosis of, 390-391 

emboloscleroagents and, 389-390 

ET form of, 386, 390 
blood pooling by, 388 

invasive investigations for, 381-382 

lesions of, 380, 384-385, 385 
therapeutic strategy for, 381 

LM and, 388 

management strategy of, 387 

secondary abnormal long-bone growth 
and, 385 

secondary phenomenon of, 389 

treatment of, 381 
ethanol and, 390 
precondition for, 382 
priority of, 389 
Venous obstructive symptoms, 355 
Venous patency restoration, 310 
Venous pressure, 417-418 
Venous severity scoring, 429 

venous ulcers and, 427 
Venous system interruption, 437 
Venous thrombectomy, 440, 443, 448 
Venous thromboembolism, 447 
Venous thrombosis, 433-438 
Venous ulcers 

arterial ulcers and, 415-416 

deep venous insufficiency and, 413-421 

deep venous thrombosis and, 426-427 

differential diagnosis of, 415 

evaluation of, 416 

investigation of, 417 

management for, 424-425 

pathophysiology behind, 430 

risk factors for, 424, 427 

role of surgery in, 429-430 



498 



Index 



Venous u\cers-cont. 

superficial venous insufficiency and, 
423-432 
Vertebral artery 
arteriogram of, 279 
dissection of 

deceleration injuries, 281 
sports injuries and, 281 
extrinsic compression of, 285 
preocclusive disease level of, 282 
Vertebral artery dissection 
diagnosis of, 279 
treatment of symptomatic, 281 
Vertebrobasilar ischemia 
arteriogram of, 278 
embolic mechanism of, 277-281 
low-flow mechanisms of, 281-285 
symptoms of, 285 



of postural origin, 284-285 

work-up of, 277 
Viscera 

inspection of, 228 

ischaemic, 226 
VLM. See Venolymphatic malformation 
VM. See Venous malformation 
Vocal cord paralysis, degenerative 
aetiology of, 264 



w 

Wall motion abnormalities, 4 
Warfarin therapy, 398 

breastfeeding and, 450 

sodium and, 414 
WBA. See Whole-blood autotransfusion 
Whole-blood autotransfusion (WBA), 19